Sei sulla pagina 1di 372

Problems and Solutions

in
Plane Trigonometry

For the use of Colleges and Schools

By

Isaac Todhunter

LATEX Edition

Edited By
Neeru Singh

Ancient Science Publishers


TEX is a trademark of the American Mathematical Society.
METAFONT is a trademark of Addison-Wesley.
Care has been taken in the preparation of this book, but makes no expressed or
test
implied warranty of any kind and assumes no responsibility for errors or omissions.
No liability is assumed for incidental or consequential damages in connection with or
arising out of the use of the information contained herein.
For comments, suggestions and or feedback, send mail to :
ancientsciencepublishers@gmail.com

Ancient Science Publishers


(ancientsciencepublishers@gmail.com)
Highly Recommended Books for Self Study and Competitions
Author : Chandra Shekhar Kumar
Conceptual Kinematics
A Companion to I. E. Irodov’s Problems in General Physics
Conceptual Geometry of Straight Line
A Companion to S. L. Loney’s Co-ordinate Geometry
Conceptual Geometry
A Companion to S. L. Loney’s Co-ordinate Geometry
Conceptual Trigonometry Part I
A Companion to S. L. Loney’s Plane Trigonometry Part I
Conceptual Trigonometry Part II
A Companion to S. L. Loney’s Plane Trigonometry Part II
Conceptual Dynamics
A Companion to S. L. Loney’s Elements of Dynamics
Conceptual Statics
A Companion to S. L. Loney’s Elements of Statics
Conceptual Particle Dynamics
A Companion to S. L. Loney’s Dynamics of A Particle
Conceptual Rigid Body Dynamics
A Companion to S. L. Loney’s Dynamics of Rigid Bodies
Conceptual School Geometry
A Companion to Hall & Stevens’ School Geometry
Conceptual School Algebra
A Companion to Hall & Knight’s Elementary Algebra
Solutions of the Examples in Higher Algebra
by
H. S. Hall, S. R. Knight, Neeru Singh & C. S. Kumar
Questions and Problems in School Physics
A Companion to I. E. Irodov’s Problems in General Physics
by
Lev Tarasov, Aldina Tarasova & Chandra Shekhar Kumar
Calculus
Basic Concepts for High Schools
by
Lev Tarasov & Chandra Shekhar Kumar
General Methods for Solving Physics Problems
A Companion to I. E. Irodov’s Problems in General Physics
by
B. S. Belikov & Chandra Shekhar Kumar
Copyright ©2018 Neeru Singh
All rights reserved. This work is protected by copyright and permission must be
obtained prior to any prohibited reproduction, storage in a retrieval system, or trans-
mission in any form or by any means, electronic, mechanical, photocopying, record-
ing, or likewise unless stated otherwise.
For permission, send mail to: ancientsciencepublishers@gmail.com
ISBN-13: 978-1533437433
ISBN-10: 1533437432
Preface

This book collects together the problems set out at end of each chapter in the author’s
Textbook of Plane Trigonometry (for the use of Colleges and Schools) along with the
possible solutions, which are linked with an explanation of the sort of reasoning used
in order to arrive at one of the answers. In many cases, several answers are given
for one question. The result is a book which can be used independently of the main
volume. This book helps in acquiring a better understanding of the basic principles
of Plane Trigonometry and in revising a large amount of the subject matter quickly.
The Keys already issued to some of the Author’s works have been found very useful
by affording assistance to private students, and by saving the labour and time of
teachers ; and this has led to the issue of the present volume. Care has been taken,
as in the former Keys, to present the solutions in a simple natural manner, in order to
meet the difficulties which are most likely to arise, and to render the work intelligible
and instructive.
St. John’s College, Isaac Todhunter.

LATEX Edition
In this edition, the entire manuscript was typeset using the LATEX 2ε document
processing system originally developed by Leslie Lamport, based on TEX typesetting
system created by Donald Knuth. The typesetting software used the XƎLATEX distribu-
tion.
It is also to be noticed, that each Example, or Problem (1000+) is here enunciated
at the head of its Solution as well as all the relevant articles (100+) are part of the
appendix; so that the book, though a fitting Companion to the textbook, is not insep-
arable from it, but may be used, as a Book of Exercises, with any other treatise on
Plane Trigonometry.
I am grateful for this opportunity to put the materials into a consistent format, and
to correct errors in the original publication that have come to my attention. I am
highly indebted to Chandra Shekhar Kumar for the fruitful discussions which led to
the idea of masterminding this entire project. He helped me put hundreds of pages
of typographically difficult material into a consistent digital format.
The process of compiling this book has given me an incentive to improve the layout,
to double-check almost all of the mathematical rendering, to correct all known errors,
to improve the original illustrations by redrawing them with Till Tantau’s marvellous
TikZ. Thus the book now appears in a form that we hope will remain useful for at least
another generation.

May, 2016. Neeru Singh.


List of Chapters

Preface i

CHAPTER I
Measurement of Angles by Degrees or Grades 1

CHAPTER II
Circular Measure of an Angle 5

CHAPTER III
Trigonometrical Ratios 7

CHAPTER IV
Application of Algebraical Signs 11

CHAPTER V
Angles with Given Trigonometrical Ratios 15

CHAPTER VI
Trigonometrical Ratios of Two Angles 17

CHAPTER VII
Formulae for the Division of Angles 27

CHAPTER VIII
Miscellaneous Propositions 35

CHAPTER IX
Construction of Trigonometrical Tables 53

CHAPTER X
Logarithms and Logarithmic Series. 63

CHAPTER XI
Use of Logarithmic And Trigonometrical Tables. 75

CHAPTER XII
Theory of Proportional Parts 83

CHAPTER XIII
Relations between the Sides of a Triangle and the Trigonometrical
Functions of the Angles 99

CHAPTER XIV
Solution of triangles 113

CHAPTER XV
Measurement of Heights and Distances 123

CHAPTER XVI
Properties of Triangles 137

CHAPTER XVII
Use of Subsidiary Angles in solving Equations and in adapting Formu-
lae to Logarithmic Computation 157
CHAPTER XVIII
Inverse Trigonometrical Functions 165

CHAPTER XIX
De Moivre’s Theorem 177

CHAPTER XX
Expansions of Some Trigonometrical Functions 183

CHAPTER XXI
Exponential Values of the Cosine and Sine 189

CHAPTER XXII
Summation of Trigonometrical series 193

CHAPTER XXIII
Resolution of Trigonometrical Expressions Into Factors 207

CHAPTER XXIV
Miscellaneous Propositions 217

CHAPTER XXV
Miscellaneous Examples 227

APPENDIX
Miscellaneous Articles and Propositions 315
CHAPTER I
Measurement of Angles by Degrees or Grades

Problem 1. The difference of two angles is 10 grades and their sum is 45 degrees;
find each angle.
Solution. Let x denote the number of degrees in the larger angle, and y the number
of degrees in the smaller angle. Then, since 10 grades are equal to 9 degrees, x−y = 9;
also x + y = 45 : hence we obtain x = 27 and y = 18.

Problem 2. Divide two-thirds of a right angle into two parts, such that the number
of degrees in one part may be to the number of grades in the other part as 3 is to 10 .
Solution. In two-thirds of a right angle there are 60 degrees; let x denote the number
of degrees in one part, then 60 − x denotes the number of degrees in the other part,
10
therefore the number of grades in this part is (60 − x).
9
Hence
10 30
x: (60 − x) :: 3 : 10; therefore 10x = (60 − x);
9 9
therefore 9x = 3(60 − x); therefore 12x = 180; therefore x = 15.

Problem 3. Divide half a right angle into two parts, such that the number of degrees
in one part may be to the number of grades in the other part as 9 is to 5 .
Solution. In half a right angle there are 45 degrees; let x denote the number of
degrees in one part, then 45 − x denotes the number of degrees in the other part,
10
therefore the number of grades in this part is (45 − x). Hence
9
10
x: (45 − x) :: 9 : 5; therefore 5x = 10(45 − x);
9
therefore 15x = 450; therefore x = 30.

Problem 4. Find the measure of 1 ′ 5 ′′ in decimals of a degree.


9
Solution. 1′ 5′′ = ·0105 of a grade; of ·0105 = ·00945.
10

Problem 5. Divide an angle of n degrees into two parts, one of which contains as
many English minutes as the other does French.
Solution. Let x denote the number of degrees in one part; then n − x denotes the
number of degrees in the other part. In x degrees there are 60x English minutes. In
10 10
n−x degrees there are (n−x) grades, and therefore ×100(n−x) French minutes.
9 9
Therefore
1000
60x = (n − x);
9
therefore 1540x = 1000n; therefore 77x = 50n;
50n 27n
therefore x= , and n − x = .
77 77

Problem 6. If one-third of a right angle be assumed as the unit of angular measure,


what number will represent 75 ◦ ?
Solution. In one-third of a right angle there are 30 degrees; if this be taken as the
75 5
unit of measurement an angle of 75 degrees must be denoted by , that is by , that
30 2
is by 2 21 .
CHAPTER I : Measurement of Angles by Degrees or Grades 2

Problem 7. Determine the number of degrees in the unit of angular measure when
an angle of 66 23 grades is represented by 20 .
Solution. Let x denote the number of grades in the unit. Then an angle of 66 32 grades
66 23
is denoted by ; and this is equal to 20. Therefore
x
2 200 10
20x = 66 = ; therefore x = .
3 3 3
9 10
Hence the number of degrees in the unit is × , that is 3.
10 3

Problem 8. The number of the sides of one equiangular polygon is two-thirds of the
number of the sides of another; and the number of grades in an angle of the first
equals the number of degrees in an angle of the second : find the angles.
Solution. Let 3x denote the number of sides in the equiangular polygon which has the
greater number of sides; then 2x denotes the number of sides in the other equiangular
polygon. All the angles of the polygon of 2x sides are equal to (4x − 4) right angles,
(4x − 4)100
that is to (4x − 4)100 grades; therefore each angle contains grades. All the
2x
angles of the polygon of 3x sides are equal to (6x − 4) right angles, that is to (6x − 4)90
(6x − 4)90
degrees ; therefore each angle contains degrees; therefore
3x
(4x − 4)100 (6x − 4)90
= ;
2x 3x
therefore (4x − 4)5 = (6x − 4)3; therefore 2x = 8; therefore x = 4. Thus one polygon
has 8 sides and the other polygon has 12 sides.

Problem 9. Show that an angle expressed in centesimal seconds will be transformed


to sexagesimal by multiplying by the factor ·324 .
Solution. It is shown in Art. 9 (page 315) that an angle expressed in centesimal
81 81 324
seconds is transformed to English seconds by multiplying by ; and = .
250 250 1000

Problem 10. Compare the angles which contain the same number of English seconds
as of French minutes.
Solution. Suppose one angle to contain x English seconds, and another to contain x
French minutes. The second angle then contains 100x French seconds, and therefore
81
× 100x English seconds. Hence the ratio of the former angle to the latter is that
250
8100 162
of 1 to , or of 1 to , or of 5 to 162.
250 5

Problem 11. Express in the French method 35 ◦ 10 ′ 3 ′′ .


Solution.
60| 3·00
60|10·05
·1675
Thus 35◦ 10′ 3′′ = 35◦ · 1675.

35·1675
3·9075
39·0750

And 39g · 0750 = 39g 7′ 50′′ .

Problem 12. Express in the English method 69 g 22 ′ 50 ′′ .


Solution. 69g 22′ 50′′ = 69g · 225.
CHAPTER I : Measurement of Angles by Degrees or Grades 3

69 · 225
6 · 9225
62 · 3025
60
18 · 1500
60
9 · 00
CHAPTER II
Circular Measure of an Angle

Problem 1. If D, G, C be respectively the number of degrees, grades, and units of


circular measure in an angle, show that
D G 2C
= = .
90 100 π
D G
Solution. It is shown in Art. 8 (page 315) that = ; and it is shown in Art. 22
90 100
D C D 2C D G 2C
(page 315) that = , so that = . Therefore = = .
180 π 90 π 90 100 π
In fact the three expressions denote the same thing, namely the ratio of the angle
considered to a right angle.

Problem 2. Find the number of degrees in the angle subtended at the centre of a
circle whose radius is 10 feet by an arc 9 inches long.
9 3
Solution. The circular measure of the angle is , that is . Therefore by Art. 22
10 × 12 40
3 180
(page 315), the number of degrees in the angle is of .
40 π

Problem 3. Find the circular measure of 5 ◦ 37 ′ 30 ′′ .


Solution. 5◦ 37′ 30′ = 337 21 minutes.
Thus the circular measure
337 12 675 135
= π= π= π
180 × 60 180 × 60 × 2 180 × 12 × 2
27 π
= π= .
36 × 12 × 2 32

Problem 4. Find the circular measure of 1 g 1 ′ .


Solution. The angle contains 1·01 grades; therefore by Art. 24 (page 316), the circular
1·01
measure is π, that is π × ·00505.
200

Problem 5. There are three angles; the circular measure of the first exceeds that of
π
the second by , the sum of the second and the third is 30 grades, and the sum of
10
the first and the second is 36 degrees. Determine the three angles.
Solution. Let x denote the number of degrees in the first angle, y the number in the
second, and z the number in the third.

The circular measure of the first angle is , and the circular measure of the
180
yπ xπ yπ π
second is ; therefore − = ; therefore x − y = 18.
180 180 180 10
10y
The number of grades in the second angle is , and the number of grades in the
9
10z 10y 10z
third is ; therefore + = 30; therefore y + z = 27.
9 9 9
Also x + y = 36.
From these three equations we have x = 27, y = 9, z = 18.

Problem 6. Express five-sixteenths of a right angle in circular measure, in degrees


and decimals of a degree, and in grades and decimals of a grade.
π
Solution. The circular measure of a right angle is ; and therefore the circular
2
5 π 5π
measure of five-sixteenths of a right angle is of , that is
16 2 32.
CHAPTER III : Trigonometrical Ratios 6

5 450
The number of degrees is of 90, that is , that is 28·125.
16 16
5 500
The number of grades is of 100, that is , that is 31·25.
16 16

Problem 7. The angles of a triangle are in arithmetical progression, and the greatest
is double the least : express the angles in degrees, in grades, and in circular measure.
Solution. Let the numbers of degrees in the three angles be denoted respectively by
x − y, x, and x + y. Then x − y + x + x + y = 180, that is 3x = 180; therefore x = 60.
Also x + y = 2(x − y); therefore 3y = x = 60; therefore y = 20.
Hence in degrees the angles are denoted by 40, 60, and 80. Therefore in grades they
400 600 800
will be denoted by , , and . And in circular measure they will be denoted
9 9 9
40π 60π 80π 2π π 4π
by , , and ; that is by , , and .
180 180 180 9 3 9

Problem 8. The angles of a triangle are in arithmetical progression, and the number
of degrees in the least is to the circular measure of the greatest as 60 is to π : find the
angles.
Solution. Let the numbers o degrees in the three angles be denoted respectively by
x − y, x, and x + y. Then x − y + x + x + y = 180, that is 3x = 180; therefore x = 60.
(x + y)π
The circular measure of the greatest angle is ; thus
180
(x + y)π (x + y)π
x−y : :: 60 : π; therefore (x − y)π = ;
180 3
x
therefore 3(x − y) = x + y; therefore y = = 30.
2
Thus the angles are 30◦ , 60◦ , and 90◦ .

Problem 9. Find the circular measure of an angle of an equiangular polygon of n


sides.
Solution. All the angles of the polygon are equal to (2n − 4) right angles, that is to
π
(2n − 4) in circular measures, that is to (n − 2)π. Hence the circular measure of each
2
(n − 2)π
angle is .
n

Problem 10. Express in each system of angular measurement the angle between the
long hand and the short hand of a watch at a quarter past twelve.
Solution. During the quarter of an hour since twelve the long hand has described
one-fourth of four right angles, that is a right angle. The short hand has described
1
one-twelfth of this, that is of a right angle. Hence the angle between the hands at
12
11
a quarter past twelve is of a right angle.
12
11 11 × 15 165 1
The measure in degrees = of 90 = = = 82 .
12 2 2 2
11 11 × 25 275 2
The measure in grades = of 100 = = = 91 .
12 3 3 3
11 π 11π
The circular measure = of = .
12 2 24
CHAPTER III
Trigonometrical Ratios

3
Problem 1. The sine of a certain angle is ; find the other Trigonometrical Ratios of
5
the angle.
3
Solution. Let sin A = . Then we have
5 √ √
√ 9 16 4
cos A = 1− sin2
A= 1− = = .
25 25 5
sin A 3 4 3 5 3
tan A = = ÷ = × = ;
cos A 5 5 5 4 4
1 4
cot A = = ;
tan A 3
1 5 1 5
sec A = = ; cosec A = = ;
cos A 4 sin A 3
4 1
vers A = 1 − cos A = 1 − = .
5 5

4
Problem 2. The tangent of a certain angle is ; find the other Trigonometrical Ratios
3
of the angle.
4
Solution. Let tan A = . Then we have
3
4
tan A 4 5 4
sin A = √ = √
= ÷ = ; 3
1 + tan2 A 16 3 3 5
1+
9
1 1 5 3
cos A = √ = √ =1÷ = ;
2
1 + tan A 16 3 5
1+
9
1 5 1 5
sec A = = ; cosec A = = ;
cos A 3 sin A 4
3 2
vers A = 1 − cos A = 1 − = .
5 5


2
Problem 3. The cosine of a certain angle is ; find the other Trigonometrical
3
Ratios of the angle. √
2
Solution. Let cos A = . Then we have
3 √ √
√ 2 1
sin A = 1 − cos2 A = 1− = ;
3 3
√ √
sin A 1 2 1
tan A = = ÷ = √ ;
cos A 3 3 2
1 √
cot A = = 2;
tan A √
1 3 1 √
sec A = = ; cosec A = = 3;
cos A 2 sin A

2
vers A = 1 − cos A = 1 − .
3
CHAPTER III : Trigonometrical Ratios 8

Problem 4. Show that sec2 θ cosec 2 θ = tan2 θ + cot2 θ + 2 .


Solution. ( )( )
sec2 θ cosec 2 θ = 1 + tan2 θ 1 + cot2 θ = 1 + tan2 θ + cot2 θ + (tan θ cot θ)2
= 1 + tan2 θ + cot2 θ + 1 = tan2 θ + cot2 θ + 2.

Problem 5. Show that


sin2 θ tan θ + cos2 θ cot θ + 2 sin θ cos θ = tan θ + cot θ.
Solution.
sin3 θ cos3 θ
sin2 θ tan θ + cos2 θ cot θ + 2 sin θ cos θ = + + 2 sin θ cos θ
cos θ ( sin θ )2
sin4 θ + cos4 θ + 2 sin2 θ cos2 θ sin2 θ + cos2 θ 1
= = =
sin θ cos θ sin θ cos θ sin θ cos θ
sin2 θ + cos2 θ sin θ cos θ
= = + = tan θ + cot θ.
sin θ cos θ cos θ sin θ

( ) ( )
Problem 6. Show that 2 sin6 θ + cos6 θ − 3 sin4 θ + cos4 θ + 1 = 0 .
Solution. ( ) ( )( )
2 sin6 θ + cos6 θ = 2 sin2 θ + cos2 θ sin4 θ − sin2 θ cos2 θ + cos4 θ
( )
= 2 sin4 θ − sin2 θ cos2 θ + cos4 θ ;
( ) ( )
therefore 2 sin6 θ + cos θ − 3 sin4 θ + cos4 θ + 1
6

= −2 sin2 θ cos2 θ − sin4 θ − cos4 θ + 1


( )2
= 1 − sin2 θ + cos2 θ = 1 − 1 = 0.

Obtain solutions of the following equations :

Problem 7. 2 sin2 θ = 3 cos θ.


Solution.
3 3
sin2 θ = cos θ; therefore 1 − cos2 θ = cos θ;
2 2
3
therefore cos2 θ + cos θ = 1.
2
1
By solving this quadratic in the usual way we obtain cos θ = or −2; but only the
2
former value is applicable, for cos θ cannot be numerically greater than unity. Hence
1 π
cos θ = , and therefore θ = .
2 3

Problem 8. sin θ + cos θ = 1 .


Solution. sin θ + cos θ = 1; therefore cos θ = 1 − sin θ; therefore cos2 θ = (1 − sin θ)2 ,
therefore 1 − sin2 θ = (1 − sin θ)2 , that is (1 − sin θ)(1 + sin θ) = (1 − sin θ)2 . Therefore
either 1 − sin θ = 0, or 1 + sin θ = 1 − sin θ.
π
Take 1 − sin θ = 0; thus sin θ = 1, therefore θ = .
2
Next take 1 + sin θ = 1 − sin θ; thus sin θ = 0, therefore θ = 0.

Problem 9. cot θ = 2 cos θ.


Solution.
cos θ
cot θ = 2 cos θ; therefore = 2 cos θ.
sin θ
1
Therefore either cos θ = 0, or = 2.
sin θ
π 1 1 π
Take cos θ = 0; then θ = . Next take = 2; thus sin θ = ; therefore θ = .
2 sin θ 2 6
CHAPTER III : Trigonometrical Ratios 9

1
Problem 10. sin2 θ − 2 cos θ + = 0.
4
1 1
Solution. sin2 θ − 2 cos θ + = 0; therefore 1 − cos2 θ − 2 cos θ + = 0; therefore
4 4
5 1
cos2 θ + 2 cos θ = . By solving this quadratic in the ordinary way we obtain cos θ = ,
4 2
5 π
or − ; but only the former value is applicable; therefore θ = .
·2 3

Problem 11. 3 sec2 θ + 8 = 10 sec2 θ.


Solution. 3 sec4 θ + 8 = 10 sec2 θ; therefore 3 sec4 θ − 10 sec2 θ + 8 = 0. By solving this
4 √ 2
quadratic in the ordinary way we obtain sec2 θ = 2 or ; therefore sec θ = 2 or √ ;
3 3
π π
therefore θ = or .
4 6

Problem 12. tan θ + cot θ = 2 .


Solution.
1
tan θ + cot θ = 2; therefore tan θ + = 2;
tan θ
therefore tan2 θ − 2 tan θ + 1 = 0, that is (tan θ − 1)2 = 0;
π
therefore tan θ = 1, therefore θ = .
4

1 1
Problem 13. Given sin(A − B) = , and cos(A + B) = , find A and B.
2 2
Solution.
1
sin(A − B) =; therefore A − B = 30◦ ,
2
1
cos(A + B) = ; therefore A + B = 60◦ ;
2
from these two equations we obtain A = 45◦ , and B = 15◦ .


Problem 14. Given tan(A + B) = 3 , and tan(A − B) = 1 , find A and B.
Solution. √
tan(A + B) = 3; therefore A + B = 60◦ ,
tan(A − B) = 1; therefore A − B = 45◦ ;
◦ ◦
from these two equations we obtain A = 52 21 , B = 7 21 .
CHAPTER IV
Application of Algebraical Signs

Problem 1. Determine the values of the Trigonometrical Ratios for an angle of 585 ◦ .
Solution. 585◦ = 360◦ + 225◦ . Thus the Trigonometrical Ratios are the same as for an
angle of 225◦ .
1
sin 225◦ = sin (180◦ + 45◦ ) = − sin 45◦ = − √ ,
2
◦ ◦ ◦ ◦ 1
cos 225 = cos (180 + 45 ) = − cos 45 = − √ .
2

Problem 2. Also for an angle of 690 ◦ .


Solution. 690◦ = 360◦ + 330◦ . Thus the Trigonometrical Ratios are the same as for an
angle of 330◦ .
1
sin 330◦ = sin (180◦ + 150◦ ) = − sin 150◦ = − sin 30◦ = − ,
√ 2
◦ ◦ ◦ ◦ ◦ 3
cos 330 = cos (180 + 150 ) = − cos 150 = cos 30 = .
2

Problem 3. Also for an angle of 930 ◦ .


Solution. 930◦ = 720◦ + 210◦ . Thus the Trigonometrical Ratios are the same as for an
angle of 210◦ .
1
sin 210◦ = sin (180◦ + 30◦ ) = − sin 30◦ = − ,
2√
3
cos 210◦ = cos (180◦ + 30◦ ) = − cos 30◦ = − .
2

Problem 4. Also for an angle of 6420 ◦ .


Solution. 6420◦ = 17 × 360◦ + 300◦ . Thus the Trigonometrical Ratios are the same as
for an angle of 300◦ .

3
sin 300◦ = sin (180◦ + 120◦ ) = − sin 120◦ = − sin 60◦ = − ,
2
1
cos 300◦ = cos (180◦ + 120◦ ) = − cos 120◦ = cos 60◦ = .
2

Problem 5. Find all the angles between 0 and 900 ◦ which satisfy the relation
tan θ = 1 .
Solution. The smallest angle is 45◦ ; the other angles are found by increasing succes-
sively by 180◦ : thus all the angles are 45◦ , 225◦ , 405◦ , 585◦ , 765◦ .

Problem 6. Find all the angles between 0 and 900 ◦ which satisfy the relation
1
cos2 θ = .
2
1 1
Solution. Since cos2 θ = , we have cos θ = ± √ .
2 2
Take the upper sign ; then the smallest value is 45◦ , and the others are 360◦ −
45◦ , 360◦ + 45◦ , 720◦ − 45◦ , 720◦ + 45◦ .
Take the lower sign ; then the smallest value is 135◦ , and the others are 360◦ −
135◦ , 360◦ + 135◦ , 720◦ − 135◦ , 720◦ + 135◦ .


Problem 7. Find all the values of versin where n is any integer.
4
nπ nπ
Solution. vers = 1 − cos .
4 4
CHAPTER IV : Application of Algebraical Signs 12

Suppose n = 0; then we have 1 − cos 0, that is 1 − 1, that is 0; next suppose n = 1,


π 1 π
then we have 1 − cos , that is 1 − √ ; next suppose n = 2, then we have 1 − cos ,
4 2 2
3π 1
that is 1 − 0, that is 1; next suppose n = 3, then we have 1 − cos ; that is 1 + √ ;
4 2
next suppose n = 4, then we have 1 − cos π, that is 1 + 1, that is 2. Then the values
5π 3π 6π 2π 7π
begin to recur in the inverse order; for cos = cos , cos = cos , cos =
4 4 4 4 4
π 8π
cos , cos = cos 2π = cos 0.
4 4
9π π
Then the whole series recurs. For cos = cos , and so on.
4 4
{ }
nπ π
Problem 8. Find all the values of sin + (−1 )n where n is any integer.
2 6
π 1
Solution. Suppose n = 0, then we have sin , that is ; next suppose n = 1, then
( ) √6 2
π π π 3
we have sin − , that is sin , that is ; next suppose n = 2, then we have
( ) 2 6 3 2 ( )
π π 1 3π π
sin π + , that is − sin , that is − ; next suppose n = 3, then we have sin − ,
6
( ) 6 2 √ 2 6
π π π 3
that is − sin − , that is − sin , that is − .
2 6 3 2 ( )
π π
Then the values recur; for suppose n = 4; then we have sin 2π + , that is sin ,
6 6
and so on.

Problem 9. Solve sin3 θ + cos3 θ = 0 .


Solution. sin3 θ = − cos3 θ. Extract the cube root of both sides; thus sin θ = − cos θ,
sin θ 3π
therefore = −1, that is tan θ = −1; therefore θ = .
cos θ 4

Problem 10. Solve 2 sin2 θ − 5 cos θ − 4 = 0 .


Solution. 2 sin2 θ − 5 cos θ − 4 = 0; therefore 2(1 − cos2 θ) − 5 cos θ − 4 = 0; therefore
1
2 cos2 θ + 5 cos θ + 2 = 0. By solving this quadratic in the usual way we obtain cos θ = −
2

or −2; but only the former value is applicable; therefore θ = .
3

Problem 11. Trace the changes in the sign and value of cos θ − sin θ as θ changes
from 0 to 2 π.
Solution. When θ = 0 we have cos θ = 1 and sin θ = 0, so that cos θ − sin θ = 1. Let
π
θ change from 0 to , then cos θ changes from 1 to 0, and sin θ from 0 to 1; therefore
2
π
cos θ − sin θ changes from 1 to −1, vanishing when θ = .
4
π
Let θ change from to π, then cos θ changes from 0 to −1 and sin θ from 1 to 0; thus
2 √
cos θ − sin θ remains negative. It has its greatest numerical value, namely − 2, when

θ= . For we have
4 ( )
(cos θ + sin θ)2 + (cos θ − sin θ)2 = 2 cos2 θ + sin2 θ = 2;
and thus (cos θ − sin θ)2 has its greatest value when cos θ + sin θ vanishes, that is when

tan θ = −1, that is when θ = .
4

Let θ change from π to ; then cos θ − sin θ goes through the same numerical
2
π
values, with a contrary sign, as when θ changes from 0 to : this follows from Art. 50
2
(page 319).
CHAPTER IV : Application of Algebraical Signs 13


Let θ change from to 2π; then cos θ − sin θ goes through the same numerical
2
π
values, with a contrary sign, as when θ changes from to π : this follows from Art. 50
2
(page 319).

Problem 12. Also of cos2 θ − sin2 θ.


π
Solution. Let θ change from 0 to ; then cos2 θ changes from 1 to 0, and sin2 θ from 0
2
to 1; therefore cos2 θ − sin2 θ changes from 1 to −1.
π
Let θ change from to π; then cos2 θ − sin2 θ changes from −1 to 1.
2

Let θ change from π to ; then cos2 θ − sin2 θ goes through the same values as
2
π
when θ changes from 0 to .
2

Let θ change from to 2π; then cos2 θ − sin2 θ goes through the same values as
2
π
when θ changes from to π.
2

Problem 13. Also of tan θ + cot θ.


1 π
Solution. tan θ + cot θ = tan θ + . Let θ change from 0 to ; then tan θ changes
tan θ 2
1
from 0 to infinity. Thus tan θ + is always positive, and is infinite both when θ = 0,
tan θ
π π
and when θ = . The least value is when θ = ; for we have
2 ( )2 ( 4 )2
1 1
tan θ + = tan θ − + 4,
tan θ tan θ
1
and thus the least value is when tan θ − vanishes, that is when tan2 θ = 1. Thus
tan θ
π
tan θ + cot θ diminishes from infinity to 2, as θ changes from 0 to ; and then increases
4
π π
from 2 to infinity, as θ changes from to .
4 2
π
Let θ change from to π; then tan θ + cot θ goes in reverse order through the same
2
π
numerical values, with a contrary sign, as when θ changes from 0 to : this follows
2
from Art. 48 (page 317).
Let θ change from π to 2π; then tan θ + cot θ goes through the same values as when
θ changes from 0 to π : this follows from Art. 50 (page 319).

4ab
Problem 14. Is sec2 θ = a possible equation if a and b are unequal ?
(a + b)2
Solution. We know by Algebra that if a and b are unequal 2ab is less than a2 + b2 ,
and therefore 4ab is less than a2 + b2 + 2ab, that is 4ab is less than (a + b)2 . Therefore
4ab
is less than unity; and cannot be equal to the secant of any angle, for a secant
(a + b)2
is never less than unity.

Problem 15. Show that


tan(A + 90◦ ) = − cot A, cot(A + 90◦ ) = − tan A,
sec(A + 90◦ ) = − cosec A, cosec (A + 90◦ ) = sec A,
vers (A + 90◦ ) = 1 + sin A.
Solution.
sin(A + 90◦ ) cos A
tan(A + 90◦ ) = = , by Art. 52 (page 320) , = − cot A,
cos(A + 90◦ ) − sin A
1 1
cot(A + 90◦ ) = =− = − tan A,
tan(A + 90◦ ) cot A
CHAPTER V : Angles with Given Trigonometrical Ratios 14

1 1
sec(A + 90◦ ) = = , by Art. 52 (page 320) , = − cosec A,
cos(A + 90◦ ) − sin A
1 1
cosec (A + 90◦ ) = = , by Art. 52 (page 320) , = sec A,
sin(A + 90◦ ) cos A
◦ ◦
vers (A + 90 ) = 1 − cos(A + 90 ) = 1 + sin A, by Art. 52 (page 320) .

Problem 16. Show that sin(270 ◦ − A) = − cos A, cos(270 ◦ − A) = − sin A.


Solution.
sin(270◦ − A) = − sin(90◦ − A), by Art. 50 (page 319) , = − cos A.
cos(270◦ − A) = − cos(90◦ − A), by Art. 50 (page 319) , = − sin A.

Problem 17. Show that sin(270 ◦ + A) = − cos A, cos(270 ◦ + A) = sin A.


Solution.
sin(270◦ + A) = − sin(90◦ + A), by Art. 50 (page 319) ,
= − cos A, by Art. 52 (page 320) .
cos(270◦ + A) = − cos(90◦ + A), by Art. 50 (page 320) ,
= −(− sin A), by Art. 52 (page 320) , = sin A.

Problem 18. Show that sin(360 ◦ − A) = − sin A, cos(360 ◦ − A) = cos A.


Solution.
sin(360◦ − A) = − sin(180◦ − A), by Art. 50 (page 319) ,
= − sin A, by Art. 48 (page 317) .
cos(360◦ − A) = − cos(180◦ − A), by Art. 50 (page 320) ,
= −(− cos A), by Art. 48 (page 317), = cos A.
CHAPTER V
Angles with Given Trigonometrical Ratios

Problem 1. Write down the general value of θ when tan θ = 1 .


θ π
Solution. tan θ = 1; the smallest value of θ is , and the general value is nπ + , by
4 4
Art. 68 (page 322).

Problem 2. Write down the general value of θ when sin θ = 1 .


π π
Solution. sin θ = 1; the smallest value of θ is , and the general value is nπ + (−1)n ,
2 2
by Art. 66 (page 321). This expression may be simplified; for first suppose n even,
π
denote it by 2m, so that we have 2mπ + ; next suppose n odd, denote it by 2m + 1,
2
π π
so that we have (2m + 1)π − , that is 2mπ + . Hence both cases are included in the
2 2
π π
expression 2mπ + , that is (4m + 1) .
2 2

Problem 3. Write down the general value of θ when cos θ = 1 .


Solution. cos θ = 1; the smallest value of θ is 0, and the general value is 2nπ, by Art. 67
(page 322).

1
Problem 4. Write down the general value of θ when cos θ = − .
2
1 2π 2π
Solution. cos θ = − ; the smallest value of θ is , and the general value is 2nπ ± ,
2 3 3
by Art. 67 (page 322).

Problem 5. Find all the values of θ which satisfy sin2 θ = sin2 α.


Solution. sin2 θ = sin2 α; therefore sin θ = ± sin α. Take the upper sign, then simplest
solution is θ = α, and the general solution is θ = nπ + (−1)n α. Take the lower sign,
then the simplest solution is θ = −α, and the general solution is θ = nπ − (−1)n α. The
two expressions are included in the single expression θ = nπ ± α.
This might also be obtained from a diagram in the manner of Arts. 66 (page 321),
67 (page 322), and 68 (page 322).

4
Problem 6. Write down the general value of θ when cosec 2 θ = .
3
4 3 π
Solution. Since cosec 2 θ = we have sin2 θ = = sin2 ; hence, by P roblem 5, the
3 4 3
π
general solution is θ = nπ ± .
3

Problem 7. Find all the values of θ which satisfy cos2 θ = cos2 α.


Solution. cos2 θ = cos2 α; therefore cos θ = ± cos α. Take the upper sign, then the
simplest solution is θ = α, and the general solution is θ = 2nπ ± α. Take the lower sign,
then the simplest solution is θ = π − α, and the general solution is θ = 2nπ ± (π − α).
The two expressions are included in the single expression θ = mπ ± α.
It will be seen that the result is the same as for P roblem 5, and this should be the
case; for if cos2 θ = cos2 α, then 1 − cos2 θ = 1 − cos2 α, that is sin2 θ = sin2 α.

Problem 8. Write down the general value of θ when sec2 θ = 2 .


1 π
Solution. Since sec2 θ = 2, we have cos2 θ = = cos2 ; hence, by P roblem 7, the
2 4
π
general solution is θ = nπ ± .
4
CHAPTER VI : Trigonometrical Ratios of Two Angles 16

Problem 9. Find all the values of θ which satisfy tan2 θ = tan2 α.


Solution. tan2 θ = tan2 α; therefore tan θ = ± tan α. Take the upper sign, then the
simplest solution is θ = α, and the general solution is θ = nπ + α. Take the lower sign,
then the simplest solution is θ = −α, and the general solution is θ = nπ − α. The two
expressions are included in the single expression θ = nπ ± α.
The result is the same as for P roblem 7, and this should be the case ; for if tan2 θ =
tan2 α then 1 + tan2 θ = 1 + tan2 α; therefore sec2 θ = sec2 α, by Art. 34 (page 316);
therefore cos2 θ = cos2 α.

1
Problem 10. Write down the general value of θ when tan2 θ = .
3
1 π π
Solution. tan2 θ = = tan2 ; hence, by P roblem 9, the general solution is θ = nπ ± .
3 6 6

Problem 11. Show that all the angles which have both the same sine and the same
cosine as α, are included in the formula 2nπ + α.
Solution. All the angles included in the expression 2nπ ± α have the same cosine as
α, by Art. 67 (page 322).
Now by Art. 45 (page 317) sin(2nπ + α) = sin α; and sin(2nπ − α) = sin(−α) = − sin α.
Thus the angles which have both the same sine and the same cosine as α are all
comprised in the expression 2nπ + α.

Problem 12. Write down the general value of θ which satisfies both

1 3
sin θ = − and cos θ = − .
2 2
( ) √ ( )
1 π 7π 3 π 7π
Solution. − = sin π + = sin , and − = cos π + = cos ;
2 6 6 2 6 6

hence, by P roblem 11, the required general value is θ = 2nπ + .
6
CHAPTER VI
Trigonometrical Ratios of Two Angles

Prove the following identities :

cos A + sin A
Problem 1. = tan 2A + sec 2A.
cos A − sin A
Solution.
cos A + sin A (cos A + sin A)2
=
cos A − sin A (cos A − sin A)(cos A + sin A)
cos2 A + sin2 A + 2 sin A cos A 1 + sin 2A
= =
cos2 A − sin2 A cos 2A
sin 2A 1
= + = tan 2A + sec 2A.
cos 2A cos 2A

Problem 2. 2 sin2 A sin2 B + 2 cos2 A cos2 B = 1 + cos 2A cos 2B.


Solution.
2 sin2 A sin2 B + 2 cos2 A cos2 B
(1 − cos 2A)(1 − cos 2B) (1 + cos 2A)(1 + cos 2B)
= +
2 2
1 − cos 2A − cos 2B + cos 2A cos 2B 1 + cos 2A + cos 2B + cos 2A cos 2B
= +
2 2
= 1 + cos 2A cos 2B.

Problem 3. tan (45 ◦ + A) − tan (45 ◦ − A) = 2 tan 2A.


Solution.
tan (45◦ + A) − tan (45◦ − A)
tan 45◦ + tan A tan 45◦ − tan A 1 + tan A 1 − tan A
= ◦
− = −
1 − tan 45 tan A 1 + tan 45◦ tan A 1 − tan A 1 + tan A
(1 + tan A)2 − (1 − tan A)2 4 tan A
= = = 2 tan 2A.
1 − tan2 A 1 − tan2 A

Problem 4. sin 3A cosec A − cos 3A sec A = 2 .


Solution.
sin 3A cosec A − cos 3A sec A
sin 3A cos 3A 3 sin A − 4 sin3 A 4 cos3 A − 3 cos A
= − = −
sin A cos A sin A cos A
= 3 − 4 sin2 A − (4 cos2 A − 3) = 6 − 4(sin2 A + cos2 A) = 6 − 4 = 2.

Problem 5. 3 sin A − sin 3A = 2 sin A(1 − cos 2A).


Solution.
3 sin A − sin 3A = 3 sin A − (3 sin A − 4 sin3 A)
= 4 sin3 A = 2 sin A × 2 sin2 A = 2 sin A(1 − cos 2A).

sin A + 2 sin 3A + sin 5A sin 3A


Problem 6. = .
sin 3A + 2 sin 5A + sin 7A sin 5A
Solution.
sin A + 2 sin 3A + sin 5A sin A + sin 5A + 2 sin 3A
=
sin 3A + 2 sin 5A + sin 7A sin 3A + sin 7A + 2 sin 5A
2 sin 3A cos 2A + 2 sin 3A
= , by Art. 84 (page 325),
2 sin 5A cos 2A + 2 sin 5A
2 sin 3A(1 + cos 2A) sin 3A
= = .
2 sin 5A(1 + cos 2A) sin 5A
CHAPTER VI : Trigonometrical Ratios of Two Angles 18

sin B sin(2A + B)
Problem 7. = − 2 cos(A + B).
sin A sin A
Solution.
sin(2A + B) sin(A + B + A) − 2 sin A cos(A + B)
− 2 cos(A + B) =
sin A sin A
sin(A + B) cos A + cos(A + B) sin A − 2 sin A cos(A + B)
=
sin A
sin(A + B) cos A − cos(A + B) sin A sin(A + B − A) sin B
= = = .
sin A sin A sin A

Problem 8. sin 4A = 4 sin A cos3 A − 4 cos A sin3 A.


Solution.
4 sin A cos3 A − 4 cos A sin3 A = 4 sin A cos A(cos2 A − sin2 A)
= 2 sin 2A cos 2A = sin 4A.

cos A − cos 3A
Problem 9. = tan 2A.
sin 3A − sin A
Solution.
cos A − cos 3A 2 sin 2A sin A
= , by Art. 84 (page 325),
sin 3A − sin A 2 cos 2A sin A
sin 2A
= = tan 2A.
cos 2A

cos 2A − cos 4A
Problem 10. = tan 3A.
sin 4A − sin 2A
Solution.
cos 2A − cos 4A 2 sin 3A sin A
= , by Art. 84 (page 325),
sin 4A − sin 2A 2 cos 3A sin A
sin 3A
= = tan 3A.
cos 3A

Problem 11. cosec 2A + cot 4A = cot A − cosec 4A.


Solution.
1 cos 4A
cosec 2A + cot 4A = +
sin 2A sin 4A
2 cos 2A cos 2A 2 cos 2A + cos 4A
= + =
2 cos 2A sin 2A sin 4A sin 4A
2 cos 2A + 2 cos2 2A − 1 2 cos 2A(1 + cos 2A) − 1
= =
sin 4A sin 4A
2 cos 2A(1 + cos 2A) 1 1 + cos 2A 1
= − = −
2 sin 2A cos 2A sin 4A sin 2A sin 4A
2 cos2 A 1
= −
2 sin A cos A sin 4A
cos A 1
= − = cot A − cosec 4A.
sin A sin 4A

Problem 12. cos2 (A − B) + cos2 B − 2 cos(A − B) cos A cos B = sin2 A.


Solution.
cos2 (A − B) + cos2 B − 2 cos(A − B) cos A cos B
= cos(A − B) {cos(A − B) − cos A cos B}
+ cos B {cos B − cos(A − B) cos A}
= cos(A − B) sin A sin B
{ ( ) }
+ cos B cos A − A − B − cos(A − B) cos A
= cos(A − B) sin A sin B + cos B sin A sin(A − B)
= sin A {cos(A − B) sin B + sin(A − B) cos B}
= sin A sin(A − B + B) = sin A sin A = sin2 A.
CHAPTER VI : Trigonometrical Ratios of Two Angles 19

Problem 13. sin2 (A − B) + sin2 B + 2 sin(A − B) sin B cos A = sin2 A.


Solution.
sin2 (A − B) + sin2 B + 2 sin(A − B) sin B cos A
= sin(A − B) {sin(A − B) + sin B cos A}
+ sin B {sin B + sin(A − B) cos A}
= sin(A − B) sin A cos B
{ ( ) }
+ sin B sin A − A − B + sin(A − B) cos A
= sin(A − B) sin A cos B + sin B sin A cos(A − B)
= sin A {sin(A − B) cos B + cos(A − B) sin B}
= sin A sin(A − B + B) = sin A sin A = sin2 A.

1 − tan2 (45 ◦ − A)
Problem 14. = sin 2A.
1 + tan2 (45 ◦ − A)
Solution.
sin2 (45◦ − A)
1−
1− tan2 (45◦
− A) cos2 (45◦ − A)
=
1 + tan2 (45◦ − A) sin2 (45◦ − A)
1+
cos2 (45◦ − A)
cos2 (45◦ − A) − sin2 (45◦ − A) cos 2(45◦ − A)
= ◦ ◦
=
cos (45 − A) + sin (45 − A)
2 2 1
= cos(90◦ − 2A) = sin 2A.

4 tan A(1 − tan2 A)


Problem 15. = sin 4A.
(1 + tan2 A)2
Solution. ( )
4 sin A sin2 A
1−
4 tan A(1 − tan2 A) cos A cos2 A
(1 + tan2 A)2
= ( )2
sin2 A
1+
cos2 A
4 sin A cos A(cos2 A − sin2 A)
= = 2 sin 2A cos 2A
(cos2 A + sin2 A)2
= sin 4A.

Problem 16. sin A(1 + tan A) + cos A(1 + cot A) = sec A + cosec A.
Solution.
sin A(1 + tan A) + cos A(1 + cot A)
( ) ( )
sin A cos A
= sin A 1 + + cos A 1 +
cos A sin A
sin2 A cos2 A
= sin A + + cos A +
cos A sin A
1 − cos2 A 1 − sin2 A
= sin A + + cos A +
cos A sin A
1 1
= sin A + − cos A + cos A + − sin A
cos A sin A
1 1
= + = sec A + cosec A.
cos A sin A

sin 3A + cos 3A 1 + 2 sin 2A


Problem 17. = tan(A − 45 ◦ ).
sin 3A − cos 3A 1 − 2 sin 2A
Solution.
sin 3A + cos 3A 3 sin A − 4 sin3 A + 4 cos3 A − 3 cos A
=
sin 3A − cos 3A 3 sin A − 4 sin3 A − 4 cos3 A + 3 cos A
CHAPTER VI : Trigonometrical Ratios of Two Angles 20

3(sin A − cos A) − 4(sin3 A − cos3 A)


=
3(sin A + cos A) − 4(sin3 A + cos3 A)
sin A − cos A 3 − 4(sin2 A + cos2 A + sin A cos A)
= ×
sin A + cos A 3 − 4(sin2 A + cos2 A − sin A cos A)
sin A − cos A −1 − 4 sin A cos A
= ×
sin A + cos A −1 + 4 sin A cos A
sin A
− 1 1 + 2 sin 2A
= cos A
sin A 1 − 2 sin 2A
+1
cos A
tan A − 1 1 + 2 sin 2A
=
tan A + 1 1 − 2 sin 2A
1 + 2 sin 2A
= tan(A − 45◦ ) .
1 − 2 sin 2A

Problem 18. cos A + cos(120 ◦ − A) + cos(120 ◦ + A) = 0 .


Solution.
cos A + cos(120◦ − A) + cos(120◦ + A)
= cos A + cos 120◦ cos A + sin 120◦ sin A + cos 120◦ cos A − sin 120◦ sin A
= cos A + 2 cos 120◦ cos A = cos A − cos A = 0.

Problem 19. 4 sin A sin(60 ◦ − A) sin(60 ◦ + A) = sin 3A.


Solution.
4 sin A sin(60◦ − A) sin(60◦ + A)
= 4 sin A(sin2 60◦ − sin2 A), by Art. 83 (page 324),
( )
3
= 4 sin A − sin2 A
4
= 3 sin A − 4 sin3 A = sin 3A.

Problem 20. 4 cos A cos(60 ◦ − A) cos(60 ◦ + A) = cos 3A.


Solution.
4 cos A cos(60◦ − A) cos(60◦ + A)
= 4 cos A(cos2 A − sin2 60◦ ), by Art. 83 (page 324),
( )
3
= 4 cos A cos2 A −
4
= 4 cos3 A − 3 cos A = cos 3A.

Problem 21. tan A tan(60 ◦ + A) tan(120 ◦ + A) = − tan 3A.


Solution.
tan A tan(60◦ + A) tan(120◦ + A)
sin A sin(60◦ + A) sin(120◦ + A)
=
cos A cos(60◦ + A) cos(120◦ + A)
sin A sin(60◦ + A) sin(60◦ + A)
=− , by Art. 48 (page 317),
cos A cos(60◦ + A) cos(60◦ + A)
sin 3A
=− , by P roblems 19 and 20, = − tan 3A.
cos 3A

Problem 22. tan A + tan(60 ◦ + A) + tan(120 ◦ + A) = 3 tan 3A.


Solution.
tan A + tan(60◦ + A) + tan(120◦ + A)
= tan A + tan(60◦ + A) − tan(60◦ − A), by Art. 48 (page 317),
tan 60◦ + tan A tan 60◦ − tan A
= tan A + ◦

1 − tan 60 tan A 1 + tan 60◦ tan A
= tan A +
CHAPTER VI : Trigonometrical Ratios of Two Angles 21

(tan 60◦ + tan A)(1 + tan 60◦ tan A) − (tan 60◦ − tan A)(1 − tan 60◦ tan A)
1 − tan2 60◦ tan2 A
2 tan2 60◦ tan A + 2 tan A
= tan A +
1 − tan2 60◦ tan2 A
8 tan A 9 tan A − 3 tan3 A
= tan A + =
1 − 3 tan A
2 1 − 3 tan2 A
= 3 tan 3A.

Problem 23. cot A + cot(60 ◦ + A) + cot(120 ◦ + A) = 3 cot 3A.


Solution.
cot A + cot(60◦ + A) + cot(120◦ + A)
1 1 1
= + −
tan A tan(60◦ + A) tan(60◦ − A)
1 1 − tan 60◦ tan A 1 + tan 60◦ tan A
= + ◦

tan A tan 60 + tan A tan 60◦ − tan A
1
= +
tan A
(1 − tan 60◦ tan A)(tan 60◦ − tan A) − (1 + tan 60◦ tan A)(tan 60◦ + tan A)
tan2 60◦ − tan2 A
1 2 tan2 60◦ tan A + 2 tan A
= −
tan A tan2 60◦ − tan2 A
1 8 tan A 3 − 9 tan2 A
= − =
tan A 3 − tan2 A 3 tan A − tan3 A
3
= = 3 cot 3A.
tan 3A

Problem 24.
cot A cot(60◦ + A) + cot(60◦ + A) cot(120◦ + A) + cot(120◦ + A) cot A = −3.
Solution.
cot A cot(60◦ + A) + cot(60◦ + A) cot(120◦ + A) + cot(120◦ + A) cot A
1 1 1
= + +
tan A tan(60◦ + A) tan(60◦ + A) tan(120◦ + A) tan(120◦ + A) tan A
tan(120◦ + A) + tan A + tan(60◦ + A)
=
tan A tan(60◦ + A) tan(120◦ + A)
3 tan 3A
= , by P roblems 21 and 22, = −3.
− tan 3A

Problem 25. sin3 A + sin3 (120 ◦ + A) + sin3 (240 ◦ + A) = − 34 sin 3A.


Solution.
1
sin3 A = {3 sin A − sin 3A},
4
1
sin3 (120◦ + A) = {3 sin(120◦ + A) − sin 3(120◦ + A)}
4
1
= {3 sin(120◦ + A) − sin 3A} ,
4
1
sin3 (240◦ + A) = {3 sin(240◦ + A) − sin 3(240◦ + A)}
4
1
= {3 sin(240◦ + A) − sin 3A} ,
4
By addition we obtain
3 3
{sin A + sin(120◦ + A) + sin(240◦ + A)} − sin 3A,
4 4
3
that is − sin 3A; for
4
sin A + sin(120◦ + A) + sin(240◦ + A)
= sin A + sin(60◦ − A) − sin(60◦ + A)
CHAPTER VI : Trigonometrical Ratios of Two Angles 22

= sin A + sin 60◦ cos A − cos 60◦ sin A − sin 60◦ cos A − cos 60◦ sin A
= sin A − 2 cos 60◦ sin A = sin A − sin A = 0.

Problem 26. sin 3A sin3 A + cos 3A cos3 A = cos3 2A.


Solution.
sin 3A sin3 A + cos 3A cos3 A
= (3 sin A − 4 sin3 A) sin3 A + (4 cos3 A − 3 cos A) cos3 A
= 3(sin4 A − cos4 A) − 4 sin6 A + 4 cos6 A
= 3(sin4 A − cos4 A)(sin2 A + cos2 A) − 4 sin6 A + 4 cos6 A
= cos6 A − 3 cos4 A sin2 A + 3 cos2 A sin4 A − sin6 A
= (cos2 A − sin2 A)3 = cos3 2A.

sin 3A cos 3A sin 4A


Problem 27. cos3 A + sin3 A = .
3 3 4
Solution.
sin 3A cos 3A
cos3 A + sin3 A
3 3
1 1
= (3 cos A + cos 3A) sin 3A + (3 sin A − sin 3A) cos 3A
12 12
1
= (sin 3A cos A + cos 3A sin A)
4
1 1
= sin(3A + A) = sin 4A.
4 4

Problem 28. cos nA cos(n + 2 )A − cos2 (n + 1 )A + sin2 A = 0 .


Solution.
cos nA cos(n + 2)A = cos {(n + 1)A − A} cos {(n + 1)A + A}
= cos2 (n + 1)A − sin2 A, by Art. 83 (page 324);
therefore cos nA cos(n + 2)A − cos2 (n + 1)A + sin2 A = 0.

sin A ± sin nA + sin(2n − 1 )A


Problem 29. = tan nA.
cos A ± cos nA + cos(2n − 1 )A
Solution.
sin A ± sin nA + sin(2n − 1)A
cos A ± cos nA + cos(2n − 1)A
sin A + sin(2n − 1)A ± sin nA
=
cos A + cos(2n − 1)A ± cos nA
2 sin nA cos(n − 1)A ± sin nA
= , by Art. 84 (page 325),
2 cos nA cos(n − 1)A ± cos nA
sin nA {2 cos(n − 1)A ± 1} sin nA
= = = tan nA.
cos nA {2 cos(n − 1)A ± 1} cos nA

Problem 30.
sin nA cosec 2 A sec A − cos nA sec2 A cosec A = 4 sin(n − 1 )A cosec 2 2A.
Solution.
sin nA cosec 2 A sec A − cos nA sec2 A cosec A
sin nA cos nA
= −
cos A sin2 A cos2 A sin A
sin nA cos A − cos nA sin A 4 sin(nA − A)
= =
sin2 A cos2 A 4 sin2 A cos A
4 sin(nA − A)
= = 4 sin(nA − A) cosec 2 2A.
sin2 2A
CHAPTER VI : Trigonometrical Ratios of Two Angles 23

Problem 31. cos 10A + cos 8A + 3 cos 4A + 3 cos 2A = 8 cos A cos3 3A.
Solution.
cos 10A + cos 8A + 3 cos 4A + 3 cos 2A
= 2 cos 9A cos A + 6 cos 3A cos A, by Art. 84 (page 325),
= 2 cos A(cos 9A + 3 cos 3A)
= 2 cos A(4 cos3 3A − 3 cos 3A + 3 cos 3A)
= 8 cos A cos3 3A.

Problem 32. cot A + cot 2A + cot 4A = cosec 4A(2 + 2 cos 2A + 3 cos 4A).
Solution.
cos A cos 2A cos 4A
cot A + cot 2A + cot 4A = + +
sin A sin 2A sin 4A
2 cos2 A cos 2A cos 4A
= + +
2 sin A cos A sin 2A sin 4A
1 + 2 cos 2A cos 4A
= +
sin 2A sin 4A
2 cos 2A(1 + 2 cos 2A) cos 4A
= +
2 sin 2A cos 2A sin 4A
1 { 2
}
= 2 cos 2A + 4 cos 2A + cos 4A
sin 4A
1
= {2 cos 2A + 2(1 + cos 4A) + cos 4A}
sin 4A
= cosec 4A{2 + 2 cos 2A + 3 cos 4A}.

2 sin 2A + 2 cos 2A
Problem 33. cosec A = .
cos A − sin A − cos 3A + sin 3A
Solution.
2 sin 2A + 2 cos 2A 2(sin 2A + cos 2A)
=
cos A − sin A − cos 3A + sin 3A cos A − cos 3A + sin 3A − sin A
2(sin 2A + cos 2A)
= , by Art. 84 (page 325),
2 sin 2A sin A + 2 cos 2A sin A
2(sin 2A + cos 2A) 1
= = .
2(sin 2A + cos 2A) sin A sin A

Problem 34. cos2 2A = (cos A − sin 3A)2 + 2 cos A sin 3A(cos A − sin A)2 .
Solution.
(cos A − sin 3A)2 + 2 cos A sin 3A(cos A − sin A)2
= cos2 A + sin2 3A − 2 cos A sin 3A + 2 cos A sin 3A(1 − 2 sin A cos A)
= cos2 A + sin2 3A − 2 cos A sin 3A sin 2A
= cos A{cos A − sin 3A sin 2A} + sin 3A{sin 3A − cos A sin 2A}
= cos A{cos(3A − 2A) − sin 3A sin 2A}
+ sin 3A{sin(2A + A) − cos A sin 2A}
= cos A cos 3A cos 2A + sin 3A sin A cos 2A
= cos 2A{cos 3A cos A + sin 3A sin A}
= cos 2A cos(3A − A) = cos 2A cos 2A = cos2 2A.

( )
Problem 35. cos6 A − sin6 A = cos 2A 1 − 14 sin2 2A .
Solution. ( )( )
cos6 A − sin6 A = cos2 A − sin2 A cos4 A + sin4 A + sin2 A cos2 A
( )
= cos 2A cos4 A + sin4 A + sin2 A cos2 A
{( )2 }
= cos 2A cos2 A + sin2 A − sin2 A cos2 A
{ }
{ } sin2 2A
= cos 2A 1 − sin A cos A
2 2
= cos 2A 1− .
4
CHAPTER VI : Trigonometrical Ratios of Two Angles 24

Problem 36. sin 5A = 5 sin A − 20 sin3 A + 16 sin5 A.


Solution.
sin 5A = sin(3A + 2A) = sin 3A cos 2A + cos 3A sin 2A
( )( ) ( )
= 3 sin A − 4 sin3 A 1 − 2 sin2 A + 4 cos3 A − 3 cos A 2 sin A cos A
( )( ) ( )
= 3 sin A − 4 sin3 A 1 − 2 sin2 A + 4 cos2 A − 3 2 sin A cos2 A
( ) ( ) ( )
= 3 sin A − 4 sin A
3
1 − 2 sin A + 1 − 4 sin A 2 sin A(1 − sin A)
2 2 2

= 5 sin A − 20 sin A + 16 sin5 A.


3

Solve the following equations :

( ) ( )
π π
Problem 37. tan −θ + cot −θ = 4.
4 4
Solution. ( ) ( )
π π
tan − θ + cot − θ = 4;
( 4 ) (4 )
π π
sin −θ cos −θ
therefore (4 )+ (4 ) = 4;
π π
cos −θ sin −θ
( ) 4( ) 4 ( ) ( )
π π π π
therefore sin 2
− θ + cos 2
− θ = 4 sin − θ cos −θ ;
4 4( ) 4 4
π
therefore 1 = 2 sin − 2θ = 2 cos 2θ;
2
1
therefore cos 2θ = ;
2
π
therefore 2θ = 2nπ ± ,
3
π
therefore θ = nπ ± .
6

Problem 38. sin 4 θ + sin θ = 0 .


Solution.
sin 4θ + sin θ = 0,
5θ 3θ
therefore 2 sin cos = 0 by Art. 84 (page 325);
2 2
5θ 3θ
therefore either sin = 0, or cos = 0.
2 2
5θ 3θ π
The former gives = nπ; and the latter gives = 2nπ ± , which may be expressed
2 2 2
3θ π
more simply as = mπ + .
2 2

Alternative Solution : Or we may proceed thus :


sin 4θ = − sin θ,
therefore sin 4θ = sin(π + θ).
Thus 4θ and π + θ must be angles which have the same sine; and therefore all the
solutions are contained in 4θ = nπ + (−1)n (π + θ).

Problem 39. sin 7 θ − sin θ = sin 3 θ.


Solution.
sin 7θ − sin θ = sin 3θ;
therefore 2 sin 3θ cos 4θ = sin 3θ;
therefore either sin 3θ = 0, or 2 cos 4θ = 1.
π
The former gives 3θ = nπ; and the latter gives 4θ = 2nπ ± .
3
CHAPTER VI : Trigonometrical Ratios of Two Angles 25

1
Problem 40. sin θ + cos θ = √ .
2
Solution.
1
sin θ + cos θ = √ ;
2
cos θ sin θ 1
therefore √ + √ = ;
2( 2) 2
π 1
therefore cos θ − = ;
4 2
π π
therefore θ − = 2nπ ± .
4 3

Problem 41. sin 5 θ = 16 sin5 θ.


Solution. By P roblem 36 we have
sin 5θ = 5 sin θ − 20 sin3 θ + 16 sin5 θ.
Thus 5 sin θ − 20 sin3 θ + 16 sin5 θ = 16 sin5 θ,
therefore 5 sin θ − 20 sin3 θ = 0,
1
therefore either sin θ = 0 or sin2 θ = .
4
π π
The former gives θ = nπ; the latter gives sin2 θ = sin2 ; and therefore θ = nπ ± , by
6 6
Chapter v : P roblem 5.

Problem 42. cos 3 θ + cos 2 θ + cos θ = 0 .


Solution.
cos 3θ + cos 2θ + cos θ = 0,
therefore cos 2θ + 2 cos 2θ cos θ = 0,
1
therefore either cos 2θ = 0, or cos θ = − .
2
π 2π
The former gives 2θ = nπ + , as in P roblem 38; and the latter gives θ = 2nπ ± .
2 3

Problem 43. sin 3 θ + sin 2 θ + sin θ = 0 .


Solution.
sin 3θ + sin 2θ + sin θ = 0,
therefore sin 2θ + 2 sin 2θ cos θ = 0,
1
therefore either sin 2θ = 0, or cos θ = − .
2

The former gives 2θ = nπ; and the latter gives θ = 2nπ ± .
3
( )
π
Problem 44. tan θ + tan +θ = 2.
4
Solution. ( )
π
tan θ + tan + θ = 2;
4
1 + tan θ
therefore tan θ + = 2,
1 − tan θ
therefore tan θ − tan θ + 1 + tan θ = 2 − 2 tan θ,
2

therefore tan2 θ − 4 tan θ + 1 = 0,


sin2 θ 4 sin θ
therefore − + 1 = 0,
cos2 θ cos θ
therefore sin θ + cos2 θ = 4 sin θ cos θ,
2

1 = 4 sin θ cos θ = 2 sin 2θ,


1 π
therefore sin 2θ = , therefore 2θ = nπ + (−1)n .
2 6
CHAPTER VII : Formulae for the Division of Angles 26

Problem 45. tan 2 θ = 8 cos2 θ − cot θ.


Solution.
tan 2θ + cot θ = 8 cot2 θ;
sin 2θ cos θ
therefore + = 8 cos2 θ,
cos 2θ sin θ
therefore sin 2θ sin θ + cos 2θ cos θ = 8 cos2 θ sin θ cos 2θ,
therefore cos(2θ − θ) = 8 cos2 θ sin θ cos 2θ;
therefore either cos θ = 0, or 1 = 8 cos θ sin θ cos 2θ.
π
The former gives θ = nπ + ; the latter gives
2
1 = 4 sin 2θ cos 2θ = 2 sin 4θ,
1 π
so that sin 4θ = , and 4θ = nπ + (−1)n .
2 6

( ) ( )
π π
Problem 46. tan +θ = 3 tan −θ .
4 4
Solution. ( ) ( )
π π
tan + θ = 3 tan −θ ,
( 4 ) (4 )
π π 3
therefore tan + θ = 3 cot +θ = ( ),
4 4 π
tan +θ
( ) 4
π π
therefore tan2 + θ = 3 = tan2 ,
4 3
π π
therefore + θ = nπ ± , by Chapter v : P roblem 9.
4 3
CHAPTER VII
Formulae for the Division of Angles

A √ √
Problem 1. Show that 2 sin = − 1 + sin A − 1 − sin A, when A lies between 450 ◦
2
and 630 ◦ .
A A
Solution. Here lies between 225◦ and 315◦ ; thus sin is negative, and is numeri-
2 2
A
cally greater than cos ; hence
2
A A √ A A √
sin + cos = − 1 + sin A, sin − cos = − 1 − sin A;
2 2 2 2
A √ √
therefore 2 sin = − 1 + sin A − 1 − sin A.
2

A A
Problem 2. Obtain cos in terms of sin A when lies between 405 ◦ and 495 ◦ .
2 2
A A
Solution. Here lies between 405◦ and 495◦ ; thus sin is negative, and is numeri-
2 2
A
cally greater than cos ; hence
2
A A √ A A √
sin + cos = 1 + sin A, sin − cos = 1 − sin A;
2 2 2 2
A √ √
therefore 2 cos = 1 + sin A − 1 − sin A.
2

A A
Problem 3. Obtain cos in terms of sin A when lies between −45 ◦ and −135 ◦ .
2 2
A A
Solution. Here lies between −45◦ and −135◦ ; thus sin is negative, and is numer-
2 2
A
ically greater than cos ; hence
2
A A √ A A √
sin + cos = − 1 + sin A, sin − cos = − 1 − sin A;
2 2 2 2
A √ √
therefore 2 sin = − 1 + sin A − 1 − sin A.
2

Problem 4. Determine the limits√ between which √A must lie in order that
2 sin A = − 1 + sin 2A + 1 − sin 2A.
Solution. The proposed formula √ must have arisen from √
sin A + cos A = − 1 + sin 2A, sin A − cos A = 1 − sin 2A;
3π 7π
the former shows that A must lie between 2nπ + and 2nπ + , and the latter shows
4 4
π 5π
that A must lie between 2mπ + and 2mπ + ; hence, by combining these results,
4 4
3π 5π
it follows that A must lie between 2nπ + and 2nπ + . See Art. 101 (page 327).
4 4

Problem 5. Determine the limits√ between which√A must lie in order that
2 cos A = − 1 + sin 2A + 1 − sin 2A.
Solution. The proposed formula √ must have arisen from √
sin A + cos A = − 1 + sin 2A, sin A − cos A = − 1 − sin 2A;
3π 7π
the former shows that A must lie between 2nπ + and 2nπ + , and the latter shows
4 4
5π 9π
that A must lie between 2mπ + and 2mπ + ; hence, by combining these results,
4 4
CHAPTER VII : Formulae for the Division of Angles 28

5π 7π
it follows that A must lie between 2nπ + and 2nπ + .
4 4

Problem 6. Determine the limits√between which √ A must lie in order that


2 cos A = 1 + sin 2A − 1 − sin 2A.
Solution. The proposed formula √ must have arisen from √
sin A + cos A = 1 + sin 2A, sin A − cos A = − 1 − sin 2A;
π 3π
the former shows that A must lie between 2nπ − and 2nπ + , and the latter shows
4 4
5π 9π 3π
that A must lie between 2mπ + and 2mπ + , that is, between 2(m + 1)π − and
4 4 4
π
2(m + 1)π + : hence, by combining these results, it follows that A must lie between
4
π π
2nπ − and 2nπ + .
4 4

Problem 7. Divide a given angle into two parts whose sines shall be in a given ratio.
Solution. Let A denote the given angle and m the given ratio. Let x denote one of
the two parts and therefore A − x the other. Then
sin x = m sin(A − x);
thus sin x = m(sin A cos x − cos A sin x).
Divide by cos x; thus
tan x = m(sin A − cos A tan x),
m sin A
therefore tan x = .
1 + m cos A
Thus tan x is known, and therefore x is known.

Problem 8. Divide a given angle into two parts whose cosines shall be in a given
ratio.
Solution. Let A denote the given angle and m the given ratio. Let x denote one of
the two parts and therefore A − x the other. Then
cos x = m cos(A − x);
thus cos x = m(cos A cos x + sin A sin x).
Divide by cos x; thus
1 = m(cos A + sin A tan x),
1 − m cos A
therefore tan x = .
m sin A
Thus tan x is known, and therefore x is known.

Problem 9. Divide a given angle into two parts whose tangents shall be in a given
ratio.
Solution. Let A denote the given angle and m the given ratio. Let x denote one of
the two parts and therefore A − x the other. Then
tan x = m tan(A − x);
m(tan A − tan x)
thus tan x = ;
1 + tan A tan x
therefore tan x(1 + tan A tan x) = m(tan A − tan x).
Thus we have a quadratic equation from which the value of tan x may be found.

Alternative Solution : Or we may proceed thus,


tan x = m tan(A − x),
sin x m sin(A − x)
therefore = ,
cos x cos(A − x)
therefore 2 sin x cos(A − x) = 2m sin(A − x) cos x,
therefore sin A + sin(2x − A) = m {sin A + sin(A − 2x)}
= m {sin A − sin(2x − A)} ,
CHAPTER VII : Formulae for the Division of Angles 29

therefore (m + 1) sin(2x − A) = (m − 1) sin A.


Thus sin(2x − A) is known, and therefore 2x − A is known, and therefore x is known.

A √
Problem 10. Given tan = 2 − 3 , find sin A.
2
Solution. By Art. 87 (page 325),
A √
2 tan 2(2 − 3)
sin A = 2 = √
1 + tan2
A 1 + (2 − 3)2
2
√ √
2(2 − 3) 2(2 − 3) 1
= √ = √ = .
1+4+3−4 3 4(2 − 3) 2

1
Problem 11. Given sin 210 ◦ = − , find cos 105 ◦ .
2
Solution. √
sin 105◦ + cos 105◦ = 1 + sin 210◦ ,

and sin 105◦ − cos 105◦ = 1 − sin 210◦ ,
√ √
therefore 2 cos 105◦ = 1 + sin 210◦ − 1 − sin 210◦
√ √ √
1 1 1 3
= 1− − 1+ = √ − √ ;
2 2 2 2

1 − 3
thus 2 cos 105◦ = √ ,
2
√ √
1− 3 3−1
and cos 105◦ = √ =− √ .
2 2 2 2

24
Problem 12. Given tan 2A = − , find sin A and cos A.
7
Solution.
2 tan A 2 tan A 24
tan 2A = ; thus =− ;
1 − tan2 A 1 − tan2 A 7
therefore 14 tan A = −24(1 − tan2 A);
therefore 24 tan2 A − 14 tan A = 24.
By solving this quadratic in the ordinary way we obtain
4 3
tan A = , or − .
3 4
tan A 1
Also sin A = √ , and cos A = √ .
1 + tan2 A 1 + tan2 A
4 4 3
If tan A = we get sin A = ± , and cos A = ± .
3 5 5
3 3 4
If tan A = − we get sin A = ± , and cos A = ∓ .
4 5 5

Problem 13. Find tan 165 ◦ from the known value of tan 330 ◦ .
Solution.
2 tan A 1
tan 2A = . Let 2A = 330◦ , then tan 2A = − √ .
1 − tan2 A 3
1 2 tan A
therefore −√ = ,
3 1 − tan2 A
√ √
therefore −1 + tan A = 2 3 tan A, therefore tan2 A − 2 3 tan A = 1.
2

By solving this quadratic in the ordinary way we obtain tan A = 3√ ± 2.

But tan 165 must be a negative quantity, and is therefore equal to 3 − 2.
CHAPTER VII : Formulae for the Division of Angles 30

A 2 sin A − sin 2A
Problem 14. Show that tan2 = .
2 2 sin A + sin 2A
Solution.
2 sin A − sin 2A 2 sin A − 2 sin A cos A 2 sin A(1 − cos A)
= =
2 sin A + sin 2A 2 sin A + 2 sin A cos A 2 sin A(1 + cos A)
2 A
1 − cos A 2 sin
= = 2 = tan2 A .
1 + cos A 2
A 2
2 cos
2

180 ◦ + A 180 ◦ − A
Problem 15. vers (180 ◦ − A) = 2 vers vers .
2 2
Solution.
1 1
2 vers (180◦ + A) vers (180◦ − A)
2 {
2 ( )} { ( )}
A A
=2 1 − cos 90◦ + 1 − cos 90◦ −
( )( 2 ) 2
A A
= 2 1 + sin 1 − sin
( 2 ) 2
A A
= 2 1 − sin2 = 2 cos2 ;
2 2
A
and vers (180◦ − A) = 1 − cos(180◦ − A) = 1 + cos A = 2 cos2 .
2
Thus the proposed expressions are equal.

A−B
Problem 16. (cos A + cos B)2 + (sin A + sin B)2 = 4 cos2 .
2
Solution.
(cos A + cos B)2 + (sin A + sin B)2
= cos2 A + cos2 B + 2 cos A cos B + sin2 A + sin2 B + 2 sin A sin B
= 2 + 2(cos A cos B + sin A sin B) = 2 + 2 cos(A − B)
1
= 2 {1 + cos(A − B)} = 4 cos2 (A − B).
2

A−B
Problem 17. (cos A − cos B)2 + (sin A − sin B)2 = 4 sin2 .
2
Solution.
(cos A − cos B)2 + (sin A − sin B)2
= cos2 A + cos2 B − 2 cos A cos B + sin2 A + sin2 B − 2 sin A sin B
= 2 − 2(cos A cos B + sin A sin B) = 2 − 2 cos(A − B)
1
= 2 {1 − cos(A − B)} = 4 sin2 (A − B).
2

Problem 18. Show that √ √


1◦ 2− 2
sin 22 = ,
2 √ 2 √
1◦ 2+ 2
cos 22 = ,
2 2
1◦ √
and tan 22 = 2 − 1.
2
Solution.
1◦
2 sin2 22 = 1 − cos 45◦ ; therefore
2
1 ◦ 2 √
4 sin2 22 = 2 − 2 cos 45◦ = 2 − √ = 2 − 2,
2
1◦
√ √ 2
therefore 2 sin 22 = 2 − 2.
2
CHAPTER VII : Formulae for the Division of Angles 31

1◦
And 2 cos2 22= 1 + cos 45◦ ; therefore
2
1 ◦ 2 √
4 cos2 22 = 2 + 2 cos 45◦ = 2 + √ = 2 + 2,
2 2
1◦
√ √
therefore 2 cos 22 = 2 + 2.
√ √2 √ √ √ √
1◦
sin 22 2 2− 2 2− 2 2− 2
Hence ◦ = √ √ = √ √ ·√ √
cos 22 12 2+ 2 2+ 2 2− 2
√ √
2− 2 2− 2 √
= √ = √ = 2 − 1,
4−2 2
1◦ √
that is tan 22 = 2 − 1.
2

( ) ( )2
A A A
Problem 19. (tan A + cot A) 2 tan 1 − tan2 = 1 + tan2 .
2 2 2
Solution. ( )
A A
(tan A + cot A)2 tan 1 − tan2
( )2 ( 2 )
sin A cos A A A
= + 1 − tan2
2 tan
cos A sin A 2 2
A 2 A − sin2 A
2
sin A + cos A 2 2 sin cos
= · 2 · 2 2
sin A cos A A A
cos cos2
2 2
A
1 2 sin
= · 2 · cos A
sin A cos A A A
cos cos2
2 2
A A
2 sin 2 sin 1
= 2 = 2 =
A A A A
sin A cos3 2 sin cos4 cos4
 2
2
2 2 2
  ( )2
1 A
= = 1 + tan2 .
 cos2 A  2
2

( )
π A sec A + tan A
Problem 20. tan2 + = .
4 2 sec A − tan A
Solution.  2  2
( )  1 + tan A   cos A + sin A 
π A 2 2 2
tan2 + = =
4 2  1 − tan A   cos A − sin A 
2 2 2
A A A A
cos2+ sin2 + 2 sin cos
= 2 2 2 2 = 1 + sin A
A A A A 1 − sin A
cos2 + sin2 − 2 sin cos
2 2 2 2
1 sin A
+
= cos A cos A = sec A + tan A .
1 sin A sec A − tan A

cos A cos A

( ) ( )
π θ π θ sin θ
Problem 21. sin − + cos − = √ .
4 2 4 2 vers θ
CHAPTER VII : Formulae for the Division of Angles 32

Solution. ( ) ( )
π θ π θ
sin − + cos −
4 2 4 2
π θ π θ π θ π θ
= sin cos − cos sin + cos cos + sin sin
4 2 4 2 4 2 4 2
θ θ θ
π θ 2 cos 2 sin cos
= 2 sin cos = √ 2 = √ 2 2
4 2 2 θ
2 sin
2
sin θ sin θ sin θ
= √ = √ = √ .
θ 1 − cos θ vers θ
2 sin 2
2

( ) { √ }2
θ θ
Problem 22. Show that 4 sin2 1 − sin = 1− 1 + sin θ .
4 2
Solution. ( ) ( )
θ θ θ θ θ θ θ
4 sin2 1 − sin = 4 sin2 + cos2 − 2 sin cos
sin2
4 2 4 4 4 4 4
( )
θ θ θ 2
= 4 sin2 sin − cos
4 4 4
( )
2 θ θ θ 2
= 2 sin − 2 sin cos
4 4
( ) 4
θ θ 2
= 1 − cos − sin .
2 2

√ θ θ θ θ θ θ
And 1 + sin θ = sin2
+ cos2 + 2 sin cos = sin + cos ;
2 2 2 2 2 2
{ √ }2 ( )
θ θ 2
therefore 1 − 1 + sin θ = 1 − sin − cos .
2 2

π 3π 5π 7π 3
Problem 23. cos4 + cos4 + cos4 + cos4 = .
8 8 8 8 2
Solution.
θ
2 cos2 = 1 + cos θ; therefore
2
θ
4 cos4 = (1 + cos θ)2 = 1 + 2 cos θ + cos2 θ
2
1 + cos 2θ 1
= 1 + 2 cos θ + = (3 + 4 cos θ + cos 2θ);
2 2
θ 1
therefore cos4 = (3 + 4 cos θ + cos 2θ).
2 8
Use this formula for each of the terms; thus
π 3π 5π 7π
cos4 + cos4 + cos4 + cos4
8 ( 8 8 8 )
12 1 π 3π 5π 7π
= + cos + cos + cos + cos
8 2( 4 4 4 4)
1 π 3π 5π 7π
+ cos + cos + cos + cos
8 2 2 2 2
3
= : see Art. 50 (page 319).
2

◦ √ √ √
Problem 24. tan 7 12 = 6− 3+ 2 − 2.
Solution. √
3−1
√ √
1◦ sin 15◦ 2 2 3−1
tan 7 = = √ = √ √
2 1 + cos 15◦ 3+1 2 2+ 3+1
1+ √
2 2
CHAPTER VII : Formulae for the Division of Angles 33

√ √ √ √ √ √
( 3 − 1)(2 2 + 1 − 3) 2 6−2 2−4+2 3
= √ √ √ √ = √
(2 2 + 3 + 1)(2 2 + 1 − 3) 6+4 2
√ √ √
6− 2−2+ 3
= √ .
3+2 2 √
Multiply both numerator and denominator
√ √ √ by 3 − 2 2; then we obtain unity for de-
nominator, and for numerator 6 − 3 + 2 − 2.

◦ √ √ √
Problem 25. tan 142 21 = 2 + 2 − 3 − 6 .
Solution.
1◦ sin 285◦ sin 105◦ cos 15◦
tan 142 = ◦
=− ◦
=−
2 1 + cos
√ 285 1 − cos 105 1 + sin 15◦
3+1
√ √
2 2 3+1
=− √ = − √ √
3−1 2 2−1+ 3
1+ √
√2 2 √ √
( 3 + 1)(2 2 − 1 − 3)
=− √ √ √ √
(2 2 − 1 + 3)(2 2 − 1 − 3)
√ √ √ √ √ √
6+ 2−2− 3 2+ 3− 2− 6
=− √ = √ .
3−2 2 √ 3−2 2
Multiply both numerator and denominator
√ √ by √ 3 + 2 2; then we obtain unity for de-
nominator, and for numerator 2 + 2 − 3 − 6.

√ x
Problem 26. If tan x = (2 + 3 ) tan , find the value of tan x.
3
Solution.
x x
− tan3
3 tan
tan x = 3 3
x ;
1 − 3 tan2
3
√ x
and since this is equal to (2 + 3 tan we obtain
3
x
3 − tan2
3 = 2 + √3;
x
1 − 3 tan2
√ ( )
3
x x
therefore 3 − tan2 = (2 + 3) 1 − 3 tan2 ;
3 3
√ x √
therefore (6 + 3 3 − 1) tan2 = 2 + 3 − 3;
√ 3√ √
2 x 3−1 ( 3 − 1)(5 − 3 3)
therefore tan = √ = √ √
3 5+3 3 (5 + 3 3)(5 − 3 3)

8 3 − 14 √
= = 7 − 4 3;
√ 25 − 27
x √ √
therefore tan = 7 − 4 3 = ±(2 − 3).
3 √ √
Hence tan x = ±(2 + 3)(2 − 3) = ±1.

( )
1 1
Problem 27. If α = n+ ± π, where n is any integer, find the value of tan α + cot α.
4 6
Solution.
sin α cos α sin2 α + cos2 α 1
tan α + cot α = + = =
cos α sin α sin α cos α sin α cos α
2 2
= = .
2 sin α cos α sin 2α
CHAPTER VIII : Miscellaneous Propositions 34

Put for α its value; then the expression


2 2 2
= ( ) = ( )= π
1 1 π π
sin 2 n + ± π sin ± cos
4 6 2 3 3
1
= 2 ÷ = 4.
2

π cos α cos 13 α
Problem 28. If α = , find the value of .
17 cos 3 α + cos 5 α
Solution.
cos α cos 13α cos α cos 13α cos 13α 1
= = =− ;
cos 3α + cos 5α 2 cos α cos 4α 2 cos 4α 2
for 13α + 4α = π, and therefore cos 13α = − cos 4α.

√ α
Problem 29. If sec(ϕ + α) + sec(ϕ − α) = 2 sec ϕ, then cos ϕ = 2 cos .
2
Solution.
sec(ϕ + α) + sec(ϕ − α) = 2 sec ϕ,
1 1 2
therefore + = ;
cos(ϕ + α) cos(ϕ − α) cos ϕ
cos(ϕ − α) + cos(ϕ + α) 2
therefore = ;
cos(ϕ + α) cos(ϕ − α) cos ϕ
2 cos ϕ cos α 2
therefore = ;
cos2 ϕ − sin2 α cos ϕ
therefore cos ϕ cos α = cos ϕ − sin2 α;
2 2

sin2 α 1 − cos2 α α
therefore cos2 ϕ = = = 1 + cos α = 2 cos2 ;
1 − cos α 1 − cos α 2
√ α
therefore cos ϕ = 2 cos .
2

( ) 12
θ 1 +c ϕ cos ϕ − c
Problem 30. If tan = tan , show that cos θ = .
2 1 −c 2 1 − c cos ϕ
Solution.
θ 1+c ϕ
tan2 = tan2 ;
2 1−c 2
2 ϕ
1 + c sin 2
1− ·
θ 1−c ϕ ϕ ϕ
1 − tan2 cos2 (1 − c) cos2 − (1 + c) sin2
therefore 2 = 2 = 2 2;
θ 2 ϕ ϕ 2 ϕ
1 + tan2 1+c sin (1 − c) cos 2 + (1 + c) sin
2 1+ · 2 2 2
1−c ϕ
cos2
2
therefore, by Art. 87 (page 325),
( )
ϕ ϕ ϕ ϕ
cos2 − sin2 − c cos2 + sin2
2 2 2 2 cos ϕ − c
cos θ = ( )= .
ϕ ϕ ϕ ϕ 1 − c cos ϕ
cos2 + sin 2 − c cos2 − sin 2
2 2 2 2
CHAPTER VIII
Miscellaneous Propositions

Prove the following formulae :

cos(α + β + γ)
Problem 1. = 1 − tan β tan γ − tan γ tan α − tan α tan β.
cos α cos β cos γ
Solution. By Art. 113 (page 328) we have
cos(α + β + γ) = cos α cos β cos γ − cos α sin β sin γ − cos β sin γ sin α
− cos γ sin α sin β;
divide both sides by cos α cos β cos γ; thus
cos(α + β + γ)
= 1 − tan β tan γ − tan γ tan α − tan α tan β.
cos α cos β cos γ

sin(α + β + γ)
Problem 2. = tan α + tan β + tan γ − tan α tan β tan γ.
cos α cos β cos γ
Solution. By Art. 113 (page 328) we have
sin(α + β + γ) = sin α cos β cos γ + sin β cos γ cos α + sin γ cos α cos β
− sin α sin β sin γ;
divide both sides by cos α cos β cos γ; thus
sin(α + β + γ)
= tan α + tan β + tan γ − tan α tan β tan γ.
cos α cos β cos γ

Problem 3.
sin(α − β) + sin(β − γ) + sin(γ − α)
α−β β−γ γ−α
+ 4 sin sin sin = 0.
2 2 2
Solution.
α−γ α − 2β + γ
sin(α − β) + sin(β − γ) = 2 sin cos
2 2
γ−α α − 2β + γ
= −2 sin cos ;
2 2
γ−α γ−α
sin(γ − α) = 2 sin cos ;
2 2
therefore sin(α − β) + sin(β − γ) + sin(γ − α)
{ }
γ−α γ−α α − 2β + γ
= 2 sin cos − cos
2 2 2
γ−α γ−β α−β
= 2 sin 2 sin sin
2 2 2
α−β β−γ γ−α
= −4 sin sin sin ;
2 2 2
therefore sin(α − β) + sin(β − γ) + sin(γ − α)
α−β β−γ γ−α
+ 4 sin sin sin =0
2 2 2

Problem 4.
4 sin(θ − α) sin(mθ − α) cos(θ − mθ)
= 1 + cos(2θ − 2mθ) − cos(2θ − 2α) − cos(2mθ − 2α).
Solution.
4 sin(θ − α) sin(mθ − α) cos(θ − mθ)
by Art. 84 (page 325),
= 2 cos(θ − mθ) {cos(θ − mθ) − cos(θ + mθ − 2α)} ,
= 2 cos2 (θ − mθ) − 2 cos(θ − mθ) cos(θ + mθ − 2α)
= 1 + cos 2(θ − mθ) − {cos(2θ − 2α) + cos(2mθ − 2α)}
CHAPTER VIII : Miscellaneous Propositions 36

= 1 + cos 2(θ − mθ) − cos(2θ − 2α) − cos(2mθ − 2α).

Problem 5. sin(α + β) cos β − sin(α + γ) cos γ = sin(β − γ) cos(α + β + γ).


Solution.
sin(α + β) cos β = sin(α + β + γ − γ) cos β
= {sin(α + β + γ) cos γ − cos(α + β + γ) sin γ} cos β,
sin(α + γ) cos γ = sin(α + β + γ − β) cos γ
= {sin(α + β + γ) cos β − cos(α + β + γ) sin β} cos γ;
∴ sin(α + β) cos β − sin(α + γ) cos γ
= cos(α + β + γ) {sin β cos γ − sin γ cos β}
= cos(α + β + γ) sin(β − γ).

Problem 6.
cos(α + β + γ) + cos(α + β − γ) + cos(α + γ − β)
+ cos(β + γ − α) = 4 cos α cos β cos γ.
Solution.
cos(α + β + γ) + cos(α + β − γ) = 2 cos(α + β) cos γ,
cos(α − β + γ) + cos(β + γ − α) = 2 cos(α − β) cos γ;
hence the sum = 2 cos γ {cos(α + β) + cos(α − β)}
= 4 cos α cos β cos γ.

Problem 7.
cos 2α + cos 2β + cos 2γ + cos 2(α + β + γ)
= 4 cos(α + β) cos(β + γ) cos(γ + α).
Solution.
cos 2α + cos 2β = 2 cos(α + β) cos(α − β),
cos 2γ + cos 2(α + β + γ) = 2 cos(2γ + α + β) cos(α + β);
hence the sum = 2 cos(α + β) {cos(α − β) + cos(2γ + α + β)}
= 2 cos(α + β)2 cos(α + γ) cos(β + γ)
= 4 cos(α + β) cos(β + γ) cos(γ + α).

Problem 8.
sin α sin β
+
sin(α − β) sin(α − γ) sin(β − γ) sin(β − α)
sin γ
+ = 0.
sin(γ − α) sin(γ − β)
Solution. Reduce the three fractions to have the common denominator
sin(α − β) sin(β − γ) sin(γ − α);
then the whole numerator
= − sin α sin(β − γ) − sin β sin(γ − α) − sin γ sin(α − β)
1
= − {cos(α − β + γ) − cos(α + β − γ)}
2
1
− {cos(β + α − γ) − cos(β + γ − α)}
2
1
− {cos(γ − α + β) − cos(γ + α − β)} = 0.
2

Problem 9.
cos(α + β) sin β − cos(α + γ) sin γ
= sin(α + β) cos β − sin(α + γ) cos γ.
Solution.
cos(α + β) sin β − cos(α + γ) sin γ
1 1
= {sin(α + β + β) − sin(α + β − β)} − {sin(α + γ + γ) − sin(α + γ − γ)}
2 2
CHAPTER VIII : Miscellaneous Propositions 37

1 1
= sin(α + 2β) − sin(α + 2γ);
2 2
sin(α + β) cos β − sin(α + γ) cos γ
1 1
= {sin(α + β + β) + sin(α + β − β)} − {sin(α + γ + γ) + sin(α + γ − γ)}
2 2
1 1
= sin(α + 2β) − sin(α + 2γ);
2 2
Thus the two expressions are equal.

Problem 10.
sin(α + β − 2γ) cos β − sin(α + γ − 2β) cos γ
= sin(β − γ) {cos(β + γ − α) + cos(α + γ − β) + cos(α + β − γ)} .
Solution.
sin(α + β − 2γ) cos β − sin(α + γ − 2β) cos γ
1
= {sin(α + 2β − 2γ) + sin(α − 2γ) − sin(α + 2γ − 2β) − sin(α − 2β)} ;
2
sin(β − γ) {cos(β + γ − α) + cos(α + γ − β) + cos(α + β − γ)}
1 1
= {sin(2β − α) + sin(α − 2γ)} + {sin α + sin(2β − 2γ − α)}
2 2
1
+ {− sin α + sin(2β − 2γ + α)}
2
1
= {sin(2β − α) + sin(α − 2γ) + sin(2β − 2γ − α) + sin(2β − 2γ + α)} .
2
Thus the two expressions are equal.

Problem 11. sin(α + β + γ) sin β = sin(α + β) sin(β + γ) − sin α sin γ.


Solution.
1
sin(α + β + γ) sin β = {cos(α + γ) − cos(α + 2β + γ)} ,
2
1
sin(α + β) sin(β + γ) = {cos(α − γ) − cos(α + 2β + γ)} ,
2
1
sin α sin γ = {cos(α − γ) − cos(α + γ)} .
2
Hence sin(α + β) sin(β + γ) − sin α sin γ
1
= {cos(α + γ) − cos(α + 2β + γ)}
2
= sin(α + β + γ) sin β.

Problem 12.
sin α sin β sin(β − α) + sin β sin γ sin(γ − β)
+ sin γ sin α sin(α − γ) + sin(β − α) sin(γ − β) sin(α − γ) = 0.
Solution.
1
sin α sin β sin(β − α) =
{cos(α − β) − cos(α + β)} sin(β − α)
2
1 1
= cos(β − α) sin(β − α) − {sin 2β − sin 2α}
2 4
1 1 1
= sin 2(β − α) − sin 2β + sin 2α.
4 4 4
Similarly we may transform sin β sin γ sin(γ − β) and sin γ sin α sin(α − γ).
Also, by P roblem 3, we have
1
sin(β − α) sin(γ − β) sin(α − γ) =
{sin 2(α − β) + sin 2(β − γ) + sin 2(γ − α)} .
4
Hence the sum of the four expressions is zero.

Problem 13.
cos(α + β) sin(α − β) + cos(β + γ) sin(β − γ)
+ cos(γ + δ) sin(γ − δ) + cos(δ + α) sin(δ − α) = 0.
CHAPTER VIII : Miscellaneous Propositions 38

Solution.
1
cos(α + β) sin(α − β) =
(sin 2α − sin 2β) ,
2
1
cos(β + γ) sin(β − γ) = (sin 2β − sin 2γ) ,
2
1
cos(γ + δ) sin(γ − δ) = (sin 2γ − sin 2δ) ,
2
1
cos(δ + α) sin(δ − α) = (sin 2δ − sin 2α)
2
hence the sum of the four expressions is zero.

Problem 14.
sin(δ − β) sin(α − γ) + sin(β − γ) sin(α − δ) + sin(γ − δ) sin(α − β) = 0.
Solution.
1
sin(δ − β) sin(α − γ) = {cos(α + β − γ − δ) − cos(α − β − γ + δ)}
2
1
sin(β − γ) sin(α − δ) = {cos(α − β + γ − δ) − cos(α + β − γ − δ)} ,
2
1
sin(γ − δ) sin(α − β) = {cos(α − β − γ + δ) − cos(α − β + γ − δ)} ;
2
hence the sum of the three expressions is zero.

If A + B + C = π, prove the following formulae contained in the problems


from 15 to 35 inclusive.

A B C A B C
Problem 15. cot + cot + cot = cot cot cot .
2 2 2 2 2 2
Solution.
A B B A A B
A B cos cos sin cos + sin cos
cot + cot = 2 + 2 = 2 2 2 2
2 2 A B A B
sin sin sin sin
2 2 2 2
1 C
sin (A + B) cos
= 2 = 2 ;
A B A B
sin sin sin sin
2  2 2 2 
C  
cos C C 1 1
2 + cot = cos +
sin
A
sin
B 2 2  sin A sin B sin
C 
2 2 2 2 2
C
cos { }
2 C A B
= sin + sin sin
A B C 2 2 2
sin sin sin
2 2 2
C
cos { }
2 1 A B
= cos (A + B) + sin sin
A B C 2 2 2
sin sin sin
2 2 2
A B C
cos cos cos
= 2 2 2 = cot A cot B cot C .
A B C 2 2 2
sin sin sin
2 2 2

A B C
Problem 16. sin A + sin B + sin C = 4 cos cos cos .
2 2 2
Solution.
1 1 C 1
sin A + sin B = 2 sin (A + B) cos (A − B) = 2 cos cos (A − B)
2 2 2 2
CHAPTER VIII : Miscellaneous Propositions 39

C C C 1
sin C = 2 sin cos = 2 cos cos (A + B);
2 2 { 2 2 }
C 1 1
∴ sin A + sin B + sin C = 2 cos cos (A − B) + cos (A + B)
2 2 2
C A B
= 2 cos 2 cos cos
2 2 2
A B C
= 4 cos cos cos .
2 2 2

A B C
Problem 17. sin A − sin B + sin C = 4 sin cos sin .
2 2 2
Solution.
1 1 B 1
sin A + sin C = 2 sin (A + C) cos (A − C) = 2 cos cos (A − C),
2 2 2 2
B B B 1
sin B = 2 sin cos = 2 cos cos (A + C);
2 2 2 2
Solution. { }
B 1 1
∴ sin A − sin B + sin C = 2 cos cos (A − C) − cos (A + C)
2 2 2
B A C
= 2 cos 2 sin sin
2 2 2
A B C
= 4 sin cos sin .
2 2 2

Problem 18. cos 2A + cos 2B + cos 2C + 4 cos A cos B cos C + 1 = 0 .


Solution. cos 2A + cos 2B = 2 cos(A + B) cos(A − B) = −2 cos C cos(A − B)
cos 2C = 2 cos2 C − 1 = −2 cos C cos(A + B) − 1;
∴ cos 2A + cos 2B + cos 2C = −2 cos C {cos(A − B) + cos(A + B)} − 1
= −2 cos C · 2 cos A cos B − 1
= −4 cos A cos B cos C − 1,
therefore cos 2A + cos 2B + cos 2C + 4 cos A cos B cos C + 1 = 0.

Problem 19. cos 4A + cos 4B + cos 4C + 1 = 4 cos 2A cos 2B cos 2C .


Solution. cos 4A + cos 4B = 2 cos 2(A + B) cos 2(A − B) = 2 cos 2C cos 2(A − B),
cos 4C = 2 cos2 2C − 1 = 2 cos 2C cos 2(A + B) − 1;
∴ cos 4A + cos 4B + cos 4C = 2 cos 2C {cos 2(A − B) + cos 2(A + B)} − 1
= 2 cos 2C · 2 cos 2A cos 2B − 1
= 4 cos 2A cos 2B cos 2C − 1;
therefore cos 4A + cos 4B + cos 4C + 1 = 4 cos 2A cos 2B cos 2C.

A B C π−A π−B π−C


Problem 20. cos + cos + cos = 4 cos cos cos .
2 2 2 4 4 4
Solution.
1 1 1
Let α= (π − A), β= (π − B), (π − C);
γ=
2 2 2
1 1
therefore α + β + γ = (3π − A − B − C) = 2π = π;
2 2
hence, by P roblem 16,
α β γ
sin α + sin β + sin γ = 4 cos
cos cos ,
2 2 2
A B C π−A π−B π−C
that is cos + cos + cos = 4 cos cos cos .
2 2 2 4 4 4

A B C π+A π−B π+C


Problem 21. cos − cos + cos = 4 cos cos cos .
2 2 2 4 4 4
CHAPTER VIII : Miscellaneous Propositions 40

Solution.
1 1 1
Let α= (π − A), β= (π − B), (π − C);
γ=
2 2 2
1 1
therefore α + β + γ = (3π − A − B − C) = 2π = π;
2 2
hence, by P roblem 17,
α β γ
sin α − sin β + sin γ = 4 sin
cos sin ,
2 2 2
A B C π−A π−B π−C
that is cos − cos + cos = 4 sin cos sin .
2 2 2 4 4 4

A B C π−A π−B π−C


Problem 22. sin + sin + sin − 1 = 4 sin sin sin .
2 2 2 4 4 4
Solution.
1 1 1
Let α= (π − A), β= (π − B), γ= (π − C);
2 2 2
1 1
therefore α + β + γ = (3π − A − B − C) = 2π = π;
2 2
hence, by Art 114 (page 328),
α β γ
cos α + cos β + cos γ − 1 = 4 sin sin sin ,
2 2 2
A B C π−A π−B π−C
that is sin + sin + sin − 1 = 4 sin sin sin .
2 2 2 4 4 4

Problem 23. sin2 A + sin2 B + sin2 C − 2 cos A cos B cos C = 2 .


Solution.
1
sin2 A + sin2 B + sin2 C = {1 − cos 2A + 1 − cos 2B + 1 − cos 2C}
2
3 1
= − {cos 2A + cos 2B + cos 2C}
2 2
3 1
= + {1 + 4 cos A cos B cos C} , by P roblem 18,
2 2
= 2 + 2 cos A cos B cos C;
therefore sin2 A + sin2 B + sin2 C − 2 cos A cos B cos C = 2.

Problem 24. sin2 2A + sin2 2B + sin2 2C + 2 cos 2A cos 2B cos 2C = 2 .


Solution.
1
sin2 2A + sin2 2B + sin2 2C = {3 − cos 4A − cos 4B − cos 4C}
2
by Problem 19 ,
3 1
= − {4 cos 2A cos 2B cos 2C − 1} ;
2 2
= 2 − 2 cos 2A cos 2B cos 2C;
therefore sin2 2A + sin2 2B + sin2 2C + 2 cos 2A cos 2B cos 2C = 2

A B B C C A
Problem 25. tan tan + tan tan + tan tan = 1.
2 2 2 2 2 2
Solution.
A B B C C A
tan tan + tan tan + tan tan
2 2 2 2 2 2
1 1 1
= + +
A B B C C A
cot cot cot cot cot cot
2 2 2 2 2 2
A B C
cot + cot + cot
= 2 2 2 = 1, by P roblem 15.
A B C
cot cot cot
2 2 2
CHAPTER VIII : Miscellaneous Propositions 41

sin A + sin B − sin C A B


Problem 26. = tan tan .
sin A + sin B + sin C 2 2
Solution.
A B C
sin A + sin B − sin C = 4 sin sin cos , by P roblem 17;
2 2 2
A B C
sin A + sin B + sin C = 4 cos cos cos , by P roblem 16;
2 2 2
therefore, by division,
A B
sin A + sin B − sin C sin sin
= 2 2 = tan A tan B .
sin A + sin B + sin C A B 2 2
cos cos
2 2

Problem 27.
1 + cos A cos B cos C = cos A sin B sin C + cos B sin A sin C
+ cos C sin A sin B.
Solution.
cos A sin B sin C + cos B sin A sin C + cos C sin A sin B
= sin C(cos A sin B + cos B sin A) + cos C sin A sin B
= sin C sin(A + B) + cos C sin A sin B
= sin2 C + cos C sin A sin B
= 1 − cos2 C + cos C sin A sin B
= 1 + cos C {cos(A + B) + sin A sin B}
= 1 + cos C cos A cos B.

Problem 28.
cot A + cot B + cot C = cot A cot B cot C + cosec A cosec B cosec C .
Solution. Take P roblem 27, and divide by sin A sin B sin C;
1 cos A cos B cos C cos A cos B cos C
therefore + = + + :
sin A sin B sin C sin A sin B sin C sin A sin B sin C
thus we obtain the required result.

C (sin B + sin C − sin A)(sin C + sin A − sin B)


Problem 29. sin2 = .
2 4 sin A sin B
Solution. By P roblem 17 we have
(sin B + sin C − sin A)(sin C + sin A − sin B)
4 sin A sin B
B C A C A B
16 sin sin cos sin sin cos
= 2 2 2 2 2 2 = sin2 C .
A A B B 2
16 sin cos sin cos
2 2 2 2

sin A
Problem 30. The expression cot A + will retain the same value if any two
sin B sin C
of the quantities A, B, C , be interchanged.
Solution.
sin A cos A sin A
cot A + = +
sin B sin C sin A sin B sin C
cos A sin B sin C + sin2 A 1 − cos2 A + cos A sin B sin C
= =
sin A sin B sin C sin A sin B sin C
1 + cos A {cos(B + C) + sin B sin C} 1 + cos A cos B cos C
= = .
sin A sin B sin C sin A sin B sin C
We have thus an expression which involves A, B, and C symmetrically; and we
shall in the same manner obtain the same result if in the original expression any two
of the quantities A, B, C be interchanged.
CHAPTER VIII : Miscellaneous Propositions 42

A B C
tan A + tan B + tan C tan tan tan
Problem 31. = 2 2 2 .
(sin A + sin B + sin C )2 2 cos A cos B cos C
Solution.
By Art. 114 (page 328), tan A + tan B + tan C = tan A tan B tan C;
A B C
by P roblem 16, sin A + sin B + sin C = 4 cos cos cos ;
2 2 2
therefore, by division,
tan A + tan B + tan C tan A tan B tan C
=
(sin A + sin B + sin C)2 A B C
16 cos2 cos2 cos2
2 2 2
A A B B C C A B C
8 sin cos sin cos sin cos tan tan tan
= 2 2 2 2 2 2 = 2 2 2 .
A B C 2 cos A cos B cos C
16 cos A cos B cos C cos2 cos2 cos2
2 2 2

nπ nA nB nC
Problem 32. sin nA + sin nB + sin nC = 4 sin cos cos cos , if n be an inte-
2 2 2 2
ger of the form 4m + 1 or 4m + 3 .
Solution.
n n
sin nA + sin nB = 2 sin (A + B) cos (A − B)
2 2
n n
= 2 sin (π − C) cos (A − B)
{ 2 2 }
nπ nC nπ nC n
= 2 sin cos − cos sin cos (A − B)
2 2 2 2 2
nπ nC n nπ
= 2 sin cos cos (A − B); since cos = 0.
2 2 2 2
nC nC n nC
Also sin nC = 2 sin cos = 2 sin (π − A − B) cos
{ 2 2 2 2 }
nπ n nπ n nC
= 2 sin cos (A + B) − cos sin (A + B) cos
2 2 2 2 2
nπ n nC
= 2 sin cos (A + B) cos .
2 2 2
Therefore sin nA + sin nB + sin nC
{ }
nπ nC n n
= 2 sin cos cos (A − B) + cos (A + B)
2 2 2 2
nπ nA nB nC
= 4 sin cos cos cos .
2 2 2 2

nπ nA nB nC
Problem 33. sin nA + sin nB + sin nC = −4 cos sin sin sin , if n be an inte-
2 2 2 2
ger of the form 4m or 4m + 2 .
Solution. Proceed as in P{ roblem 32. Thus }
nπ nC nπ nC n
sin nA + sin nB = 2 sin cos − cos sin cos (A − B)
2 2 2 2 2
nπ nC n nπ
= −2 cos sin cos (A − B); since sin = 0.
2 2 2 2
nC nC n nC
Also sin nC = 2 sin cos = 2 cos (π − A − B) sin
{ 2 2 2 2 }
nπ n nπ n nC
= 2 cos cos (A + B) + sin sin (A + B) sin
2 2 2 2 2
nπ nC n
= 2 cos sin cos (A + B).
2 2 2
Therefore sin nA + sin nB + sin nC
{ }
nπ nC n n
= −2 cos sin cos (A − B) − cos (A + B)
2 2 2 2
nπ nC nA nB
= −4 cos sin sin sin .
2 2 2 2
CHAPTER VIII : Miscellaneous Propositions 43

A B C B+C A+C A+B


Problem 34. cos + cos + cos = 4 cos cos cos .
2 2 2 4 4 4
Solution. By P roblem 20,
A B C π−A π−B π−C
cos + cos + cos = 4 cos cos cos
2 2 2 4 4 4
B+C C+A A+B
= 4 cos cos cos .
4 4 4

Problem 35.
tan A tan B tan C tan A tan B tan C
+ + + + +
tan B tan C tan A tan C tan A tan B
= sec A sec B sec C − 2.
Solution. ( )
tan B tan C 1 sin B sin C sin(B + C)
+ = + =
tan A tan A tan A cos B cos C tan A cos B cos C
sin A cos A
= = .
tan A cos B cos C cos B cos C
In this was we see that the given expression
cos A cos B cos C
= + +
cos B cos C cos C cos A cos A cos B
cos2 A + cos2 B + cos2 C 3 − sin2 A − sin2 B − sin2 C
= =
cos A cos B cos C cos A cos B cos C
1 − 2 cos A cos B cos C
= , by P roblem 23,
cos A cos B cos C
= sec A sec B sec C − 2.

Problem 36. If the sum of four angles be two right angles, the sum of their tangents
is equal to the sum of the products of the tangents taken three and three.
Solution.
Suppose A + B + C + D = 180◦ ; then A + B = 180◦ − C − D;
therefore tan(A + B) = − tan(C + D), by Art. 48 page 317);
tan A + tan B tan C + tan D
∴ =− ;
1 − tan A tan B 1 − tan C tan D
∴ (tan A + tan B)(1 − tan C tan D) = −(tan C + tan D)(1 − tan A tan B),
therefore tan A + tan B + tan C + tan D
= (tan A + tan B) tan C tan D + (tan C + tan D) tan A tan B
= tan B tan C tan D + tan A tan C tan D + tan A tan B tan D + tan A tan B tan C.

tan(A − B) sin2 C
Problem 37. If + = 1 , prove that tan A tan B = tan2 C .
tan A sin2 A
Solution.
sin2 C tan(A − B) sin(A − B) cos A
=1− =1−
sin2 A tan A cos(A − B) sin A
sin A cos(A − B) − cos A sin(A − B) sin {A − (A − B)}
= =
cos(A − B) sin A cos(A − B) sin A
sin B 2 sin A sin B
= ; therefore sin C = .
cos(A − B) sin A cos(A − B)
sin A sin B
Hence cos2 C = 1 − sin2 C = 1 −
cos(A − B)
cos(A − B) − sin A sin B cos A cos B
= = .
cos(A − B) cos(A − B)
2
sin C sin A sin B cos A cos B sin A sin B
Therefore = ÷ = ,
cos2 C cos(A − B) cos(A − B) cos A cos B
that is tan2 C = tan A tan B.
CHAPTER VIII : Miscellaneous Propositions 44

tan2 α cos β(cos x − cos α)


Problem 38. Given = ,
tan2 β cos α(cos x − cos β)
x α β
show that tan2 = tan2 tan2 .
2 2 2
Solution.
tan2 α cos β(cos x − cos α)
= ,
tan2 β cos α(cos x − cos β)
cos x − cos α tan2 α cos α sin2 α cos β
therefore = = ;
cos x − cos β tan2 β cos β sin2 β cos α
sin2 β cos2 α − sin2 α cos2 β
therefore cos x = 2
(sin β cos α) − sin2 α cos
( β )
1 − cos2 β cos2 α − 1 − cos2 α cos2 β
=
(1 − cos2 β) cos α − (1 − cos2 α) cos β
cos2 α − cos2 β cos α + cos β
= = .
(cos α − cos β) (1 + cos α cos β) 1 + cos α cos β
1 − cos x 1 + cos α cos β − cos α − cos β
Hence =
1 + cos x 1 + cos α cos β + cos α + cos β
(1 − cos α) (1 − cos β)
= ;
(1 + cos α) (1 + cos β)
x α β
therefore tan2 = tan2 tan2 , by Art. 82 (page 324).
2 2 2

cos α cos α′ tan θ tan α


Problem 39. If cos2 θ = , cos2 θ′ = , and = ,
cos β cos β ′ tan θ′ tan α′
α α ′ β
show that tan2 tan2 = tan2 .
2 2 2
Solution.
tan2 θ tan2 α 1 − cos2 θ cos β − cos α

= ; but tan2 θ = = ,
2
tan θ tan2 α′ cos2 θ cos α
1 − cos θ
2 ′ cos β − cos α ′
and tan2 θ′ = = ;
cos2 θ′ cos α′
cos β − cos α cos α ′ 2
tan α
therefore · = ;
cos β − cos α′ cos α tan2 α′
cos β − cos α 2
sin α cos α ′
therefore = ,
cos β − cos α′ sin2 α′ cos α
2 ′
sin α cos α − sin α cos α
2 2 2 ′
∴ cos β = 2 ′ ′
(sin α cos α) − sin2 α cos
( α )
1 − cos2 α′ cos2 α − 1 − cos2 α cos2 α′
=
(1 − cos2 α′ ) cos α − (1 − cos2 α) cos α′
cos2 α − cos2 α′ cos α + cos α′
= = .
(cos α − cos α′ ) (1 + cos α cos α′ ) 1 + cos α cos α′
Hence
1 − cos β 1 + cos α cos α′ − cos α − cos α′ (1 − cos α) (1 − cos α′ )
= ′ ′
= ;
1 + cos β 1 + cos α cos α + cos α + cos α (1 + cos α) (1 + cos α′ )
β α α′
therefore tan2 = tan2 tan2 .
2 2 2

Problem 40. If cos α = cos β cos ϕ = cos β ′ cos ϕ′ , and


ϕ ϕ′ α β β′
sin α = 2 sin sin , show that tan2 = tan2 tan2 .
2 2 2 2 2
cos α ′ cos α
Solution. cos ϕ = , cos ϕ = ;
cos β cos β ′
cos β − cos α cos β ′ − cos α
therefore 1 − cos ϕ = , 1 − cos ϕ′ = ;
cos β cos β ′
CHAPTER VIII : Miscellaneous Propositions 45

ϕ cos β − cos α ϕ′ cos β ′ − cos α


therefore 2 sin2 = , 2 sin2 = ;
2 cos β 2 cos β ′
ϕ ϕ ′ ′
(cos β − cos α) (cos β − cos α)
therefore 4 sin2 sin2 = .
2 2 cos β cos β ′
(cos β − cos α) (cos β ′ − cos α)
Thus sin2 α = ;
cos β cos β ′( )
therefore cos β cos β sin α = cos β cos β − cos α cos β + cos β ′ + cos2 α,
′ 2 ′
( )
therefore cos β cos β ′ cos2 α = cos α cos β + cos β ′ − cos2 α;
( ′
)
therefore cos α 1 + cos β cos β = cos β + cos β ′ ;
cos β + cos β ′
therefore cos α = .
1 + cos β cos β ′
1 − cos α (1 − cos β) (1 − cos β ′ )
Hence = ;
1 + cos α (1 + cos β) (1 + cos β ′ )
α β β ′
therefore tan2 = tan2 tan2 .
2 2 2

sin(α − β) sin(α + θ)
Problem 41. If = , show that
sin β sin θ
cot β − cot θ = cot(α + θ) + cot(α − β).
Solution. The proposed result is true if
cot β − cot(α + θ) = cot θ + cot(α − β),
that is if
cos β cos(α + θ) cos θ cos(α − β)
− = + ,
sin β sin(α + θ) sin θ sin(α − β)
that is if
sin(α + θ) cos β − cos(α + θ) sin β sin(α − β) cos θ + cos(α − β) sin θ
= ,
sin β sin(α + θ) sin θ sin(α − β)
that is if
sin(α + θ − β) sin(α − β + θ)
= ,
sin β sin(α + θ) sin θ sin(α − β)
that is if
sin θ sin(α − β) = sin β sin(α + θ);
and this is true by supposition.

( )2
tan α tan β tan β
Problem 42. If − = tan2 α − tan2 β, then cos θ = .
sin θ tan θ tan α
Solution. ( )
tan α − cos θ tan β 2
= tan2 α − tan2 β; therefore
sin θ ( )( )
(tan α − cos θ tan β)2 = 1 − cos2 θ tan2 α − tan2 β ;
therefore ( )( )
tan2 α − 2 cos θ tan α tan β + cos2 θ tan2 β = 1 − cos2 θ tan2 α − tan2 β ;
therefore tan2 β − 2 cos θ tan α tan β + cos2 θ tan2 α = 0,
that is (tan β − cos θ tan α)2 = 0;
tan β
therefore tan β − cos θ tan α = 0; therefore cos θ = .
tan α

ϕ
Problem 43. If tan ϕ = cos θ tan α, and tan α′ = tan θ sin ϕ, then one value of tan2 is
2
α + α′ α − α′
tan tan .
2 2
CHAPTER VIII : Miscellaneous Propositions 46

Solution.
tan ϕ tan2 α − tan2 ϕ
cos θ = ; therefore tan2 θ = ;
tan α tan2 ϕ
tan α − tan ϕ
2 2 2
tan α ′
therefore = ;
tan2 ϕ sin2 ϕ
cos ϕ tan α − sin ϕ
2 2 2 2
tan α ′
therefore 2ϕ
= ;
sin
( ) sin2 ϕ
2 ′
therefore cos ϕ tan α − 1 − cos ϕ = tan α ;
2 2 2

1 + tan2 α′ cos2 α
therefore cos2 ϕ = = ;
1 + tan α2 cos2 α′
cos α
therefore cos ϕ = ± .
cos α′
cos α
Take the upper sign; thus cos ϕ = ; therefore
cos α′
1 1
1 − cos ϕ cos α′ − cos α 2 sin (α − α′ ) sin (α + α′ )
= = 2 2
1 + cos ϕ cos α′ + cos α 1 1
2 cos (α − α ) cos (α + α′ )

2 2
1 1
= tan (α − α′ ) tan (α + α′ ).
2 2

Problem 44. Find the relation between the angles α, β, γ, when the cosines are
connected by the relation
1 − cos2 α − cos2 β − cos2 γ + 2 cos α cos β cos γ = 0 .
Solution.
1 − cos2 α − cos2 β − cos2 γ + 2 cos α cos β cos γ
= 1 − (cos α − cos β cos γ)2 + cos2 β cos2 γ − cos2 β − cos2 γ
( )( )
= 1 − cos2 β 1 − cos2 γ − (cos α − cos β cos γ)2
= sin2 β sin2 γ − (cos α − cos β cos γ)2
= (sin β sin γ − cos α + cos β cos γ)(sin β sin γ + cos α − cos β cos γ)
= {− cos α + cos(β − γ)} {cos α − cos(β + γ)}
α+β−γ α−β+γ α+β+γ β+γ−α
= 4 sin sin sin sin
2 2 2 2
Hence in order that the proposed expression may be zero one of the four sines last
written must be zero, and thus one of the four angles must be zero or a multiple of
two right angles.

tan(θ + α) tan(θ + β) tan(θ + γ)


Problem 45. If = = , then will
x y z
x +y y + z z +x
sin2 (α − β) + sin2 (β − γ) + sin2 (γ − α) = 0 .
x −y y−z z −x
1
Solution. Let denote the common value of the three fractions; so that
k
x = k tan(θ + α), y = k tan(θ + β), z = k tan(θ + γ).
x+y
Then sin2 (α − β)
x−y
tan(θ + α) + tan(θ + β)
= sin2 (α − β)
tan(θ + α) − tan(θ + β)
sin(θ + α) cos(θ + β) + sin(θ + β) cos(θ + α)
= sin2 (α − β)
sin(θ + α) cos(θ + β) − sin(θ + β) cos(θ + α)
sin(2θ + α + β)
= sin2 (α − β) = sin(2θ + α + β) sin(α − β)
sin(α − β)
1
= {cos(2θ + 2β) − cos(2θ + 2α)} .
2
y+z 1
Similarly sin2 (β − γ) = {cos(2θ + 2γ) − cos(2θ + 2β)} ,
y−z 2
CHAPTER VIII : Miscellaneous Propositions 47

z+x 1
and sin2 (γ − α) = {cos(2θ + 2α) − cos(2θ + 2γ)} .
z−x 2
Thus the sum of the three terms is zero.

tan2 θ tan2 ϕ sin θ sin ϕ


Problem 46. If 2
+ = 1 , and = ,
tan α tan2 β sin α sin β
± sin α
show that sin θ = √ .
1 ± cos α cos β
Solution. From the second given equation
sin2 β sin2 θ
sin2 ϕ = ,
sin2 α
sin β sin2 θ
2
therefore tan2 ϕ = .
sin α − sin2 β sin2 θ
2

Substitute in the first given equation; thus


tan2 θ cos2 β sin2 θ
+ = 1;
tan2 α sin2 α − sin2 β sin2 θ
2
tan θ sin α − sin β sin θ − cos2 β sin2 θ
2 2 2
therefore =
2
tan α sin2 α − sin2 β sin2 θ
sin2 α − sin2 θ
=
sin2 α − sin2 β sin2 θ
sin2 θ cos2 α sin2 α − sin2 θ
therefore ( ) = ;
1− sin2 θ sin2 α sin2 α − sin2 β sin2 θ
therefore ( ) ( )( )
sin2 θ cos2 α sin2 α − sin2 β sin2 θ = sin2 α − sin2 θ 1 − sin2 θ sin2 α;
therefore ( )
sin4 θ sin2 α + cos2 α sin2 β
( )
− sin2 θ cos2 α sin2 α + sin2 α + sin4 α + sin4 α = 0;
( )
therefore sin4 θ 1 − cos2 α cos2 β − 2 sin2 θ sin2 α + sin4 α = 0.
By solving this quadratic in the ordinary way we obtain
1 ± cos α cos β sin2 α
sin2 θ = sin2 α = .
1 − cos2 α cos2 β 1 ∓ cos α cos β

sin(θ − α) a cos(θ − α) a′
Problem 47. If = and = ′,
sin(θ − β) b cos(θ − β) b
aa ′ + bb′
then cos(α − β) = .
ab′ + a ′ b
Solution.
sin {θ − β − (α − β)} a
= ,
sin(θ − β) b
sin(θ − β) cos(α − β) − cos(θ − β) sin(α − β) a
therefore = ;
sin(θ − β) b
a
therefore cos(α − β) − sin(α − β) cot(θ − β) = .
b
cos {θ − β − (α − β)} a′
Again = ′,
cos(θ − β) b
cos(θ − β) cos(α − β) + sin(θ − β) sin(α − β) a′
therefore = ′;
cos(θ − β) b
a′
therefore cos(α − β) + tan(θ − β) sin(α − β) = ′ .
b
Hence sin(α − β) cot(θ − β) sin(α − β) tan(θ − β)
{ }{ ′ }
a a
= cos(α − β) − − cos(α − β) ;
b b′
CHAPTER VIII : Miscellaneous Propositions 48

( )
aa′ a a′
therefore sin2 (α − β) = − + + ′ cos(α − β) − cos2 (α − β);
bb′ b
( b ′)
aa′ a a
therefore 1+ ′ = + ′ cos(α − β);
bb b b
aa′ + bb′
therefore cos(α − β) = ′ .
ab + a′ b

sin θ cos θ′ ϕ
Problem 48. Having given tan ϕ = , show that one of the values of tan
( ) sin θ′ + cos θ 2
θ π θ′
is tan tan − .
2 4 2
Solution.
ϕ
2 tan
tan ϕ = 2 ; thus
ϕ
1 − tan2
2
ϕ
2 tan sin θ cos θ′
2 = ;
ϕ sin θ′ + cos θ
1 − tan2
2 ( )
ϕ( ) ϕ
therefore 2 tan sin θ′ + cos θ = 1 − tan2 sin θ cos θ′ ;
2 2
ϕ ϕ( )
therefore sin θ cos θ′ tan2 + 2 tan sin θ′ + cos θ = sin θ cos θ′ ;
2 2
By solving this quadratic in the ordinary way we obtain
ϕ − (sin θ′ + cos θ) ± (1 + sin θ′ cos θ)
tan =
2 sin θ cos θ′
Take the upper sign; thus
ϕ (1 − sin θ′ )(1 − cos θ)
tan = .
2 sin θ cos θ′
θ
1 − cos θ 2 sin2 θ
Now = 2 = tan ,
sin θ θ θ 2
2 sin cos
2 ( 2 )
π
′ 1 − cos − θ′ ( )
1 − sin θ π θ′
and similarly ′
= ( 2 ) = tan − ;
cos θ π 4 2
sin − θ′
2
( )
ϕ θ π θ′
thus tan = tan tan − .
2 2 4 2
In like manner with the lower sign we shall find that
( )
ϕ θ π θ′
tan = − cot cot − .
2 2 4 2
ϕ
The product of the two values of tan is −1, as it should be by the nature of
2
quadratic equations.

Problem 49. Given cos θ = cos α cos β, cos θ′ = cos α′ cos β,


θ θ′ β
tan tan = tan , show that sin2 β = (sec α − 1 )(sec α′ − 1 ).
2 2 2
Solution. cos θ = cos α cos β;
1 − cos θ 1 − cos α cos β
therefore = ;
1 + cos θ 1 + cos α cos β
θ 1 − cos α cos β
therefore tan2 = .
2 1 + cos α cos β
θ′ 1 − cos α′ cos β
Similarly tan2 = .
2 1 + cos α′ cos β
CHAPTER VIII : Miscellaneous Propositions 49

(1 − cos α cos β) (1 − cos α′ cos β) β 1 − cos β


Hence = tan2 = ;
(1 + cos α cos β) (1 + cos α′ cos β) 2 1 + cos β
′ ′
1 − (cos α + cos α ) cos β + cos α cos α cos β 2 1 − cos β
therefore = ;
1 + (cos α + cos α′ ) cos β + cos α cos α′ cos2 β 1 + cos β
(cos α + cos α′ ) cos β
therefore = cos β;
1 + cos α cos α′ cos2 β
( )
therefore cos α + cos α′ = 1 + cos α cos α′ 1 − sin2 β ;
therefore sin2 β cos α cos α′ = 1 − cos α − cos α′ + cos α cos α′
( )
= (1 − cos α) 1 − cos α′ ;
( )( )
1 1
therefore sin2 β = −1 ′
− 1
cos α ( cos α )
= (sec α − 1) sec α′ − 1 .

Problem 50. Having given that sin(B + C − A), sin(C + A − B), and
sin(A + B − C ) are in arithmetical progression, show that tan A, tan B and tan C , are
in arithmetical progression.
Solution. Here
sin(C + A − B) − sin(B + C − A) = sin(A + B − C) − sin(C + A − B);
therefore 2 sin(A − B) cos C = 2 sin(B − C) cos A;
therefore (sin A cos B − cos A sin B) cos C = (sin B cos C − cos B sin C) cos A.
Divide by cos A cos B cos C; thus
tan A − tan B = tan B − tan C;
therefore tan A, tan B, and tan C are in Arithmetical Progression.

Problem 51. If the sines of the angles of a triangle be in arithmetical progression,


the cotangents of the half angles are also in arithmetical progression.
Solution. Suppose sin A, sin B, and sin C to be in Arithmetical Progression, so that
sin B − sin A = sin C − sin B.
B−A B+A C−B C+B
Thus 2 sin cos = 2 sin cos ;
2 2 2 2
B−A C C−B A
therefore sin sin = sin sin ;
( 2 2 2) 2
B A B A C
therefore sin cos − cos sin sin
( 2 2 2 2 ) 2
C B C B A
= sin cos − cos sin sin .
2 2 2 2 2
A B C
Divide by sin sin sin ; thus
2 2 2
A B B C
cot − cot = cot − cot ;
2 2 2 2
C B A
thus cot , cot and cot are in Arithmetical Progression.
2 2 2

Problem 52. If the sum of the squares of the cosines of the angles of a triangle = 1 ,
the difference between the greatest and least angle is equal to the mean angle.
Solution.
Suppose cos2 A + cos2 B + cos2 C = 1;
therefore 3 − sin2 A − sin2 B − sin2 C = 1;
therefore sin2 A + sin2 B + sin2 C = 2;
therefore by P roblem 23 we have cos A cos B cos C = 0; therefore one of the three
angles is a right angle, and this will be the largest angle. Suppose it to be A, so that
A = 90◦ ; therefore B + C = 90◦ = A; therefore A − C = B.
CHAPTER VIII : Miscellaneous Propositions 50

( )
C C
Problem 53. If A + B + C = 180 ◦ , and sin A + = n sin ,
2 2
A B n−1
show that tan tan = .
2 2 n+1
Solution. ( ) ( )
C 180◦ − A − B
sin A + = sin A +
2 ( 2 )
◦ B−A B−A
= sin 90 − = cos ;
2 2
C A+B
and sin = cos ;
2 2
B−A A+B
thus cos = n cos ;
2 ( 2 )
A B A B A B A B
therefore cos cos + sin sin = n cos cos − sin sin ;
2 2 2 2 2 2 2 2
A B A B
therefore (n + 1) sin sin = (n − 1) cos cos ;
2 2 2 2
A B
sin sin
therefore 2 2 = n − 1,
A B n+1
cos cos
2 2
A B n−1
therefore tan tan = .
2 2 n+1

sin A sin B sin C


Problem 54. If A + B + C = 180 ◦ , and = = ,
x y z
C A B
then (x − y) cot + (y − z) cot + (z − x) cot = 0.
2 2 2
1 sin A sin B sin C
Solution. Suppose to denote the value of , and ; then
k x y z
x = k sin A, y = k sin B, z = k sin C.
C C
∴ (x − y) cot = k(sin A − sin B) cot
2 2
1 1 C
= 2k sin (A − B) cos (A + B) cot
2 2 2
1 C C
= 2k sin (A − B) sin cot
2 2 2
1 C
= 2k sin (A − B) cos
2 2
1 1
= 2k sin (A − B) sin (A + B)
{ 2 2 }
2 1 2 1
= 2k sin A − sin B , by Art. 83 page 324.
2 2
{ }
A 1 1
Similarly (y − z) cot = 2k sin2 B − sin2 C ,
2 { 2 2 }
B 2 1 2 1
and (z − x) cot = 2k sin C − sin A .
2 2 2
Thus the sum of the three terms is zero.

Problem 55. If A + B + C = mπ where m is any integer, then


tan A + tan B + tan C = tan A tan B tan C .
Solution. tan(A + B + C) = tan mπ = 0; and therefore, by Art. 113 (page 328),
tan A + tan B + tan C − tan A tan B tan C = 0.

Problem 56. Show that if α, β, γ, and x are any angles


sin(2α + x) + sin(2β + x) + sin(2γ + x) − sin(2α + 2β + 2γ + 3x)
= 4 sin(α + β + x) sin(β + γ + x) sin(γ + α + x).
CHAPTER VIII : Miscellaneous Propositions 51

Solution.
sin(2α + x) + sin(2β + x) = 2 sin(α + β + x) cos(α − β),
sin(2γ + x) − sin(α + 2β + 2γ + 3x) = −2 sin(α + β + x) cos(α + β + 2γ + 2x);
2 sin(α + β + x) {cos(α − β) − cos(α + β + 2γ + 2x)}
= 2 sin(α + β + x)2 sin(β + γ + x) sin(α + γ + x)
= 4 sin(α + β + x) sin(β + γ + x) sin(γ + α + x).

Problem 57. From the preceding result deduce two special cases by supposing re-
π
spectively that x = 0 and that x = ; and from these cases obtain the first two rela-
2
tions of Art. 114 (page 328).
Solution. If x = 0 we have
sin 2α + sin 2β + sin 2γ − sin(2α + 2β + 2γ) = 4 sin(α + β) sin(β + γ) sin(γ + α).
If then α + β + γ = π we have sin(2α + 2β + 2γ) = 0;
also sin(α + β) = sin γ, sin(β + γ) = sin α, sin(γ + α) = sin β,
so that sin 2α + sin 2β + sin 2γ = 4 sin γ sin α sin β.
π
If x = we have
2
cos 2α + cos 2β + cos 2γ + cos(2α + 2β + 2γ)
= 4 cos(α + β) cos(β + γ) cos(γ + α).
π
If then α + β + γ = we have cos(2α + 2β + 2γ) = −1,
2
also cos(α + β) = sin γ, cos(β + γ) = sin α, cos(γ + α) = sin β,
so that cos 2α + cos 2β + cos 2γ − 1 = 4 sin α sin β sin γ.

Problem 58. If α, β, γ be any angles, show that


α β γ
sin α + sin β + sin γ − 4 cos cos cos
2 {2 2
α+β+γ−π 3α − β − γ + π 3β − α − γ + π
= 2 sin cos + cos
4 4 4 }
3γ − α − β + π α+β+γ−π
+ cos + cos .
4 4
Solution. { }
α β γ γ 1 1
4 cos cos cos = 2 cos cos (α − β) + cos (α + β)
2 2 2 2 2 2
1 1 1 1
= cos (γ + α − β) + cos (γ + β − α) + cos (α + β + γ) + cos (α + β − γ).
2 2 2 2
Thus the left-hand member of the proposed expression
1 1
= sin α + sin β + sin γ − cos (α + β + γ) − cos (β + γ − α)
2 2
1 1
− cos (α + γ − β) − cos (α + β − γ).
2 2
Again
α+β+γ−π 3α − β − γ + π β+γ−α−π
2 sin cos = sin α + sin
4 4 2
β+γ−α
= sin α − cos ;
2
so also
α+β+γ−π 3β − α − γ + π α+γ−β
2 sin cos = sin β − cos ,
4 4 2
α+β+γ−π 3γ − α − β + π α+β−γ
2 sin cos = sin γ − cos ,
4 4 2
and
α+β+γ−π α+β+γ−π α+β+γ−π α+β+γ
2 sin cos = sin = − cos .
4 4 2 2
Thus the result is established.
CHAPTER VIII : Miscellaneous Propositions 52

Problem 59. Express cos 5 θ in terms of cos θ.


Solution.
cos 5θ = cos(3θ + 2θ) = cos 3θ cos 2θ − sin 3θ sin 2θ
( )( ) ( )
= 4 cos3 θ − 3 cos θ 2 cos2 θ − 1 − 3 sin θ − 4 sin3 θ 2 sin θ cos θ
( )( ) ( )
= 4 cos3 θ − 3 cos θ 2 cos2 θ − 1 − 2 sin2 θ 3 − 4 sin2 θ cos θ
( ) ( ) ( ) ( )
= 4 cos3 θ − 3 cos θ 2 cos2 θ − 1 − 2 1 − cos2 θ 4 cos2 θ − 1 cos θ
( )
= 8 cos θ − 10 cos θ + 3 cos θ − 2 −4 cos θ + 5 cos θ − 1 cos θ
5 3 4 2

= 16 cos5 θ − 20 cos3 θ + 5 cos θ.

( )
Problem 60. Show that sin 6 θ = 2 sin θ 16 cos5 θ − 16 cos3 θ + 3 cos θ .
Solution. ( )( )
sin 6θ = 2 sin 3θ cos 3θ = 2 3 sin θ − 4 sin3 θ 4 cos3 θ − 3 cos θ
( )( )
= 2 sin θ 3 − 4 sin2 θ 4 cos3 θ − 3 cos θ
( ) ( )
= 2 sin θ 4 cos2 θ − 1 4 cos3 θ − 3 cos θ
( )
= 2 sin θ 16 cos θ − 16 cos θ + 3 cos θ .
5 3
CHAPTER IX
Construction of Trigonometrical Tables

Problem 1. Let P be any point in a semicircle whose diameter is AB and centre C ;


draw PM perpendicular to AB, and draw PA, PB; from this construction, observing
that the angles BPM and PAM are each equal to half of PCB, deduce the formula
1 − cos A A
= tan2 .
1 + cos A 2
1 1
Solution. Let P CB = A, so that BP M = A and P AM = A. Then
2 2
MB 1 PM 1
= tan A, and = tan A,
PM 2 AM 2
1 MB P M MB CB − CM
so that tan2 A = · = =
2 P M AM AM CA + CM
CM
CP − CM 1 −
= = CP = 1 − cos A .
CP + CM CM 1 + cos A
1+
CP

θ ϕ
a cos ϕ − b tan tan
Problem 2. If cos θ = , then √ 2 = √ 2 .
a − b cos ϕ a+b a−b
a cos ϕ − b
Solution. cos θ = ; therefore
a − b cos ϕ
1 − cos θ a − b cos ϕ − a cos ϕ + b (a + b)(1 − cos ϕ)
= = ;
1 + cos θ a − b cos ϕ + a cos ϕ − b (a − b)(1 + cos ϕ)
θ a+b ϕ
therefore tan2 = tan2 ;
2 a−b 2
θ ϕ
tan2 tan2
therefore 2 = 2.
a+b a−b

Problem 3. If tan2 θ = 2 tan2 ϕ + 1 , then cos 2 θ + sin2 ϕ = 0 .


Solution.
1 1 1 1
cos2 θ = = = = cos2 ϕ;
1 + tan2 θ 1 + 2 tan2 ϕ + 1 2(1 + tan2 ϕ) 2
and cos 2θ = 2 cos2 θ − 1 = cos2 ϕ − 1 = − sin2 ϕ,
therefore cos 2θ + sin2 ϕ = 0.

Problem 4. If sec 2 θ = 2 sec θ cosec θ, then cosec 2 θ = cosec 2 θ − sec2 θ.


Solution.
sec 2θ = 2 sec θ cosec θ;
1 2
therefore = ;
cos 2θ cos θ sin θ
2 cos 2θ
therefore 1= ;
cos θ sin θ
1 2 cos 2θ cos 2θ
therefore = =
sin 2θ sin 2θ cos θ sin θ sin2 θ cos2 θ
cos2 θ − sin2 θ 1 1
= = − .
sin2 θ cos2 θ sin2 θ cos2 θ
Thus cosec 2θ = cosec θ − sec θ.
2 2
CHAPTER IX : Construction of Trigonometrical Tables 54

(n − 1 )2
Problem 5. If tan θ = n tan ϕ, show that tan2 (θ − ϕ) cannot exceed .
4n
Solution.
tan θ − tan ϕ (n − 1) tan ϕ n−1
tan(θ − ϕ) = = = ;
1 + tan θ tan ϕ 1 + n tan2 ϕ cot ϕ + n tan ϕ
(n − 1) 2 (n − 1) 2
therefore tan2 (θ − ϕ) = = .
cot2 ϕ + 2n + n2 tan2 ϕ (n tan ϕ − cot ϕ)2 + 4n
The greatest value of this fraction is when the denominator is least, that is when the
term n tan ϕ − cot ϕ vanishes.

Problem 6. Reduce sin θ + sin ϕ − cos θ sin(θ + ϕ) to a single term.


Solution.
sin θ + sin ϕ − cos θ sin(θ + ϕ)
1 1 1 1
= 2 sin (θ + ϕ) cos (θ − ϕ) − 2 cos θ sin (θ + ϕ) cos (θ + ϕ)
2 { 2 2 } 2
1 1 1
= 2 sin (θ + ϕ) cos (θ − ϕ) − cos θ cos (θ + ϕ)
2 { (2 ) 2 }
1 θ+ϕ 1
= 2 sin (θ + ϕ) cos θ − − cos θ cos (θ + ϕ)
2 2 2
1 1 1
= 2 sin (θ + ϕ) sin θ sin (θ + ϕ) = 2 sin θ sin2 (θ + ϕ).
2 2 2

sin β cos α(tan α + tan β) sin 12 (α − β)


Problem 7. Show that + = 1.
1 − cos(α + β) cos β sin 12 (α + β)
Solution. { }
sin β cos α(tan α + tan β) sin β cos α sin α sin β
= +
1 − cos(α + β) 1 cos α cos β
2 sin2 (α + β)
2
sin β cos α sin(α + β)
= ·
2 1 cos α cos β
2 sin (α + β)
2
1 1
sin β · 2 sin (α + β) cos (α + β)
= 2 2
1
2 sin2 (α + β) cos β
2
1
sin β cos (α + β)
= 2 ;
1
sin (α + β) cos β
2
1 1
sin β cos (α + β) sin (α − β)
and 2 + 2
1 1
sin (α + β) cos β sin (α + β) cos β
( 2) 2
α+β 1 1
sin − β + sin β cos (α + β) sin (α + β) cos β
2 2 2
= = = 1.
1 1
sin (α + β) cos β sin (α + β) cos β
2 2

Problem 8. Find approximately the height of an object which at the distance of a


mile subtends at the eye an angle of one minute.
x
Solution. Let x denote the height in yards; then = tan 1′ , therefore x = 1760 tan 1′ .
1760
The value of tan 1 is approximately equal to the circular measure of 1′ , that is to

π 1760π
; therefore x = approximately.
180 × 60 180 × 60

Problem 9. Find approximately the distance at which a circular plate of six inches
CHAPTER IX : Construction of Trigonometrical Tables 55

diameter must be placed so as just to conceal the Moon, supposing the apparent
diameter of the Moon to be half a degree.
3 1◦
Solution. Let x denote the distance in inches; then = tan ; and taking the tangent
x 4
as approximately equal to the circular measure we have
3 π 12 × 180
= ; therefore x = .
x 180 × 4 π

Problem 10. If sin 3A = n sin A be true for any value of A besides zero, or two right
angles, or a multiple of two right angles, show that n must lie between 3 and −1 ;
solve the equation when n = 2 .
Solution. We have 3 sin A − 4 sin3 A = n sin A; as we suppose that A is not zero nor a
multiple of two right angles we may divide by sin A; thus 3 − 4 sin2 A = n; therefore
3−n
sin2 A = , and as this must lie between zero and unity, n must lie between 3 and
4
−1.
1 π π
If n = 2 we have sin2 A = = sin2 ; therefore A = mπ ± , where m is zero or any
4 6 6
integer.

n sin α cos α
Problem 11. If tan β = , show that tan(α − β) = (1 − n) tan α.
1 − n sin2 α
Solution.
tan α − tan β
tan(α − β) =
1 + tan α tan β
n sin α cos α
tan α −
1 − n sin2 α
=
n sin α cos α
1 + tan α ·
1 − n sin2 α
sin α(1 − n sin2 α) − n sin α cos2 α
=
cos α(1 − n sin2 α) + n sin2 α cos α
sin α − n sin α (1 − n) sin α
= = = (1 − n) tan α.
cos α cos α

Problem 12. If sin 3 θ be given, determine the number of values of tan θ.


Solution. All the angles which have the same sine as 3θ are included in the formula
nπ + (−1)n 3θ. Therefore any expression which gives the value of tan θ in terms of
sin 3θ may be expected to give the value of the tangent of every angle included in the
1
formula tan {nπ + (−1)n 3θ} .
3
Now n must be of one of the following forms :
6m, 6m + 1, 6m + 2, 6m + 3, 6m + 4, 6m + 5.
1
The corresponding values of tan {nπ + (−1)n 3θ} are, by Art. 45 (page 317),
( 3 ) ( )
π 2π
tan θ, tan − θ , tan + θ , tan(π − θ),
( 3 ) 3( )
π 2π
tan π + + θ , tan π + −θ .
3 3
Thus we have six distinct values. They may also by Arts. 48 (page 317) and 50
(page 319) be expressed thus: ( ) ( )
π 2π
± tan θ, ± tan + θ , ± tan +θ .
3 3

( )
Problem 13. Prove that 64 cos8 A + sin8 A = cos 8A + 28 cos 4A + 35 .
1
Solution. cos2 A = (1 + cos 2A); therefore
2
1( )
cos4 A = 1 + 2 cos 2A + cos2 2A
4
CHAPTER IX : Construction of Trigonometrical Tables 56

1 1 1 + cos 4A
= + cos 2A +
4 2 8
3 1 1
= + cos 2A + cos 4A.
8 2 8
1
Similarly sin2 A = (1 − cos 2A);
2
1( )
sin4 A = 1 − 2 cos 2A + cos2 2A
4
3 1 1
= − cos 2A + cos 4A.
8 2 8
∴ cos8 A + sin8 A
( )2 ( )2
3 1 1 3 1 1
= + cos 2A + cos 4A + − cos 2A + cos 4A
{
8 2
( )2 (
8
)2 (
8 2
)2
8 }
3 1 1 3 1
=2 + cos 2A + cos 4A + 2 · · cos 4A
8 2 8 8 8
{ }
9 1 1 3
=2 + cos2 2A + cos2 4A + cos 4A
64 4 64 32
9 1 1 3
= + (1 + cos 4A) + (1 + cos 8A) + cos 4A
32 4 64 16
1
= {cos 8A + 28 cos 4A + 35} .
64

Problem 14. Find all the values of θ and ϕ which satisfy


cos θ cos ϕ + 1 = 0 .
Solution. cos θ cos ϕ = −1.
As the cosine of an angle is never numerically greater than unity, we must have cos θ
and cos ϕ both numerically equal to unity, one being positive and the other negative.
Hence one of the angles must be zero or an even multiple of π, and the other must be
an odd multiple of π.

Problem 15. If n 2 sin2 (α + β) = sin2 α + sin2 β − 2 sin α sin β cos(α − β), show that
1 ±n
tan α = tan β.
1 ∓n
Solution.
sin2 α + sin2 β − 2 sin α sin β cos(α − β)
= sin α{sin α − sin β cos(α − β)} + sin β{sin β · sin α cos(α − β)}
= sin α{sin(α − β + β) − sin β cos(α − β)}
( )
+ sin β{sin α − α − β − sin α cos(α − β)}
= sin α sin(α − β) cos β − sin β cos α sin(α − β)
= sin(α − β){sin α cos β − sin β cos α} = sin2 (α − β).
Thus sin2 (α − β) = n2 sin2 (α + β);
therefore sin(α − β) = ±n sin(α + β);
therefore sin α cos β − cos α sin β = ±n(sin α cos β + cos α sin β);
divide by cos α cos β; thus tan α − tan β = ±n(tan α + tan β);
therefore (1 ∓ n) tan α = (1 ± n) tan β;
1±n
therefore tan α = tan β.
1∓n

sin 4 θ cot θ
Problem 16. Find the limit of , when θ is indefinitely diminished.
vers 2 θ cot2 2 θ
Solution.
sin 4θ cot θ sin 4θ sin2 2θ cos θ 2 sin3 2θ cos 2θ cos θ
= =
2
vers 2θ cot 2θ (1 − cos 2θ) cos 2θ sin θ
2 2 sin3 θ cos2 2θ
3
2(2 sin θ cos θ) cos θ 4
8 cos θ
= = .
2 sin3 θ cos 2θ cos 2θ
CHAPTER IX : Construction of Trigonometrical Tables 57

When θ = 0 the value is therefore 8.

Solve the following equations :


Problem 17. sin θ + cos θ = 2.
Solution.
√ sin θ cos θ
sin θ + cos θ = therefore √ + √ = 1;
2;
( ) 2 2
π π
therefore cos θ − = 1; therefore θ − = 2nπ.
4 4

√ √
Problem 18. 3 sin θ − cos θ = 2.
Solution. √
√ √ 3 1 1
3 sin θ − cos θ = therefore
2; sin θ − cos θ = √ ;
2 2 2

1 3 1
therefore cos θ − sin θ = − √ ;
2
( 2 ) 2
π 1
therefore cos θ + = −√ ;
3 2
π 3π
therefore θ + = 2nπ ± .
3 4

Problem 19. sin 2 θ = cos θ. ( )


π
Solution. sin 2θ = cos θ; therefore cos − 2θ = cos θ;
2
π
Therefore − 2θ and θ are angles having the same cosine; therefore all the solu-
2
π
tions are contained in − 2θ = 2nπ ± θ.
2

Problem 20. cos θ − cos 2 θ = sin 3 θ.


Solution. cos θ − cos 2θ = sin 3θ; therefore
3θ θ 3θ 3θ
2 sin sin = 2 sin cos ;
2 2 2 2

3θ θ 3θ
Therefore either sin = 0, or sin = cos .
2 2 2
3θ 3θ
If sin = 0, then = nπ.
2 2 ( )
θ 3θ π θ 3θ
If sin = cos , then cos − = cos ;
2 2 2 2 2
π θ 3θ
and therefore − = 2nπ ± .
2 2 2


Problem 21. (4 − 3 )(sec θ + cosec θ) = 4 (sin θ tan θ + cos θ cot θ).
Solution. √
(4 − 3)(sec θ + cosec θ) = 4(sin θ tan θ + cos θ cot θ);
( )
√ ( 1 1
)
sin2 θ cos2 θ
therefore (4 − 3) + =4 + ;
cos θ sin θ cos θ sin θ
√ ( )
therefore (4 − 3)(sin θ + cos θ) = 4 sin3 θ + cos3 θ
( )
= 4(sin θ + cos θ) sin2 θ + cos2 θ − sin θ cos θ ;
therefore either sin θ + cos θ = 0,

or 4 − 3 = 4(1 − sin θ cos θ).
If sin θ + cos θ = 0, then sin θ = − cos θ; therefore tan θ = −1;
CHAPTER IX : Construction of Trigonometrical Tables 58


therefore θ = nπ +.
√ √4
If 4 − 3 = 4(1 − sin θ cos θ), then 3 = 4 sin θ cos θ = 2 sin 2θ;

3 π
therefore sin 2θ = ; therefore 2θ = nπ + (−1)n .
2 3

Problem 22. cot θ − tan θ = cos θ + sinθ.


Solution.
cos θ sin θ
cot θ − tan θ = cos θ + sin θ; therefore − = cos θ + sin θ;
sin θ cos θ
therefore cos2 θ − sin2 θ = sin θ cos θ(cos θ + sin θ);
therefore either cos θ + sin θ = 0, or cos θ − sin θ = sin θ cos θ.
If sin θ + cos θ = 0, then sin θ = − cos θ; therefore tan θ = −1;

therefore θ = nπ + .
4
If cos θ − sin θ = sin θ cos θ, then by squaring
1 − 2 sin θ cos θ = sin2 θ cos2 θ;
sin2 2θ
therefore 1 − sin 2θ = .
4 √
By solving this quadratic in the usual way we obtain sin 2θ = −2 ± 2 2; the upper sign
must be taken, for the lower sign would make sin 2θ numerically greater than unity.

Problem 23. 2 sin2 θ + sin2 2 θ = 2 .


Solution.
2 sin2 θ + sin2 2θ = 2; therefore sin2 2θ = 2 − 2 sin2 θ = 2(1 − sin2 θ);
therefore 4 sin2 θ cos2 θ = 2 cos2 θ;
1
therefore either cos2 θ = 0, or sin2 θ = .
2
π
If cos2 θ = 0, then θ = nπ + .
2
2 1 2 2 π
If sin θ = , then sin θ = sin ;
2 4
π
therefore θ = nπ ± .
4

( )
θ
Problem 24. tan θ + 2 cot 2 θ = sin θ 1 + tan θ tan .
2
Solution. ( )
θ
tan θ + 2 cot 2θ = sin θ 1 + tan θ tan
 2
θ

sin θ 2 cos 2θ sin θ sin
therefore + = sin θ 1 + 2 ;
cos θ sin 2θ θ
cos θ cos
( ) 2
θ
cos θ −
sin2 θ + cos 2θ 2 sin θ
therefore = sin θ · = ;
sin θ cos θ θ cos θ
cos θ cos
2
therefore sin2 θ + cos 2θ = sin2 θ; therefore cos 2θ = 0;
π
therefore 2θ = nπ + .
2

π
Problem 25. sin2 2 θ − sin2 θ = sin2 .
6
Solution.
π 1
sin2 2θ − sin2 θ = sin2 = ; therefore
6 4
CHAPTER IX : Construction of Trigonometrical Tables 59

1
4 sin2 θ cos2 θ − sin2 θ = ;
4
1
therefore 4 sin2 θ(1 − sin2 θ) − sin2 θ = ;
4
1
therefore 4 sin4 θ − 3 sin2 θ +
= 0.
4
By solving this quadratic in the usual way we obtain

3± 5
sin2 θ = .
8

Taking the upper sign we have sin2 θ = sin2 , and therefore
10

θ = nπ ± .
10
2 2 π
Taking the lower sign we have sin θ = sin , and therefore
10
π
θ = nπ ± .
10

θ
Problem 26. cosec θ = cosec .
2
Solution.
θ 1 1
cosec θ = cosec ; therefore ;=
2 θ
sin θ
sin
2
θ θ θ θ
therefore sin = sin θ; therefore sin = 2 sin cos ;
2 2 2 2
θ θ 1
therefore either sin = 0, or cos = .
2 2 2
θ θ
If sin = 0, then = nπ.
2 2
θ 1 θ π
If cos = , then = 2mπ ± .
2 2 2 3

Problem 27. cos θ cos 3 θ = cos 5 θ cos 7 θ.


Solution.
cos θ cos 3θ = cos 5θ cos 7θ;
therefore cos 4θ + cos 2θ = cos 12θ + cos 2θ;
therefore cos 4θ = cos 12θ; therefore 12θ = 2nπ ± 4θ;
2nπ nπ
taking the upper sign we obtain θ= = ,
8 4
2nπ nπ
and taking the lower sign we obtain θ = = .
16 8
It is obvious however that the second expression includes the first.

1
Problem 28. sin θ sin 3 θ = .
2
Solution.
1 1
sin θ sin 3θ = ; therefore sin θ(3 sin θ − 4 sin3 θ) = ;
2 2
1
therefore 4 sin4 θ − 3 sin2 θ +
= 0.
2
By solving this quadratic in the usual way we obtain
3±1 1 1
sin2 θ = = or .
8 2 4
1 2 π π
If 2
sin θ = , then 2
sin θ = sin , and θ = nπ ± .
2 4 4
1 π π
If sin2 θ = , then sin2 θ = sin2 , and θ = nπ ± .
4 6 6
CHAPTER IX : Construction of Trigonometrical Tables 60

See P roblem 5 of Chapter v.

Problem 29. 4 sin2 θ + sin2 2 θ = 3 .


Solution.
4 sin2 θ + sin2 2θ = 3; therefore 4 sin2 θ + 4 sin2 θ(1 − sin2 θ) = 3;
therefore 4 sin4 θ − 8 sin2 θ + 3 = 0.
1 3
By solving this quadratic in the usual way we obtain sin2 θ = or ; and only the
2 2
π π
former value is admissible. Thus sin2 θ = sin2 ; therefore θ = nπ ± .
4 4

Problem 30. (1 − tan θ)(1 + sin 2 θ) = 1 + tan θ.


Solution.
(1 − tan θ)(1 + sin 2θ) = 1 + tan θ;
( )
sin θ sin θ
therefore 1− (sin θ + cos θ)2 = 1 + .
cos θ cos θ
therefore (cos θ − sin θ)(cos θ + sin θ) = cos θ + sin θ;
2

therefore either cos θ + sin θ = 0, or (cos θ − sin θ)(cos θ + sin θ) = 1;


If cos θ + sin θ = 0, then sin θ = − cos θ;
therefore tan θ = −1;

therefore θ = nπ + .
4
If (cos θ − sin θ)(cos θ + sin θ) = 1, then cos2 θ − sin2 θ = 1;
therefore cos 2θ = 1;
therefore 2θ = 2nπ.

Problem 31. sin θ + sin 2 θ + sin 3 θ + sin 4 θ = 0 .


Solution.
sin θ + sin 2θ + sin 3θ + sin 4θ = 0;
therefore sin θ + sin 4θ + sin 2θ + sin 3θ = 0;
5θ 3θ 5θ θ
therefore 2 sin cos + 2 sin cos = 0;
2 (2 2 ) 2
5θ 3θ θ
therefore 2 sin cos + cos = 0;
2 2 2
5θ θ
therefore 4 sin cos cos θ = 0.
2 2
Thus there are three cases :
5θ 5θ
If sin = 0, then = nπ,
2 2
θ θ π
If cos = 0, then = nπ + ,
2 2 2
π
If cos θ = 0, then θ = nπ + .
2

Problem 32. sin θ − cos θ = 4 sin θ cos2 θ.


Solution.
sin θ − cos θ = 4 sin θ cos2 θ;
therefore sin θ − 4 sin θ(1 − sin2 θ) = cos θ;
therefore 4 sin3 θ − 3 sin θ = cos θ;
( )
π
therefore cos θ = − sin 3θ = cos 3θ + ;
2
π
therefore 3θ + = 2nπ ± θ.
2

√ √
Problem 33. (cot θ − tan θ)2 (2 − 3 ) = 4 (2 + 3 ).
CHAPTER IX : Construction of Trigonometrical Tables 61

Solution. √ √
(cot θ − tan θ)2 (2 −3) = 4(2 + 3);
( ) √
cos θ sin θ 2 4(2 + 3)
therefore − = √ ;
sin θ cos θ 2− 3
( )2 √ √
cos2 θ − sin2 θ 2+ 3 (2 + 3)2
therefore = √ = √ √ ;
2 sin θ cos θ 2− 3 (2 − 3)(2 + 3)
( ) √
cos 2θ 2
therefore = (2 + 3)2 ;
sin 2θ
π
therefore cot2 2θ = cot2 ;
12
π
therefore 2θ = nπ ± .
12


( )
π
Problem 34. 2 2 cos − θ (1 + sin θ) = 1 + cos 2 θ.
4
Solution.
√ ( )
π
2 2 cos − θ (1 + sin θ) = 1 + cos 2θ;
√ ( )4
π
therefore 2 2 cos − θ (1 + sin θ) = 2 cos2 θ = 2(1 − sin2 θ);
4
√ ( )
π
therefore either 1 + sin θ = 0, or 2 cos − θ = 1 − sin θ.
4

If 1 + sin θ = 0, then sin θ = −1; therefore θ = nπ + (−1)n , which may be expressed
2
π
more simply as (4m + 3) .
√ ( )2 √ ( 1 )
π 1
If 2 cos − θ = 1 − sin θ, then 2 √ cos θ + √ sin θ = 1 − sin θ;
4 2 2
therefore 2 sin θ = 1 − cos θ;
θ θ θ
therefore 4 sin cos = 2 sin2 ;
2 2 2
θ θ
therefore either sin = 0, or tan = 2.
2 2
θ θ
If sin = 0, then = nπ.
2 2
θ θ
If tan = 2, then = nπ + α, where α is such that tan α = 2.
2 2

Problem 35. sin 9 θ + sin 5 θ + 2 sin2 θ = 1 .


Solution. sin 9θ + sin 5θ + 2 sin2 θ = 1; therefore
2 sin 7θ cos 2θ = 1 − 2 sin2 θ = cos 2θ;
1
therefore either cos 2θ = 0, or sin 7θ = .
2
π
If cos 2θ = 0, then 2θ = nπ + .
2
1 π
If sin 7θ = , then 7θ = nπ + (−1)n .
2 6
CHAPTER X
Logarithms and Logarithmic Series.


Problem 1. Find the logarithm of 128 to the base 3 4.
Solution. Let x denote the required logarithm; then
(√3
)x x 2x
128 = 4 , that is 27 = 4 3 = 2 3 ;
2x 21
therefore = 7; therefore x = .
3 2
√ √
Problem 2. Find the logarithm of 243 3 9 to the base 3 .
Solution. Let x denote the required logarithm; then
√3
(√ )x √
3 x 2 x
243 9 = 3 , that is 35 9 = 3 2 , that is 35+ 3 = 3 2 ;
x 17 34
therefore = ; therefore x = .
2 3 3

Problem 3. Find the following logarithms, log3 2187 , log10 ·0001 , log2 cos 45 ◦ .
Solution. Let x denote the logarithm of 2187 to the base 3; then 2187 = 3x , that is
37 = 3x ; therefore x = 7.
1
Let x denote the logarithm of ·0001 to the base 10; then ·0001 = 10x , that is =
104
10x , that is 10−4 = 10x ; therefore x = -4.
1
Let x denote the logarithm of cos 45◦ to the base 2; then cos 45◦ = 2x , that is √ = 2x ,
2
−21 1
that is 2 = 2 ; therefore x = − .
x
2

Problem 4. Find approximately the value of x from the equation 5 6 −4x = 2 x+3 , hav-
ing given log 2 = ·301030 .
Solution. 56−4x = 2x+3 ; therefore (6 − 4x) log 5 = (x + 3) log 2;
10
therefore (6 − 4x) log = (x + 3) log 2;
2
therefore (6 − 4x)(1 − log 2) = (x + 3) log 2;
therefore x(4 − 3 log 2) = 6 − 9 log 2;
6 − 9 log 2 3·29073
therefore x= = = 1·06 . . .
4 − 3 log 2 3·09691

Problem 5. Given log ·224 = a and log 125 = b, find log 2 and log 7 .
Solution.
224 7 × 32
Here a = log ·224 = log = log = log 7 + 5 log 2 − 3;
1000 1000
1000
b = log 125 = log = 3 − 3 log 2.
8
1
From the second equation we have log 2 = (3 − b); and then substituting in the first
3
5
equation we have log 7 = a + 3 − (3 − b.)
3

Problem 6. Required the characteristics of log6 725 , and of log6 5 ·0725.
Solution. 725 lies between 63 and 64 ; and therefore the characteristic of the logarithm
of 725 to the base 6 is 3.
√ 1 1 725
Then log 5 (·0725) = log ·0725 = log ;
5 5 10000
CHAPTER X : Logarithms and Logarithmic Series 64

725 1 1 1 725
and lies between and , that is between 6−1 and 6−2. Hence log to
10000 6 36 5 10000
1 2
the base 6 lies between − and − ; and thus the characteristic will be −1, since by
5 5
supposition the decimal part of a logarithm is positive.

Problem 7. Given log 2 = ·301030 , log 405 = 2 ·607455 , find log ·003 .
( )
10 34 × 10
Solution. log 405 = log(81 × 5) = log 81 × = log = 4 log 3 + 1 − log 2;
2 2
therefore 4 log 3 = log 405 + log 2 − 1 = 8·908485;
therefore log 3 = ·477121.

( )1
4 2
Problem 8. Given log 2 = ·301030 , log 7 = ·845098 , find log 98 and log .
( ) 343
Solution. log 98 = log 2 × 7 = log 2 + 2 log 7 = ·301030 + 1·690196 = 1·991226;
2

( ) 12
4 1 4 1 22 1
log = log = log 3 = (2 log 2 − 3 log 7)
343 2 343 2 7 2
= −·966617 = 1̄·033383.

1
Problem 9. Given log 2 = ·30103 , log 3 = ·47712 , find log(·0020736 ) 3 .
Solution.
1 1 1 20736
log(·0020736) 3 = log ·0020736 = log
3 3 107
1 34 × 28 1
= log = {4 log 3 + 8 log 2 − 7}
3 107 3
= −·89443 = 1̄·10557.

Problem 10. Determine the sum of the series


2 4 6
+ + + . . . ad inf.
3 5 7
2 1 1 4 1 1 6 1 1
Solution. = − , = − , = − , ...
3 2 3 5 4 5 7 6 7
1 1 1 1
thus we see that the series = − + − + . . . = e−1 .
2 3 4 5

Problem 11. Show that


e 1 1+2 1+2+3 1+2+3+4
= + + + + . . . ad inf.
2 2 3 4 5
Solution.
1 1 1·2 1
= · = ,
2 2 2 2
1+2 1 2·3 1 1
= · = · ,
3 2 3 2 1
1+2+3 1 3·4 1 1
= · = · ,
4 2 4 2 2
1+2+3+4 1 4·5 1 1
= · = · ,
5 2 5 2 3
1 + 2 + 3 + ... + n 1 n(n + 1) 1 1
and generally = · = ·
n+1 2 n+1 2 n−1 .
{ }
1 1 1 1 e
Thus we see that the series = 1+ + + + ... = .
2 1 2 3 2
CHAPTER X : Logarithms and Logarithmic Series 65

Find x from the following six equations :

Problem 12. 4 sin x sin(x − α) = 2 cos α − 1 .


Solution. 4 sin x sin(x − a) = 2 cos α − 1;
therefore 2 {cos α − cos(2x − α)} = 2 cos α − 1;
1
therefore cos(2x − α) = ;
2
π
therefore 2x − α = 2nπ ± .
3


√β a − x + a sin α = x sin β.
Problem 13. cos 2 2

Solution. cos β (a2 − x2 ) = x sin β − a sin α;


therefore cos2 β(a2 − x2 ) = x2 sin2 β − 2xa sin β sin α + a2 sin2 α;
therefore x2 − 2xa sin β sin α = a2 cos2 β − a2 sin2 α;

therefore (x − a sin β sin α)2 = a2 cos2 β − a2 sin2 α + a2 sin2 β sin2 α


= a2 cos2 β − a2 sin2 α cos2 β = a2 cos2 β cos2 α;
therefore x − a sin β sin α = ±a cos β cos α;
therefore x = a(sin β sin α ± cos β cos α) = a cos(β − α) or − a cos(β + α).

Problem 14. sin α + sin(x − α) + sin(2x + α) = sin(x + α) + sin(2x − α).


Solution. sin α + sin(x − α) + sin(2x + α) = sin(x + α) + sin(2x − α);
therefore sin α = sin(x + α) − sin(x − α) + sin(2x − α) − sin(2x + α)
= 2 sin α cos x − 2 sin α cos 2x;
therefore 1 = 2 cos x − 2 cos 2x = 2 cos x − 2(2 cos2 x − 1);
therefore 4 cos2 x − 2 cos x − 1 = 0.

1± 5
By solving this quadratic in the usual way we obtain cos x = .
4
π π
Taking the upper sign we have cos x = cos , and therefore x = 2nπ ± .
5 5
3π 3π
Taking the lower sign we have cos x = cos , and therefore x = 2nπ ± .
5 5
( ) ( )
3 1
Problem 15. cos x + α + cos x + α = sin α.
( )2 ( ) 2
3 1
Solution. cos x + α + cos x + α = sin α;
2 2
α α α
therefore 2 cos(x + 1)α cos = sin α = 2 sin cos ;
2 ( 2 ) 2
α π α
therefore cos(x + 1)α = sin = cos − .
2 2 2
Hence all the solutions are contained in ( )
π α
(x + 1)α = 2nπ ± − .
2 2

( )
β β
Problem 16. x 2 cos α cos α − + x cos(α − β) = 2 cos .
( )2 2
β β
Solution. x2 cos α cos α − + x cos(α − β) = 2 cos ;
2 2
β
x cos(α − β) 2 cos
therefore x2 + ( )= (2 );
β β
cos α cos α − cos α cos α −
2 2
CHAPTER X : Logarithms and Logarithmic Series 66

 2
  β
cos(α − β) 2 cos
∴ x+ ( ) = (2 )
 2 cos α cos α −
β  cos α cos α −
β
2 2
cos2 (α − β)
+ ( )
β
4 cos2 α cos2 α −
( ) 2
β β
cos2 (α − β) + 8 cos α cos
cos α −
2 2
= ( )
β
4 cos α cos α −
2 2
2
cos (α − β) + 4 cos α{cos α + cos(α − β)}
2
= ( )
β
4 cos2 α cos2 α −
2
{cos(α − β) + 2 cos α}2
= ( );
β
4 cos2 α cos2 α −
2
cos(α − β) cos(α − β) + 2 cos α
therefore x+ ( ) =± ( ).
β β
2 cos α cos α − 2 cos α cos α −
2 2
Taking the upper sign we have
( )
2 cos α β
x= ( ) = sec α − .
β 2
2 cos α cos α −
2
Taking the lower sign we have
cos α + cos(α − β)
x=− ( )
β
cos α cos α −
( )2
β β
2 cos α − cos
2 β
=− ( )2 = −2 cos sec α.
β 2
cos α cos α −
2
Or we may write the proposed equation in this form
{ ( ) } { ( ) }
β β β
x cos α x cos α − − 1 + 2 x cos α − − 1 cos = 0;
2 2 2
and then the two values of x which satisfy it are obvious.

−1
Problem 17. cot 2 x α − cot 2 x α = cosec 3 α.
Solution. cot 2x−1 α − cot 2x α = cosec 3α;
put y for 2x−1 α; thus cot y − cot 2y = cosec 3α;
cos y cos 2y
therefore − = cosec 3α;
sin y sin 2y
sin(2y − y) 1
therefore = cosec 3α = ;
sin y sin 2y sin 3α
x
therefore sin 2y = sin 3α, that is sin 2 α = sin 3α.
Thus the general solution is 2x α = nπ + (−1)n 3α.

Problem 18. Solve the equation m vers θ = n vers (α − θ).


Solution. m vers θ = n vers (α − θ);
therefore m(1 − cos θ) = n{1 − cos(α − θ)};
θ α−θ
therefore 2m sin2 = 2n sin2 ;
2 2
( ) 12
α−θ m θ
therefore sin = sin ;
2 n 2
CHAPTER X : Logarithms and Logarithmic Series 67

( ) 12
α θ α θ m θ
therefore sin cos − cos sin = sin ;
2 2 2 2 n 2
θ θ
Divide by cos ; thus we obtain a simple equation for finding tan .
2 2

Problem 19. Solve the equation cos nθ + cos(n − 2 )θ = cos θ.


Solution. cos nθ + cos(n − 2)θ = cos θ;
therefore 2 cos(n − 1)θ cos θ = cos θ;
1
therefore either cos θ = 0, or cos(n − 1)θ = .
2
π
If cos θ = 0, then θ = mπ + .
2
1 π
If cos(n − 1)θ = , then (n − 1)θ = 2mπ ± .
2 3

Problem 20. Solve the following equation, and show that there are seven positive
values of θ greater than 0 and less than 2 π,
sin θ + sin 3 θ = sin 2 θ + sin 4 θ.
Solution. sin θ + sin 3θ = sin 2θ + sin 4θ;
therefore 2 sin 2θ cos θ = 2 sin 3θ cos θ;
therefore either cos θ = 0, or sin 2θ = sin 3θ.
π
If cos θ = 0, then θ = nπ + .
2
θ 5θ
If sin 2θ = sin 3θ, then sin 2θ − sin 3θ = 0; therefore 2 sin cos = 0;
2 2
θ 5θ θ θ
therefore either sin = 0, or cos = 0 : taking sin = 0 we have = nπ,
2 2 2 2
5θ 5θ π
and taking cos = 0 we have = nπ + .
2 2 2
The seven values greater than 0 and less than 2π are
π 3π 5π 7π 9π π 3π
, , , , , and .
5 5 5 5 5 2 2

Problem 21. Find tan x from the equation tan x = tan β tan(α + x); and show that in or-
der that tan x may be real, tan β must not lie between (sec α − tan α)2 and (sec α + tan α)2 .
tan β(tan x + tan α)
Solution. tan x = tan β tan(x + α) = ;
1 − tan x tan α
therefore tan x(1 − tan x tan α) = tan β(tan x + tan α);
therefore tan2 x tan α + (tan β − 1) tan x + tan α tan β = 0.
By solving this quadratic in the usual way we obtain the values of tan x. It is known
by the theory of quadratic equations that for the values to be real we must have
(tan β − 1)2 − 4 tan2 α tan β positive or zero.
And (tan β − 1)2 − 4 tan2 α tan β.
= tan2 β − 2 tan β − 4 tan2 α tan β + 1
{ }2 ( )2
= tan β − (1 + 2 tan2 α) + 1 − 1 + 2 tan2 α
{ }2
1 + sin2 α 4 sin2 α
= tan β − −
cos2 α cos4 α
{ }{ }
1 + sin2 α 2 sin α 1 + sin2 α 2 sin α
= tan β − − tan β − +
cos2 α cos2 α cos2 α cos2 α
{ ( ) }{ ( ) }
1 + sin α 2 1 − sin α 2
= tan β − tan β − .
cos α cos α
This expression then must be positive or zero, and therefore tan β must not lie
( ) ( )
1 − sin α 2 1 + sin α 2
between and .
cos α cos α
CHAPTER X : Logarithms and Logarithmic Series 68

Problem 22. Find the least value of θ which satisfies


( ) ( ) ( √ ) 21
π π 8 2
tan −θ + tan +θ = √ .
4 4 1+ 2
( ) ( )
π π
Solution. tan − θ + tan +θ
4( ) 4 ( )
π π
sin −θ sin +θ
= (4 )+ (4 )
π π
cos −θ cos +θ
(4 ) ( 4) ( ) ( )
π π π π
sin − θ cos + θ + sin + θ cos −θ
4
= (4 ) ( 4 ) 4
π π
cos − θ cos +θ
4 4
π
sin 1
= ( ) 2 ( )= ( ) ( )
π π π π
cos − θ cos +θ sin + θ cos +θ
4 4 4 4
2 2
= ( )= .
π cos 2θ
sin + 2θ
2
( √ ) 12
2 8 2
Thus = √ ;
cos 2θ 1+ 2
( √ )1
cos 2θ 1+ 2 2
therefore = √ ;
2 8 2

1+ 2
therefore cos2 2θ = √ ;
2 2

1+ 2 1
therefore 2 cos2 2θ − 1 = √ −1= √ ;
2 2
1 π
therefore cos 4θ = √ = cos ;
2 4
π
therefore the least value of θ is given by 4θ = .
4

Problem 23. Given sin2 (n + 1 )θ = sin2 nθ + sin2 (n − 1 )θ where (n + 1 )θ, nθ, and
(n − 1 )θ are the angles of a triangle, find an integral value of n.
Solution.
sin2 (n + 1)θ = sin2 nθ + sin2 (n − 1)θ;
therefore sin2 (n + 1)θ − sin2 (n − 1)θ = sin2 nθ;
therefore sin 2nθ sin 2θ = sin2 nθ. (Art. 83) (page 324)
But (n + 1)θ + (n − 1)θ + nθ = π;
π
therefore 3nθ = π; therefore nθ = ;
3
2π π
therefore sin 2θ sin = sin2 ;
3 3
π
therefore sin 2θ = sin ;
3
π π π
thus 2θ = ; therefore θ = . But nθ = ; and therefore n = 2.
3 6 3

Problem 24. Reduce to its simplest form and solve the equation
cos2 θ − cos2 α = 2 cos3 θ(cos θ − cos α) − 2 sin3 θ(sin θ − sin α).
Solution. cos2 θ − cos2 α = 2 cos3 θ(cos θ − cos α) − 2 sin3 θ(sin θ − sin α);

cos 3θ + 3 cos θ
therefore cos2 θ − cos2 α = (cos θ − cos α)
2
CHAPTER X : Logarithms and Logarithmic Series 69

3 sin θ − sin 3θ
− (sin θ − sin α);
2
therefore 2(cos2 θ − cos2 α) = cos 3θ cos θ + sin 3θ sin θ
− cos 3θ cos α − sin 3θ sin α
+ 3 cos2 θ − 3 sin2 θ − 3 cos θ cos α
+ 3 sin θ sin α;

∴ cos(3θ − θ) − cos(3θ − α) − 3 cos(θ + α) = 3 sin2 θ − cos2 θ − 2 cos2 α;

∴ cos 2θ − cos(3θ − α) − 3 cos(θ + α) = 3 − 4 cos2 θ − 2 cos2 α


= 3 − 2(1 + cos 2θ) − (1 + cos 2α)
= −2 cos 2θ − cos 2α;

therefore 3 cos 2θ − 3 cos(θ + α) − cos(3θ − α) + cos 2α = 0;


3θ + α α−θ (3θ + α) 3θ − 3α
therefore 3 sin sin + sin sin = 0;
2 { 2 2 }2
3θ + α 3(θ − α) θ−α
therefore sin sin − 3 sin = 0;
2 2 2
3θ + α θ−α
therefore 4 sin sin3 = 0.
2 2
3θ + α θ−α 3θ + α
Hence either sin = 0, or sin = 0; the former gives = nπ, and the
2 2 2
θ−α
latter gives = nπ.
2

Problem 25. ) all( the angles


( Show that ) which have the same sine as α are included in
1 π
the formula 2n + π± −α .
2 2
( Let )θ denote
Solution. ( an angle
) having the same sine as α, so that sin θ = sin α;
π π π
thus cos θ − = cos − α ; therefore all the solutions are comprised in θ − =
( )
2 2 2
π
2nπ ± −α .
2

(
Problem 26. Show ) all the angles
that ( which) have the same cosine as α are included
1 π
in the formula n + π + (−1 ) n α− .
2 2
Solution.( Let )
θ denote ( an )
angle having the same cosine as α, so that cos θ = cos α;
π π π
thus sin θ − = sin α − ; therefore all the solutions are comprised in θ − =
( 2 ) 2 2
π
nπ + (−1)n α − .
2

A A √
Problem 27. In the formula cos − sin = ± 1 − sin A the ambiguous ± may be
2 2
270 + A
replaced by (−1 )m , where m is the greatest integer contained in , the angle
360
A being expressed in degrees.
A
Solution. By Art. 101 (page 327) it follows that the upper sign ought to be taken if
2
lies between n 360◦ + 225◦ and n 360◦ + 405◦ ; in this case A lies between 2n 360◦ + 450◦ ,
and 2n 360◦ + 810◦ , and A + 270◦ lies between 2n 360◦ + 720◦ and 2n 360◦ + 1080◦ , and
A + 270◦
therefore lies between 2n+2 and 2n+3 : thus the integral part of this fraction
360◦
is an even number, so that denoting it by m we have (−1)m positive.
In precisely the same manner we find that the present problem agrees with Art. 101
(page 327) for the case in which m is odd.
CHAPTER X : Logarithms and Logarithmic Series 70


A ± 1 + tan2 A − 1
Problem 28. In the formula tan = the ambiguity ± may be re-
2 tan A
90 + A
placed by (−1 )m , where m is the greatest integer contained in , the angle A
180
being expressed in degrees.
Solution. First suppose the number of degrees in A to lie between n 360 and n 360+90;
A
then tan A and tan are both positive, and therefore the upper sign must be taken in
2
A + 90 n 360 + 90 n 360 + 180
the ambiguity. Also in this case lies between and , that
180 180 180
1
is between 2n + and 2n + 1; so that m is even.
2
Next suppose the number of degrees in A to lie between n 360 + 90 and n 360 + 180;
A
then tan A is negative, and tan is positive; and therefore the lower sign must be
2
A + 90
taken in the ambiguity. Also in this case lies between 2n + 1 and 2n + 2, so that
180
m is odd.
Similarly we may proceed if the number of degrees in A lies between n 360 + 180
and n 360 + 270, or between n 360 + 270 and n 360 + 360.
It will be observed that in this and the preceding problem the greatest integer in
a certain expression means that integer which with a positive proper fraction consti-
tutes the whole expression.
Or we might treat the example thus :

√ 1 1
± (1 + tan2 A) = ± =± ;
cos2 A cos A
1
A 1 − cos A −1
but tan = = cos A :
√ 2 sin A tan A
hence the ambiguity in ± (1 + tan A) must be so taken as to ensure that the sign is
2

the same as the sign of cos A, and it is easy to show that (−1)m is of the same sign as
cos A, when m has the prescribed value.

Problem 29. If tan(cot x) = cot(tan x), show that the real values of x are given by
4
sin 2x = , where n is any integer except −1 .
(2n + 1 )π
Solution.
tan(cot x) = cot(tan x);
{ }
π
therefore tan(cot x) = tan − tan x ;
2
therefore, by Art. 68 (page 322), all the possible solutions are comprised in
π
cot x = nπ + − tan x;
2
π
therefore cot x + tan x = nπ + ;
2
cos x sin x π
therefore + = nπ + ;
sin x cos x 2
1 (2n + 1)π
therefore = ;
sin x cos x 2
2
therefore sin x cos x = ;
(2n + 1)π
4
therefore sin 2x = .
(2n + 1)π
The value n = −1 would make sin 2x greater than unity.

A
Problem 30. Show how to express cos in terms of cos A, where n is any positive
2n
integer.
CHAPTER X : Logarithms and Logarithmic Series 71

Solution.
A
2 cos2 = 1 + cos A;
2
A
therefore 4 cos2 = 2 + 2 cos A;
2
A √
therefore 2 cos = (2 + 2 cos A).
2
A A
Again 2 cos2 = 1 + cos ;
4 2
A A
therefore 4 cos2 = 2 + 2 cos ;
4 √ (
2
) √{ }
A A √
therefore 2 cos = 2 + 2 cos ) = 2+ (2 + 2 cos A) .
4 2
√[ √{
√ }]
A
Similarly 2 cos = 2+ 2+ (2 + 2 cos A) .
8
and this process may be continued to any extent.


1 + cos 2x
Problem 31. From the equation cos x = ± deduce the formula for sin x in
2
terms of sin 2x, and show how the proper signs for the radicals may be determined.
π π
Solution. Change x successively to − x and + x; thus
( ) √ {
4
(
4
)} √
π 1 π 1 + sin 2x
cos x − =± 1 + cos 2x − =± ,
4 2 2 2
( ) √ { ( )} √
π 1 π 1 − sin 2x
and cos +x =± 1 + cos + 2x =± .
4 2 2 2
( ) ( )
π π
Then putting for cos −x and cos +x their values we have
4 4 √
1 1 1 + sin 2x
√ cos x + √ sin x = ± (1)
2 2 2

1 1 1 − sin 2x
and √ cos x − √ sin x = ± (2)
2 2 2
Hence by subtraction we find the required )
( expression for sin x. In (1) the upper
π
or lower sign must be taken according as cos x − is positive or negative, that is
4
π 1 1 1
according as x − lies between 2nπ − π and 2nπ + π, or between 2nπ + π and
4 2 2 2

2nπ + . Similarly we can determine the sign to be taken in (2) .
2

A cos(θ + α) + B sin(θ + β)
Problem 32. If the expression retain the same value for
A′ sin(θ + α) + B ′ cos(θ + β)
all values of θ, then will
AA′ − BB ′ = (A′ B − AB ′ ) sin(α − β).
Solution. Let k denote the value which the expression retains for all values of θ, so
that
A cos(θ + α) + B sin(θ + β)
= k;
A′ sin(θ + α) + B ′ cos(θ + β)
then A cos(θ + α) + B sin(θ + β) = k{A sin(θ + α) + B ′ cos(θ + β)};

therefore cos θ(A cos α + B sin β) + sin θ(B cos β − A sin α)


= k cos θ(A′ sin α + B ′ cos β) + k sin θ(A′ cos α − B ′ sin β)}
{ }
therefore cos θ A cos α + B sin β − k(A′ sin α + B ′ cos β)
{ }
+ sin θ B cos β − A sin α − k(A′ cos α − B ′ sin β) = 0.
CHAPTER X : Logarithms and Logarithmic Series 72

π
Now this is to be true for all values of θ. Put for θ in succession 0 and ; thus we
2
obtain the following two results:
A cos α + B sin β = k(A′ sin α + B ′ cos β),
B cos β − A sin α = k(A′ cos α − B ′ sin β);
and it is obvious that if these hold the original expression does always retain the same
value.
By cross multiplication we obtain
(A cos α + B sin β)(A′ cos α − B ′ sin β) = (A′ sin α + B ′ cos β)(B cos β − A sin α);
therefore AA′ cos2 α − BB ′ sin2 β + (A′ B − AB ′ ) cos α sin β
= BB ′ cos2 β − AA′ sin2 α + (A′ B − AB ′ ) sin α cos β;
therefore AA′ − BB ′ = (A′ B − AB ′ ) sin(α − β).

Problem 33. If the sum of two angles is given, show that the sum of their sines is
numerically greatest when the angles are equal. If the cosine of the given sum is
positive show that the sum of the tangents is numerically least when the angles are
equal.
Solution. Let A denote the sum of the two angles x and y. Then
x+y x−y A x−y
sin x + sin y = 2 sin cos = 2 sin cos ;
2 2 2 2
x−y
and the numerically greatest value of this expression is when cos is greatest,
2
that is when x − y = 0, that is when x = y.
sin x sin y sin(x + y)
Again tan x + tan y = + =
cos x cos y cos x cos y
sin A 2 sin A
= =
cos x cos y 2 cos x cos y
2 sin A 2 sin A
= = ;
cos(x − y) + cos(x + y) cos(x − y) + cos A
and if cos A is positive the numerically least value of this is when
cos(x − y) = 1, that is when x = y.

Problem 34. If A + B + C = 90 ◦ , show that unity is the least value of


tan2 A + tan2 B + tan2 C .
Solution. By Art. 114 (page 328) we have
tan A tan B + tan B tan C + tan C tan A = 1;
1
∴ tan2 A + tan2 B + tan2 C = 1 + (tan A − tan B)2
2
1 1
+ (tan B − tan C)2 + (tan C − tan A)2 .
2 2
Hence the least value of the expression is when tan A − tan B, tan B − tan C, and
tan C − tan A all vanish; and the value is then unity.

Problem 35. If A + B + C = 180 ◦ , show that unity is the least value of


cot2 A + cot2 B + cot2 C .
Solution. By Art. 114 (page 328) we have
tan A + tan B + tan C = tan A tan B tan C;
1 1 1 1
therefore + + = ;
cot A cot B cot C cot A cot B cot C
therefore cot B cot C + cot A cot C + cot A cot B = 1;
therefore cot2 A + cot2 B + cot2 C
1 1 1
= 1 + (cot A − cot B)2 + (cot B − cot C)2 + (cot C − cot A)2 .
2 2 2
Hence the least value of the expression is when cot A − cot B, cot B − cot C, and
cot C − cot A all vanish; and the value is then unity.
CHAPTER X : Logarithms and Logarithmic Series 73

Problem 36. If A, B, C are the angles of a triangle show that 2 cot A + 2 cot B + 2 cot C
is never less than cosec A + cosec B + cosec C .
cos B cos C 1
Solution. cot B + cot C − cosec A = + −
sin B sin C sin A
sin(B + C) 1 sin A 1 sin2 A − sin B sin C
= − = − = .
sin B sin C sin A sin B sin C sin A sin A sin B sin C

Proceeding in this way we find that the difference of the two given expressions is
equivalent to a fraction with the denominator sin A sin B sin C, while the numerator is
sin2 A + sin2 B + sin2 C − sin B sin C − sin C sin A − sin A sin B,
1 1 1
that is (sin A − sin B)2 + (sin B − sin C)2 + (sin C − sin A)2 .
2 2 2
This expression is never negative.

Problem 37. Show that the sum of the three acute angles which satisfy the equation
cos2 A + cos2 B + cos2 C = 1 is less than 180 ◦ .
Solution. Suppose A, B, C to be three acute angles such that
cos2 A + cos2 B + cos2 C = 1,
then cos2 A = 1 − cos2 C − cos2 B = sin2 C − cos2 B
= − cos(C − B) cos(C + B).
This shows that C + B must be greater than a right angle. Now if we take A′ =
180◦ − C − B we shall have cos2 A′ numerically equal to cos2 (B + C), and therefore
numerically less than cos(C − B) cos(C + B); for we may suppose C not less than B,
and then C − B is less than 180◦ − C − B. Hence cos2 A is greater than cos2 A′ , and A
is less than A′ , and therefore A + B + C is less than 180◦ .

Problem 38. If each of the angles A, B, C be less than 90 ◦ , then sin(A + B + C ) is


less than sin A + sin B + sin C .
Solution. By Art. 113 (page 328) we have sin A + sin B + sin C − sin(A + B + C)
= sin A(1 − cos B cos C) + sin B(1 − cos C cos A) + sin C(1 − cos A cos B)
+ sin A sin B sin C;
and as A, B, and C are acute this expression is necessarily positive.

( )n 2
α
Problem 39. Find the limit of cos when n is increased indefinitely.
n
( )n2
α
Solution. Let u = cos ;
n
( )
α n2 α
therefore log u = n2 log cos = log 1 − sin2
n 2 n
{ }
n2 2 α 1 4 α 1 α
=− sin + sin + sin6 + . . . .
2 n 2 n 3 n
α
α sin
Now n sin = α α n , and this is equal to α when n is definitely increased; and
n
n
α
therefore n2 sin2 is equal to α2 .
n
α α α
Then n2 sin4 = n2 sin2 × sin2 ; and this vanishes when n is indefinitely in-
n n n
creased. Similarly the other terms in log u vanish, and as in Art. 150 (page 334) their
α2 α2
sum vanishes also; and thus log u = − ultimately. Therefore u = e− 2 .
2

( )n 3
α
Problem 40. Find the limit of cos when n is increased indefinitely.
n
CHAPTER XI : Use of Logarithmic And Trigonometrical Tables 74

( )n3
α
Solution. Let u = cos ; therefore
n
( )
α n3 α
log u = n3 log cos
= log 1 − sin2
n 2 n
{ }
n3 2 α 1 4 α 1 α
=− sin + sin + sin6 + . . . .
2 n 2 n 3 n
α
Now we have shown in solving the preceding Problem that n2 sin2 = α2 ulti-
n
α
mately; hence n3 sin2 = nα2 , and so becomes infinite. Thus the logarithm of u is
n
negative infinity, and therefore u vanishes ultimately.

tan3 θ
Problem 41. Show that sin θ is greater than tan θ − if θ is positive and less
2
π
than .
2
1 1
Solution. sin θ − (tan θ − tan3 θ) = sin θ − tan θ + tan3 θ
2 { 2 }
sin θ 1 sin3 θ sin θ 1
= sin θ − + 3
= 3
cos3 θ − cos2 θ + sin2 θ
cos θ 2 cos θ cos θ 2
sin θ { }
= 2 cos 3
θ − 2 cos 2
θ + 1 − cos2
θ
2 cos3 θ
sin θ(1 − cos θ) { }
= 3
1 + cos θ − 2 cos2 θ
2 cos θ
sin θ(1 − cos θ)(1 − cos θ)(1 + 2 cos θ)
=
2 cos3 θ
sin θ(1 − cos θ)2 (1 + 2 cos θ)
= , which is positive.
2 cos3 θ

( )
x −1 x
Problem 42. Show that continually increases as x increases from unity to
x
( limit)ofx the expression when x is increased indefinitely.
infinity; and find the
x−1
Solution. Let u = ; then
x ( )
x−1 1
log u = x log = x log 1 −
{ x x}
1 1 1
= −x + 2
+ + . ..
{ x 2x 3x3 }
1 1
=− 1+ + + ... .
2x 3x2
Thus the logarithm is always negative, and as x increases the logarithm diminishes
numerically, and so u increases; when x is infinite log u = −1; and therefore u = e−1 .
CHAPTER XI
Use of Logarithmic And Trigonometrical Tables.

Problem 1.
Given log 12440 = 4·0948204,
log 12441 = 4·0948553,
find log 12440·35.
Solution.
4·0948553
4·0948204 1 : ·35 :: ·0000349 : x;
·0000349
this gives x = ·0000122;
therefore log 12440·35 = 4·0948326.

Problem 2.
Given log 1·0686 = ·0288152,
log 1·0687 = ·0288558,
find the number of which the logarithm is ·0288355 .
Solution.
·0288558 ·0288355
·0288152 ·0288152 ·0000406 : ·0000203 :: ·0001 : x;
·0000406 ·0000203
this gives x = ·00005;
therefore log 1·06865 = ·0288355.

Problem 3.
Given log 23456 = 4·3702540,
log 23457 = 4·3702725,
form a table of proportional parts for the intermediate numbers,
and find log ·2345638 .
Solution.
1 185
2 370
3 555
4·3702725 4 740
4·3702540 5 925
·0000185 6 1110
7 1295
8 1480
9 1665
log 23456 =4·3702540
add for 3 555
8 1480
=4·370261030
therefore retaining 7 places of decimals
log 23456·38 = 4·3702610, and log ·2345638 = 1̄·3702610.

Problem 4. Find the number whose logarithm is −(1 ·8753145 ), having given
log 1·3325 = ·1246672, log 1·3326 = ·1246998.
Solution. −(1·8753145) = 2̄·1246855.
CHAPTER XI : Use of Logarithmic And Trigonometrical Tables 76

·1246998 ·1246855
·1246672 ·1246672 ·0000326 : ·0000183 :: ·0001 : x;
·0000326 ·0000183
this gives x = ·000056;
therefore log 1·332556 = ·1246855,
therefore log ·01332556 = 2̄·1246855.

Problem 5.
Given log 3·855 = ·5860244,
log 3·8551 = ·5860356,
1
find log(·00385504) 4 .
Solution.
·5860356
·5860244 ·0001 : ·00004 :: ·0000112 : x;
·0000112
this gives x = ·0000045;
therefore log 3·85504 = ·5860289;
therefore log ·00385504 = 3̄·5860289;
1 1 1
therefore log(·00385504) 4 = (3̄·5860289) = (−4 + 1·5860289) = 1̄·3965072.
4 4

Problem 6.
Given log 24 = 1·3802112,
log 4·8989 = ·6900986,
log 4·8990 = ·6901074,
1
find (24 ) 2 to six places of decimals.
Solution.
1 1
log(24) 2 = log 24 = .6901056.
2
·6901074 ·6901056
·6900986 ·6900986 ·0000088 : ·0000070 :: ·0001 : x;
·0000088 ·0000070
this gives x = ·000079;
therefore log 4·898979 = ·6901056;
1
therefore (24) 2 = 4·898979.

Problem 7.
Given log 14271 = 4·1544544,
log 20313 = 4·3077741,
log 20314 = 4·3077954,
1
find (142 ·71 ) 2 .
Solution.
1 1
log(142·71) 7 = × 2·1544544 = ·3077792.
7
·3077954 ·3077792
·3077741 ·3077741 ·0000213 : ·0000051 :: 1 : x;
·0000213 ·0000051
this gives x = ·24;
therefore log 20313·24 = 4·3077792;
therefore log 2·031324 = ·3077792;
1
therefore (142·71) 7 = 2·031324.
CHAPTER XI : Use of Logarithmic And Trigonometrical Tables 77

Problem 8.
Given log 7 = ·8450980,
log 58751 = 4·7690153,
log 58752 = 4·7690227,
1
find (·07 ) 5 to seven significant figures.
Solution.
1 1 1 1
log(·07) 5 = log ·07 = (2̄·8450980) = (−5 + 3·8450980) = 1̄·7690196.
5 5 5
·7690227 ·7690196
·7690153 ·7690153 ·0000074 : ·0000043 :: 1 : x;
·0000074 ·0000043
this gives x = ·58;
therefore log 58751·58 = 4·7690196;
therefore log ·5875158 = 1̄·7690196;
1
therefore (·07) 5 = ·5875158.

Problem 9. Given log 2 = ·3010300 , log 5 ·743491 = ·7591760 , find the fifth root of ·0625 .
Solution.
( ) 15 ( )1 ( )1
1 625 125 5 25 5
log(·0625) 5 = log = log = log
10000 2000 400
( ) 15
1 1 4
= log = − log 16 = − log 2 = −·2408240
16 5 5
= 1̄·7591760 = log ·5743491;
1
therefore (·0625) 5 = ·5743491.

1
Problem 10. Given log 2 ·7 = ·4313638 , log 5 ·172818 = ·7137272 , find the value of 27 − 5 .
Solution.
1 1 1
log(27)− 5 = − log 27 = − (1·4313638) = −·2862728
5 5
= 1̄·7137272 = log ·5172818;
1
therefore (27)− 5 = ·5172818.

Problem
√ 11. Given log 71968 = 4 ·8571394 , diff. for 1 = ·0000060 , find the value of
8
·0719686.
6
Solution. log 71968·6 = 4·8571394 + of ·0000060 = 4·8571430;
10
1 1 1
log(·0719686) 8 = (2̄·8571430) = (−8 + 6·8571430) = 1̄·8571429.
8 8
1
But log ·719686 = 1̄·8571430; therefore (·0719686) 8 = ·719686.

Problem 12. Given log 103 = 2 ·0128372 , log 7440942 = 6 ·871628 ,


find (1 ·03 )−10 .
Solution. log(1·03)−10 = −10 × ·0128372 = −·128372 = 1̄·871628 = log ·7440942;
therefore (1·03)−10 = ·7440942.

{ }
Problem 13. Find the value of 64 1 − (1 ·05 )−20 , having given
log 105 = 2 ·0211893 , log 37689 = 4 ·5762140 .
Solution. log(1·05)−20 = −20 × ·0211893 = −·423786 = 1̄·576214 = log ·37689;
therefore (1·05)−20 = ·37689;
{ }
therefore 64 1 − (1·05)−20 = 64{1 − ·37689}
= 64 × ·62311 = 39·87904.
CHAPTER XI : Use of Logarithmic And Trigonometrical Tables 78


Problem 14. Find approximately 5 5 , having given
log 2 = ·301030, log 1·562944 = ·193943,
log 349485 = 5·543428, log 3·655 = ·562887,
log 3·656 = ·563006.
√ √ √
Solution. Denote it by u; then log u = 5 log 5 = 2 5 log( √5; )

therefore log(log u) = log 2 + log 5 + log log 5 .
√ 1 1 10 1
Now log 5= log 5 = log = (1 − log 2)
2 2 2 2
1 1
= (1 − ·301030) = (·698970) = ·349485,
2 2
( √ )
log log 5 = log ·349485 = 1̄·543428.
Therefore log(log u) = ·301030 + ·349485 + 1̄·543428 = ·193943.
Therefore log u = 1·562944.
·563006 ·562944
·562887 ·562887 ·000119 : ·000057 :: ·001 : x;
·000119 ·000057
this gives x = ·00048; therefore u = 36·5548.

Problem 15. Having given


log 12 = 1·0791812, log 1·257915 = ·0996512,
log 1·121568 = ·0498256,
find the value of
(1 ·44 )−6 − (1 ·44 )−12 .
Solution. log 144 = log 122 = 2 log 12 = 2·1583624;
log(1·44)−6 = −6 log 1·44 = −6(·1583624) = −·9501744
= 1̄·0498256 = log ·1121568;

therefore (1·44)−6 = ·1121568.

log(1·44)−12 = −12 log 1·44 = −12(·1583624) = −1·9003488


= 2̄·0996512 = log ·01257915;

therefore (1·44)−12 = ·01257915;


−6 −12
therefore (1·44) − (1·44) = ·1121568 − ·01257915 = ·09957765.

Problem 16. Having given


log 105 = 2·0211893, log 5303214 = 6·7245391,
log 3768894 = 6·576214,
find the value of { }
1 1 1
− .
·05 (1 ·05 )13 (1 ·05 )20
Solution.
1
log = −13 log 1·05 = −13(·0211893) = −·2754609
(1·05)13
= 1̄·7245391 = log ·5303214;

1
therefore = ·5303214;
(1·05)13
1
log = −20 log 1·05 = −20(·0211893) = −·423786
(1·05)20
= 1̄·576214 = log ·3768894;

1
therefore = ·3768894;
(1·05)20
CHAPTER XI : Use of Logarithmic And Trigonometrical Tables 79

{ }
1 1 1
therefore − = 20{·5303214 − ·3768894}
·05 (1·05)13 (1·05)20
= 20 × ·153432 = 3·06864.

Problem 17.
Given sin 47◦ = ·7313537,
sin 48◦ = ·7431448,
sin 47◦ 1′ .
Solution.
·7431448
·7313537 60′ : 1′ :: ·0117911 : x;
·0117911
this gives x = ·0001965;
therefore sin 47◦ 1′ = ·7313537 + ·0001965 = ·7315502.

Problem 18.
Given sin 7◦ 17′ = ·1267761,
sin 7◦ 18′ = ·1270646,
find sin 7◦ 17′ 25′′ .
Solution.
·1270646
·1267761 60′′ : 25′′ :: ·0002885 : x;
·0002885
this gives x = ·0001202;
therefore sin 7◦ 17′ 25′′ = ·1267761 + ·0001202 = ·1268963.

Problem 19.
Given L sin 17◦ 1′ = 9·4663483,
L sin 17◦ = 9·4659353,
find L sin 17◦ 0′ 12′′ .
Solution.
9 · 4663483
9 · 4659353 60′′ : 12′′ :: ·0004130 : x;
· 0004130
this gives x = ·0000826;
therefore L sin 17◦ 0′ 12′′ = 9·4659353 + ·0000826 = 9·4660179.

Problem 20.
Given L sin 26◦ 24′ = 9·6480038,
L sin 26◦ 25′ = 9·6482582,
find L sin 26◦ 24′ 12′′ .
Solution.
9·6482582
9·6480038 60′′ : 12′′ :: ·0002544 : x;
·0002544
this gives x = ·0000509;
therefore L sin 26◦ 24′ 12′′ = 9·6480038 + ·0000509 = 9·6480547.

Problem 21.
Given L cot 72◦ 15′ = 9·5052891,
L cot 72◦ 16′ = 9·5048538,
CHAPTER XI : Use of Logarithmic And Trigonometrical Tables 80

find L cot 72◦ 15′ 35′′ .


Solution.
9·5052891
9·5048538 60′′ : 35′′ :: ·0004353 : x;
·0004353
this gives x = ·0002539;
therefore L cot 72◦ 15′ 35′′ = 9·5052891 − ·0002539 = 9·5050352.

Problem 22. Given L cot 81 ◦ 46 ′ = 9 ·1604569 , diff. for 10 ′′ = ·0001486 , find the angle
whose L cot is 9 ·1603493 .
Solution.
9·1604569
9·1603493 ·0001486 : ·0001076 :: 10 : x;
·0001076
this gives x = 7; therefore the required angle is 81◦ 46′ 7”.

Problem 23. Given L cos 20 ◦ 35 ′ 20 ′′ = 9 ·9713351 , diff. for 10 ′′ = ·0000079 , find the
angle whose L cos is 9 ·9713383 .
Solution.
9·9713383
9·9713351 ·0000079 : ·0000032 :: 10 : x;
·0000032
this gives x = 4; therefore the required angle is 20◦ 35′ 20′′ − 4′′ , that is 20◦ 35′ 16′′ . For
as the L cosine increases the angle diminishes.

Problem 24. Given L cos 34 ◦ 24 ′ = 9 ·9165137 , diff. for 1 ′ = ·0000865 ,


find L cos 34 ◦ 24 ′ 26 ′′ , and also the angle whose L cos is 9 ·9165646 .
Solution.
60′′ : 26′′ :: ·0000865 : x;
this gives x = ·0000375;
therefore L cos 34◦ 24′ 26′′ = 9·9165137 − ·0000375 = 9·9164762.
Again
9·9165646
9·9165137 ·0000865 : ·0000509 :: 60 : x;
·0000509
this gives x = 35; therefore the required angle is 34◦ 24′ − 35′′ , that is 34◦ 23′ 25′′ .

Problem 25.
Given L sin 37◦ 19′ = 9·7826301, diff. for 1′ = ·0001657,
L cos 37◦ 19′ = 9·9005294, diff. for 1′ = ·0000963,
find L sec 37◦ 19′ 47′′ , and L tan 37◦ 19′ 47′′ .
Solution. Since sec θ × cos θ = 1, we have log sec θ + log cos θ = 0;
therefore L sec θ + L cos θ − 20 = 0; therefore L sec θ = 20 − L cos θ.
We shall first find L cos 37◦ 19′ 47′′ .
60′′ : 47′′ :: ·0000963 : x;
this gives x = ·0000754;
therefore L cos 37◦ 19′ 47′′ = 9·9005294 − ·0000754 = 9·9004540.
Then L sec 37◦ 19′ 47′′ = 20 − 9·9004540 = 10·0995460.
Next find L sin 37◦ 19′ 47′′ .
60′′ : 47′′ :: ·0001657 : x;
this gives x = ·0001298;
therefore L sin 37◦ 19′ 47′′ = 9·7826301 + ·0001298 = 9·7827599.
sin θ
Then tan θ = ; therefore log tan θ = log sin θ − log cos θ;
cos θ
therefore L tan θ − 10 = L sin θ − 10 − (L cos θ − 10) = L sin θ − L cos θ;
CHAPTER XI : Use of Logarithmic And Trigonometrical Tables 81

therefore L tan θ = 10 + L sin θ − L cos θ.


Thus L tan 37◦ 19′ 47′′ = 10 + 9·7827599 − 9·9004540 = 9·8823059.

Problem 26.
Given L sin 32◦ 18′ = 9·7278277, diff. for 1′ = ·0001998,
L cos 32◦ 18′ = 9·9269913, diff. for 1′ = ·0000799,
find L sine, L cosine, and L tangent of 32◦ 18′ 24′′ ·6.
Solution.
60′′ : 24′′ .6 :: ·0001998 : x;
this gives x = ·0000819;
therefore L sin 32◦ 18′ 24′′ ·6 = 9·7278277 + ·0000819 = 9·7279096.
60′′ : 24′′ ·6 :: ·0000799 : x;
this gives x = ·0000328;
therefore L cos 32◦ 18′ 24′′ ·6 = 9·9269913 − ·0000328 = 9·9269585.
And L tan 32◦ 18′ 24′′ ·6 = 10 + L sin 32◦ 18′ 24′′ ·6 − L cos 32◦ 18′ 24′′ ·6
= 9·8009511.
CHAPTER XII
Theory of Proportional Parts

Problem 1. From one of the angles of a rectangle a perpendicular is drawn to its


diagonal, and from the point of their intersection lines are drawn perpendicular to
the sides which contain the opposite angle; show that if p and p′ be the lengths of the
perpendiculars last drawn, and c the diagonal of the rectangle,
2 2 2
p 3 + p′ 3 = c 3 .
Solution. Let ABCD denote the rectangle. From A draw AP perpendicular to the
diagonal BD; and from P draw P M perpendicular to BC, and P N perpendicular to
CD.
Let the angle DBA be denoted by α; then
AB = c cos α, BP = AB cos α = c cos2 α,
P M = BP cos BP M = BP cos α = c cos3 α.
Thus denoting P M by p we have p = c cos3 α.
Similarly
AD = c sin α, P D = AD sin P AD = AD sin α = c sin2 α,
P N = P D sin P DN = P D sin α = c sin3 α.
Thus q = c sin3 α.
2 2 ( ) 32 ( ) 32 2 ( ) 2
∴ p 3 + q 3 = c cos3 α + c sin3 α = c 3 cos2 α + sin2 α = c 3 .

Problem 2. If two circles whose radii are a and b touch each other externally, and if
θ be the angle contained by the two common tangents to these circles, show that

4(a − b) ab
sin θ = .
(a + b)2
Solution. Let a denote the radius of the larger circle, and b the radius of the smaller
circle. Let x denote the distance of the point of intersection of the two common
tangents from the centre of the larger circle; therefore x − a − b denotes the distance
of this point from the centre of the smaller circle.
θ a θ b
Then sin = , and also sin = ;
2 x 2 x−a−b
a b
therefore x = , and x − a − b = ;
θ θ
sin sin
2 2
a−b
Therefore, by subtraction, a + b = ;
θ
sin
2 √
θ a−b θ 2 ab
∴ sin = ; ∴ cos = ;
2 a+b 2 a +√b
θ θ 4(a − b) ab
∴ sin θ = 2 sin cos = .
2 2 (a + b)2

Problem 3. Given sec α sec θ + tan α tan θ = sec β, find tan θ.


Solution.
sec α sec θ + tan α tan θ = sec β
∴ sec α sec θ = sec β − tan α tan θ;
∴ sec2 α sec2 θ = (sec β − tan α tan θ)2 ;
( )
∴ sec α 1 + tan2 θ = sec2 β − 2 sec β tan α tan θ + tan2 α tan2 θ;
2
( )
∴ sec2 α − tan2 α tan2 θ + 2 sec β tan α tan θ = sec2 β − sec2 α;
∴ tan2 θ + 2 sec β tan α tan θ = sec2 β − sec2 α;
CHAPTER XII : Theory of Proportional Parts 84

∴ (tan θ + tan α sec β)2 = sec2 β − sec2 α + tan2 α sec2 β


= sec2 β sec2 α − sec2 α = tan2 β sec2 α;
∴ tan θ + tan α sec β = ± tan β sec α;
∴ tan θ = − tan α sec β ± tan β sec α
sin α sin β − sin α ± sin β
=− ± = .
cos α cos β cos β cos α cos α cos β

θ
sin cos 2θ tan2 θ
Problem 4. Find the limit when θ = 0 of 2 , and of . Kindly note
vers θ cot θ sec 2θ − 1
that vers θ = 1 − cos θ.
Solution.
θ θ θ θ
sin cos 2θ sin cos 2θ sin θ 2 sin2 cos cos 2θ
2 = 2 = 2 2
vers θ cot θ vers θ cos θ (1 − cos θ) cos θ
θ θ θ
2 sin2 cos cos 2θ cos cos 2θ
= 2 2 = 2
θ cos θ
2 sin2 cos θ
2
and the value of this is unity when θ = 0.

tan2 θ sin2 θ sin2 θ cos 2θ


= =
sec 2θ − 1 cos2 θ(sec 2θ − 1) cos2 θ(1 − cos 2θ)
sin2 θ cos 2θ cos 2θ
= = ;
2 cos2 θ sin2 θ 2 cos2 θ
1
and the value of this is when θ = 0.
2

θ
Problem 5. Show that cot is greater than 1 + cot θ for all values of θ between 0 and
π. 2
Solution.
θ
θ θ cos
cot − (1 + cot θ) = cot − cot θ − 1 = 2 − cos θ − 1
2 2 θ sin θ
sin
2 ( )
θ θ θ
sin θ cos − cos θ sin sin θ −
2 2 −1= 2
= −1
θ θ
sin sin θ sin sin θ
2 2
1
= − 1.
sin θ
π
Now this is always positive as θ changes from 0 to π, except when θ = , and when
2
it is zero.

θ tan θ + c − 1 θ
Problem 6. If tan = , find tan .
2 tan θ + c + 1 2
Solution.
θ tan θ + c − 1
tan =
2 tan θ + c + 1
θ
∴ tan(tan θ + c + 1) = tan θ + c − 1
 2
θ
 θ
θ 2 tan 2 tan
∴ tan  2 + c + 1 = 2 +c−1
2 θ θ
1 − tan2 1 − tan2
( 2 ) 2 ( )
θ θ θ θ θ
∴ 2 tan2 + (c + 1) 1 − tan2 tan = 2 tan + (c − 1) 1 − tan2
2 2 2 2 2
CHAPTER XII : Theory of Proportional Parts 85

θ θ θ
∴ (c + 1) tan3 − (1 + c) tan2 + (1 − c) tan + (c − 1) = 0
2 ( 2 ) ( 2 )
θ θ θ
∴ (c + 1) tan2 tan − 1 = (c − 1) tan − 1
2 2 2
θ θ
∴ tan − 1 = 0, or (c + 1) tan2 = c − 1
2 √ 2
θ c−1
∴ tan = 1, or ± .
2 c+1

Problem 7. Find the condition necessary that the same value of θ may satisfy both
the equations
a sec2 θ − b sec θ = 2a,
b cos2 θ − a sec θ = 2b.
Solution.
a sec2 θ − b sec θ = 2a
∴ a − b cos3 θ = 2a cos2 θ
∴ b cos3 θ = a − 2a cos2 θ. (3)
Again
b cos2 θ − a sec θ = 2b
∴ b cos3 θ = 2b cos θ + a. (4)
From (3) and (4):
a − 2a cos2 θ = 2b cos θ + a
b
∴ −a cos θ = b; ∴ cos θ = − .
a
Substitute this value of cos θ in either of the gien equations, for instance the first;
thus
a3 b2
+ = 2a
b2 a
∴ a + b − 2a2 b2 = 0
4 4

∴ a2 = b2 .

Problem 8. Eliminate α and β from the equations


a = sin α cos β sin θ + cos α cos θ,
b = sin α cos β cos θ − cos α sin θ,
c = sin α sin β sin θ.
Solution.
a2 + b2 = (sin α cos β sin θ + cos α cos θ)2 + (sin α cos β cos θ − cos α sin θ)2
= sin2 α cos2 β + cos2 α
and
c2
= sin2 α sin2 β.
sin2 θ
c2
∴ a2 + b2 + = sin2 α cos2 β + sin2 α sin2 β + cos2 α
sin2 θ
= sin2 α + cos2 α = 1.

Problem 9. Eliminate α and β from the equations


b + c cos α = u cos(α − θ),
b + c cos β = u cos(β − θ),
α − β = 2δ.
and show that
u2 − 2uc cos θ + c2 = b2 sec2 δ.
Solution. Adding the first two equations
2b + c(cos α + cos β) = u cos(α − θ) + u cos(β − θ)
CHAPTER XII : Theory of Proportional Parts 86

( )
α+β α−β α+β α−β
∴ b + c cos cos = u cos − θ cos
2 2 2 2
( )
α−β α+β α+β
b sec = u cos − θ − c cos (5)
2 2 2
Again from the first two equations, by subtraction,
c(cos α − cos β) = u cos(α − θ) − u cos(β − θ)
( )
β−α α+β β−α α+β
∴ c sin sin = u sin sin −θ
2 2
( )2 2
α+β α+β
∴ 0 = u sin − θ − c sin (6)
2 2
Square and add (5) and (6); thus
b2 sec2 δ = u2 + c2
{ ( ) ( ) }
α+β α+β α+β α+β
− 2uc cos − θ cos + sin − θ sin
2 2 2 2
= u2 + c2 − 2uc cos θ.

Problem 10. Eliminate x from the equations


a tan2 θ − x 2a tan θ
= = a − x;
tan 2α tan 2α′ tan 2α + tan 2α′
′ π ′
and show that θ = α + α , or +α+α .
2
Solution.
2a tan θ tan 2α tan 2α′
a tan2 θ − x =
tan 2α + tan 2α′
2a tan θ
a−x=
tan 2α + tan 2α′
therefore, by subtraction,
( ) 2a tan θ(1 − tan 2α tan 2α′ )
a 1 − tan2 θ =
tan 2α + tan 2α′
tan 2α + tan 2α′ 2 tan θ
∴ ′
=
1 − tan(2α tan 2α) 1 − tan2 θ
∴ tan 2α + 2α′ = tan 2θ.

Problem 11. Eliminate θ and ϕ from the equations


sin θ + sin ϕ = a,
cos θ + cos ϕ = b,
cos(θ − ϕ) = c.
Solution. Square and add the first two equations
2 + 2(cos θ cos ϕ + sin θ sin ϕ) = a2 + b2
∴ 2 + 2 cos(θ − ϕ) = a2 + b2
∴ 2 + 2c = a2 + b2 .

Problem 12. Eliminate θ and ϕ from the equations


x cos θ + y sin θ = a,
x cos(θ + 2ϕ) − y sin(θ + 2ϕ) = a,
b sin(θ + ϕ) = a sin ϕ.
Subtracting the second equation from the first one,
x {cos θ − cos(θ + 2ϕ)} + y {sin θ + sin(θ + 2ϕ)} = 0
∴ x sin(θ + ϕ) sin ϕ + y sin(θ + ϕ) cos ϕ = 0
∴ x sin ϕ + y cos ϕ = 0 (7)
Again, by addition,
x {cos θ + cos(θ + 2ϕ)} + y {sin θ − sin(θ + 2ϕ)} = 2a
∴ x cos(θ + ϕ) cos ϕ − y cos(θ + ϕ) sin ϕ = 0
a
∴ x cos ϕ − y sin ϕ = (8)
cos(θ + ϕ)
CHAPTER XII : Theory of Proportional Parts 87

Square and add (7) and (8),


a2 a2 a2
x2 + y 2 = = =
(
cos θ + ϕ) 1 − sin2 (θ + ϕ) a2
1 − 2 sin2 ϕ
( ) b
a2
∴ (x2 + y 2 ) 1− sin2 ϕ = a2 . (9)
b2
But from (7) ( )
x2 sin2 ϕ = y 2 cos2 ϕ = y 2 1 − sin2 ϕ
y2
∴ sin2 ϕ = .
x2 + y2
Substituting this in (9), ( )
a2 y 2
(x2 + y 2 ) 1− = a2
b (x2 + y 2 )
2

a2 y 2
∴ x2 + y 2 = a2 + .
b2

Problem 13. Eliminate x and y from the equations


tan x + tan y = a,
cot x + cot y = b,
x + y = c.
Solution.
tan x + tan y
tan c = tan(x + y) = .
1 − tan x tan y
Now
tan x + tan y = a, and cot x + cot y = b
1 1
∴ + =b
tan x tan y
∴ tan x + tan y = b tan x tan y
a
∴ a = b tan x tan y; ∴ tan x tan y =
b
a ab
∴ tan c = a =
1− b−a
b
b−a 1 1
∴ cot c = = − .
ab a b

Problem 14. Eliminate θ from the equations


x sec2 θ − cos2 θ
= ,
a sec2 θ + cos2 θ
2b
= sec2 θ + cos2 θ.
y
Solution.
x sec2 θ − cos2 θ 1 − cos4 θ
= 2 2
= ,
a sec θ + cos θ 1 + cos4 θ
2b 4
1 + cos θ
= sec2 θ + cos2 θ =
y cos2 θ
( )2 ( )2
x2 y2 1 − cos4 θ 4 cos4 θ 1 + cos4 θ
∴ 2 + 2 = + = = 1.
a b 1 + cos4 θ (1 + cos4 θ)2 1 + cos4 θ

Problem 15. Eliminate θ from the equations


(a + b) tan(θ − ϕ) = (a − b) tan(θ + ϕ),
a cos 2ϕ + b cos 2θ = c.
Solution.
(a + b) tan(θ − ϕ) = (a − b) tan(θ + ϕ),
CHAPTER XII : Theory of Proportional Parts 88

∴ (a + b) sin(θ − ϕ) cos(θ + ϕ) = (a − b) sin(θ + ϕ) cos(θ − ϕ)


∴ b {sin(θ + ϕ) cos(θ − ϕ) + sin(θ − ϕ) cos(θ + ϕ)}
= a {sin(θ + ϕ) cos(θ − ϕ) − sin(θ − ϕ) cos(θ + ϕ)}
∴ b sin 2θ = a sin 2ϕ, and
b cos 2θ = c − a cos 2ϕ.
Square and add; thus b2 = c2 + a2 − 2ac cos 2ϕ.

Problem 16. Given


x2 y2 z2
cos θ = 2 cos θ + 2 cos θ′ ,
a2 a b
x y z
and = = .
sin(θ + θ′ ) sin(θ − θ′ ) sin 2θ
Show that
sin θ b2
= 2.
sin θ′ a
Solution.
z sin(θ + θ′ ) z sin(θ − θ′ )
x= , y= .
sin 2θ sin 2θ
Square and substitute in the first given equation; thus
z 2 sin2 (θ + θ′ ) z 2 sin2 (θ − θ′ ) z2
cos θ = cos θ + 2 cos θ′
a2 sin2 2θ a2 sin2 2θ b
sin2 (θ + θ′ ) cos θ − sin2 (θ − θ′ ) cos θ cos θ′
∴ =
a2 sin2 2θ b2
(sin θ cos θ + cos θ sin θ ) − (sin θ cos θ − cos θ sin θ′ )2
′ ′ 2 ′ cos θ′
∴ 2 2 2
cos θ =
4a sin θ cos θ b2
4 sin θ cos2 θ sin θ′ cos θ′ cos θ′
∴ =
4a2 sin2 θ cos2 θ b2
sin θ ′ a 2
∴ = 2.
sin θ b

Problem 17. Eliminate ϕ from the equations


y cos ϕ − x sin ϕ = a cos 2ϕ,
y sin ϕ + x cos ϕ = 2a sin 2ϕ.
And show that
2 2 2
(x + y) 3 + (x − y) 3 = 2a 3 .
Solution. Given equations are
y cos ϕ − x sin ϕ = a cos 2ϕ, (10)
y sin ϕ + x cos ϕ = 2a sin 2ϕ. (11)
Multiply (10) by cos ϕ, and (11) by sin ϕ, and add; thus
y = a cos 2ϕ cos ϕ + 2a sin 2ϕ sin ϕ
( ) ( )
= a cos ϕ cos 2ϕ + 4 sin2 ϕ = a cos ϕ cos2 ϕ + 3 sin2 ϕ .
Again multiply (11) by cos ϕ, and (10) by sin ϕ, and subtract : thus
x = 2a sin 2ϕ cos ϕ − a cos 2ϕ sin ϕ
( ) ( )
= a sin ϕ 4 cos2 ϕ − cos 2ϕ = a sin ϕ 3 cos2 ϕ + sin2 ϕ .
( )
∴ x + y = a sin3 ϕ + cos3 ϕ + 3 sin2 ϕ cos ϕ + 3 cos2 ϕ sin ϕ
= a(sin ϕ + cos ϕ)3 ;
( )
∴ x − y = a sin3 ϕ − 3 sin2 ϕ cos ϕ + 3 cos2 ϕ sin ϕ − cos3 ϕ
= a(sin ϕ − cos ϕ)3 ;
2 2 2 { } 2
∴ (x + y) 3 + (x − y) 3 = a 3 (sin ϕ + cos ϕ)2 + (sin ϕ − cos ϕ)2 = 2a 3 .

Problem 18. Eliminate θ and ϕ from the equations


sin β
cos θ = ,
sin α
CHAPTER XII : Theory of Proportional Parts 89

sin γ
cos ϕ = ,
sin α
cos(θ − ϕ) = sin β sin γ.
And show that
tan2 α = tan2 β + tan2 γ.
Solution.
cos θ cos ϕ + sin θ sin ϕ = sin β sin γ
∴ sin2 θ sin2 ϕ = (sin β sin γ − cos θ cos ϕ)2
( )( )
∴ 1 − cos2 θ 1 − cos2 ϕ = (sin β sin γ − cos θ cos ϕ)2
( )( ) ( )2
sin2 β sin2 γ sin β sin γ
∴ 1− 1− = sin β sin γ −
sin2 α sin2 α sin2 α
( )( ) ( )2
∴ sin2 α − sin2 β sin2 α − sin2 γ = sin2 β sin2 γ sin2 α − 1
( ) ( )
∴ sin4 α − sin2 α sin2 β + sin2 γ = sin2 β sin2 γ sin4 α − 2 sin2 α
( )
∴ sin2 α − sin2 β − sin2 γ = sin2 β sin2 γ sin2 α − 2
( )
∴ sin2 α 1 − sin2 β sin2 γ = sin2 β cos2 γ + cos2 β sin2 γ
sin2 β cos2 γ + cos2 β sin2 γ
∴ sin2 α =
1 − sin2 β sin2 γ
1 − sin 2 β sin2 γ − sin2 β cos2 γ − cos2 β sin2 γ
∴ cos2 α =
1 − sin2 β sin2 γ
( )( )
sin2 β + cos2 β sin2 γ + cos2 γ − sin2 β sin2 γ − sin2 β cos2 γ − cos2 β sin2 γ
=
1 − sin2 β sin2 γ
cos2 β cos2 γ
= .
1 − sin2 β sin2 γ
sin2 β cos2 γ + cos2 β sin2 γ
∴ tan2 α =
cos2 β cos2 γ
sin2 β sin2 γ
= 2
+ = tan2 β + tan2 γ.
cos β cos2 γ

Problem 19. Eliminate θ from the equations


m = cosec θ − sin θ,
n = sec θ − cos θ.
Solution.
1 1 − sin2 θ cos2 θ
m = cosec θ − sin θ = − sin θ = =
sin θ sin θ sin θ
1 1 − cos2 θ sin2 θ
n = sec θ − cos θ = − cos θ = =
cos θ cos θ cos θ
cos2 θ sin2 θ
∴ mn = = cos θ sin θ
sin θ cos θ
mn mn
∴ sin θ = , and cos θ =
cos θ cos θ
cos3 θ sin3 θ
∴m= , and n =
mn mn
( ) 13 ( ) 13
∴ cos θ = m2 n , and sin θ = mn2
( ) 32 ( ) 32
∴ cos2 θ + sin2 θ = m2 n + mn2
2
{ 2 2
}
∴ 1 = (mn) 3 m3 + n3 .

Problem 20. Eliminate θ from the equations



x sin θ − y cos θ = x2 + y 2 ,
cos2 θ sin2 θ 1
+ = 2 .
a2 b2 x + y2
CHAPTER XII : Theory of Proportional Parts 90

Solution.
(x sin θ − y cos θ)2 = x2 + y 2
∴ x2 + y 2 − (x sin θ − y cos θ)2 = 0
∴ x cos2 θ + 2xy sin θ cos θ + y 2 sin2 θ = 0
2

∴ (x cos θ + y sin θ)2 = 0


∴ x cos θ + y sin θ = 0
x
∴ tan θ = − .
y
y 2 x2
Hence we obtain cos2 θ = 2 2
and sin2 θ = 2 .
x +y x + y2
Substitute in the second given equation
( :)thus
1 y2 x2 1
+ 2 = ;
x2 + y2 a2 b x2 + y 2
y2 x2
∴ 2
+ 2 = 1.
a b

Problem 21. Eliminate θ and θ′ from the equations


a sin2 θ + a′ cos2 θ = b,
a′ sin2 θ′ + a cos2 θ′ = b′ ,
a tan θ = a′ tan θ′ .
And show that
1 1 1 1
+ ′ = + ′.
b b a a
Solution.
a sin2 θ + a′ cos2 θ = b
( )
∴ a sin2 θ + a′ 1 − sin2 θ = b
b − a′
∴ sin2 θ =
a − a′
a−b
∴ cos θ =
2
a − a′
b − a′
∴ tan2 θ = .
a−b
b′ − a
Similarly we find tan2 θ′ = .
a′ − b′
∵ a2 tan2 θ = a′2 tan2 θ′
b − a′ b′ − a
∴ a2 = a′2 ′
a−b a − b′
∴ a2 (b − a′ )(b′ − a′ ) = a′2 (b′ − a)(b − a)
{ } { }
∴ a2 bb′ − a′ (b + b′ ) = a′2 bb′ − a(b + b′ )

( ′2
)
∴ bb a2 − a = aa′ (a − a′ )(b + b′ )
∴ bb (a + a ) = aa′ (b + b′ ).
′ ′

1 1 1 1
Divide by aa′ bb′ ; thus ′ + = ′ + .
a a b b

Problem 22. Given


x2 + y 2 = a2 + b2 ,
xy = ab sin α,
cos2 θ sin2 θ 1
+ = 2,
x2 y2 a
show that
a2
± cot 2θ = cot 2α + cosec 2α.
b2
CHAPTER XII : Theory of Proportional Parts 91

Solution.
x2 y 2 a2 b2 sin2 α
y 2 cos2 θ + x2 sin2 θ = = = b2 sin2 α
a2 a2
y2 x2
∴ (1 + cos 2θ) + (1 − cos 2θ) = b2 sin2 α
2 ( 2 )
∴ x2 + y 2 + y 2 − x2 cos 2θ = 2b2 sin2 α
{ } 21
∴ a2 + b2 + (y 2 + x2 )2 − 4y 2 x2 cos 2θ = 2b2 sin2 α
{ } 12
∴ a2 + b2 + (a2 + b2 )2 − 4a2 b2 sin2 α cos 2θ = 2b2 sin2 α
2b2 sin2 α − b2 − a2
∴ cos 2θ = { } 12
(a2 + b2 )2 − 4a2 b2 sin2 α
−4a2 b2 sin2 α + 4b2 sin2 α(b2 + a2 ) − 4b4 sin4 α
∴ sin2 2θ =
(a2 + b2 )2 − 4a2 b2 sin2 α
4b4 sin2 α(1 − sin2 α)
= 2
(a + b2 )2 − 4a2 b2 sin2 α
2b2 sin α cos α
∴ ± sin 2θ = { }1
(a2 + b2 )2 − 4a2 b2 sin2 α 2
Hence by division
2b2 sin2 α − b2 − a2 a2 + b2 cos 2α
± cot 2θ = =−
2b2 sin α cos α b2 sin 2α
a2
= − cot 2α − 2 cosec 2α,
b
which we may also express thus
a2
± cot 2θ = cot 2α + 2 cosec 2α.
b

cos x cos 2x cos 3x


Problem 23. If = = , show that
a1 a2 a3
x 2a2 − a1 − a3
sin2 = .
2 4a2
cos x cos 2x cos 3x 1
Solution. Let , , and each be equal to ; then
a1 a2 a3 k
a1 = k cos x, a2 = k cos 2x, and a3 = k cos 3x.
2a2 − a1 − a3 2 cos 2x − cos x − cos 3x
∴ =
4a2 4 cos 2x
2 cos 2x − 2 cos 2x cos x 1 − cos x x
= = = sin2 .
4 cos 2x 2 2

sin x sin 3x sin 5x


Problem 24. If = = , show that
a1 a3 a5
a1 − 2a3 + a5 a3 − 3a1
= .
a3 a1
sin x sin 3x sin 5x 1
Solution. Let , , and each be equal to ; then
a1 a3 a5 k
a1 = k sin x, a3 = k sin 3x, and a5 = k sin 5x.
a1 − 2a3 + a5 sin x − 2 sin 3x + sin 5x 2 sin 3x cos 2x − 2 sin 3x
∴ = =
a3 sin 3x sin 3x
= 2(cos 2x − 1) = −4 sin2 x.
a3 − 3a1 sin 3x − 3 sin x 3 sin x − 4 sin3 x − 3 sin x
and = = = −4 sin2 x.
a1 sin x sin x
a1 − 2a3 + a5 a3 − 3a1
∴ = .
a3 a1
CHAPTER XII : Theory of Proportional Parts 92

Problem 25. Given


cos x cos(x + θ) cos(x + 2θ) cos(x + 3θ)
= = = ,
a1 a2 a3 a4
show that
a1 + a3 a2 + a4
= .
a2 a3
1
Solution. Let denote the value of the fractions which are given equal; thus
k
a1 = k cos x, a2 = k cos(x + θ), a3 = k cos(x + 2θ), a4 = k cos(x + 3θ).
a1 + a3 cos x + cos(x + 2θ) 2 cos(x + θ) cos θ
∴ = = = 2 cos θ, and
a2 cos(x + θ) cos(x + θ)
a2 + a4 cos(x + θ) + cos(x + 3θ) 2 cos(x + 2θ) cos θ
= = = 2 cos θ;
a3 cos(x + 2θ) cos(x + 2θ)
thus the required result is established.

( )
π
tan ±α
cos 2α cos 2α′ ϕ
Problem 26. If sin2 ϕ = , then tan2 = (4 ).
cos2 (α + α′ ) 2 π
tan ± α′
4
Solution.
cos 2α cos 2α′
sin2 ϕ =
cos2 (α + α′ )
cos2 (α + α′ ) − cos 2α cos 2α′
∴ cos2 ϕ =
cos2 (α + α′ )
1 + cos 2(α + α′ ) − cos 2(α + α′ ) − cos 2(α − α′ ) sin2 (α − α′ )
= ′
=
2
2 cos (α + α ) cos2 (α + α′ )
sin(α − α′ )
∴ cos ϕ = ±
cos(α + α′ )
sin(α − α′ )
Take the upper sign ; then cos ϕ = ;
cos(α + α′ )
( )
π
′ ′ sin − α − α′ − sin(α − α′ )
1 − cos ϕ cos(α + α ) − sin(α − α )
∴ = = (2 )
1 + cos ϕ cos(α + α′ ) + sin(α − α′ ) π
sin − α − α′ + sin(α − α′ )
( ) ( ) 2( )
π π ′ π
2 sin − α cos −α tan −α
= (4 ) (4 )= (4 )
π ′
π π
2 sin − α cos −α tan − α′
4 4
( ) 4
π
tan −α
ϕ
∴ tan2 = (4 ).
2 π
tan − α′
4
sin(α − α′ )
Take the lower sign ; then cos ϕ = − ;
cos(α + α′ )
( )
π
′ ′ sin − α − α′ + sin(α − α′ )
1 − cos ϕ cos(α + α ) + sin(α − α )
∴ = = (2 )
1 + cos ϕ cos(α + α′ ) − sin(α − α′ ) π
sin − α − α′ − sin(α − α′ )
( ) ( ) 2( ) ( )
π π π π
2 sin − α′ cos −α cot −α tan +α
= (4 ) ( 4 )= (4 )= (4 )
π π ′
π ′
π
2 sin − α cos −α cot −α tan + α′
4 4
( )4 4
π
tan +α
ϕ
∴ tan2 = (4 ).
2 π
tan + α′
4
CHAPTER XII : Theory of Proportional Parts 93

Problem 27.
If
sin(θ − β) cos α cos(α + θ) sin β
+ =0
sin(ϕ − α) cos β cos(ϕ − β) sin α
and
tan θ tan α cos(α − β)
+ = 0,
tan ϕ tan β cos(α + β)
show that
1 1
tan θ = (tan β + cot α) , tan ϕ = (tan α − cot β) .
2 2
Solution.
sin(θ − β) cos α cos(α + θ) sin β
∵ + =0
sin(ϕ − α) cos β cos(ϕ − β) sin α
sin(θ − β) cos α sin(ϕ − α) sin β
∴ + =0
cos(α + θ) cos β cos(ϕ − β) sin α
(sin θ cos β − cos θ sin β) cos α (sin ϕ cos α − cos ϕ sin α) sin β
∴ + =0
(cos α cos θ − sin α sin θ) cos β (cos ϕ cos β + sin ϕ sin β) sin α
(tan θ cos β − sin β) cos α (tan ϕ cos α − sin α) sin β
∴ + =0
(cos α − sin α tan θ) cos β (cos β + tan ϕ sin β) sin α
tan θ − tan β tan ϕ cot α − 1
∴ + =0
1 − tan α tan θ cot β + tan ϕ
∴ (tan θ − tan β)(cot β + tan ϕ) + (tan ϕ cot α − 1)(1 − tan α tan θ) = 0
∴ tan θ(cot β + tan α) + tan ϕ(cot α − tan β) = 2.
But
tan β cos(α − β)
tan θ = − tan ϕ · ;
tan α cos(α + β)
tan β cos(α − β)
∴ − tan ϕ · (cot β + tan α) + tan ϕ(cot α − tan β) = 2
tan α cos(α + β)
∴ − tan ϕ(cot α + tan β) cos(α − β) + tan ϕ(cot α − tan β) cos(α + β)
= 2 cos(α + β)
∴ tan ϕ {cot α [cos(α + β) − cos(α − β)] − tan β [cos(α + β) + cos(α − β)]}
= 2 cos(α + β)
∴ tan ϕ {cot α sin α sin β + tan β cos α cos β} = − cos(α + β).
cos(α + β) 1
∴ tan ϕ = − = (tan α − cot β), and
2 cos α sin β 2
tan β cos(α − β)
tan θ = − tan ϕ
tan α cos(α + β)
cos(α − β) 1
= = (cot α + tan β).
2 sin α cos β 2
CHAPTER XII : Theory of Proportional Parts 94

Problem 28.
If
2 sin β sin θ tan(θ − α)
= = ,
1+x cos(β − θ) cot β
prove that
( )( )
α α
x2 = − 2 cot β
cot tan + 2 cot β .
2 2
2 sin β sin θ sin β sin θ 1
Solution. If = = = .
1+x cos(β − θ) cos β cos θ + sin β sin θ cot β cot θ + 1
1+x
∴ cot β cot θ + 1 =
2
1+x x−1
∴ cot β cot θ = −1= (12)
2 2
Again
2 tan(θ − α) (tan θ − tan α) tan β
= =
1+x cot β 1 + tan θ tan α
∴ 2(1 + tan θ tan α) = (1 + x)(tan θ − tan α) tan β;
2 + (1 + x) tan α tan β
∴ tan θ = (13)
(1 + x) tan β − 2 tan α
From (12) and (13) by multiplication
2 + (1 + x) tan α tan β x − 1
cot β = ·
(1 + x) tan β − 2 tan α 2
( )
∴ 2 cot β {(1 + x) tan β − 2 tan α} = 2(x − 1) + x2 − 1 tan α tan β
( )
∴ 2(1 + x) − 4 cot β tan α = 2(x − 1) + x2 − 1 tan α tan β
∴ x2 tan α tan β = 4 − 4 cot β tan α + tan α tan β
x2 = 4 cot α cot β − 4 cot2 β + 1
( )
α α
= 2 cot − tan cot β − 4 cot2 β + 1
( 2 2) ( )
α α
= cot − 2 cot β tan + 2 cot β .
2 2

α
Problem 29. Given sin θ sin ϕ = sin α sin β, tan ϕ cos β = cot , prove that one of the
2
θ α
values of sin is sin sin β.
2 2
Solution.
θ θ sin α sin β
∵ sin θ sin ϕ = sin α sin β; ∴ 2 sin cos =
2 2 sin ϕ
2 θ 4 θ sin2 α sin2 β
∴ 4 sin − 4 sin =
2 2 sin2 ϕ
θ θ sin2 α sin2 β
∴ 4 sin4 − 4 sin2 + 1 = 1 −
2 2 sin2 ϕ
2 α
cot
∵ sin2 ϕ = 2
2
α
cot + cos2 β
2( )
α
sin2 α cot2 + cos2 β sin2 β
θ θ 2
∴ 4 sin4 − 4 sin2 + 1 = 1 − α
2 2 cot2
( 2 )
α α
= 1 − 4 sin4 cot2 + cos2 β sin2 β
2 ( 2 )
α α
= 1 − 4 sin4 cot2 + 1 − sin2 β sin2 β
2 2
α α β
= 1 − 4 sin2 sin2 β + 4 sin4 sin4
2 2 2
CHAPTER XII : Theory of Proportional Parts 95

( )
θ α
∴ 2 sin2
− 1 = ± 1 − 2 sin2 sin2 β .
2 2
2 θ 2 α 2
Taking the lower sign we have sin = sin sin β.
2 2

Problem 30. Given sin ϕ = n sin θ, tan ϕ = 2 tan θ, find the limiting values of n that
these equations may coexist. √
Solution. sin ϕ = n sin θ; ∴ cos ϕ = 1 − n2 sin2 θ.
n sin θ
∴ tan ϕ = √ .
1 − n2 sin2 θ
n sin θ 2 sin θ
∴ √ = 2 tan θ =
1 − n2 sin2 θ cos θ

∴ n cos θ = 2 1 − n2 sin2 θ
( ) ( )
∴n 2
1 − sin θ = 4 1 − n2 sin2 θ
2

4 − n2
∴ 3n2 sin2 θ = 4 − n2 ; ∴ sin2 θ = .
3n2
This must lie between 0 and 1, so that 4 − n2 must lie between 0 and 3n2 , therefore 4
must lie between n2 and 4n2 ; therefore n2 must lie between 1 and 4.

Problem 31. Show by means of a Trigonometrical formula that


if
x + y + z = xyz,
then
2x 2y 2z 2x 2y 2z
+ + = · · .
1 − x2 1 − y2 1 − z2 1 − x2 1 − y 2 1 − z 2
Solution. Assume x = tan A and y = tan B; then by Art. 114 (page 328), we have
z = tan C, where A + B + C = 180◦ .
Therefore 2A + 2B + 2C = 360◦ ; therefore tan(2A + 2B + 2C) = 0;
and therefore, as in Art. 114, (page 328)
tan 2A + tan 2B + tan 2C = tan 2A tan 2B tan 2C;
2 tan A 2 tan B 2 tan C 2 tan A 2 tan B 2 tan C
∴ + + = · · .
1 − tan2 A 1 − tan2 B 1 − tan2 C 1 − tan2 A 1 − tan2 B 1 − tan2 C

Problem 32. Find the values of v, x, y, z from the equations


sin x sin y sin z
v= = = ; x + y + z = 2π.
sin a sin b sin c
Solution.
v sin c = sin z = sin(2π − x − y) = − sin(x + y)
= − sin x cos y − cos x sin y = −v sin a cos y − v sin b cos x.
∴ v = 0; or sin c = − sin a cos y − sin b cos x.
Take the latter, thus sin a cos y = − sin c − sin b cos x;
but sin a sin y = sin b sin x;
square and add, thus
sin2 a = sin2 b + sin2 c + 2 sin b sin c cos x;
sin2 a − sin2 b − sin2 c
∴ cos x = .
2 sin b sin c
Similarly cos y and cos z may be found, and then v.
If v = 0, we have sin x = 0, sin y = 0, and sin z = 0. This will give us three solutions;
x = 0, y = π, z = π; x = π, y = 0, z = π; x = π, y = π, z = 0 : and also three solutions,
x = 0, y = 0, z = 2π; x = 0, y = 2π, z = 0; x = 2π, y = 0, z = 0.

2
Problem 33. Find the limit of (cos αx)cosec βx when x is zero.
CHAPTER XII : Theory of Proportional Parts 96

2
Solution. Let u = (cos αx)cosec βx ;
therefore
1 ( )
log u = cosec βx log cos αx = cosec 2 βx log 1 − sin2 αx
2
{ 2 }
1 1 1
=− 2
sin2 αx + sin4 αx + sin6 αx + . . . .
2 sin βx 2 3
sin αx α sin αx βx
Now = · · ;
sin βx β αx sin βx
sin αx βx
when x is zero the value of is unity, and so also if the value of ;
αx sin βx
sin αx α sin2 αx α2
thus = ; therefore = 2.
sin βx β sin2 βx β
4
sin αx
The limit of is zero, and so also the other terms in log u vanish, and as
sin4 βx
α2
in Art. 150 (page 334), their sum vanishes also. Hence log u = − 2 , and therefore

2
− α2
u=e 2β .

Problem 34. From a table of natural tangents which goes to 7 places of decimals,
1
show that an angle may be determined within about 200 th part of a second when the
angle is nearly 60◦ .
Solution. By Art. 188 (page 334), if h is very small we have tan(θ + h) − tan θ = h sec2 θ;
thus if θ be nearly equivalent to 60◦ we have approximately
tan(θ + h) − tan θ = 4h.
Since the tables extend to 7 places of decimals it follows that we can discriminate
angles which are near 60◦ , by means of their tangents, when the circular measure h
1 1
of the difference is such that 4h = .0000001. Thus h = of ; the corresponding
4 107
1 1 180 18 × 9 1
value in seconds is × 7 × × 60 × 60, that is , that is about .
4 10 π 10000π 200

Problem 35. When an angle is very nearly equal to 64◦ 36′ , show that the angle can
be determined from its L sine within about 10 1
th of a second; having given loge 10 ·
tan 64◦ 36′ = 4.8492, and the tables going to 7 places of decimals.
Solution. By Art. 196 (page 335), if h is very small we have
h
L sin(θ + h) − L sin θ = µh cot θ = ;
(loge 10) tan θ
◦ ′
thus if θ be nearly equivalent to 64 36 , we have approximately
h
L sin(θ + h) − L sin θ = ;
4.8492
Since the tables extend to 7 places of decimals it follows that we can discriminate
angles which are near 64◦ 36′ , by means of their L sines, when the circular measure
h 4.8492
of the difference is such that = .0000001. Thus h = ; the corresponding
4.8492 107
value in seconds is
4.8492 180
× × 60 × 60;
107 π
1
this will be found to be about .
10

Problem 36.( Show that )( )( )


α α α α
1 − tan2 1 − tan2 2 1 − tan2 3 . . . ad inf. = .
2 2 2 tan α
Solution.
α α
cos2 − sin2
2 α 2 2 = cos α ,
1 − tan = α α
2 cos2 cos2
2 2
CHAPTER XII : Theory of Proportional Parts 97

α α α
α cos2 − sin2 cos
1 − tan 2
= 4 4 = 2 ,
4 α α
cos2 cos2
4 4
and so on.
In this way we find that the proposed expression
α α α
cos α cos cos 2 cos 3 . . .
= 2 2 2
α α α
cos2 cos2 2 cos2 3 . . .
2 2 2
cos α
= α α α
cos cos 2 cos 3 . . .
2 2 2
sin α α
= cos α ÷ = . See Art. 129 (page 331).
α tan α

Problem 37. If A, B, C be positive angles which satisfy the equation


sin2 A + sin2 B + sin2 C = 1,
prove that A + B + C is greater than 90◦ .
Solution. We have universally
sin2 (A + B) = (sin A cos B + cos A sin B)2
= sin2 A cos2 B + cos2 A sin2 B + 2 sin A cos A sin B cos B
( ) ( )
= sin2 A 1 − sin2 B + sin2 B 1 − sin2 A + 2 sin A cos A sin B cos B
= sin2 A + sin2 B + 2 sin A sin B (cos A cos B − sin A sin B)
∴ sin2 (A + B) = sin2 A + sin2 B + 2 sin A sin B cos(A + B) (14)
Also in the present case
sin2 A + sin2 B = 1 − sin2 C = cos2 C (15)
If A + B is greater than 90◦ , then a fortiori A + B + C is greater than 90◦ .
If A + B is less than 90◦ , then sin2 (A + B) is greater than sin2 A + sin2 B by (14), and
therefore greater than cos2 C by (15); and therefore A + B is greater than 90◦ − C, so
that A + B + C is greater than 90◦ .

Problem 38. A circle is drawn touching the tangent and secant of a given angle α,
as well as the corresponding arc; find its radius and explain the double value. If one
π
value be equal to the radius of the original circle, show that α = .
3
Solution. Take the diagram of Art. 71 (page 323). Let α be the angle P AB. Suppose
a circle having its centre O within the space bounded by P B, BT and T P ; let it touch
the arc P B, the tangent BT, and the secant AP T. Let ρ denote the radius of this circle,
and r the radius of the original circle.
OT will bisect the angle AT B, and OA will pass through the point of contact of the
circles. Let N be the point of contact of the secant AP T and the circle with centre O.
Then ( )
1 π
N T = ρ cot − α ; OA = r + ρ;
√2 2 √
∴ AN = (r + ρ)2 − ρ2 = r2 + 2rρ.
√ ( )
π α
∴ r2 + 2rρ + ρ cot − = AT = r sec α;

4 2 ( )
π α
∴ r2 + 2rρ = r sec α − ρ cot − .
4 2
By squaring we obtain a quadratic equation for determining ρ. The reason why we
have a quadratic equation is that another circle can also be drawn, which may be
said to fulfil the conditions. For produce P A through A to meet the original circle
again at p; then we may have a circle outside the arc Bp, touching this arc, touching
T B produced through B, and touching T p produced through p. The corresponding
equation would be ( ) √
π α
ρ cot − − r2 + 2rρ = r sec α.
4 2
CHAPTER XII : Theory of Proportional Parts 98

This differs from the former only in the sign of the radical, and therefore leads to the
same quadratic equation.
Suppose ρ = r; then ( )
π α
√ cos −
1
± 3= − ( 4 2)
cos α π α
sin −
4 2
α α
1 cos + sin
= − 2 2
cos α α α
cos − sin
( 2 2 )
α α 2
cos + sin
1 2 2
= − α α
cos α cos2 − sin2
2 2
1 1 + sin α
= − = − tan α.
cos α cos α
√ π
Hence taking 3 = tan α, we have α = .
3

Problem 39.
If
l cos(θ − β) − m cos(θ − α) = n,
show that √
l sin(θ − β) − m sin(θ − α) = l2 + m2 − n2 − 2lm cos(α − β).
Solution. Let x denote the value of l sin(θ − β) − m sin(θ − α); so that
l cos(θ − β) − m cos(θ − α) = n, l sin(θ − β) − m sin(θ − α) = x.
Square and add; thus
l2 + m2 − 2lm {cos(θ − β) cos(θ − α) + sin(θ − β) sin(θ − α)} = n2 + x2 ;
∴ l2 + m2 − 2lm cos(α − β) = n2 + x2 ;

∴x= l2 + m2 − n2 − 2lm cos(α − β).

π
Problem 40. Show that θ − sin θ is less than tan θ − θ if θ lies between 0 and .
2
Solution. θ − sin θ is less than tan θ − θ if 2θ is less than sin θ + tan θ, that is if 2θ is less
than tan θ(1 + cos θ), that is if 2θ is less than
θ
2 tan 2
2 × ,
θ θ
1 − tan2 1 + tan2
2 2
θ
θ tan
that is if is less than 2 , and this is obviously the case, because θ is less
2 θ 2
1 − tan4
2
θ
than tan .
2
CHAPTER XIII
Relations between the Sides of a Triangle and the
Trigonometrical Functions of the Angles

Problem 1. The sides of a triangle are x2 + x + 1, 2x + 1, and x2 − 1, show that the


greatest angle is 120◦ .
Solution. The greatest angle is opposite to the greatest side; thus the cosine
( )2 ( )2
x2 − 1 + (2x + 1)2 − x2 + x + 1
=
2 (x2 − 1) (2x + 1)
( )
x4 − 2x2 + 1 + 4x2 + 4x + 1 − x4 + x2 + 1 + 2x3 + 2x2 + 2x
=
2 (x2 − 1) (2x + 1)
−2x3 − x2 + 2x + 1 1
= =− .
2 (2x3 + x2 − 2x − 1) 2
Therefore the angle is 120◦ .

sin A
Problem 2. If cos B = , show that the triangle is isosceles.
2 sin C
Solution.
2 sin C cos B = sin A = sin(B + C) = sin B cos C + cos B sin C;
∴ sin C cos B = sin B cos C;
∴ sin(C − B) = 0; ∴ B = C.

Problem 3. In a right-angled triangle of which C is the right angle,


A b+c
cot = .
2 a
b a
Solution. We have cos A = , sin A = ;
c c
1 + cos A c+b A b+c
∴ = ; ∴ cot = .
sin A a 2 a

A+B
Problem 4. If in a triangle a tan A + b tan B = (a + b) tan , show that A = B.
2
Solution.
A+B
a tan A + b tan B = (a + b) tan
( ) ( 2 )
A+B A+B
∴ a tan A − tan = b tan − tan B
( 2 ) ( 2 )
A+B A+B A+B A+B
a sin A cos − cos A sin b sin cos B − cos sin B
2 2 2 2
∴ =
A+B A+B
cos A cos cos B cos
2 2
A−B A−B
a sin b sin
∴ 2 = 2
cos A cos B
a cos A
∴ = .
b cos B
a sin A
∵ =
b sin B
sin A cos A
∴ =
sin B cos B
∴ tan A = tan B; ∴ A = B.
CHAPTER XIII : Relations ≬ the Sides of a Triangle and the Trig Func of the Angles 100

Problem 5. The angles of a plane triangle form a geometrical progression of which


1 π
the common ratio is ; show that the greatest side is to the perimeter as 2 sin is to
2 14
unity.
Solution. Let 2α denote the least angle; then the other angles are 4α and 8α respec-
π
tively: therefore 2α + 4α + 8α = π; therefore α = .
14
Then by Art. 214 (page 336), the ratio of the greatest side to the perimeter
sin 8α
=
sin 2α + sin 4α + sin 8α
sin 8α 2 sin 4α cos 4α
= = ;
sin 2α + sin 4α + sin 6α 2 sin 3α cos α + 2 sin 3α cos 3α
π
but 4α + 3α = , ∴ cos 4α = sin 3α; hence this expression
2
sin 4α 2 sin 2α cos 2α sin 2α
= = = = 2 sin α.
cos α + cos 3α 2 cos α cos 2α cos α

Problem 6. If A′ , B ′ , C ′ are the external angles of a triangle, show that


2bc vers A′ + 2ca vers B ′ + 2ab vers C ′ = (a + b + c)2 .
Solution.
2bc vers A′ + 2ca versB ′ + 2ab vers C ′
= 2bc(1 − cos A′ ) + 2ca(1 − cos B ′ ) + 2ab(1 − cos C ′ )
= 2bc(1 + cos A) + 2ca(1 + cos B) + 2ab(1 + cos C)
A B C
= 4bc cos2 + 4ca cos2 + 4ab cos2
2 2 2
= 4s(s − a) + 4s(s − b) + 4s(s − c)
= 4s(3s − a − b − c) = 4s2 = (2s)2 = (a + b + c)2 .

Problem 7. From the angle A of any triangle ABC a perpendicular AD is drawn


to the base, and from D perpendiculars DE, DF are drawn to AB, AC respectively :
show that
AE · EB · cos2 C = AF · F C · cos2 B.
Solution. Let AD = p. Suppose the angles B and C to be acute, as in the left-handed
diagram of Art. 214 (page 336). Then
AE = p cos(90◦ − B) = p sin B,
DE = p sin(90◦ − B) = p cos B,
EB = DE cot B = p cos B cot B;
AE · EB = p2 cos2 B.
Similarly AF · F C = p2 cos2 C.
∴ AE · EB cos2 C = AF · F C cos2 B.
Next suppose one of the angles B and C to be obtuse, say the angle C, as in the
right-hand diagram of Art. 214 (page 336).
Then
AE · EB = p2 cos2 B as before,
AF = p cos(C − 90◦ ) = p sin C,
DF = p sin(C − 90◦ ) = −p cos C,
F C = DF cot(180◦ − C) = −DF cot C = p cos C cot C;
∴ AF · F C = p2 cos2 C, as before.

Problem 8. Let a, b, c be the sides of a triangle and the opposite angles be 2θ, 3θ, 4θ.
Show that
( )2
2b
tan2 θ = − 1.
a+c
Solution.
sin 2θ + sin 4θ a+c
= ;
sin 3θ b
CHAPTER XIII : Relations ≬ the Sides of a Triangle and the Trig Func of the Angles 101

a+c a+c
∴ 2 cos θ = ; ∴ cos θ = ;
b ( )2b2
1 2b
∴ tan2 θ = −1= − 1.
cos2 θ a+c

Problem 9. ABC is a triangle of which C is an obtuse angle. Show that tan A tan B
is less than unity.
Solution. Since C is obtuse, A + B is less than 90◦ ; therefore cos(A + B) is positive,
therefore cos A cos B −sin A sin B is positive; therefore sin A sin B is less than cos A cos B;
sin A sin B
therefore is less than unity, that is tan A tan B is less than unity.
cos A cos B

Problem 10. If the sides a, b, c of a triangle be in arithmetical progression, show


that
A−C B
cos= 2 sin , and
2 2
2 C 2 A 3b
a cos + c cos = .
2 2 2
Solution. Since a, b, c are in Arithmetical Progression, so are sin A, sin B, sin C.
∴ sin A + sin C = 2 sin B;
A+C A−C B B B A+C
∴ sin cos = 2 sin cos = 2 sin sin ;
2 2 2 2 2 2
A−C B
∴ cos = 2 sin .
2 2
Again
C A
a cos2 + c cos2
2 2
a c
= (1 + cos C) + (1 + cos A)
2 2
1 1 1 b
= (a + c) + (a cos C + c cos A) = (a + c) + , by Art. 216 (page 337),
2 2 2 2
b
= b + , by hypothesis,
2
3b
= .
2

Problem 11. If D be the middle point of the side BC of a triangle


cot BAD − cot B = 2 cot A.
Solution. From the triangle ABD we have
sin ADB AB 2c
= = .
sin BAD BD a
Put ϕ for BAD; thus
sin(ϕ + B) 2c 2 sin C
= = ;
sin ϕ a sin A
sin ϕ cos B + cos ϕ sin B 2 sin C
∴ = ;
sin ϕ sin A
2 sin C 2 sin(A + B)
∴ cot B + cot ϕ = =
sin A sin B sin A sin B
= 2 cot A + 2 cot B;
∴ cot ϕ − cot B = 2 cot A.

Problem 12. If an angle of a triangle be divided into two parts such that the sines
are in the ratio of the sides adjacent to them respectively, show that the difference of
their cotangents is equal to the difference of the cotangents of the angles opposite to
their sides.
Solution. Let the angle A of a triangle be divided into two parts by a straight line
CHAPTER XIII : Relations ≬ the Sides of a Triangle and the Trig Func of the Angles 102

sin ϕ c
AD; denote BAD by ϕ and CAD by ψ, and suppose that = .
sin ψ b
sin(A − ψ) c sin C
∴ = = ;
sin ψ b sin B
sin C
∴ sin A cot ψ − cos A = ;
sin B
sin(A + B)
∴ cot ψ = cot A + = 2 cot A + cot B.
sin A sin B
Similarly
cot ϕ = 2 cot A + cot C.
∴ cot ψ − cot ϕ = cot B − cot C.

Problem 13. If the cotangents of the angles of a triangle be in arithmetic progres-


sion, the squares of the sides will also be in arithmetic progression.
Solution. Suppose cot A + cot B = 2 cot B;
cos A cos C 2 cos B
∴ + = ;
sin A sin C sin B
sin(A + C) 2 cos B
∴ = ;
sin A sin B sin B
sin2 B
∴ = 2 cos B;
sin A sin C
b2 a2 + c2 − b2
∴ = ;
ac ac
∴ 2b2 = a2 + c2 .
Thus a2 , b2 , c2 are in Arithmetical Progression.

Problem 14. Given the vertical angle and the ratio between the base and altitude
of a triangle, find the tangents of the angles into which the vertical angle is divided
by the perpendicular drawn from it to the base.
Solution. Let a perpendicular AD be drawn from the angle A of a triangle on the
base BC. Let BAD = ϕ, and CAD = ψ. Let m denote the ratio of the base BC to the
perpendicular AD.
Then in the case of the left-hand diagram of Art. 214 (page 336), we have
BD CD
tan ϕ = , tan ψ = ;
AD AD
BD + CD BC
∴ tan ϕ + tan ψ = = = m; (16)
AD AD
Also ϕ + ψ = A; thus
tan ϕ + tan ψ
tan A = tan(ϕ + ψ) = (17)
1 − tan ϕ tan ψ
Hence from (16) and (17), we can find tan ϕ and tan ψ.
Similarly in the case of the right-hand diagram of Art. 214 (page 336), we have
tan ϕ − tan ψ = m, and
tan ϕ − tan ψ
tan A = tan(ϕ − ψ) = .
1 + tan ϕ tan ψ

Problem 15. If the base of a triangle be divided into three equal parts, and t1 , t2 , t3
be the tangents of the angles which they subtend (at the vertex
)
( )( )
1 1 1 1 1
+ + =4 1+ 2 .
t1 t2 t2 t3 t2
Solution. Let the base BC of a triangle be divided at D and E, so that BD = DE = EC.
Let the angle BAD be denoted by ϕ1 , the angle DAE by ϕ2 , and the angle EAC by ϕ3 .
Then from the triangle AEB, we have
sin(ϕ1 + ϕ2 ) BE 2 a
= = · ,
sin AEB AB 3 c
CHAPTER XIII : Relations ≬ the Sides of a Triangle and the Trig Func of the Angles 103

and from the triangle AEC, we have


sin ϕ3 EC 1 a
= = · ;
sin AEC AC 3 b
therefore by division
sin(ϕ1 + ϕ2 ) 2b
= .
sin ϕ3 c
In the same manner we wee that
sin(ϕ3 + ϕ2 ) 2c
= .
sin ϕ1 b
sin(ϕ1 + ϕ2 ) sin(ϕ3 + ϕ2 ) ( )
∴ = 4 = 4 sin2 ϕ2 + cos2 ϕ2 ;
sin ϕ1 sin ϕ3 ( )
∴ (cos ϕ2 + sin ϕ2 cot ϕ1 ) (cos ϕ2 + sin ϕ2 cot ϕ3 ) = 4 sin2 ϕ2 + cos2 ϕ2 ;
( )
∴ (cot ϕ2 + cot ϕ1 ) (cot ϕ2 + cot ϕ3 ) = 4 1 + cot2 ϕ2 .

Problem 16. If the sines of the angles of a triangle be in arithmetical progression,


1
the product of the tangents of half the greatest and half the least is .
3
Solution. Suppose that sin A + sin C = 2 sin B,
A+C A−C B B A+C A+C
2 sin cos = 4 sin cos = 4 cos sin ;
2 2 2 2 2 2
A−C A+C
∴ cos = 2 cos ;
2 2
A C A C A C A C
∴ cos cos + sin sin = 2 cos cos − 2 sin sin ;
2 2 2 2 2 2 2 2
A C A C
∴ 3 sin sin = cos cos ;
2 2 2 2
A C 1
∴ tan tan = .
2 2 3

Problem 17. If the side BC of a triangle be bisected at D and AD be drawn, show


that
2bc sin A
tan ADB = 2 .
b − c2
Solution. Denote ADB by ϕ. From the triangle ABD we have
sin BAD BD a
= = ;
sin ADB AB 2c
sin(ϕ + B) a
∴ = ;
sin ϕ 2c
a
∴ cos B + sin B cot ϕ = ;
2c
a
− cos B
∴ cot ϕ = 2c ;
sin B
2c sin B 2ac sin B
tan ϕ = = 2
a − 2c cos B a − (a2 + c2 − b2 )
2ac sin B 2bc sin A
= 2 = 2 .
b − c2 b − c2

A B C
Problem 18. If A, B, C be the angles of a triangle and cot , cot , cot in arith-
2 2 2
metical progression, show that
A C
cot cot = 3.
2 2
CHAPTER XIII : Relations ≬ the Sides of a Triangle and the Trig Func of the Angles 104

A C B
Solution. Here cot + cot = 2 cot ;
2 2 2
A C B A+C
cos cos 2 cos 2 sin
∴ 2 + 2 = 2 = 2 ;
A C B A+C
sin sin sin cos
2 2 2 2
A+C A+C
sin 2 sin
∴ 2 = 2 ;
A C A+C
sin sin cos
2 2 2
A+C A C
∴ cos = 2 sin sin ;
2 2 2
A C A C A C
∴ cos cos − sin sin = 2 sin sin ;
2 2 2 2 2 2
A C A C
∴ cos cos = 3 sin sin ;
2 2 2 2
A C
∴ cot cot = 3.
2 2

Problem 19. Straight lines are drawn from the angles A and B of a triangle dividing
the angles respectively into parts whose sines are in the ratio of 1 to n; these straight
lines intersect at D. Show that DC either bisects the angle C or divides it into parts
whose sines are in the ratio of 1 to n2 .
sin DAC 1 sin DBC 1
Solution. First suppose that = , and that = .
sin DAB n sin DBA n
We have
sin DCB BD
= , and
sin DBC DC
sin DBC 1
= ;
sin DBA n
sin DCB BD 1
∴ = · .
sin DBC DC n
Similarly
sin DCA AD 1
= · .
sin DAB DC n
sin DCB sin DAB BD
∴ · = ;
sin DCA sin DBA DA
sin DCB DB BD
∴ · = ;
sin DCA DA DA
sin DCB
∴ = 1.
sin DCA
In this case the angle C is bisected by DC.
sin DAC 1 sin DBA 1
Next suppose that = , and that = ; thus the angle B is divided
sin DAB n sin DBC n
into two parts equal to the two former, but differently situated.
Then proceeding as before we have
sin DCB BD
= , and
sin DBC DC
sin DBC
= n;
sin DBA
sin DCB n · BD
∴ = .
sin DBA DC
sin DCA AD 1
Also = .
sin DAB DC n
Hence we find that
sin DCB
= n2 .
sin DCA

Problem 20. If l be the length of the straight line which bisects the angle A of a
triangle and is terminated by the base, θ the angle which it makes with the base, 2s
CHAPTER XIII : Relations ≬ the Sides of a Triangle and the Trig Func of the Angles 105

the perimeter of the triangle, show that


( )
A A
s sin θ − sin
= l cos sin θ.
2 2
Solution. Let the straight line which bisects the angle A of a triangle meet the base
at D.
Then
∠ADC = ∠B + ∠BAD;
( )
A
∴ sin θ = sin B + .
2
( ) { ( ) }
A A A
∴ s sin θ − sin = s sin B + − sin
2 2 2
B+A B C B
= 2s cos sin = 2s sin sin ;
2 2 2 2
C B
Put for sin and sin their values by Art. 217 (page 337); thus we have
2 2 √
C B 2s (s − b)(s − c)
2s sin sin = (s − a)
2 2 a bc
2s(s − a) A 2bc A A
= sin = cos2 sin
a 2 a 2 2
bc A
= cos sin A.
a 2
Again l sin θ = the perpendicular from A on BC
= b sin C.
A A
∴ l sin θ cos = b sin C cos
2 2
bc A
= sin A cos , by Art. 214 (page 336).
( a )2
A A
∴ s sin θ − sin = l sin θ cos .
2 2

Problem 21. If θ and ϕ be the greatest and least angles of a triangle, the sides of
which are in arithmetical progression, show that
4(1 − cos θ)(1 − cos ϕ) = cos θ + cos ϕ.
Solution. The third angle of the triangle will be π − θ − ϕ; and as the sines of the
angles must be in Arithmetical Progression, we have
sin θ + sin ϕ = 2 sin(π − θ − ϕ) = 2 sin(θ + ϕ);
θ+ϕ θ−ϕ θ+ϕ θ+ϕ
∴ 2 sin cos = 4 sin cos ;
2 2 2 2
θ+ϕ θ−ϕ
∴ 2 cos = cos ;
( 2 ) 2
θ ϕ θ ϕ θ ϕ θ ϕ
∴ 2 cos cos − sin cos = cos cos + sin cos ;
2 2 2 2 2 2 2 2
θ ϕ θ ϕ
∴ cos cos = 3 sin sin ;
2 2 2 2
θ 2 ϕ 2 θ 2 ϕ
∴ cos 2
cos = 9 sin sin ;
2 2 ( 2 2) ( )
θ ϕ
= 9 1 − cos2 1 − cos2 ;
2 2
( )( ) ( )( )
θ ϕ θ ϕ θ ϕ
∴ 8 1 − cos2 1 − cos2 = cos2 cos2 − 1 − cos2 1 − cos2
2 2 2 2 2 2
θ ϕ θ ϕ
= cos2 + cos2 − 1 = cos2 − sin2 ;
2 2 2 2
θ ϕ θ+ϕ θ−ϕ
∴ 8 sin2 sin2 = cos cos ;
2 2 2 2
θ+ϕ θ−ϕ
∴ 4(1 − cos θ)(1 − cos ϕ) = 2 cos cos ;
2 2
CHAPTER XIII : Relations ≬ the Sides of a Triangle and the Trig Func of the Angles 106

= cos θ + cos ϕ.
a2 + b2 − c2 a+c
Or thus, cos θ = , and b = ;
2ab 2
a−c b a−c a+c 5a − 3c
∴ cos θ = + = + = .
a 2a a 4a 4a
Similarly
5c − 3a
cos ϕ = .
4c
(3c − a)(3a − c) 10ac − 3a2 − 3c2
∴ 4(1 − cos θ)(1 − cos ϕ) = = ; and
4ac 4ac
5a − 3c 5c − 3a 10ac − 3a − 3c
2 2
cos θ + cos ϕ = + = .
4a 4c 4ac

Problem 22. From the angular points of a triangle ABC straight lines are drawn
making each the same angle α towards the same parts with the sides of the triangle
taken in order. Show that these straight lines will form another triangle similar to
the former, and that the linear dimensions of the two triangles are in the ratio of
cos α − sin α(cot A + cot B + cot C) to 1.
Solution. Draw from A, B, C respectively straight lines to meet the opposite sides
at D, E, F, so that the ∠BAD = ∠CBE = ∠ACF = α.
Let LM N be the triangle formed by the straight lines thus drawn : so that A, L, M, D
are in one straight line; B, M, N, E on another; and C, N, L, F on a third. Then will
the triangle LM N be similar to the triangle ABC.
For the ∠M LN = ∠M AC + ∠LCA = A − α + α = A; similarly ∠N M L = B, and
∠LN M = C. Thus the triangle LM N is equivalent to the original triangle, and there-
fore similar to it.

Again
BN sin BCN sin(C − α) sin(C − α)
= = = ;
BC sin BN C sin(π − C) sin C
a sin(C − α)
∴ BN = ,
sin C
and
BM sin BAM sin α sin α
= = = ;
BA sin BM A sin(π − B) sin B
c sin α
∴ BM = .
sin B
a sin(C − α) c sin α
∴ MN = −
sin C sin B
a sin C
= a cos α − a cot C sin α − sin α
sin A sin B
a sin(A + B)
= a cos α − a cot C sin α − sin α
sin A sin B
= a cos α − a sin α(cot C + cot B + cot A).
The ratio of this to a is the same as the ratio of
cos α − sin α(cot A + cot B + cot C) to unity.

Show that in any triangle the relations given in the following Examples, from 23 to
42, hold:

Problem 23. a(b cos C − c cos B) = b2 − c2 .


a2 + b2 − c2 a2 + c2 − b2
Solution. ab cos C − ac cos B = − = b 2 − c2 .
2 2

Problem 24. a(cos B cos C + cos A) = b(cos A cos C + cos B) = c(cos A cos B + cos C).
CHAPTER XIII : Relations ≬ the Sides of a Triangle and the Trig Func of the Angles 107

Solution.
a(cos B cos C + cos A) = a {cos B cos C − cos(B + C)}
a
= a sin B sin C = sin A sin B sin C.
sin A
Similarly
b
sin A sin B sin C; and
b(cos A cos C + cos B) =
sin B
c
c(cos A cos B + cos C) = sin A sin B sin C.
sin C
Thus the three expressions are equal by Art. 214 (page 336).

A B C
Problem 25. (b + c − a) tan = (c + a − b) tan = (a + b − c) tan .
2 2 2
Solution. √
A A (s − b)(s − c)
(b + c − a) tan = 2(s − a) tan = 2(s − a)
2 2 s(s − a)

(s − a)(s − b)(s − c)
2

= .
s
Similarly the other two proposed expressions reduce to the same symmetrical form.

Problem 26. b cos B + c cos C = a cos(B − C).


Solution.
a sin B a sin C
b cos B + c cos C = cos B + cos C
sin A sin A
a
= (sin 2B + sin 2C)
2 sin A
2a sin(B + C) cos(B − C)
= = a cos(B − C).
2 sin(B + C)

Problem 27. (a + b) cos C + (b + c) cos A + (c + a) cos B = a + b + c.


Solution. By Art. 216 (page 337),
c cos B + b cos C = a, a cos C + c cos A = b, b cos A + a cos B = c;
therefore by addition
c(cos B + cos A) + b(cos A + cos C) + a(cos C + cos B) = a + b + c.

Problem 28. (a2 − b2 ) cot C + (b2 − c2 ) cot A + (c2 − a2 ) cot B = 0.


a b c
Solution. Let k stand for , , and which we know are all equal. Then
sin A sin B sin C
(a − b ) cot C + (b − c ) cot A + (c − a ) cot B
2 2 2 2 2 2
{( ) ( ) ( ) }
= k2 sin2 A − sin2 B cot C + sin2 B − sin2 C cot A + sin2 C − sin2 A cot B
= k2 {sin(A + B) sin(A − B) cot C + sin(B + C) sin(B − C) cot A
+ sin(C + A) sin(C − A) cot B}
= k2 {sin(A − B) cos C + sin(B − C) cos A + sin(C − A) cos B}
= −k2 {sin(A − B) cos(A + B) + sin(B − C) cos(B + C) + sin(C − A) cos(C + A)}
k2
=− {sin 2A − sin 2B + sin 2B − sin 2C + sin 2C − sin 2A}
2
= 0.

C B A
Problem 29. (a − b) cot + (c − a) cot + (b − c) cot = 0.
2 2 2
Solution. Let k have the same meaning as in the preceding solution; then
C B A
(a − b) cot + (c − a) cot + (b − c) cot
{ 2 2 2 }
C B A
= k (sin A − sin B) cot + (sin C − sin A) cot + (sin B − sin C) cot
2 2 2
CHAPTER XIII : Relations ≬ the Sides of a Triangle and the Trig Func of the Angles 108

{ }
A−B A+B C−A C+A B−C B+C
= 2k sin sin + sin sin + sin sin
{ 2 2 2 2 2 } 2
2 A 2 B 2 C 2 A 2 B 2 C
= 2k sin − sin + sin − sin + sin − sin
2 2 2 2 2 2
= 0.

A B 2c
Problem 30. 1 − tan tan = .
2 2 a+b+c
Solution. √ √
A B (s − b)(s − c) (s − a)(s − c)
1 − tan tan =1− ×
2 2 s(s − a) s(s − b)
s−c a+b−c 2c
=1− =1− = .
s a+b+c a+b+c

Problem 31.
(a + b + c)(cos A + cos B + cos C)
A B C
= 2a cos2 + 2b cos2 + 2c cos2 .
2 2 2
Solution.
(a + b + c)(cos A + cos B + cos C)
= a cos A + b cos B + c cos C
+ a cos B + b cos A + a cos C + c cos A + b cos C + c cos B
= a cos A + b cos B + c cos C + c + b + a, by Art. 216 (page 337),
= a(1 + cos A) + b(1 + cos B) + c(1 + cos C)
A B C
= 2a cos2 + 2b cos2 + 2c cos2 .
2 2 2

sin2 A cos A cos B cos A cos C cos B cos C


Problem 32. = + + .
a2 ab ac bc
Solution. Let k have the same meaning as in the solution of Problem 28; then
cos A cos B cos A cos C cos B cos C
+ +
ab {ac bc }
1 cos A cos B cos A cos C cos B cos C
= 2 + +
k sin A sin B sin A sin C sin B sin C
1
= 2 {cot A cot B + cot A cot C + cot B cot C}
k
1 tan A + tan B + tan C 1
= 2 = 2 , by Art. 114(page 328),
k tan A tan B tan C k
sin2 A
= .
a2

Problem 33. a cos A + b cos B + c cos C = 2a sin B sin C.


Solution.
a cos A + b cos B + c cos C
a sin B a sin C
= a cos A + cos B + cos C
sin A sin A
a(sin 2B + sin 2C) 2a sin(B + C) cos(B − C)
= a cos A + = a cos A +
2 sin A 2 sin A
= a cos A + a cos(B − C) = −a cos(B + C) + a cos(B − C)
= 2a sin B sin C.

2a sin B sin C
Problem 34. cos A + cos B + cos C = 1 + .
a+b+c
CHAPTER XIII : Relations ≬ the Sides of a Triangle and the Trig Func of the Angles 109

Solution.
2a sin B sin C 2 sin B sin C 2 sin B sin C
= =
a+b+c b c sin B sin C
1+ + 1+ +
a a sin A sin A
2 sin A sin B sin C
=
sin A + sin B + sin C
2 sin A sin B sin C
= , by Chapter viii. Problem 16,
A B C
4 cos cos cos
2 2 2
A B C
= 4 sin sin sin
2 2 2
= cos A + cos B + cos C − 1, by Art. 114 (page 328);
2a sin B sin C
∴ cos A + cos B + cos C = 1 + .
a+b+c

Problem 35. a2 − 2ab cos(60◦ + C) = c2 − 2bc cos(60◦ + A).


a2 − 2ab cos(60◦ + C) = a2 − 2ab (cos 60◦ cos C − sin 60◦ sin C)
= a2 − ab cos C + 2ab sin 60◦ sin C
a2 + b2 − c2
= a2 − + 2cb sin 60◦ sin A
2
c2 + b2 − a2
= c2 − + 2bc sin 60◦ sin A
2
= c2 − bc cos A + 2bc sin 60◦ sin A
= c2 − 2bc cos (60◦ + A) .

A A B C
Problem 36. cot − cosec : cot + cot :: b + c − a : 2a.
4 2 2 2
b+c−a sin B + sin C − sin A
=
2a 2 sin A
B+C B−C A A
2 sin cos − 2 sin cos
= 2 2 2 2
A A
4 sin cos
2 2
B−C A B−C B+C B C
cos − sin cos − cos sin sin
= 2 2 = 2 2 = 2 2 .
A A A
2 sin 2 sin sin
2 2 2
Again
A A A
A A cos 2 cos2 −1 cos
cot − cosec = 4 − 1 = 4 = 2 ; and
4 2 A A A A
sin sin sin sin
4 2 2 2
B C B+C A
B C cos cos sin cos
cot + cot = 2 + 2 = 2 = 2 ;
2 2 B C B C B C
sin sin sin sin sin sin
2 2 2 2 2 2
A A B C
cot − cosec sin sin
∴ 4 2 = 2 2 = b + c − a.
B C A 2a
cot + cot sin
2 2 2

Problem 37.
A B C ∑( A
)(
B
)(
C
)
cos2 cos2 cos2 =4 Σ − cos Σ − cos Σ − cos ,
2 2 2 2 2 2
where
A B C
2Σ = cos + cos + cos .
2 2 2
CHAPTER XIII : Relations ≬ the Sides of a Triangle and the Trig Func of the Angles 110

Solution. ∑( A
)(
B
)(
C
)
4 Σ − cos Σ − cosΣ − cos = the product of
( 2 2 )( 2 )
1 A B C B C A
cos + cos + cos cos + cos − cos
4 ( 2 2 2 )( 2 2 2 )
A C B A B C
into cos + cos − cos cos + cos − cos .
2 2 2 2 2 2
Now substitute for the trinomial expressions results given by Chapter viii. Problems
20 and 21; thus we obtain
{ }
π−A π−B π−C π+A π+B π+C 2
8 cos cos cos cos cos cos ,
4 4 4 4 4 4
{ }2
π−A π−B π−C π−A π−B π−C
that is 8 cos cos cos sin sin sin ,
4 4 4 4 4 4
{ }2
π−A π−B π−C
that is sin sin sin ,
2 2 2
A B C
that is cos2 cos2 cos2 .
2 2 2

A B A+B
Problem 38. The perimeter of any triangle is 2c cos cos sec .
2 2 2
c sin A c sin B
Solution. The perimeter = a + b + c = + +c
sin C sin C
A B C
c(sin A + sin B + sin C) 4 cos cos cos
= = 2 2 2 , by Example viii. 16,
sin C sin C
A B A B
2 cos cos 2 cos cos
= 2 2 = 2 2
C A+B
sin cos
2 2
A B A+B
= 2 cos cos sec .
2 2 2

Problem 39. If y sin2 A + x sin2 B = z sin2 B + y sin2 C = x sin2 C + z sin2 A, then


x : y : z :: sin 2A : sin 2B : sin 2C.
Solution. Let h = y sin2 A( + x sin2 B = z ) sin2 B + y sin2 C = x sin2 C + z sin2 A.
∴ h sin C − sin A = x sin2 B sin2 C − z sin2 A sin2 B, and
2 2

h = x sin2 C + z sin2 A;
( )
∴ h sin C − sin A + h sin B = 2x sin2 B sin2 C,
2 2 2

∴ h sin(C − A) sin(C + A) + h sin2 B = 2x sin2 B sin2 C;


∴ h sin(C − A) + h sin(C + A) = 2x sin B sin2 C;
∴ x sin B sin2 C = h sin C cos A.
h cos A h sin 2A
∴x= = .
sin B sin C 2 sin A sin B sin C
Similarly
h sin 2B
y= , and
2 sin A sin B sin C
h sin 2C
z= .
2 sin A sin B sin C

A B C
Problem 40. 8 sin sin sin is less than 1, except when A = B = C.
2 2 2
A B C
Solution. Since A + B + C = π, we may show that 8 sin sin sin has its greatest
2 2 2
value when A, B and C are all(equal. ) ( )
A B A+B A−B A+B A−B
For sin sin = sin + sin sin − sin
2 2 4 4 4 4
CHAPTER XIII : Relations ≬ the Sides of a Triangle and the Trig Func of the Angles 111

A+B A−B
= sin2 − sin2 ;
4 4
A B
thus whatever may be the value of C, it follows that sin sin has its greatest value
2 2
A+B
when A = B; for sin does not change while A and B change in such a manner
4
as to leave C unchanged. In this way we see that the greatest value of the expression
π
is when all the angles are equal, and the value then is 8 sin3 , that is 1.
6

Problem 41.
a sin(B − C) cos(B + C − A) + b sin(C − A) cos(C + A − B)
+ c sin(A − B) cos(A + B − C) = 0.
Solution. Let k have the same meaning as in the solution of Example 28; then
a sin(B − C) cos(B + C − A) = k sin A sin(B − C) cos (180◦ − 2A)
= −k sin(B + C) sin(B − C) cos 2A
( )( )
= k sin2 B − sin2 C 2 sin2 A − 1
( ) ( )
= 2k sin2 A sin2 B − sin2 C − k sin2 B − sin2 C .
Similarly the other two terms of the proposed expression may be transformed; and
(
then the whole )
vanishes because ( ) ( )
sin2 A sin2 B − sin2 C + sin2 B sin2 C − sin2 A + sin2 C sin2 A − sin2 B = 0,
and sin2 B − sin2 C + sin2 C − sin2 A + sin2 A − sin2 B = 0.

Problem 42.
sin A sin B sin C sin A sin B sin C
+ + + + +
cos B cos C cos A cos C cos A cos B
= sin A + sin B + sin C + (cos A + cos B + cos C) tan A tan B tan C.
Solution.
sin A sin B sin A cos A + sin B cos B sin 2A + sin 2B
+ = =
cos B cos A cos A cos B 2 cos A cos B
2 sin(A + B) cos(A − B) sin C
= = (cos A cos B + sin A sin B)
2 cos A cos B cos A cos B
= sin C + cos C tan A tan B tan C.
Similarly
sin B sin C
+ = sin A + cos A tan A tan B tan C,
cos C cos B
sin C sin A
and + = sin B + cos B tan A tan B tan C.
cos A cos C
Hence by addition we obtain the required result.
CHAPTER XIV
Solution of triangles

Problem 1. Find the values of the angle A having given sin B = ·25 , a = 5 , b = 2 ·5 .
a 5 1
Solution. sin A = sin B = ·25 = ; therefore A = 30◦ or 150◦ .
b 2·5 2

Problem 2. One side of a triangle is half another and the included angle is 60 ◦ : find
the other angles.
1
Solution. Suppose c = b, and A = 60◦ ; then, by Art. 229 (page 338),
2
1
1−
1
tan (B − C) =
b−c
cot
A
= 2 cot 30◦ = 1 · √3 = √1 ;
2 b+c 2 1 3 3
1+
2
1 1
therefore (B − C) = 30◦; and (B + C) = 60◦ .
2 2
Hence B = 90◦ and C = 30◦ .

√ √
Problem 3. The sides of a triangle are in the ratio of 2 : 6 : 1 + 3 : determine the
angles.
Solution. Let a, b, c denote these sides in order. Then
√ √
b2 + c2 − a2 6 + (1 + 3)2 − 4 6+2 3
cos A = = √ √ = √ √
2bc 2(1 + 3) 6 2(1 + 3) 6
√ √
3(1 + 3) 1
= √ √ = √ ; therefore A = 45◦ .
(1 + 3) 6 2
√ √
a2 + c2 − b2 4 + (1 + 3)2 − 6 2+2 3 1
cos B = = √ = √ = ;
2ac 4(1 + 3) 4(1 + 3) 2
therefore B = 60◦ .
√ √ √
a2 + b2 − c2 4 + 6 − (1 + 3)2 6−2 3 3− 3
cos C = = √ = √ = √
2ab 4 6 4 6 2 6

3−1 ◦
= √ ; therefore C = 75 .
2 2

Problem 4. If A = 30 ◦ , b = 100 , a = 40 , is there any ambiguity ?


b 100 1 5
Solution. sin B = sin A = · = ; but this is impossible, for a sine cannot be
a 40 2 4
greater than unity.


Problem 5. Having given A = 18 ◦ , a = 4 , b = 4 + 80 , solve the triangle.
Solution. √
b 4 + (80) √
sin B = sin A = sin 18◦ = (1 + 5) sin 18◦
a √ √ 4
(1 + 5)( 5 − 1)
= = 1; therefore B = 90◦ .
4 { √ } 2
Thus C = 72◦ ; and c2 = b2 − a2 = 4 + (80) − 16
√ √
= 80 + 8 (80) = 16(5 + 2 5);
√ √
therefore c = 4 (5 + 2 5).
CHAPTER XIV : Solution of triangles 114

◦ √
Problem 6. Having given A = 15 √ , a = 4 , b = 4 + 48 √, solve the triangle.
b 4 + 48 ◦
√ 3−1 1
Solution. sin B = sin A = sin 15 = (1 + 3) √ = √ ;
a 4 2 2 2
therefore B = 45◦ or 135◦ .
√ √
a sin C 2 2 3
If B = 45◦ , then C = 120◦ ; and c = =4· √ ·
√ √ √
sin A 3 − 1 2
4 6 4 6( 3 + 1) √ √
= √ = = 2 6( 3 + 1).
3−1 2

a sin C 2 2 1
If B = 135◦ , then C = 30◦ ; and c = =4· √ ·

sin A
√ √ 3 − 1 2
4 2 4 2( 3 + 1) √ √
= √ = = 2 2( 3 + 1).
3−1 2

Problem 7. If a, b, A be given, and a be less than b, and if c, c ′ be the two values


found for the third side of the triangle, then
c 2 − 2cc ′ cos 2A + c ′2 = 4a 2 cos2 A.
Solution. With the first diagram of Art. 234 (page 339) we may put c = AB and
c′ = AB ′ ; thus
c = b cos A − a cos CBB ′ , and c′ = b cos A + a cos CBB ′ ;
therefore c + c′ = 2b cos A,
and cc′ = b2 cos2 A − a2 cos2 CBB ′ = b2 cos2 A − a2 cos2 B
= b2 (1 − sin2 A) − a2 (1 − sin2 B) = b2 − a2 .
Hence (c + c′ )2 = 4b2 cos2 A,
4cc′ cos2 A = 4(b2 − a2 ) cos2 A;
therefore c + 2cc′ + c′2 − 4cc′ cos2 A = 4a2 cos2 A,
2

that is c2 − 2cc′ cos 2A + c′2 = 4a2 cos2 A.

Problem 8. Find the sum of the areas of the two triangles which satisfy the conditions
of the problem in the ambiguous case. See Art. 247 (page 342).
Solution. With the notation of the preceding solution the area of the smaller triangle
c c′
is b sin A, and the area of the larger triangle is b sin A; hence the sum of the areas
2 2
1
= (c + c′ )b sin A = b2 sin A cos A.
2

Problem 9. If B1 , C1 , and B2 , C2 are the angles of the two triangles in the ambiguous
case, then
sin C1 sin C2
+ = 2 cos A.
sin B1 sin B2
Solution. With the notation of the two preceding solutions we have
sin C1 c sin C2 c′
= and = ;
sin B1 b sin B2 b
sin C1 sin C2 c + c′ 2b cos A
therefore + = = = 2 cos A.
sin B1 sin B2 b b

Problem 10. As in the solution of P roblem 8, we have


1 ′ n
c b sin A = cb sin A;
2 2
therefore c′ = nc.
And as in the solution of P roblem 7,
c′ + c 2b cos A
= ;
c′ − c 2a cos CBB ′
b n + 1 cos CBB ′
therefore = · ;
a n−1 cos A
CHAPTER XIV : Solution of triangles 115

cos CBB ′
but the angle CBB ′ is greater than A, and therefore is less than unity. Hence
cos A
b n+1
is less than .
a n−1

Problem 11. If log a + 10 = log b + L sin A, can the triangle be ambiguous ?


b
Solution. sin B = sin A; therefore L sin B − 10 = log b + L sin A − 10 − log a.
a
Thus if log a + 10 = log b + L sin A we have L sin B − 10 = 0; therefore L sin B = 10,
therefore log sin B = 0, therefore sin B = 1, therefore B = 90◦ , and the triangle is not
ambiguous.

a−b
Problem 12. If θ be an angle determined from the equation cos θ = , prove that
c
in any triangle
A−B (a + b) sin θ A+B c sin θ
cos = √ , cos = √ .
2 2 ab 2 2 ab
Solution.
1 1
a+b sin A + sin B 2 sin (A + B) cos (A − B)
= = 2 2
c sin C C C
2 sin cos
2 2
1
cos (A − B)
= 2 ;
1
sin C
2
C
1 (a + b) sin
therefore cos (A − B) = 2 ,
2 c
a−b
Now assume cos θ = ; therefore
c
c 2 − (a − b)2 a2 + b2 − 2ab cos C − (a − b)2
sin2 θ = =
c2 c2
2ab(1 − cos C) 4ab 2 C
= = 2 sin ;
c2 c 2

2 (ab) C
therefore sin θ = sin ;
c 2
1 (a + b) sin θ
therefore cos (A − B) = √ .
2 2(ab)
√ √
2 (ab) (ab)
C 2 1
And sin θ = sin = cos (A + B);
c c2 2
1 c sin θ
therefore cos (A + B) = √ .
2 2 (ab)


2 ab C
Problem 13. If tan ϕ = sin , then c = (a − b) sec ϕ.
a−b 2
Solution. ( )
C
c2 = a2 + b2 − 2ab cos C = a2 + b2 − 2ab 1 − 2 sin2
2
C
= (a − b)2 + 4ab sin2 = (a − b)2 + (a − b)2 tan2 ϕ
{ 2}
= (a − b)2 1 + tan2 ϕ = (a − b)2 sec2 ϕ.

A
Problem 14. In a triangle ABC in which a = 18 , b = 20 , c = 22 , find L tan , having
2
CHAPTER XIV : Solution of triangles 116

given log 2 = ·3010300 , log 3 = ·4771213 .


Solution. Here s = 30, s − a = 12, s − b = 10, s − c = 8.
√ √ √ √
A (s − b)(s − c) 10 × 8 8 2
tan = = = = ;
2 s(s − a) 30 × 12 36 9

A 2 1
therefore L tan = 10 + log = 10 + log 2 − log 3 = 9·6733937.
2 9 2

Problem 15. The sides of a triangle are 32 , 40 , 66 : find the greatest angle, having
given
log 207 = 2·3159703, log 1073 = 3·0305997,
L cot 66◦ 18′ = 9·6424342, diff. for 1′ = ·0003431.
Solution. The greatest angle is opposite to the side 66; denote this angle by C. Then

C s(s − c)
cot = .
2 (s − a)(s − b)
Here s = 69, s − a = 37, s − b = 29, s − c = 3;
√ √
C 69 × 3 207
therefore cot = = ;
2 37 × 29 1073

C 207
therefore L cot = 10 + log
2 1073
1
= 10 + (log 207 − log 1073) = 9·6426853.
2
9·6426853
9·6424342 ·0003431 : ·0002511 :: 60′′ : x′′ ;
·0002511
C
this gives x = 44; therefore = 66◦ 18′ − 44′′ = 66◦ 17′ 16′′ ;
2
therefore C = 132◦ 34′ 32′′ .

Problem 16. The sides of a triangle are 4 , 5 , 6 : find B, having given


log 2 = ·3010300 , L cos 27 ◦ 53 ′ = 9 ·9464040 , diff. for 1 ′ = ·0000669 .
15 7 5 3
Solution. Here s = , s−a= , s−b= , s−c= .
2 √ 2 √ 2 √ 2√
B s(s − b) 15 × 5 25 100
cos = = = = ;
2 ac 8 × 12 32 27

B 100 1
therefore L cos = 10 + log = 10 + (log 100 − log 27 )
2 27 2
7
= 10 + 1 − log 2 = 9·9463950.
2
9·9464040
9·9463950 ·0000669 : ·0000090 :: 60′′ : x′′ ;
·0000090
B
this gives x = 8; therefore = 27◦ 53′ 8′′ ; therefore B = 55◦ 46′ 16′′ .
2


A s(s − a)
Problem 17. Apply the formula cos = to find the greatest angle in a
2 bc
triangle whose sides are 5 , 6 , 7 feet respectively, having given
log 6 = ·7781513,
L cos 39◦ 14′ = 9·8890644, diff. for 60′′ = ·0001032.
CHAPTER XIV : Solution of triangles 117

Solution. Here a = 7, s = 9, s − a = 2; therefore


√ √ √
A 9×2 3 6
cos = = = ;
2 5×6 5 10

A 6 1
therefore L cos = 10 + log = 10 + (log 6 − log 10)
2 10 2
1 1
= 10 + log 6 − = 9·8890756.
2 2
9·8890756
9·8890644 ·0001032 : ·0000112 :: 60′′ : x′′ ;
·0000112
A
this gives x = 6.5; therefore = 39◦ 14′ − 6′′ ·5 = 39◦ 13′ 53′′ ·5; therefore A =
2
78◦ 27′ 47′′ .

Problem 18. Two sides of a triangle are 18 and 2 feet respectively, and the included
angle is 55 ◦ : find the other angles, having given
log 2 = ·3010300, L cot 27◦ 30′ = 10·2835233,
L tan 56◦ 56′ = 10·1863769, diff. for 1′ = ·0002763.
Solution. As in Art. 229 (page 338) we have
1 18 − 2 A 8
tan (B − C) = cot = cot 27◦ 30′ ;
2 18 + 2 2 10
1
therefore L tan (B − C) = L cot 27◦ 30′ + log 8 − log 10
2
= L cot 27◦ 30′ + 3 log 2 − 1 = 10·1866133.
10·1866133
10·1863769 ·0002763 : ·0002364 :: 60′′ : x′′ ;
·0002364
1
this gives x = 51; therefore (B − C) = 56◦ 56′ 51′′ .
2
1
And (B + C) = 62◦ 30′ ; therefore B = 119◦ 26′ 51′′ , C = 5◦ 33′ 9′′ .
2

Problem 19. Two sides of a triangle are in the ratio of 9 to 7 , and the included angle
is 64 ◦ 12 ′ : find the other angles, having given
log 2 = ·3010300, L tan 57◦ 54′ = 10·2025255,
L tan 11◦ 16′ = 9·2993216, L tan 11◦ 17′ = 9·2999804.
Solution.
1 9−7 A 1
tan (B − C) = cot = cot 32◦ 6′′ ;
2 9+7 2 8
1 ◦ ′
therefore L tan (B − C) = L cot 32 6 − log 8
2
= L tan 57◦ 54′ − 3 log 2 = 9.2994355.

9·2999804 9·2994355
9·2993216 9·2993216 ·0006588 : ·0001139 :: 60′′ : x′′ ;
·0006588 ·0001139
1
this gives x = 10; therefore (B − C) = 11◦ 16′ 10′′ .
2
1
And (B + C) = 57◦ 54′ ; therefore B = 69◦ 10′ 10′′ , C = 46◦ 37′ 50′′ .
2

Problem 20. If a = 70 , b = 35 , C = 36 ◦ 52 ′ 12 ′′ , find the other angles, having given


log 3 = ·4771213 , L cot 18 ◦ 26 ′ 6 ′′ = 10 ·4771213 .
CHAPTER XIV : Solution of triangles 118

1 a−b C 70 − 35 C 1
Solution. tan (A − B) = cot = cot = cot 18◦ 26′ 6′′ ;
2 a+b 2 70 + 35 2 3
1 ◦ ′ ′′
therefore L tan (A − B) = L cot 18 26 6 − log 3 = 10;
2
1
therefore log tan (A − B) = 0;
2
1 1
therefore tan (A − B) = 1; therefore (A − B) = 45◦ .
2 2
1
And (A + B) = 71◦ 33′ 54′′ ; therefore A = 116◦ 33′ 54′′ , B = 26◦ 33′ 54′′ .
2

Problem 21. The ratio of two sides of a triangle is 9 to 7 , and the included angle is
47 ◦ 25 ′ : find the other angles, having given
log 2 = ·3010300, L tan 66◦ 17′ 30′′ = 10·3573942,
L tan 15◦ 53′ = 9·4541479, diff. for 1′ = ·0004797.
1 9−7 A 1
Solution. tan (B − C) = cot = cot 23◦ 42′ 30′′ ;
2 9+7 2 8
1
therefore L tan (B − C) = L cot 23◦ 42′ 30′′ − log 8
2
= L tan 66◦ 17′ 30′′ − 3 log 2 = 9·4543042.
9·4543042
9·4541479 ·0004797 : ·0001563 :: 60′′ : x′′ ;
·0001563
1
this gives x = 20′′ ; therefore (B − C) = 15◦ 53′ 20′′ .
2
1
And (B + C) = 66◦ 17′ 30′′ ; therefore B = 82◦ 10′ 50′′ , C = 50◦ 24′ 10′′ .
2

Problem 22. In a triangle a = 30 , b = 20 , and the contained angle = 22 ◦ : find the


other angles, having given
L cot 11◦ = 10·7113477, L tan 45◦ 48′ = 10.0121294,
L tan 45◦ 49′ = 10·0123821, log 2 = ·3010300.
1 a−b C 30 − 20 C 2
Solution. tan (A − B) = cot = cot = cot 11◦ ;
2 a+b 2 30 + 20 2 10
1
therefore L tan (A − B) = L cot 11◦ + log 2 − log 10
2
= L cot 11◦ + log 2 − 1 = 10·0123777.
10·0123821 10·0123777
10·0121294 10·0121294 ·0002527 : ·0002483 :: 60′′ : x′′ ;
·0002527 ·0002483
1
this gives x = 59; therefore (A − B) = 45◦ 48′ 59′′ .
2
1
And (A + B) = 79◦ ; therefore A = 124◦ 48′ 59′′ , B = 33◦ 11′ 1′′ .
2

Problem 23. Given b = 14 , c = 11 , A = 60 ◦ , show that B = 71 ◦ 44 ′ 29 ′′ , having given


L tan 11◦ 44′ 29′′ = 9.31774, log 2 = ·30103, log 3 = ·47712.
Solution. √
1 b−c A 3 3 3
tan (B − C) = cot = cot 30◦ = ;
2 b+c √2 25 25
1 3 3 3
therefore L tan (B − C) = 10 + log = 10 + log 3 − log 25
2 25 2
3 100 3
= 10 + log 3 − log = 10 + log 3 − 2 + 2 log 2 = 9·31774;
2 4 2
1
therefore (B − C) = 11◦ 44′ 29′′ .
2
1
And (B + C) = 60◦ ; therefore B = 71◦ 44′ 29′′ .
2
CHAPTER XIV : Solution of triangles 119

Problem 24. The sides of a triangle are 7 , 8 , 9 : determine all the angles, having
given
log 2 = ·3010300,
L tan 24◦ 5′ 40′′ = 9·6505069, L tan 24◦ 5′ 50′′ = 9·6505634,
L tan 29◦ 12′ 20′′ = 9·7474183, L tan 29◦ 12′ 30′′ = 9·7474677.
Solution. Let a = 7, b = 8, c = 9; then s = 12, s − a = 5, s − b = 4, s − c = 3.
√ √ √ √
A (s − b)(s − c) 4×3 1 2
tan = = = = .
2 s(s − a) 12 × 5 5 10

A 2 1
L tan = 10 + log = 10 + (log 2 − log 10)
2 10 2
1
= 10 + (log 2 − 1) = 9·6505150.
2
9·6505634 9·6505150
9·6505069 9·6505069 ·0000565 : ·0000081 :: 10′′ : x′′ ;
·0000565 ·0000081
A
this gives x = 1.5; therefore = 24◦ 5′ 41′′ ·5; therefore A = 48◦ 11′ 23′′ .
√ 2 √ √ √
B (s − a)(s − c) 5×3 5 10
tan = = = = .
2 s(s − b) 12 × 4 16 32

B 10 1
L tan = 10 + log = 10 + (log 10 − log 32)
2 32 2
1 5
= 10 + − log 2 = 9·7474250.
2 2
9·7474677 9·7474250
9·7474183 9·7474183 ·0000494 : ·0000067 :: 10′′ : x′′ ;
·0000494 ·0000067
B
this gives x = 1·5; therefore = 29◦ 12′ 21′′ ·5; therefore B = 58◦ 24′ 43′′ .
2
Hence C = 180◦ − 48◦ 11′ 23′′ − 58◦ 24′ 43′′ = 73◦ 23′ 54′′ .

Problem 25. In a right-angled triangle the hypotenuse c = 6953 and b = 3 : find B,


having given
log 3·475 = ·5409548, log 6·953 = ·8421722,
L sin 44◦ 59′ 15′′ = 9·8493902, diff. for 1′′ = ·0000021.
Solution. As in Art. 238 (page 340) we have
( ) √ √ ( )
B ◦ 1 − sin B 1 3
sin 45 − = = 1−
2 2 2 6953
√ √
1 6950 3475
= × = ;
2 6953 6953
( ) √
B 3475
therefore L sin 45◦ − = 10 + log
2 6953
1
= 10 + log(3475 − log 6953)
2
1
= 10 − (·3012174) = 9·8493913.
2
9·8493913
9·8493902 ·0000021 : ·0000011 :: 1′′ : x′′ ;
·0000011
B B
this gives x = ·5; therefore 45◦ − = 44◦ 59′ 15′′ ·5; therefore = 44′′ ·5; therefore
2 2
B = 1′ 29′′ .
CHAPTER XIV : Solution of triangles 120

Problem 26. Two sides are 80 and 100 feet, and the included angle 60 ◦ : find the
other angles, having given
log 3 = ·47712, L tan 10◦ 53′ 36′′ = 9·28432.
Solution.
Let b = 100, c = 80;

1 b−c A 1 3 3
tan (B − C) = cot = cot 30◦ = = 3− 2 ;
2 b+c 2 9 9
1 3 3
therefore L tan (B − C) = 10 + log 3− 2 = 10 − log 3 = 9·28432;
2 2
1
therefore (B − C) = 10◦ 53′ 36′′ .
2
1
And (B + C) = 60◦ ; therefore B = 70◦ 53′ 36′′ , C = 49◦ 6′ 24′′ .
2

Problem 27. Two sides of a triangle are 3 and 5 feet, and the included angle is 120 ◦ :
find the other angles, having given
log 4.8 = ·6812412,
L tan 8◦ 12′ = 9·1586706, diff. for 60′′ = ·0008940.
Solution.
Let b = 5, c = 3;
1 b−c A 1 1 1
tan (B − C) = cot = cot 60◦ = √ = √ ;
2 b+c 2 4 4 3 (48)
1 1 1
therefore L tan (B − C) = 10 + log √ = 10 − log 48
2 48 2
1
= 10 − (1.6812412) = 9·1593794.
2
9·1593794
9·1586706 ·0008940 : ·0007088 :: 60′′ : x′′ ;
·0007088
1
this gives x = 48; therefore (B − C) = 8◦ 12′ 48′′ .
2
1
And (B + C) = 30◦ ; therefore B = 38◦ 12′ 48′′ and C = 21◦ 47′ 12′′ .
2

Problem 28. A side of a base of a square pyramid is 200 feet and each edge is 150
feet: find the slope of each face, having given
log 2 = ·30103 , L tan 26 ◦ 33 ′ = 9 ·69868 , L tan 26 ◦ 34 ′ = 9 ·69900 .
Solution. Let ABCD denote the square base, P the vertex. From P suppose a per-
pendicular P Q drawn to the ground, and from Q draw QR perpendicular to AB. Let
PQ
ϕ denote the required inclination; then tan ϕ = .
QR
Now QR = 100. Also P Q2 + QR2 = P R2 , and P R2 + AR2 = AP 2 ; thus
P Q2 = P R2 − QR2 = AP 2 − AR2 − QR2 = (150)2 − (100)2 − (100)2 = 2500;
50 1
therefore P Q = 50. Therefore tan ϕ = = .
100 2
1
Hence L tan ϕ = 10 + log = 10 − log 2 = 9·69897.
2
9·69900 9·69897
9·69868 9·69868 ·00032 : ·00029 :: 60′′ : x′′ ;
·00032 ·00029
this gives x = 54; therefore ϕ = 26◦ 33′ 54′′ .

Problem 29. Find the other angles, having given


a
= 1·2̇, C = 60◦ , log 3 = ·4771213,
b
L cot 9◦ 49′ = 10·7618797, diff. for 1′ = ·0007514.
CHAPTER XIV : Solution of triangles 121


1 a−b C 12 − 1 1 3
Solution. tan (A − B) = cot = 92 cot 30◦ = cot 30◦ = ;
2 a+b 2 19 + 1 10 10

1 3 1
therefore L tan (A − B) = 10 + log = 10 + log 3 − 1 = 9·2385606.
2 10 2
Now L cot 9◦ 49′ = 10·7618797; and as tan θ × cot θ = 1, we have
log tan θ + log cot θ = 0;
therefore L tan θ − 10 + L cot θ − 10 = 0;
therefore L tan θ = 20 − L cot θ.
Thus L tan 9◦ 49′ = 9·2381203.

9·2385606
9·2381203 ·0007514 : ·0004403 :: 60′′ : x′′ ;
·0004403
1
this gives x = 35; therefore (A − B) = 9◦ 49′ 35′′ .
2
1
And (A + B) = 60◦ ; therefore A = 69◦ 49′ 35′′ , B = 50◦ 10′ 25′′ .
2

Problem 30. If a = 2 , c = 3 , L sin A = 9 ·5228787 , find C ; log 3 being ·4771213 .


Solution.
c
sin C = sin A; L sin C = L sin A + log c − log a
a
= L sin A + log 3 − log 2
= 9·5228787 + ·4771213 − log 2 = 10 − log 2;
1
therefore log sin C = − log 2 = log ;
2
1
therefore sin C = ; therefore C = 30◦ or 150◦.
2

Problem 31. Show how to solve a triangle having given the base, the height, and
the difference of the angles at the base; these angles being supposed both acute.
Solution. Let c be the given base and let h denote the given height. With the left-hand
diagram of Art. 214 (page 336) we have
BD CD
cot B = and cot C = ;
h h
BD + CD c
therefore cot B + cot C = = (18)
h h
Also B − C is supposed given, so that cot(B − C) is known; call it m : thus
cot B − cot C
=m (19)
1 + cot B cot C
From (18) and (19) we can find cot A and cot B.

Problem 32. Show how to solve a triangle having given the three perpendiculars
from the angles on the opposite sides.
Solution. Let a, b, c denote the sides; and l, m, n the perpendiculars on them re-
spectively from the opposite angles. Then al = bm = cn; for each of these expressions
denotes twice the area of the triangles. Hence the sides a, b, c are respectively in-
versely proportional to l, m, n. Thus the ratios of the sides are known; and hence
the angles of the triangle can be calculated by Art. 217 (page 337). Then the actual
lengths of the sides can be found; for l = c sin B, and l and B are known, so that c
can be found; and then a and b can be deducted as the ratios of the sides are already
known.
CHAPTER XV
Measurement of Heights and Distances

Problem 1. A person standing on the bank of a river observes the angle subtended
by a tree on the opposite bank to be 60 ◦ , and when he retires 40 feet from the river’s
bank he finds the angle to be 30 ◦ : determine the height of the tree and the breadth
of the river.
Solution. Take the diagram of Art. 240 (page 340). The angle P BC = 60◦ , the angle
P AC = 30◦ ; therefore the angle AP B = 30◦ . Also AB = 40 feet.
Since the angle P AB = the angle AP B, we have BP = AB = 40. Then

3 √
P C = BP sin 60◦ = 40 = 20 3;
2
1
and BC = BP cos 60◦ = 40 · = 20.
2

Problem 2. From a station B at the base of a mountain its summit A is seen at an


elevation of 60 ◦ ; after walking one mile towards the summit up a plane making an
angle of 30 ◦ with the horizon to another station C , the angle BCA is observed to be
135 ◦ . Find the height of the mountain in yards.
Solution. Let AC produced through C meet the horizontal plane which contains B
at D. Then the angle ABD = 60◦ , and the angle CBD = 30◦ ; therefore the angle
ABC = 30◦ . The angle ACB = 135◦ . Hence
the angle BAC = 180◦ − 30◦ − 135◦ = 15◦ .

AB sin ACB sin 135◦ 1 3−1 2
= = = √ ÷ √ = √ ;
BC sin BAC sin 15◦ 2 2 2 3−1
2 × 1760
therefore AB = √ yards.
3−1

3
The height of the mountain = AB sin 60◦ = AB
√ 2
√ √
1760 3 1760 3( 3 + 1)
= √ = √ √
3−1 ( 3 − 1)( 3 + 1)
√ √
1760 3( 3 + 1) √
= = 880(3 + 3).
2

Problem 3. The altitude of a tower is observed to be 30 ◦ at the end of a horizontal


base of 100 yards measured from its foot. Find the height of the tower.
Solution. Let h denote the height of the tower in yards; then
h 1 100
= tan 30◦ = √ ; therefore h = √ .
100 3 3

Problem 4. The angular elevation of a tower at a place A due south of it is 30 ◦ ; and


at a place B, due west of A, and at the distance a from it, the elevation is 18 ◦ : show
that the height of the tower is
a
√ √ .
2 +2 5
Solution. Let h denote the height of the tower, x the distance of the foot from A, and
y the distance of the foot from B. Then
x = h cot 30◦ , and y = h cot 18◦ .
( )
But y 2 − x2 = a2 ; therefore h2 cot2 18◦ − cot2 30◦ = a2 ;
{ √ }
10 + 2 5
therefore h2 √ − 3 = a2 ;
( 5 − 1)2
CHAPTER XV : Measurement of Heights and Distances 124

{ √ }
5
5+
therefore h2 √ − 3 = a2 ;
3− 5
√ √
therefore 4h ( 5 − 1) = a2 (3 − 5);
2

√ √ √
3− 5 2 (3 − 5)(3 + 5)
therefore h2 = √ a = √ √
4( 5 − 1) 4( 5 − 1)(3 + 5)
4a2 a2
= √ = √ .
4(2 + 2 5) 2+2 5

Problem 5. A spherical balloon whose radius is r feet subtends at an observer’s eye


an angle α, when the angular elevation of its centre is β : determine the height of the
centre of the balloon.
Solution. Let A denote the eye of the spectator, and B the centre of the balloon. The
angle α is formed by straight lines drawn from A in the vertical plane which contains
B, so as to touch the balloon. Hence
r α α
= sin ; therefore AB = r cosec .
AB 2 2
α
And the height of the centre of the balloon = AB sin β = r sin β cosec .
2

Problem 6. A person wishing to ascertain the distances between three inaccessible


objects A, B, C , places himself in a straight line with A and B; he then measures the
distances along which he must walk in a direction at right angles to AB until A, C and
B, C respectively are in a straight line with him, and also observes in those positions
their angular bearings : show how he can find the distances between A, B, C .
Solution. Let O denote the station which is in the same straight line as A and B; let
P be the station which is in the same straight line as A and C; and let Q be the station
which is in the same straight line as B and C. Then O, P and Q are in a straight line
which is at right angles to AB. Let OP = p, OQ = q; let AP O = α, and BQO = β.
Then OA = p tan α, and OB = q tan β. Thus AB = q tan β − p tan α. And the angles of the
π π
triangle ABC are known; for ABQ = − β, and OAP = − α. Hence AC and BC can
2 2
be found.

Problem 7. Two posts AB and CD are placed at the edge of a river at a distance
AC = AB, the height of CD being such that AB and CD subtend equal angles at E, a
point on the other bank exactly opposite to A : show that the square of the breadth of
AB 4
the river is equal to , and that AD and BC subtend equal angles at E.
CD − AB 2
2
AB
Solution. The tangent of the angle which AB subtends E is ; and the tangent of
AE
CD AB CD
the angle which CD subtends at E is ; therefore = ;
CE AE CE
AE · CD AE 2 · CD 2
therefore CE = ; therefore CE 2 = ;
AB AB 2
AE 2 · CD 2
therefore CA2 + AE 2 = :
AB 2
AB 4
but CA2 = AB 2 ; therefore AE 2 = .
CD 2 − AB 2
EA EB + EC − BC 2
2 2
Again cos DEA = ; and cos BEC =
ED 2EB · EC
EA2 + AB 2 + EA2 + AC 2 − (AB 2 + AC 2 ) EA2
= = .
2EB · EC EB · EC
EA EC
But by hypothesis the cosine of BEA is equal to the cosine of DEC, that is = ;
EB ED
EA2 EA
therefore EA · ED = EB · EC; therefore = .
EB · EC ED
CHAPTER XV : Measurement of Heights and Distances 125

Problem 8. A flag-staff a feet high stands on the top of a tower b feet high. Find at
what point on a horizontal plane passing through the base of the tower an observer
must place himself so that the tower and the flag-staff may subtend equal angles, the
height of the eye being h.
Solution. Let A be the top of the flag staff, B the top of the tower, C the foot of the
tower, E the eye. From E draw a perpendicular ED on the horizontal plane which
contains C. Then the angle BEC is to equal to the angle BEA.
sin BEC BC sin BEA AB
Now = , and = ;
sin EBC EC sin EBA AE
BC AB
therefore = .
EC AE

This coincides with Euclid vi. 3. √
Let CD = x; then EC = (h2 + x2 ), EA = (a + b − h)2 + x2 ;
b a
therefore √ = √ ;
h2 + x2 (a + b − h)2 + x2
therefore {(a + b − h)2 + x2 }b2 = (h2 + x2 )a2 ;
b2 (a + b − h)2 − h2 a2
therefore x2 = ;
a2 − b2
h2 (a2 − b2 ) + b2 (a + b − h)2 − h2 a2
therefore EC 2 =
a2 − b2
b2 {(a + b − h)2 − h2 } b2 (a + b)(a + b − 2h) b2 (a + b − 2h)
= = = ;
a2 − b2 ( a2 − b2
) a−b

a + b − 2h
therefore EC = b .
a−b

Problem 9. A tower situated on a horizontal plane leans towards the North; at two
points due South and distant a, b, respectively from the base, the angular altitudes
of the tower are α and β. Show that if θ be the inclination of the tower, and h the
perpendicular height,
b−a b−a
tan θ = , h= .
b cot α − a cot β cot β − cot α
Solution. Let P denote the top of the tower; from P draw P Q perpendicular to the
ground; then P Q = h. Let x denote the distance of Q from the base of the tower; x + a
is the distance of Q from one point of observation, and x + b is the distance of Q from
the other point of observation.
x x+a x+b
Thus cot θ = , cot α = , cot β = ;
h h h
therefore h cot α = x + a, h cot β = x + b;
b−a
therefore h= ;
cot β − cot α
(b − a) cot α b cot α − a cot β
and x = h cot α − a = −a= .
cot β − cot α cot β − cot α
h b−a
Thus tan θ = = .
x b cot α − a cot β

Problem 10. An object a feet high placed on the top of a tower subtends an angle γ
at a place whose horizontal distance from the foot of the tower is b feet : determine
the height of the tower.
Solution. Let x denote the required height; and suppose θ the angle which the tower
subtends: then
x = b tan θ, x + a = b tan(θ + γ);
b(tan θ + tan γ) x + b tan γ
therefore x+a= =
1 − tan θ tan γ 1−
x tan γ
b
thus we have a quadratic equation for finding x.
CHAPTER XV : Measurement of Heights and Distances 126

Problem 11. On the bank of a river there is a column 200 feet high supporting a
statue 30 feet high; the statue to an observer on the opposite bank subtends an equal
angle with a man 6 feet high standing at the base of the column : required the breadth
of the river.
Solution. Let x denote the breadth of the river in feet; let α denote the angle sub-
tended by the column, and β the angle subtended by the column and statue.
200 230
Thus tan α = , and tan β = ;
x x
230 200
− 30x
therefore tan(β − α) = x x = 2 .
200 × 230 x + 46000
1+ 2
x
6
But, by hypotheses, tan(β − α) = ; therefore
x
6 30x
= 2 ; therefore x2 + 46000 = 5x2 ;
x x + 46000 √
therefore x2 = 11500; therefore x = 10 115.

Problem 12. The height of a house subtends a right angle at an opposite window, the
top being 60 ◦ above a horizontal straight line : find the height of the house, taking
the breadth of the street to be 30 feet.
Solution. The part of the house above the horizontal straight line subtends an angle
of 60◦ , and thus the height of the top of the house above the window is 30 tan 60◦ feet.
The part of the house below the horizontal straight line subtends an angle of 30◦ , and
thus the depth of the foot of the house below the window is 30 tan 30◦ feet. Hence the
distance from the foot of the house to the(top of the)house in feet
√ 1 4 √
= 30(tan 60◦ + tan 30◦ ) = 30 3+ √ = √ 30 = 40 3.
3 3

Problem 13. Two chimneys are of equal height. A person standing between them in
the straight line joining their bases observes the elevation of the nearer one to him to
be 60 ◦ . After walking 80 feet in a direction at right angles to the straight line joining
their bases he observes the elevations of the two to be respectively 45 ◦ and 30 ◦ . Find
their height and the distance between them.
Solution. Let x denote the height of each chimney in feet, and y the distance be-
tween them. The distance of the first point of observation from the nearer chimney
is x cot 60◦ , and therefore the distance of the second point of observation is

(80)2 + x2 cot2 60◦ .
Thus
x
√ = tan 45◦ = 1;
(80)2 + x2 cot2 60◦
x2
∴ x2 = (80)2 + x2 cot2 60◦ = (80)2 + ; therefore 2x2 = 3(80)2 ;
3 √
∴ x2 = 6(40)2 ; therefore x = 40 6.
The distance of the first point of observation from the further chimney is y −

√cot 60 , and therefore the distance of the second point of observation is
x
(80) + (y − x cot 60◦ )2 . Thus
2

x 1
√ = tan 30◦ = √ ;
2 ◦
(80) + (y − x cot 60 ) 2 3
∴ 3x2 = (80)2 + (y − x cot 60◦ )2 ; therefore 14(40)2 = (y − x cot 60◦ )2 ;
√ √ √
therefore y = x cot 60◦ + 40 14 = 40( 2 + 14).

Problem 14. An object is observed at three points A, B, C lying in a horizontal


straight line which passes directly underneath the object ; the angular elevation at B
is twice that at A, and at C is three times that at A; AB = a, BC = b : show that the
CHAPTER XV : Measurement of Heights and Distances 127

height of the object is


a √
{(a + b)(3b − a)} .
2b
If the tangent of the angle of elevation at A be 13 , show that 5a = 13b.
Solution. Let P be the object, P Q the perpendicular from P on the horizontal plane
which contains A, B, and C.
Let P Q = x, CQ = y. Suppose θ the angle P AQ, then P BQ = 2θ, and P CQ = 3θ.
Thus
x x x
tan θ = , tan 2θ = , tan 3θ = ;
y+a+b y+b y
therefore y + a + b = x cot θ, y + b = x cot 2θ, y = x cot 3θ;
therefore a = x(cot θ − cot 2θ), b = x(cot 2θ − cot 3θ);
( )
cos θ cos 2θ x sin(2θ − θ) x
therefore a=x − = = ,
( sin θ sin 2θ ) sin θ sin 2θ sin 2θ
cos 2θ cos 3θ x sin(3θ − 2θ) x sin θ
and b=x − = =
sin 2θ sin 3θ sin 2θ sin 3θ sin 2θ sin 3θ
x
= .
sin 2θ(3 − 4 sin2 θ)
x x a
Thus sin 2θ = , and 3 − 4 sin2 θ = = ;
a b sin 2θ ( b )
a 1 a
therefore 3 − 2(1 − cos 2θ) = ; therefore cos 2θ = −1 .
b 2 b
( )2
x2 1 a
Hence + −1 = 1;
a2 4 b
( )2
x 2 1 a 4b − (a − b)2
2
therefore 2
=1− −1 =
a 4 b 4b2
3b2 + 2ab − a2 (3b − a)(a + b)
= = ;
a√
4b 2 4b2
therefore x= (a + b)(3b − a).
2b
2
1 2 tan θ 3 3 x
If tan θ = , then sin 2θ = = = , and sin 2θ = ;
3 1 + tan2 θ 1 5 a
1+
√ 9
3 (a + b)(3b − a)
thus = ;
5 2b
therefore 36b2 = 25(a + b)(3b − a) = 25(3b2 + 2ab − a2 );
therefore 39b2 + 50ab − 25a2 = 0;
therefore (13b − 5a)(3b + 5a) = 0; therefore 13b − 5a = 0.

Problem 15. A vertical tower whose base is in the same horizontal plane with the
observer, is observed from a station A to bear directly North and to subtend an angle
of 15 ◦ ; the observer then walks 100 yards so that the tower always subtends the same
angle, and then it bears North-East : find its height and distance from A.
Solution. Let x denote the height of the tower in yards; then the distance from A to
the foot of the tower is x cot 15◦ . The observer moves so that the tower always subtends
the same angle, hence he must describe the arc of a circle having its centre at the
foot of the tower; and as the bearing of the tower changes from north to north-east
he must describe one-eighth part of the circumference; therefore
2πx cot 15◦ 400 tan 15◦
= 100; therefore x = .
8 π

Problem 16. A person walking along a straight road observes that the greatest
angle which two objects subtend is α; from the spot where this is the case he walks
a distance c, and the objects now appear as one, their direction making an angle β
CHAPTER XV : Measurement of Heights and Distances 128

with the road. Show that the distance between the objects is
2c sin α sin β
.
cos α + cos β
Solution. Let A denote the object which is further from the road, B that which is
nearer to the road, C the point where AB subtends the greatest angle, D the second
point of observation.
It is known that the point C is such that a circle described round A, B, and C will
touch CD at C. Therefore the angle BCD is equal to the angle BAC; denote it by θ.
Then the angle ABC = θ + β, and also = π − θ − α; therefore 2θ = π − α − β.
BC sin β c sin β AB sin α
Now = , therefore BC = , and = ;
CD sin(θ + β) sin(θ + β) BC sin θ
c sin α sin β 2c sin α sin β
therefore AB = =
sin θ sin(θ + β) cos β − cos(2θ + β)
2c sin α sin β 2c sin α sin β
= = .
cos β − cos(π − α) cos β + cos α

Problem 17. A fortress was observed by a ship at sea to bear East-north-east, and
after sailing 4 miles to the East it was observed to bear North-north-east : show that
the
√( distance of the√ship from the fortress at the first and second observation was
√ ) ( √ )
16 + 8 2 and 16 − 8 2 miles respectively.
Solution. Let A denote the fortress, B the first position of the ship, C the second;

produce BC through C to any point E. Then the angle ABC = 22 21 , and the angle

ACE = 67 12 ; therefore the angle BAC = 45◦ .
( ◦ )
AB sin ACB sin 180◦ − 67 12
= =
BC sin BAC √ 45◦ √ √
√ √ √
(2 + 2) 1 (2 + 2) 2+ 2
= ÷√ = √ = ;
√ 2 √ 2 2 2
2+ 2
√ √
therefore AB = 4 = 16 + 8 2.
2
And √
◦ √
AC sin ABC sin 22 12 (2 − 2) 1
= = = ÷√
BC sin BAC sin 45◦ 2
√ √
2
2− 2
= ;
2
√ √
2− 2 √ √
therefore AC = 4 = (16 − 8 2). See Chapter viii : P roblem 18.
2

Problem 18. A ship sailing towards the North observes two light-houses in a line
due West; and after an hour’s sailing the bearings of the light-houses are observed to
be South-west and South-south-west. The distance between the light-houses being 8
miles, find the rate at which the ship is sailing.
Solution. Let P be the first position of the ship, A the nearer lighthouse, and B the
further lighthouse; let Q be the second position of the ship. Then the angle BQP =
◦ ◦
45◦ , and the angle AQP = 22 12 ; therefore the angle QAP = 67 12 .
( ◦ )
◦ ◦
BQ sin BAQ sin 180◦ − 67 21 sin 67 12 cos 22 12 1◦
= = ◦ = ◦ = ◦ = cot 22
BA sin BQA sin 22 12 sin 22 12 sin 22 12 2
√ √
= 2 + 1, by Chapter viii : P roblem 18; therefore BQ = 8( 2 + 1).

8( 2 + 1) √
And P Q = BQ sin 45◦ = √ = 8 + 4 2.
2
CHAPTER XV : Measurement of Heights and Distances 129

Problem 19. From the top of the mast of a ship 64 feet above the level of the sea the
light of a distant lighthouse is just seen in the horizon; and after the ship has sailed
directly towards the light for 30 minutes it is seen from the deck of the ship, which
is 16 feet above the sea. Find the rate at which the ship is sailing, considering the
earth as a sphere of 4000 miles radius.
Solution. Let A denote the top of the lighthouse, P the top of the mast at the first
observation, C the centre of the earth. Draw a straight line from P to A and let it
touch the earth at B.
√ √ of the earth in√
Let r denote the radius feet; then

P B = P C 2 − BC 2 = (r + 64)2 − r2 = 2r × 64 + (64)2 = 2r × 64 very nearly, for
r is very large compared with (64) . 2

In precisely
√the same manner if Q denote the deck of the ship at the second obser-
vation, QB = 2r × 16.
Now, since P CB is a very small angle, we may, by the principle that tan θ is nearly
equal to θ when θ is very small, consider the straight line P B to be equal to the
arc which measures the distance of the ship from B at the first observation; and
similarly we may consider QB to be equal to the arc which measures the distance of
the ship from B at the √ second observation.
√ Thus between
√ the two observations the
√ over 2r × 64 − 2r × 16, that is, 4√ 2r; that is, in half-an-hour it has
ship has sailed
sailed
√ over 4 8000 × 5280 feet, so that the
√ rate is 8 8000 × 5280 feet per √hour, that is,
8 8000 × 5280 800 50
miles per hour, that is 8 miles per hour, that is, 8 miles per
5280 √ 528 33
3
hour; this is very nearly 8 miles per hour.
2

Problem 20. A man ascends a mountain by a path which is the shortest distance
between the base and the vertex. The inclination of the path to the horizon at first is
α, but afterwards suddenly increases to β, and then continues the same. On reaching
the vertex he finds by the barometer he has ascended n feet in altitude, and observes
the angle of depression γ of the point from which he started. Show that the distance
he traveled in the ascent is ( )
α+β
n cos −γ
2
.
β−α
cos sin γ
2
Solution. Let A denote the summit of the mountain, B the base, BC the first part of
the path, CA the second part. From A draw AE perpendicular to the horizontal plane
which contains B; then AE = n.
The following are the angles:
π π
BAE = − γ, CBE = α, CAE = − β;
2 2
therefore BAC = β − γ, ABC = γ − α, ACB = π + α − β.

AE n
AB = = ,
sin γ sin γ
BC sin BAC sin(β − γ)
= = ,
AB sin ACB sin(β − α)
AC sin ABC sin(γ − α)
= = ;
AB sin ACB sin(β − α)
BC + AC sin(β − γ) + sin(γ − α)
∴ =
AB sin(β − α)
( ) ( )
β−α α+β α+β
2 sin cos −γ cos −γ
2 2 2
= = ;
sin(β − α) β−α
cos
2
CHAPTER XV : Measurement of Heights and Distances 130

( )
α+β
cos −γ
n 2
therefore BC + AC = · .
sin γ β−α
cos
2

Problem 21. If from two points in a horizontal plane an object be seen at angles of
elevation α, β, and if from a third point between the two points and in the straight
line joining them and at distances a, b from them respectively the object be seen at
an angle of elevation γ, show that the height of the object above the horizontal plane
is
1
sin α sin β sin γ {ab(a + b)} 2
{ ( ) ( )} 1 .
a sin2 α sin2 γ − sin2 β + b sin2 β sin2 γ − sin2 α 2
Solution. Let O denote the foot of the object; and let A, B, and C denote the three
points of observation. Let x denote the height of the object; then OA = x cot α, OB =
x cot β, and OC = x cot γ.
From the triangle AOC we have
x2 cot2 α = x2 cot2 γ + a2 − 2ax cot γ cos ACO,
and from the triangle BOC we have
x2 cot2 β = x2 cot2 γ + b2 − 2bx cot γ cos BCO.
Multiply the first equation
( by b and the ) second by a, and add; thus
x2 b cot2 α + a cot2 β = ab(a + b) + x2 (a + b) cot2 γ;

∴ x2 =
ab(a + b) sin2 α sin2 β sin2 γ
( ) ( )
a cos2 β sin2 γ− cos2 γ sin2 β sin2 α + b cos2 α sin2 γ − cos2 γ sin2 α sin2 β
ab(a + b) sin2 α sin2 β sin2 γ
= ( ) ( ) .
a sin2 γ − sin2 β sin2 α + b sin2 γ − sin2 α sin2 β

Problem 22. A person walking along a straight road observes the angles of elevation
α, α′ of the summits of two hills in ront of him, one behind and partially hid by the
other. After walking c miles the farther hill becomes entirely hidden, and on observing
the elevation of the lower hill after walking another mile he finds it to be β. Find the
heights of the two hills.
Solution. Let P be the summit of the lower hill, Q the summit of the higher hill; let A
be the first point of observation, B the second, C the third. From P and Q draw P M
and QN, respectively perpendicular to the horizontal plane which contains A, B, and
C.
Let P M = h, and QN = h′ .
Then AM = h cot α, and AM = AB + BC + CM = c + 1 + h cot β;
therefore h cot α = c + 1 + h cot β; therefore h(cot α − cot β) = c + 1;
(c + 1) sin α sin β
therefore h= .
sin(β − α)
And by similar triangles
h′ QN BN AN − AB h′ cot α′ − c
= = = = ;
h PM BM AM − AB h cot α − c
thus since h is known we can find h . ′

Problem 23. A tower is surrounded by a circular moat. At noon on a certain day


the shadow of the top of the tower is observed to project 45 feet beyond the edge of
the moat. When the sun is due West on the same day the shadow projects 120 feet
beyond the moat. The distance between the extremities of the shadow is 375 feet.
The angle of elevation of the top of the tower from any point of the edge of the moat
is 60 ◦ . Find the height of the tower and the altitude of the sun at noon.
CHAPTER XV : Measurement of Heights and Distances 131

Solution. Let h be the height of the tower in feet, α the altitude of the sun at noon.
The distance between the foot of the tower and the edge of the moat is h cot 60◦ ;
hence the distance between the foot of the tower and the extremity of the shadow is
h cot 60◦ + 45 at noon, and h cot 60◦ + 120 when the sun is due west. The directions of
the shadows include a right angle;
therefore (h cot 60◦ + 45)2 + (h cot 60◦ + 120)2 = (375)2 .
2h2 2h
Therefore + √ · 165 + (45)2 + (120)2 = (375)2 ;
3 3
2h2 2h
therefore + √ 165 = 124200.
3 3 √ √
By solving this quadratic in the usual way we obtain h = 180 3 or −345 3; only the

positive value is applicable. Then h cot α − h cot 60 = 45;
45 1 45 1 1 5
therefore cot α = cot 60◦ + = √ + √ = √ + √ = √ ;
h 3 180√ 3 3 4 3 4 3
4 3
therefore tan α = .
5

Problem 24. A tower stands upon an inclined plane, meeting it at a point A; at a


point C in the plane the tower is observed to subtend an angle α; on proceeding to a
point D in the straight line AC such that CD = AC , the tower is observed to subtend
an angle β : if ϕ be the angle between the tower and AC , show that
cot ϕ = 2 cot α − cot β.
Also if similar observations be made in another straight line AC ′ D ′ , it is found that
tan α′ = 2 tan β ′ ; the angle CAC ′ = γ : prove that if θ be the inclination of the plane to
the horizon,
sin θ sin γ = cos ϕ.
Solution. Let P denote the top of the tower. Then ϕ is the angle between P A and CA
produced through A. Thus the angle CP A = ϕ − α, and the angle DP C = α − β.
DC sin DP C sin(α − β)
Then = = ,
CP sin CDP sin β
CA sin CP A sin(ϕ − α) sin(ϕ − α)
= = = ;
CP sin CAP sin(π − ϕ) sin ϕ
sin(α − β) sin(ϕ − α)
therefore = ;
sin β sin ϕ
therefore sin α cot β − cos α = cos α − sin α cot ϕ;
therefore cot ϕ = 2 cot α − cot β.
Now let α′ , β ′ , and ϕ′ correspond to observations made in another straight line
AC ′ D′ ; then cot ϕ′ = 2 cot α′ − cot β ′ ; but by supposition 2 tan β ′ = tan α′ ; therefore
π
cot ϕ′ = 0; therefore ϕ′ = . Thus AC ′ D′ makes a right angle with AP ; and therefore
2
AC ′ D′ is a horizontal straight line.
From D draw DM perpendicular to AD′ , and from M draw M N perpendicular to
the horizontal plane which contains D; and produce P A through A to meet the same
plane at Q.
MN DM AQ MN
Then sin θ = , sin γ = , cos ϕ = = ;
MD DA AD AD
therefore cos ϕ = sin θ sin γ.


Problem 25. In a triangle ABC having given A = 30 ◦ , b = 3 3 , a = 3 , solve the tri-
′′
angle; and supposing that an error of 2 is made in observing the angle A, find ap-
proximately the corresponding error
√ in the angle B.
b 3 3 √
Solution. sin B = sin A = sin A = 3 sin A;
a 3
π √ 1 π 2π
thus if A = we have sin B = 3 · ; therefore B = or .
6 2 3 3
CHAPTER XV : Measurement of Heights and Distances 132

π
Suppose however that A = ± h, where h is the circular measure of 2′′ ; then
√ ( ) √ { 6 }
π π π π
sin B = 3 sin ± h = 3 sin ± h cos very nearly. Suppose that B = ± k;
√ { }
6 6 6 3
π π π π
then approximately sin ± k cos = 3 sin ± h cos ;
3 3 6 6
π √ π √ π
therefore ±k cos = ±h 3 cos ; therefore k = h 3 · cot = 3h.
3 6 6

In the same way if B = ± k we find that k = −3h. Thus the approximate error
3
in B is 6 seconds.

Problem 26. The distance between two objects on the opposite bank of a river is
known to be c. An equal distance is taken anywhere along the bank on this side and
the angles subtended by c at the extremities of this distance are α and β. Find the
breadth of the river, the sides being parallel.
Solution. Let A and B be the two objects on the opposite bank of the river; and
suppose P and Q two points on this bank, such that P Q = AB; and let P correspond
to A and Q to B, so that AP is equal and parallel to BQ. Let AQ and BP intersect at
C.
Then α = the angle AP B, and β = the angle AQB = the angle P AQ.
PC sin β AC sin α
Therefore = , = ;
PA sin(α + β) PA sin(α + β)
but P Q = P C + QC − 2P C · QC · cos P CQ; and QC = AC;
2 2 2

sin2 β + sin2 α − 2 sin α sin β cos(α + β)


therefore c2 = P A 2 .
sin2 (α + β)
1
Let x denote the breadth of the river; then the area of the triangle AP B = xc;
2
and this area is also equal to
1 P A2 sin α sin β
P A · P B sin AP B = P A · P C sin α =
2 sin(α + β)
c2 sin α sin β sin(α + β)
= .
sin2 β + sin2 α − 2 sin α sin β cos(α + β)

Problem 27. A person wishing to obtain the breadth of a square fort on a distant hill,
observes that when he is due South of one corner, the face towards him subtends an
angle α. He then walks due West, and at a distance of a feet from his first position,
finds that the face subtends the same angle as before. On walking b feet further, he
is due South of the other corner of the face. Show that the breadth of the fort is
b tan α
(a + b) sec ϕ feet, where tan ϕ = .
a+b
Solution. Let AB denote a side of the fort, C the position due south of A; let D be
the second position, so that CD = a, and the angle ACD = 90◦ ; also A, B, D, and C
will lie on the circumference of a circle. Let E be the third position, so that E is on
CD produced through D, and DE = b; and the angle BED is a right angle.
Let ϕ be the angle between AB produced through B and CE produced through E.
Then a + b = AB cos ϕ; therefore AB = (a + b) sec ϕ.
And BE = EC tan BCE, and = ED tan BDE;
∴ (a + b) tan(90◦ − α) = b tan BAC = b tan(90◦ − ϕ). (Euclid iii. 22)

Problem 28. A and A′ are the peaks of two mountains, and BC is a straight horizontal
road; show that if the nearer of the two peaks just conceals the more distant at some
point of the road, then sin α sin β ′ = sin α′ sin β, where α is the altitude of A as seen
from any point B of the road, β is the angle ABC , and α′ , β ′ are similar quantities for
the peak A′ as seen from any point B ′ of the road.
Solution. From A draw AM perpendicular to the horizontal plane which contains
CHAPTER XV : Measurement of Heights and Distances 133

the road, and draw AN perpendicular to the straight road.


AM AN
Then sin α = , and sin β = .
AB AB
Similarly from A′ draw A′ M ′ perpendicular to the horizontal plane, and A′ N ′ per-
pendicular to the straight road.
A′ M ′ A′ N ′
Then sin α′ = ′ ′ and sin β ′ = ′ ′ .
AB AB
AM A′ N ′ A′ M ′ AN
Thus we have to show that · ′ ′ = ′ ′ · ,
AB A B AB AB
AM A′ M ′
or that AM · A′ N ′ = A′ M ′ · AN, or that = ′ ′.
AN AN
Now if A is just hidden by A′ at some point of the road, the straight line A′ A if

produced through A will intersect the road; and then AA and the road will lie in one
plane; the sine of the inclination of this plane to the horizontal plane is expressed by
AM A′ M ′
and also by ′ ′ ; so that these are equal.
AN AN

Problem 29. A and B are two objects in the same horizontal plane, P is a point in the
same plane at which the angle α subtended by AB is observed; from P two persons
walk in this plane in directions at right angles to PA, PB respectively, to points Q, R,
at each of which the angle subtended by AB is α; the distanced PQ, PR are a, b. Find
the length of AB.
Solution. There are two cases. Suppose the angles AP Q and BP R to be on the same
sides of AP and BP respectively; then the angle QP R = the angle AP B = α. Suppose
the angles AP Q and BP R not to fall on the same sides of AP and BP respectively;
then the angle RP Q = π − α. In both cases AB = RQ; for the diameter of the circle
AB RQ
which goes round the five points A, B, P, Q and R = and also = .
sin AP B sin RP Q

In the former case AB = (a2 + b2 − 2ab cos α), and in the latter case

AB = (a2 + b2 + 2ab cos α).

Problem 30. A, C , B are three objects in the same plane as an observer; AC = CB,
and AC , CB are at right angles to each other. At the point O, AC , CB subtend angles
α, β respectively. The observer moves from O in the direction OO ′ at right angles
to CO through a space OO ′ = d; here he finds that AC , CB subtend angles α′ , β ′
respectively. Find the distance AB.
Solution. Suppose both straight lines OC and O′ C to fall within the angle ACB. Let
AC = α, ACO = ϕ; then from the triangles ACO and BCO we get
a sin(ϕ + α) a cos(ϕ − β)
OC = and OC = ;
sin α sin β
therefore OC sin α = a(sin ϕ cos α + cos ϕ sin α),
OC sin β = a(cos ϕ cos β + sin ϕ sin β).
OC sin α(cos β − sin β)
Hence a sin ϕ = ,
cos(α + β)
OC sin β(cos α − sin α)
a cos ϕ =
cos(α + β)
Square and add; thus { }
a2 cos2 (α + β) = OC 2 sin2 α(cos β − sin β)2 + sin2 β(cos α − sin α)2
= OC 2 {sin2 α + sin2 β − 2 sin α sin β sin(α + β)}.
a2 cos2 (α + β)
Thus OC 2 = .
sin α + sin β − 2 sin α sin β sin(α + β)
2 2

A similar expression will be found for ′ 2 ′ ′ ′ 2


√ O C in terms of α and β . Then O C =
OC 2 + d2 . This finds a; and then AB = a 2.
Similarly the problem may be solved for any other positions of the lines OC, O′ C.
CHAPTER XV : Measurement of Heights and Distances 134

Problem 31. A tower 150 feet high throws a shadow 75 feet long upon the horizontal
plane upon which it stands. Find the Sun’s altitude, having given
log 2 = ·3010300, L tan 63◦ 26′ = 10·3009994,
L tan 63◦ 27′ = 10·3013153.
150
Solution. Let α denote the Sun’s altitude; then tan α = = 2;
75
therefore L tan α = 10 + log 2 = 10·3010300.
10·3013153 10·3010300
10·3009994 10·3009994 ·0003159 : ·0000306 :: 60′′ : x′′ ;
·0003159 ·0000306
this gives x = 6; therefore α = 63◦ 26′ 6′′ .

Problem 32. A person standing at the edge of a river observes that the top of a tower
on the edge of the opposite side subtends an angle of 55 ◦ with a horizontal straight
line drawn from his eye; receding backwards 30 feet he then finds it to subtend an
angle of 48 ◦ . Determine the breadth of the river, having given
L sin 7◦ = 9·08589, L sin 35◦ = 9·75859,
L sin 48◦ = 9·87107, log 3 = ·47712,
log 1·0493 = ·02089.
Solution. Take the diagram of Art. 240 (page 340). Here P BC = 55◦ , P AC =
48◦ , AB = 30 feet.
PB sin P AB sin 48◦ 30 sin 48◦
= = ; therefore P B = ;
BA sin AP B sin 7◦ sin 7◦
30 sin 48 ◦ sin 35◦
BC = BP cos P BC = BP cos 55◦ = BP sin 35◦ = ;
sin 7◦
log BC = log 30 + L sin 48◦ − 10 + L sin 35◦ − 10 − (L sin 7◦ − 10)
= 1.47712 + 9.87107 + 9.75859 − 9.08589 − 10 = 2.02089;
therefore BC = 104.93.

Problem 33. A rope-dancer wishes to ascend a tower 100 feet high, by means of a
rope 196 feet long. If he can do so, find at what inclination he must be able to walk
up the rope, having given
log 2 = ·30103, L sin 30◦ 40′ = 9·70761,
log 7 = ·84510, L 30◦ 41′ = 9·70782.
100 100
Solution. Let α denote the inclination; then sin α = = ;
196 4 × 49
therefore L sin α = 10 + log 100 − log(4 × 49) = 12 − 2 log 2 − 2 log 7 = 9.70774.
9·70782 9·70774
9·70761 9·70761 ·00021 : ·00013 :: 60′′ : x′′ ;
·00021 ·00013
this gives x = 37; therefore α = 30◦ 40′ 37′′ .

Problem 34. Two hills rise at the same point, with inclinations of 60 ◦ and 40 ◦ to the
horizon. At a distance of 64 feet from the base of the latter hill the angles of elevation
of the bottom and top of a vertical object on the former hill are 40 ◦ and 70 ◦ . Find the
height of the object, having given
L tan 20◦ = 9·5610659, L cos 40◦ = 9·8842540,
log 2 = ·3010300; log 26940031 = 7·4303981.
Solution. Let A be the point of intersection of the hills, B the point of observation
on the hill, P the top of the object, C the bottom. Produce P B through B to meet at
D the horizontal straight line which contains A; produce DA through A to any point
E. Then AB = 64 feet; and the following are the given angles:
CAE = 60◦ , BAD = 40◦ , BDA = 70◦ , BP C = 90◦ − 70◦ = 20◦ .
Therefore BAC = 80◦ , BCA = 20◦ , P BC = 30◦ .
CHAPTER XV : Measurement of Heights and Distances 135

BC sin BAC sin 80◦


= = ,
BA sin BCA sin 20◦
PC sin P BC sin 30◦ PC sin 80◦ sin 30◦
= = ◦
; therefore = ;
BC sin BP C sin 20 BA sin2 20◦
64 sin 80◦ sin 30◦ 64 sin 40◦ cos 40◦ 128 cos 40◦
therefore PC = = = ;
sin2 20◦ sin2 20◦ tan 20◦
◦ ◦
therefore log P C = 7 log 2 + L cos 40 − L tan 20 = 2.4303981;
therefore P C = 269.40031.

Problem 35. A vessel observed another α◦ from the North sailing in a direction
parallel to its own. After an hour’s sailing its bearing was β ◦ , and after another hour
γ ◦ from the North. Find in what direction the vessels were sailing.
Solution. Let A, B, C be the three successive positions of the ship from which the
observations are made; let P, Q, R be the corresponding positions of the other ship.
Then the straight line ABC is parallel to the straight line P QR; also AB = BC, and
P Q = QR.
Let θ be the angle between the North direction and the direction of sailing.
From B draw a straight line parallel to AP, meeting P Q at M ; then
QM sin QBM sin QBM sin(β − α)
= = = .
BM sin BQM sin BQP sin(θ − β)
Again, from C draw a straight line parallel to AP, meeting QR at N ; then
RN sin RCN sin RCN sin(γ − α)
= = = .
CN sin CRN sin CRP sin(θ − γ)
But BM = CN ; and RN = 2QM, for RN is the difference of the paths of the ships
in two hours, and QM is the difference in one hour.
2 sin(β − α) sin(γ − α)
Therefore = ;
sin(θ − β) sin(θ − γ)
therefore 2 sin(θ − γ) sin(β − α) = sin(γ − α) sin(θ − β),
therefore
2(sin θ cos γ − cos θ sin γ) sin(β − α) = (sin θ cos β − cos θ sin β) sin(γ − α).
Divide by cos θ; thus we obtain the value of tan θ.

Problem 36. In the problem discussed in Art. 242 (page 341), show that if
π
α + β + C = π, then ϕ = ,
4
and the solution cannot be obtained from the data.
Solution. If α + β + C = π, then x + y = π; therefore sin x = sin y; therefore tan ϕ = 1.
We might as in Art. 242 (page 341) say that ( )
sin x − sin y π
= tan ϕ − ,
sin x + sin y 4
x−y x+y
2 sin cos ( )
2 2 π
that is = tan ϕ − .
x+y x−y 4
2 sin cos
2 2
x+y
But as cos is now zero we cannot divide both numerator and denominator of
2
the last fraction by it, and thus we cannot proceed further. In fact in this case a circle
would go round P, A, C, and B, and P may be at any point of the arc between A and
B.
CHAPTER XVI
Properties of Triangles

Problem 1. The sides of a plane triangle are 24 , 30 , 18 : find the area.


Solution. Here s = 36, s − a =
√12, s − b = 6, s − c√
= 18.
The area of the triangle = 36 × 12 × 6 × 18 = 36 × 36 × 36 = 63 = 216.

Problem 2. Two angles of a triangle are 15 ◦ and 45 ◦ , and the included side is 10
feet : find the area.
Solution. The third angle of the triangle = 180◦ − 60◦ = 120◦ .
10 × sin 15◦ 10 × sin 45◦
One of the containing sides = ◦
, and the other = .
sin 120 sin 120◦
Hence the area √
1 (10)2 sin 15◦ sin 45◦ ◦ 50 sin 15◦ sin 45◦ 50( 3 − 1) 1 2
= sin 120 = = √ ×√ ×√
2 sin2 120◦ sin 120◦ 2 2 2 3

25( 3 − 1)
= √ .
3

Problem 3. The sides of a triangle are equal to 3 and 12 respectively, and the con-
tained angle is 30 ◦ : find the hypotenuse of an equal right-angled isosceles triangle.
1 36
Solution. The area of the triangle = × 3 × 12 × sin 30◦ = = 9.
2 4
Let x denote the hypotenuse of the right-angled triangle; then each of the equal
( )2
x 1 x x2
sides is √ , and the area is × √ , that is .
2 2 2 4
x 2
Hence = 9; ∴ x2 = 36; ∴ x = 6.
4

1 ( 2 )
Problem 4. The area of a triangle = a sin 2B + b2 sin 2A .
4
Solution. From the angle C of a triangle draw a perpendicular CD to the side AB,
or AB produced.
First suppose A and B acute, so that D is between A and B. Then CD = b sin A, AD =
1 1
b cos A; thus the area of ACD = b2 sin A cos A = b2 sin 2A.
2 4
1 1
Similarly the area of BCD = a2 sin B cos B = a2 sin 2B.
2 4
1( 2 )
Therefore the area of the whole triangle = a sin 2B + b2 sin 2A .
4
Next suppose the angle B obtuse, so that D falls on AB produced through D. Then
1
as before the area of ACD = b2 sin 2A.
4
And the area of CBD
1 2
= a sin (180◦ − B) cos (180◦ − B)
2
a2
= sin (360◦ − 2B) .
4
Therefore the area of ABC
1{ 2 } 1( 2 )
= b sin 2A − a2 sin (360◦ − 2B) = b sin 2A + a2 sin 2B .
4 4
This mode of solution shows the geometrical meaning of the two parts of the ex-
pression. We may proceed more briefly thus :
1( 2 )
a sin 2B + b2 sin 2A
4
1
= (a sin B a cos B + b sin A b cos A)
2
CHAPTER XVI : Properties of Triangles 138

1
= a sin B(a cos B + b cos A), by Art. 214 (page 336),
2
1
= ac sin B, by Art. 216 (page 337),
2
= the area of the triangle by Art. 247 (page 342),

a 2 − b2 sin A sin B
Problem 5. The area of a triangle = .
2 sin(A − B)
Solution. { }
a2 − b2 sin A sin B sin A sin B c2 sin2 A c2 sin2 B
= −
2 sin(A − B) 2 sin(A − B) 2
sin C sin2 C
( )
c2 sin A sin B sin2 A − sin2 B
=
2 sin(A − B) sin2 C
c2 sin A sin B sin(A + B) sin(A − B)
=
2 sin(A − B) sin2 C
c2 sin A sin B
=
2 sin C
= area of the triangle, by Art. 247 (page 342).

2abc A B C
Problem 6. The area of a triangle = cos cos cos .
a+b+c 2 2 2
Solution. √ √ √
2abc A B C 2abc s(s − a) s(s − b) s(s − c)
cos cos cos = × ×
a+b+c 2 2 2 2s bc ac ab

= s(s − a)(s − b)(s − c)
= S = the area of the triangle.

Problem 7. Show that( the triangle


) (whose sides
) are proportional to
gh k 2 + l 2 , kl g 2 + h 2 , (hk + gl)(hl − gk).
has its area and the trigonometry ratios of its angles rational.
Solution. ( ) ( )
Here 2s = gh k2 + l2 + kl g 2 + h2 + (hk + gl)(hl − gk)
( ) ( )
= gh k2 + l2 + kl g 2 + h2 + h2 kl + ghl2 − hgk2 − klg 2
= 2ghl2 + 2h2 kl.
∴ s = ghl2 + klh2 = hl(gl + hk);
∴ s − a = klh2 − ghk2 = kh(lh − gk),
s − b = ghl2 − klg 2 = gl(lh − gk),
s − c = hgk2 + klg 2 = kg(hk + lg).
Thus s(s − a)(s − b)(s − c) = g 2 h2 k2 l2 (lh − gk)2 (hk + lg)2 ;
∴ S = ghkl(lh − gk)(hk + lg).
Therefore by Art. 218 (page 338) the sines of the angles of the triangle are rational
quantities; and by Art. 215 (page 336) the cosines of the angles are rational quantities.

Problem 8. The sides of a triangle are in arithmetical progression, and its area is
to that of an equilateral triangle of the same perimeter as 3 is to 5 . Find the ratio of
the sides and the value of the largest angle.
Solution. Let a, b, c be in Arithmetical Progression; then 2b = a + c. Thus the perime-
ter = 3b, and the side of an equilateral triangle of equal perimeter is b.
√ √
3 1 2 ◦ 3 3 2
Thus s(s − a)(s − b)(s − c) = · b sin 60 = b ,
5 2 20 √
√ 3 3 2
∴ (a + b + c)(b + c − a)(a + c − b)(a + b − c) = b ,
5
CHAPTER XVI : Properties of Triangles 139

√ √
3 3 2
∴ 3b2 (b + c − a)(a + b − c) =b ,
5
√ 3
∴ (b + c − a)(b + a − c) = b,
√ 5
3c − a 3a − c 3
∴ × = (a + c);
2 2 10
9
∴ (3c − a(3a − c) = (a + c)2 ;
25
( ) 9 ( 2 )
∴ 10ac − 3 a2 + c2 = a + 2ac + c2 ;
( ) 25
∴ 84 a2 + c2 − 232ac = 0,
( )
a2 a
∴ 21 + 1 = 58 .
c2 c
a 7 3
By solving this quadratic in the usual way, we obtain = or .
c 3 7
a 7
Take = ; thus a, b, and c are proportional to 7, 5, and 3 respectively.
c 3
b2 + c2 − a2 52 + 32 − 72 1
Then cos A = = =− ;
2bc 2×5×3 2
∴ A = 120◦ .

Problem 9. If the alternate angles of a regular hexagon be joined so as to form


another regular hexagon, and again the alternate angles of the latter hexagon be
S
joined, and so on, show that the sum of the areas of all the figures so formed = ,
2
where S is the area of the original figure.
Solution. Let A, B, C, D, E be five consecutive angles of the hexagon; draw
AC, BD, CE; let AC and BD intersect at P, and let BD and CE intersect at Q. Then
P Q is the side of the second regular hexagon.
The angle DBC is half of the angle which DC would subtend at the centre of the
π
circle circumscribing the regular hexagon, and is therefore . Similarly the angle
6
π
ACB is .
6
π π
PC sin sin 1 BC
Then = ( 6 )= 6 =
2π π ; ∴ PC = π.
BC 2π
sin π − sin 2 cos 2 cos
6 6 6 6
1 π π
And P Q = 2P C sin P CQ = 2P C sin = BC tan .
2 6 6
BC
Thus P Q = √ . And the areas of similar polygons are as the squares of their homo-
3
logus sides; so that if S denote the area of the first hexagon the area of the second is
S 1 S S
. In like manner the area of the next hexagon is of , that is ; and so on. Hence
3 3 3 9
S S S 1 S
the sum of the areas of all the derived figures is + + + . . . , that is , that
3 9 27 3 1
1−
3
S
is .
2

Problem 10. If we proceed with a regular figure of n sides, and of area S, as with
the hexagon of Example 9 , and Σ denote the sum of the areas so formed, show that
3π π 2π
Σ sin sin = S cos2 .
n n n
Explain the cases where n = 3 or 4 .
Solution. Suppose that the original figure instead of being a hexagon is a regular
CHAPTER XVI : Properties of Triangles 140

polygon of n sides. Proceed as before and we have


π π
PC sin sin
= ( n )= n = 1 .
BC 2π 2π π
sin π − sin cos
n n n
1
Then P Q = 2P C sin P CQ;
2
π π
and the angle P CQ = (n − 4) ; ∴ P Q = 2P C sin(n − 4)
n 2n

( ) cos
π 2π 2π n
= 2P C sin − = 2P C cos = BC π .
2 n n cos
n

S cos2
Thus the area of the second polygon is n
π ;
cos2
n

( 2 3
) cos2
n
and Σ = S 1 + m + m + . . . where m stands for π ;
cos2
n
S cos2 2π S cos2 2π S cos2 2π
Sm n n n .
thus Σ= = = =
1−m π 2π 2π π 3π π
cos2 − cos2 sin2 − sin2 sin sin
n n n n n n
If n = 3 this becomes infinite; for sin π = 0; in this case the original figure is a triangle,
and the second figure is the same triangle, and so on : thus the sum of the areas is
infinite.

If n = 4 the expression vanishes; for cos = 0; in this case the original figure is a
4
square, and the second figure is only a point, and so on : thus the sum of the areas is
zero.

Problem 11. If an equilateral triangle be described with its angular points on


the sides of a given right-angled isosceles triangle, and one side parallel to the hy-
potenuse, its area will be 2a 2 sin 60 ◦ (sin 15 ◦ )2 , where a is a side of the given triangle.
Solution. Let ABC denote the right-angled isosceles triangle where C is the right
angle. Let F be the middle point of AB; let D be on BC, and E on AC, such that DE
is parallel to AB, and the triangle DEF is equilateral.
Then the angle DEC = 45◦ , and the angle DEF = 60◦ ; therefore the angle AEF =
◦ FE sin F AE sin 45◦
75 . Now = = ;
FA sin F EA sin 75◦
F A sin 45◦ a sin 45◦ a 1 a sin 15◦
∴ FE = = √ · = =
sin 75◦ 2 cos 15◦ 2 cos 15◦ 2 cos 15◦ sin 15◦
a sin 15◦ ◦
= = 2a sin 15 .
sin 30◦
Therefore the area of the equilateral triangle
1
= (2a sin 15◦ )2 sin 60◦ = 2a2 sin2 15◦ sin 60◦ .
2

Problem 12. Show that, with the notation of Arts. 248 (page 342) and 250 (page 344),
A B C
r1 r2 r3 = r 3 cot2 cot2 cot2 .
2 2 2
S3 S3
Solution. r1 r2 r3 = , r3 = 3 ;
(s − a)(s − b)(s − c) s
r1 r2 r3 s3
therefore = ;
r3 (s − a)(s − b)(s − c)
A B C s(s − a) s(s − b) s(s − c)
and cot2 cot2 cot2 = × ×
2 2 2 (s − b)(s − c) (s − a)(s − c) (s − a)(s − b)
CHAPTER XVI : Properties of Triangles 141

s3
= ;
(s − a)(s − b)(s − c)
r1 r2 r3 A B C
therefore = cot2 cot2 cot2 .
r3 2 2 2

Problem 13. The straight lines which bisect the angles A, C of a triangle ABC meet
the circumference of the circumscribing circle at the points A′ , C ′ . Show that A′ C ′ is
divided by CB, BA into three parts, which are in the proportion
A A B C C
sin2 : 2 sin sin sin : sin2 .
2 2 2 2 2
Solution. Let C ′ A′ intersect AB at E and CB at F.
The angle A′ FC is equal to the sum of the angles FC ′ C and FCC ′ ,
that is to the sum of the angles A′ AC and FCC ′ ,
1 1 1
that is to A + C ; the angle BCA′ = ∠BAA′ = A.
2 2 2
A A
FA ′ sin sin
Thus = 2 = 2 .
CA′ (A + C) B
sin cos
2 2
Let R be the radius of the circle; then
A
A 2R sin2
′ ′ 2 .
A C = 2R sin ; ∴ F A =
2 B
cos
2
C
2R sin2
In the same manner EC ′ = 2 .
B
cos
2
(A + C) B
And A′ C ′ = 2R sin = 2R cos .
( 2 ) 2
A C
2R sin2 + sin2
B 2 2
∴ EF = 2R cos −
2 B
cos
{ 2 }
2R B A C
= cos2 − sin2 − sin2
B 2 2 2
cos
2
R
= {1 + cos B − (1 − cos A) − (1 − cos C)}
B
cos
2
R
= (cos A + cos B + cos C − 1)
B
cos
2
2R A B C
= × 2 sin sin sin , by Art.114 (page 328).
B 2 2 2
cos
2

Problem 14. If h be the difference between the sides containing the right angle of
a right-angled
√ triangle, and S its area, the diameter of the circumscribing circle is
equal to h 2 + 4S.
Solution. Let a denote √ one side of the right-angled
√ triangle, and a + h the other side;
then the hypotenuse = a2 + (a + h)2 = h2 + 2a(a + h).
1
But S = half the product of the sides = a(a + h); therefore 4S = 2a(a + h).
√ 2
Thus the hypotenuse = h2 + 4S; and the hypotenuse is a diameter of the circum-
scribing circle.
CHAPTER XVI : Properties of Triangles 142

Problem 15. The sides of a plane triangle are 3 , 5 , 6 . Compare the radii of the
inscribed and circumscribed circles.

S abc R sabc
Solution. r= , R= ; ∴ = .
s 4S √r 4S 2
Now s = 7, s − a = 4, s − b = 2, s − c = 1; ∴ S = 7 × 4 × 2; thus
R 7×3×5×6 45
= = .
r 4×7×4×2 16

Problem 16. O is the centre of the circle circumscribed round an acute-angled


triangle, and AO is produced to meet BC at D. Show that
DO cos(B − C) = AO cos A.
π
Solution. The angle ABO = the angle BAO = − C; and therefore the angle BOD =
2
π
π − 2C; the angle OBD = − A;
2
π π π
therefore the angle BDO = 2C + A − = A + C + B + C − B − = + C − B.
2( ) 2 2
π
sin −A
DO sin DBO cos A
Thus = = ( 2 )= ,
BO sin BDO π cos(C − B)
sin +C−B
2
and BO = AO; ∴ DO cos(B − C) = AO cos A.

Problem 17. A circle is inscribed within a given triangle, and another triangle
formed by joining the points of contact; within this latter triangle a circle is inscribed,
and another triangle is formed as before, and so on continually. Show that the trian-
gles thus formed ultimately become equilateral.
Solution. Take the diagram of Art. 248 (page 342); draw F D, DE, and EF.
1 1
The angle F DB = (π − B), the angle EDC = (π − C); therefore the angle F DE =
2 2
1 1 1
(B + C). Similarly the angle DEF = (C + A), and the angle EF D = (A + B).
2 2 2
Suppose A, B, C in ascending order of magnitude; then
1 1 1
(A + B), (A + C), (B + C),
2 2 2
are in ascending order of magnitude; and
1 1 1
(B + C) − (A + B) = (C − A).
2 2 2
Thus the difference between the greatest and least angles of the first derived triangle
is half the difference between the greatest and least angles of the original triangle.
In like manner the difference between the greatest and least angles of the second
derived triangle is half the difference between the greatest and least angles of the
first derived triangle, and therefore a fourth of the difference between the greatest
and least angles of the original triangle. Proceeding in this way we see that the
triangles thus formed ultimately become equilateral.

Problem 18. The sum of the diameters of the inscribed and circumscribed circles
of any plane triangle is equal to
a cot A + b cot B + c cot C .
Solution.
a cot A + b cot B + c cot C
a b c
= cos A + cos B + cos C
sin A sin B sin C
= 2R(cos A + cos B + cos C) = 2R + 2R(cos A + cos B + cos C − 1)
A B C
= 2R + 8R sin sin sin , by Art. 114, (page 328),
2 2 2
CHAPTER XVI : Properties of Triangles 143

√ √ √
(s − b)(s − c) (s − a)(s − c) (s − a)(s − b)
= 2R + 8R · × ×
bc ac ab
8R S 2 2S
= 2R + = 2R + = 2R + 2r.
abc s s

Problem 19. Perpendiculars are drawn from the angles A, B, C of an acute-angled


triangle on the opposite sides, and produced to meet the circumscribing circle; if
those produced parts be α, β, γ respectively, show that
a b c
+ + = 2 (tan A + tan B + tan C ).
α β γ
Solution. From A draw AD perpendicular to BC, and produce AD to meet the cir-
cumference of the circle at L.
Then the angle ALB = the angle ACB = C;
α = DL = BD cot ALB = BD cot C
c cos B cos C a cos B cos C
= = ;
sin C sin A
a sin A sin(B + C)
therefore = = = tan B + tan C.
α cos B cos C cos B cos C
b c
Similarly = tan A + tan C, and = tan C + tan A.
β γ
a b c
Therefore + + = 2(tan A + tan B + tan C).
α β γ

Problem 20. In any triangle the area of the inscribed circle is to the area of the
A B C
triangle as π is to cot cot cot .
2 2 2
Solution. The area of the inscribed circle is to the area of the triangle as πr 2 is to S,
S
that is, as π is to 2 . Thus we have to show that
r
S A B C
= cot cot cot .
r2 2 2 2
Now √ √ √
A B C s(s − a) s(s − b) s(s − c)
cot cot cot = × ×
2 2 2 (s − b)(s − c) (s − a)(s − c) (s − a)(s − b)

s s s2 s2 S
= √ = = S × 2 = 2.
(s − a)(s − b)(s − c) S S r

Problem 21. On each side of an acute-angled triangle as base an isosceles triangle


is constructed externally, the sides of each being equal to the radius of the circum-
scribed circle : if the vertices of these be joined a triangle will be formed equal and
similar to the original.
Solution. Let the triangle constructed on BC have its vertex at L, let that constructed
on CA have its vertex at M, and that constructed on AB have its vertex at N.
Take the diagram of Art. 252 (page 344). The triangle CLB will be equal to the
π
triangle COB in all respects; therefore the angle BCL = the angle OCB = − A.
2
π
In the same manner the angle ACM = − B;
2
π π
therefore the angle LCM = − A + − B + C = 2C.
2 2
Then (LM )2 = R2 + R2 − 2R2 cos 2C = 2R2 (1 − cos 2C) = 4R2 sin2 C;
therefore LM = 2R sin C = c.
In a similar manner we find that M N = a, and N L = b. Thus the triangle LM N is
in all respects equal to the triangle ABC.
CHAPTER XVI : Properties of Triangles 144

Problem 22. If R be the radius of the circumscribed circle of a triangle,


a cos A + b cos B + c cos C = 4R sin A sin B sin C .
Solution.
a cos A + b cos B + c cos C = 2R sin A cos A + 2R sin B cos B + 2R sin C cos C
= R(sin 2A + sin 2B + sin 2C)
= 4R sin A sin B sin C, by Art. 114 (page 328).

Problem 23. O is the centre of the circle circumscribed about a triangle ABC ; from
O the perpendiculars
( OD, OE, OF )are drawn to the sides. Show that
4 OD 2 + OE 2 + OF 2 = a 2 cot2 A + b2 cot2 B + c 2 cot2 C .

a2 a2
Solution. OD2 = R2 cos2 A = cos2 A = cot2 A,
4 sin2 A 4
b2 b2
OE 2 = R2 cos2 B = cos2 B = cot2 B,
4 sin2 B 4
c2 c2
OF 2 = R2 cos2 C = cos2 C = cot2 C.
( 2 ) 4 sin 2C 4
∴ 4 OD + OE 2 + OF 2 = a2 cot2 A + b2 cot2 B + c2 cot2 C.

Problem 24. If r be the radius of the circle inscribed in a triangle, and ra the radius
of the circle inscribed between this circle and the sides containing the angle A, show
that ( )
A A A 2
1 − sin cos − sin
2 =r 4 4
ra = r ( ) .
A A A 2
1 + sin cos + sin
2 4 4
Solution. Take the diagram of Art. 248 (page 342). The circle which is to be drawn
will have its centre, and its point of contact with the circle already drawn, on the
A
straight line OA. Thus the length of OA = r + ra + ra cosec ; and this distance also
2
A
= r cosec ; therefore
2 ( ) ( )
A A
ra 1 + cosec = r cosec −1 ;
2 2
( ) ( )
A A A 2
r 1 − sin r cos − sin
2 4 4
∴ ra = = ( ) .
A A A 2
1 + sin cos + sin
2 4 4

Problem 25. If r be the radius of the circle inscribed in a triangle, and ra , rb , rc , the
radii of the circles inscribed between this circle and the sides containing the angles
A, B, C respectively. Show that
√ √ √
ra rb + rb rc + rc ra = r.
Solution. By P roblem 24 we have
( ) ( )
A A 2 B B 2
r2 cos − sin cos − sin
4 4 4 4
ra rb = ( ) ( ) ;
A A 2 B B 2
cos + sin cos + sin
( 4 4 )( 4 4)
A A B B
r cos − sin cos − sin
√ 4 4 4 4
∴ ra rb = ( )( );
A A B B
cos + sin cos + sin
4 4 4 4
CHAPTER XVI : Properties of Triangles 145

( )( )( )
A A B B C C
r cos − sin cos − sin cos + sin
4 4 4 4 4 4
= ( )( )( );
A A B B C C
cos + sin cos + sin cos + sin
4 4 4 4 4 4
A+π B+π C−π
r cos cos cos
= 4 4 4
A−π B−π C−π
cos cos cos
( 4 4 )4
A B C
r cos + cos − cos
2 2 2
= , by Examples viii. 20 and 21.
A B C
cos + cos + cos
2 2 √ 2 √
Similar expressions can be found for rb rc and rc ra ; and the sum of the three ex-
pressions = r.

Problem 26. If a triangle A′ B ′ C ′ be formed by joining the feet of the perpendiculars


let fall from A, B, C on the opposite sides, show that B ′ C ′ is numerically equal to
R sin 2A, where R is the radius of the circle circumscribed about ABC .
Solution. Suppose A to be acute; then AB ′ = c cos A, AC ′ = b cos A, and
( )2
B′C ′ = AB ′2 + AC ′2 − 2AC ′ · AB ′ cos A
( )
= cos2 A c2 + b2 − 2bc cos A
= a2 cos2 A;
∴ B ′ C ′ = a cos A = 2R sin A cos A = R sin 2A.
If A is obtuse we find that AB ′ = c cos(π − A), AC ′ = b cos(π − A), and (B ′ C ′ )2 =
a2 cos2 A as before.

Problem 27. Perpendiculars drawn from the angular points of an acute-angled


triangle to the opposite sides meet those sides at the points D, E, F. Show that if
R and R1 be the radii of the circles described about the triangles ABC and DEF
respectively, and r1 the radius of the circle inscribed in the latter triangle,
1
R1 = R, and r1 = 2R cos A cos B cos C .
2
Solution. Let P denote the point of intersection of AD and BE.
Then since P EC and P DC are right angles a circle would go round P ECD; there-
π π
fore the angle P DE = the angle P CE = − A. Similarly P DF = − A. Therefore
2 2
F DE = π − 2A.
FE FE R sin 2A 1
R1 = = = , by Example 26, = R.
2 sin F DE 2 sin 2A 2 sin 2A 2
And
area of F DE F D · ED sin 2A
r1 = =
semi-perimeter of F DE R(sin 2A + sin 2B + sin 2C)
R sin 2A sin 2B sin 2C
= , by P roblem 26,
sin 2A + sin 2B + sin 2C
R sin 2A sin 2B sin 2C
= , by Art. 114 (page 328), = 2R cos A cos B cos C.
4 sin A sin B sin C

Problem 28. If r, r1 , r2 , r3 denote the radii of the inscribed and escribed circles of
a triangle, show that
A rr1
tan2 = .
2 r2 r3
Solution.
rr1 S2 S2 (s − b)(s − c) A
= ÷ = = tan2 .
r2 r3 s(s − a) (s − b)(s − c) s(s − a) 2

Problem 29. If A be the area of the circle inscribed in a triangle, A1 , A2 , A3 the


CHAPTER XVI : Properties of Triangles 146

areas of the escribed circles, then


1 1 1 1
√ = √ +√ +√ .
A A1 A2 A3
Solution.
1 1 1 1 1 s
√ = √ = √ · = √ · .
A πr 2 π r π S
1 1 s−a 1 1 s−b 1 1 s−c
Similarly √ = √ · , √ = √ · , √ = √ · ,
π S π S π
A1 A2
( A3
) S
1 1 1 1 s−a s−b s−c
∴ √ +√ +√ = √ + +
A1 A2 A3 π S S S
1 3s − a − b − c 1 s
= √ · = √ · .
π S π S

Problem 30. If the sides of a triangle be in arithmetical progression, the perpen-


dicular on the mean side from the opposite angle, and the radius of the circle which
touches the mean side and the other two sides produced, are each equal to three
times the radius of the inscribed circle.
Solution. Suppose a, b, c to be in Arithmetic Progression; so that 2b = a + c.
The perpendicular on the mean side from the opposite angle
ab sin C 2S
= a sin C = = .
b b
The radius of the circle which touches the mean side and the other two sides pro-
S 2S 2S
duced = = = .
s−b a+c−b b
S 2S 2S
The radius of the inscribed circle = = = .
s a+b+c 3b
The first and the second of these are each three times the third.

Problem 31. The distances of the centre of the circle inscribed in a triangle from
the centres of the three escribed circles are respectively equal to
A B C
a sec , b sec , c sec .
2 2 2
Solution. Let O denote the centre of the inscribed circle, and P the centre of the
escribed circle which is opposite to the angle A. Then O and P are both on the straight
line which bisects the angle A.
1 1 π
∠OBP = B + (π − B) = .
2 2 2
OB OB OB
Thus OP = = = ;
cos BOP 1 C
cos (A + B) sin
2 2
1 1 1
AB sin A c sin A c sin A
and OB = ( 2 )= 2 = 2 .
A B A+B C
sin π − − sin cos
2 2 2 2
1 1 1
c sin A 2c sin A 2a sin A a
∴ OP = 2 = 2 = 2 = .
C C sin C sin A A
sin cos cos
2 2 2

Problem 32. Two similar triangles have a common escribed circle touching sides
not homologous a1 , b2 . Show that
a1 : a2 = sin B + sin C − sin A : sin A + sin C − sin B.
Solution. Let a1 , b1 c1 be the sides of one triangle, S1 its area; let a2 , b2 , c2 be the
sides of the other triangle, S2 its area.

S1 S2
Then, by hypothesis, = ;
b1 + c1 − a1 a2 + c2 − b2
CHAPTER XVI : Properties of Triangles 147

sin B sin C
S1 b1 + c1 − a1 a1 + a1 − a1
∴ = = sin A sin A
S2 a2 + c2 − b2 sin C sin B
a2 + a2 − a2
sin A sin A
a1 sin B + sin C − sin A
= · .
a2 sin A + sin C − sin B
But the areas of similar triangles are as the square of their homologous sides; thus
S1 a2
= 12 ; therefore, finally,
S2 a2
a1 sin B + sin C − sin A
= .
a2 sin A + sin C − sin B

Problem 33. If O1 , O2 , O3 are the centres of the escribed circles of a triangle, then
the area of the triangle O1 O2 O3
{ }
a b c
= area of triangle ABC 1 + + + .
b+c−a a+c−b a+b−c
Solution. The points O2 , O3 , and A are in a straight line; similarly O3 , O1 , and B are
in a straight line; and O1 , O2 , and C are in a straight line.
The triangle O1 O2 O3 consists of four parts; namely ABC, O1 BC, O2 CA, and O3 AB.
1 aS aS
The area of O1 BC = ar1 = = .
2 2(s − a) b+c−a
Similar expressions hold for the ( areas of O 2 CA and O3 AB. )
a b c
Thus the area of O1 O2 O3 = S 1 + + + .
b+c−a a+c−b a+b−c

Problem 34. The centres of the three escribed circles of a triangle are joined. Show
abc
that the area of the triangle thus formed is , where r is the radius of the inscribed
2r
circle of the original triangle.
Solution. Here we have another expression for the area of the triangle considered
in the preceding solution.
1 B
O1 C sin (π − B) cos
We have = 2 = 2 ;
BC 1 A
sin (B + C) cos
2 2
B B
a cos 2R sin A cos
∴ O1 C = 2 = 2 = 4R sin A cos B .
A A 2 2
cos cos
2 2
B A
Similarly O2 C = 4R sin cos ;
( 2 2 )
A B B A
∴ O1 O2 = 4R sin cos + sin cos
2 2 2 2
A+B C
= 4R sin = 4R cos .
2 2
B
In like manner O1 O3 = 4R cos .
2
1
Then area of O1 O2 O3 = O1 O2 × O1 O3 × sin O2 O1 O3
2
C B B+C
= 8R2 cos cos sin
2 2 2
A B C
= 8R2 cos cos cos .
√ 2 2 √2 √
s(s − a) s(s − b) s(s − c)
= 8R2 × ×
bc ac ab
2
8R sS abcs abc
= = = .
abc 2S 2r
CHAPTER XVI : Properties of Triangles 148

Problem 35. A′ , B ′ , C ′ are the centres of the escribed circles of a triangle; A′ , B ′ , C ′


are joined so as to form a triangle. If r and r ′ be the radii of the circles inscribed in
ABC and A′ B ′ C ′ respectively,
A B C
r′ cot cot cot
= 2 2 2 .
r A B C
cos + cos + cos
2 2 2
Solution. We have
abc
area of A′ B ′ C ′

r = = ( 2r ),
semi-perimeter of A′ B ′ C ′ A B C
2R cos + cos + cos
2 2 2
by the solution of the preceding Problem,
abc S
= ( )= ( )
A B C A B C
4Rr cos + cos + cos r cos + cos + cos
2 2 2 2 2 2
s
= .
A B C
cos + cos + cos
2 2 2
A B C s2 s
Also cot cot cot = = ;
2 2 2 S r
(see the solution of Problem 20 : page 143),
A B C
r cot cot cot
′ 2 2 2 .
∴r =
A B C
cos + cos + cos
2 2 2

Problem 36. If r be the radius of the circle inscribed in a triangle ABC , 2s the sum
of the sides, r ′ , 2s′ similar quantities for the triangle which is formed by joining the
centres of the escribed circles, show that
rs A B C
= 2 sin sin sin .
r ′ s′ 2 2 2
Solution.
area of A′ B ′ C ′
We have r′ = ;
s′
abc
∴ r′ s′ = area of A′ B ′ C ′ = , by P roblem 34 (page 147).
2r
Again, rs = area of ABC = S.
rs 2rS 2S 2
∴ ′ ′ = = .
rs abc abcs
And √
A B C (s − b)(s − c) (s − c)(s − a) (s − a)(s − b)
2 sin sin sin =2 × ×
2 2 2 bc ac ab
2S 2 rs
= = ′ ′.
abcs rs

Problem 37. Let α, α1 be the distances of the angle A of a triangle from the centres
of the inscribed circle, and the circle touching the side a and the other two sides
produced; β, β1 similar quantities for the angle B; γ, γ1 similar quantities for the
angle C . Show that
αβγα1 β1 γ1 = (abc)2 .

A A
Solution. We have α = r cosec , α1 = r1 cosec ,
2 2
B B
β = r cosec , β1 = r2 cosec ,
2 2
C C
γ = r cosec , γ1 = r3 cosec ,
2 2
CHAPTER XVI : Properties of Triangles 149

∴ αβγα1 β1 γ1
A B C
= r3 r1 r2 r3 cosec 2 cosec 2 cosec 2
2 2 2
S3 S3
= 3 ×
s (s − a)(s − b)(s − c)
bc ca ab
× × ×
(s − c)(s − b) (s − a)(s − c) (s − a)(s − b)
S 6 a2 b2 c2
= = a2 b2 c2 .
S6

bc ca ab
Problem 38. Show also that + 2 + 2 = 1.
α21 β1 γ1
Solution.
A B C
bc ca ab bc sin2 ca sin2 ab sin2
+ 2 + 2 = 2 + 2 + 2
α21 β1 γ r12 r22 r2
{ 1 } 3
1 A B C
= 2 bc cos2 + ca cos2 + ab cos2 , by Art. 251 (page 344),
s 2 2 2
1 1
= 2 {s(s − a) + s(s − b) + s(s − c)} = (s − a + s − b + s − c)
s s
1
= (3s − a − b − c) = 1.
s

( ) ( ) ( )
1 1 1 1 1 1
Problem 39. Show also that α2 − + β2 − + γ2 − = 0.
c b a c b a
Solution. ( ) ( ) ( )
1 1 1 1 1 1
α2 − + β2 − + γ2 −
c b a c b a
r2 (b − c) r2 (c − a) r2 (a − b)
= + +
A B C
bc sin2 ca sin2 ab sin2
2 2 2 
 
r2 a(b − c) b(c − a) c(a − b)
= + +
abc  sin2 A sin2
B
sin2
C 

2 { 2 2 2 }
4Rr A B C
= (b − c) cot + (c − a) cot + (a − b) cot
abc 2 2 2
= 0, by Chapter xiii : P roblem 29 (page 107).

b−c c−a a−b


Problem 40. Show also that + + = 0.
aα21 bβ12 cγ12
b−c c−a a−b
Solution. + +
aα21 bβ12 cγ12
b−c A c−a B a−b C
= sin2 + sin2 + sin2
ar12 2 br22 2 cr32 2
{ }
1 b−c A c − a B a − b C
= 2 cos2 + cos2 + cos2 , by Art. 251 (page 344),
s {a 2 b 2 c 2 }
1 A B C
= 2
(b − c) cot + (c − a) cot + (a − b) cot
4Rs 2 2 2
= 0, by Chapter xiii : P roblem 29 (page 107).

Problem 41. There is only one point within a triangle, such that if perpendiculars
be drawn from it to the sides, circles can be inscribed in each of the three resulting
quadrilaterals : prove this, and if ρ1 , ρ2 , ρ3 be the radii of those circles, and ρ that of
CHAPTER XVI : Properties of Triangles 150

the inscribed
( circle
) ( of the) triangle,
( then)( ) ( )( )
1 1 1 1 1 1 1 1 1 1 1 1 1
− − + − − + − − = 2.
ρ1 ρ ρ2 ρ ρ2 ρ ρ3 ρ ρ3 ρ ρ1 ρ ρ
Solution. In order that it may be possible to inscribe a circle within a quadrilateral
the sum of one pair of opposite sides must be equal to the sum of the other pair. Now
if we take the point O of the diagram of Art. 248 (page 342), we see that the condition
is satisfied for OF AE, OECD, and ODBF ; since OE + AF = OF + AE, and so on. We
have then to show that no other point but O can be taken.
Take any other point P ; from it draw P M perpendicular to AC and P N perpendic-
ular to AB. The centre of a circle inscribed within P M AN must be on the straight
line which bisects the angle A; and also on the straight line which bisects the angle
N P M ; but unless P is on AO, the latter straight line will be parallel to AO, the former
straight line, and therefore cannot meet it. Thus P must be on AO; similarly it must
be on BO and on CO.
Then take the circle inscribed in OF AE, and draw perpendiculars from the centre
on the sides of the quadrilateral. Thus we have
ρ1 (AF + F O + OE + EA) = twice the area of OF AE;
{ }
A A
∴ ρ1 ρ + ρ cot = ρ2 cot ;
2 2
A A
ρ cot 1 + cot
∴ ρ1 = 2 ; ∴ 1 = 2 .
A ρ1 A
1 + cot ρ cot
2 2
B
1 1 + cot
Similarly = 2 .
ρ2 B
ρ cot
( )( ) 2
1 1 1 1 1 1 A B
Thus − − = = 2 tan tan .
ρ1 ρ ρ2 ρ 2
A B ρ 2 2
ρ cot cot
2 2
In this manner we find that the proposed expression
{ }
1 A B B C C A
= 2 tan tan + tan tan + tan tan
ρ 2 2 2 2 2 2
1
= 2 , by Chapter viii : P roblem 25.
ρ

Problem 42. A circle is inscribed in a plane triangle ABC . Another circle is in-
scribed so as to touch the two sides AB, AC , and the last circle; again, a third circle
is inscribed so as to touch the same two sides AB, AC , and the second circle, and so
on. Circles are also inscribed in the same way so as to touch BC , BA and CA, CB.
Show that the area of the inscribed circle is to the sum of the areas of all the other
circles as 1 is to
B+C A C +A B A+B C
sin4 cosec + sin4 cosec + sin4 cosec .
4 2 4 2 4 2
Solution. As in P roblem 24 (page 144) we shall find that the radii of the circles
successively inscribed in the angle A are lr, l2 r, l3 r, . . . where
A
1 − sin
l= 2 .
A
1 + sin
2
Hence the sum of the (areas of all these circles ) is
2 2 4 2 6 2
= π l r + l r + l r + ... ;
( )2
A
π 1 − sin r2
πl2 r2 2
= = ,
1 − l2 A
4 sin
2
CHAPTER XVI : Properties of Triangles 151

( )
B+C 2 2
π 1 − cos r
2 B+C A
= = πr 2 sin4 cosec .
A 4 2
4 sin
2
Similarly we find the areas of the circles inscribed within the angles B and C. Thus
{ the areas is
the sum of all }
B+C A C+A B A+B C
πr 2 sin4 cosec + sin4 cosec + sin4 cosec .
4 2 4 2 4 2

Problem 43. O and O ′ are respectively the centres of the circles described about
and inscribed in a plane triangle ABC . Join OA, OB, OC , O ′ A, O ′ B, O ′ C , and let Ra ,
Rb , Rc , ra , rb , rc , be respectively the radii of the circles circumscribing the triangles
BOC , COA, AOB, BO ′ C , CO ′ A, AO ′ B. If R be the radius of the circle circumscribing
the given triangle ABC , show that
ra rb rc R a b c abc
= , and + + = 3.
abc a+b+c Ra Rb Rc R
BC a
Solution. Ra = = ;
2 sin BOC 2 sin 2A
b c
Similarly Rb = , and Rc = .
2 sin 2B 2 sin 2C
a b c
Thus + + = 2(sin 2A + sin 2B + sin 2C)
Ra Rb Rc
= 8 sin A sin B sin C, by Art. 114 (page 328),
a b c abc
= × × = 3.
R R R R
Again,
BC a A a
ra = = ( )= = ;
2 sin BO′ C B C B+C A
2 sin π − − 2 sin 2 cos
2 2 2 2
b c
Similarly, rb = , and rc = .
B C
2 cos 2 cos
2 2
ra rb rc 1
∴ =
abc A B C
8 cos cos cos
2 2 2
1
= , by Chapter viii : P roblem 16,
2(sin A + sin B + sin C)
R R
= = .
2R sin A + 2R sin B + 2R sin C a+b+c

Problem 44. From any point P within or without a triangle ABC , perpendiculars
PA′ , PB ′ , PC ′ are dropped on the sides BC , CA, AB; and circles are described about
the triangles PA′ B ′ , PB ′ C , PC ′ A′ . Show that the area of the triangle formed by joining
the centres of these circles is one-fourth of the area of the triangle ABC .
Solution. Since the angles at B ′ and C ′ are right angles it will follow that A will be
on the circumference of the circle which is described round PB ′ C ′ , and that PA is a
1
diameter of the circle. Let O1 denote the centre of the circle, then P O1 = P A.
2
In a similar manner if O2 is the centre of the circle round P C ′ A′ , and O3 the centre
of the circle round P A′ B ′ , we have
1 1
P O2 = P B, and P O3 = P C.
2 2
Then is the triangle P O2 O3 we have
O2 O32 = P O22 + P O32 − 2P O2 P O3 cos O2 P O3 ;
and in the triangle P BC we have
BC 2 = P B 2 + P C 2 − 2P B · P C cos BP C.
1
Hence O2 O3 = BC. Or this might be obtained by Euclid vi. 2, and vi. 4.
2
CHAPTER XVI : Properties of Triangles 152

1 1
Similarly O3 O1 = CA, and O1 O2 = AB. Thus the area of O1 O2 O3 is one-fourth
2 2
of the area of ABC.

Problem 45. Three circles touch each other externally. Prove that the square of the
area of the triangle formed by joining their centres is equal to the product of the sum
and product of their radii.
Solution. Let r1 , r2 , r3 denote the radii of the circles; then the sides of the triangle
are respectively r2 + r3 , r3 + r1 , and r1 + r2 . Thus
s = r1 + r2 + r3 , s − a = r1 , s − b = r2 , s − c = r3 .
∴ S 2 = (r1 + r2 + r3 ) r1 r2 r3 .

Problem 46. If the sides of a triangle be in geometrical progression, and the perpen-
diculars from the angles on the opposite sides be taken as the sides of a new triangle,
then the angles of this new triangle will be equal to those of the original triangle.
Solution. Suppose a, b, c in Geometrical Progression, so that b2 = ac; let p1 , p2 , p3
denote the perpendiculars from the opposite angles on a, b, c respectively.
1 2S 2S 2S
Then p1 a = S, so that p1 = ; similarly p2 = , and p3 = .
2 a b c
Let A1 , B1 , C1 be the angles opposite p1 , p2 , p3 respectively in the new triangle.
1 1 1 b2 + c2 1
p22 + p23 − p21 2
+ 2 − 2 2 c2
− 2
Then cos A1 = = b c a = b a
2p2 p3 2 2
bc bc
a2 (b2 + c2 ) − b2 c2 b2 (a2 − c2 ) + a2 c2
= 2
=
2a bc 2a2 bc
a2 − c2 + ac a2 + b2 − c2
= = = cos C.
2ab 2ab
Thus A1 = C. Similarly C1 = A. Therefore B1 = B.

Problem 47. If α, β, γ be the ratios which the sides a, b, c of a triangle bear to the
perpendiculars on them from the opposite angles A, B, C then
α2 + β 2 + γ 2 − 2 (αβ + βγ + γα) + 4 = 0 .

a sin A
Here α= = .
c sin B sin B sin C
sin B sin C
Similarly β= , and γ = .
sin C sin A sin A sin B
1
Therefore 2(βγ + γα + αβ) − α2 − β 2 − γ 2 = the product of into
{ sin }
2 A sin2 B sin2 C
2 sin B sin C + 2 sin C sin A + 2 sin A sin B − sin A − sin B − sin C .
2 2 2 2 2 2 4 4 4

The expression within brackets is equal to


(sin A + sin B + sin C)(sin A + sin B − sin C)
(sin A − sin B + sin C)(sin B + sin C − sin A),
as we know from a similar process in Art. 218 (page 338).
Then, by Chapter viii : P roblems 16 and 17, we obtain
A B C A B C
44 sin2 sin2 sin2 cos2 cos2 cos2 = 4 sin2 A sin2 B sin2 C.
2 2 2 2 2 2
Hence 2(βγ + γα + αβ) − α2 − β 2 − γ 2 = 4,
and therefore α2 + β 2 + γ 2 − 2(βγ + γα + αβ) + 4 = 0.

Problem 48. On the sides of any triangle equilateral triangles are described exter-
nally, and their centres are joined. Show that the triangle thus formed is equilateral.
Solution. Let P, Q, R be the centres of the equilateral triangles described on
BC, CA, AB respectively.
Then P Q2 = P C 2 + QC 2 − 2P C · QC cos P CQ;
CHAPTER XVI : Properties of Triangles 153

a b
also P C = √ , and QC = √ .
3 3
Thus 3P Q2 = a2 + b2 − 2ab cos (C + 60◦ )
= a2 + b2 − 2ab (cos C cos 60◦ − sin C sin 60◦ )

= a2 + b2 − ab cos C + ab sin C 3
a2 + b2 − c2 √
= a2 + b2 − + ab sin C 3
2
a2 + b2 + c2 √
= + 2S 3.
2
We shall obtain the same symmetrical expression for 3QR2 and 3RP 2 .
Thus P Q = QR = RP.

Problem 49. The sides of a triangle are 65 and 25 , and the difference of the opposite
angles is 60 ◦ . Find all the angles, having given
log 3 = .4771213 , log 2 = .3010300 ,
L tan 52 ◦ 24 ′ = 10 .1134508 , L tan 52 ◦ 25 ′ = 10 .1137122 .
Solution.
B−C b−c A
We have tan = cot ;
2 b+c 2 √
A 65 + 25 9 1 3 3
∴ cot = tan 30◦ = · √ = ;
2 65 − 25 4 3 4
A 3
∴ L cot = 10 + log 3 − 2 log 2 = 10.1136219.
2 2
10.1137122 10.1136219
10.1134508 10.1134508

.0002614 .0001711 .0002614 : .0001711 :: 60′′ : x′′ ;


A
This gives x = 39; ∴ = 37◦ 36′ − 39′′ = 37◦ 35′ 21′′ . Therefore A = 75◦ 10′ 42′′ . Thus
2
B + C = 180 − 75 10 42 ; and B − C = 60◦ . Therefore B = 82◦ 24′ 39′′ and C = 22◦ 24′ 39′′ .
◦ ◦ ′ ′′

Problem 50. If perpendiculars be drawn from the angles of an acute-angled triangle


to the opposite sides, show that the sides of the triangle formed by joining the feet of
those perpendiculars are a cos A, b cos B, and c cos C ; and thence show that
a 2 cos2 A − b2 cos2 B − c 2 cos2 C
= cos 2A.
2bc cos B cos C
Solution. In the solution of P roblem 26 (page 145), it is shown that the sides of the
new triangle are a cos A, b cos B, and c cos C respectively.
In the solution of P roblem 27 (page 145), it is shown that the angles of the new
triangle are π − 2A, π − 2B, and π − 2C respectively. Then, by Art. 215 (page 336),
b2 cos2 B + c2 cos2 C − a2 cos2 A
cos(π − 2A) = ;
2bc cos B cos C
but cos(π − 2A) = − cos 2A. Therefore
a2 cos2 A − b2 cos2 B − c2 cos2 C
cos 2A = .
2bc cos B cos C

Problem 51. Six circles are inscribed between the three escribed circles of a trian-
gle and the angular points, each touching a side and a side produced. Show that the
products of their radii taken alternately are equal.
Solution. Let ρ1 denote the radius of the circle which touches BD, BF and the arc
DF in the diagram of Art. 250 (page 344). Let ρ2 denote the radius of the circle which
touches CD, CE, and the arc DE.
The angle DBF = π − B. Hence, by the method of P roblem 24 (page 144), we have
π−B B
1 − sin 1 − cos
ρ1 = r1 2 = r1 2 = r tan2 B .
1
π−B B 4
1 + sin 1 + cos
2 2
CHAPTER XVI : Properties of Triangles 154

C
Similarly ρ2 = r1 tan2 .
4
In this was we see that the product of three of the radii
B C A
= r1 tan2 × r2 tan2 × r3 tan2 ;
4 4 4
and the product of the other three
C A B
= r1 tan2 × r2 tan2 × r3 tan2 .
4 4 4
The two products are equal.

Problem 52. Straight lines are drawn from the angles A, B, C of a triangle through
any point P meeting the opposite sides of the triangle at the points A′ , B ′ , C ′ respec-
tively. Show that
AB ′ · BC ′ · CA′ = AC ′ · BA′ · CB ′ .
Solution.
AB ′ sin AP B ′ AP sin AP B ′
= ′
; ∴ AB ′ = .
AP sin AB P sin AB ′ P
BP sin BP C ′ CP sin CP A′
Similarly BC ′ = ′
, and CA′ = .
sin BC P sin CA′ P
AP · BP · CP sin AP B sin BP C sin CP A′
′ ′
Thus AB ′ · BC ′ · CA′ = .
sin AB ′ P · sin BC ′ P · sin CA′ P
In like manner
AP · BP · CP sin AP C ′ sin BP A′ sin CP B ′
AC ′ · BA′ · CB ′ = .
sin AC ′ P · sin BA′ P · sin CB ′ P
The two expressions are obviously equal; for sin AP B ′ = sin BP A′ , sin BP C ′ = sin B ′ P C,
and sin CP A′ = sin C ′ P A. Also, sin AB ′ P = sin CB ′ P, and so on.

Problem 53. Show conversely that if the relation just expressed holds then the
straight lines AA′ , BB ′ , CC ′ meet at a point.
Solution. Let P denote the intersection of AA′ and BB ′ ; then, if CC ′ does not pass
through P, let a straight line be drawn from C through P, and let it meet AB at C1 .
Then, by the Example, we have
AB ′ · BC1 · CA′ = AC1 · BA′ · CB ′ .
But by hypothesis,
AB ′ · BC ′ · CA′ = AC ′ · BA′ · CB ′ .
BC1 AC1
∴ = ;
BC ′ AC ′

BC − C1 C ′ AC + C1 C ′

∴ ′
= ,
BC AC ′
C1 C ′ C1 C ′
∴ − = ;
BC ′ AC ′

∴ C1 C = 0;
Therefore C1 must coincide with C ′ .

Problem 54. Show that the perpendiculars from the angles of a triangle on the
opposite sides meet at a point.
Solution. Let the feet of the perpendiculars from A, B, C be denoted by A′ , B ′ , C ′
respectively. If all the angles are acute, we have
AB ′ = c cos A, BC ′ = a cos B, CA′ = b cos C,
AC ′ = b cos A, BA′ = c cos B, CB ′ = a cos C;
thus AB ′ · BC ′ · CA′ = AC ′ · BA′ · CB ′ .
Therefore, by P roblem 53 (page 154), the straight lines AA′ , BB ′ and CC ′ meet at a
point.
Next suppose one angle obtuse, say C. Then
CA′ = b cos (180◦ − C) , and CB ′ = a cos (180◦ − C) ;
the other expressions remain as before, and the result holds as before.
CHAPTER XVI : Properties of Triangles 155

Problem 55. Show that the straight lines which bisect the internal angles of a
triangle meet at a point.
Solution. Let the straight lines which bisect the angles A, B, C respectively meet
the opposite sides at A′ , B ′ , C ′ respectively. Then
B C A
AB ′ sin ′ sin ′ sin
= 2 , BC = 2 , CA = 2 ;
BB ′ sin A CC ′ sin B AA′ sin C
A B C
sin sin sin
∴ AB ′ · BC ′ · CA′ = AA′ · BB ′ · CC ′ 2 2 2 ;
sin A sin B sin C

the same value may be obtained for AC · BA · CB . ′ ′

Therefore, by P roblem 53 (page 154), the straight lines AA′ , BB ′ , and CC ′ meet at
a point.

Problem 56. Show that the straight lines which join the angles of a triangle with
the middle points of the opposite sides meet at a point.
Solution. Let A′ , B ′ , C ′ denote the middle points of BC, CA, AB respectively. Then
b c a abc
AB ′ · BC ′ · CA′ = × × = .
2 2 2 8
abc
Similarly AC ′ · BA′ · CB ′ = .
8
Therefore, by P roblem 53 (page 154), the straight lines AA′ , BB ′ , and CC ′ meet at a
point.

Problem 57. Show that the straight lines which join the angles of a triangle with
the points where the inscribed circle touches the opposite sides respectively, meet at
a point.
Solution. Let the points of contact opposite to A, B, C respectively be denoted by
A′ , B ′ , C ′ respectively.
A B C
Then AB ′ = r cot , BC ′ = r cot , CA′ = r cot .
2 2 2
′ ′ ′ A B C
Thus AB · BC · CA = r cot cot
3
cot .
2 2 2
′ ′ ′ A B C
Similarly AC · BA · CB = r cot cot
3
cot .
2 2 2
Therefore, by P roblem 53 (page 154), the straight lines AA′ , BB ′ and CC ′ meet at a
point.

Problem 58. Let a straight line be drawn from the angle A of a triangle to the
point where the escribed circle opposite to the angle A touches the side opposite to
it; let similar straight lines be drawn from B and C with respect to the other escribed
circles. Show that these straight lines meet at a point.
Solution. Let the points of contact opposite to A, B, C respectively be denoted by
A′ , B ′ , C ′ respectively.
π−A A
Then AB ′ = r2 cot = r2 tan ,
2 2
B
BC ′ = r3 tan ,
2
C
CA′ = r1 tan ;
2
A B C
∴ AB ′ · BC ′ · CA′ = r1 r2 r3 tan tan tan .
2 2 2
A B C
Similarly AC ′ · BA′ · CB ′ = r1 r2 r3 tan tan tan .
2 2 2
Therefore, by P roblem 53 (page 154), the straight lines AA′ , BB ′ and CC ′ meet at a
point.

Problem 59. In the figure of Art. 250 (page 344) show that the straight lines BE, CF
CHAPTER XVI : Properties of Triangles 156

and AD meet at a point.


Solution.
Here AE = AF, CE = CD, BD = BF ;
∴ AE · BF · CD = AF · BD · CE.
Therefore, by P roblem 53 (page 154), the straight lines AD, BE and CF meet at a
point.

Problem 60. A quadrilateral figure is so taken that a circle can be described about
it and inscribed in it. It its sides be produced in both directions, and ra , rb , rc , rd be
the radii of the circles, inscribed in the triangles formed on two sides, and escribed
on the other two sides, then ra rb rc rd = r 4 , where r is the radius of the circle inscribed
in the quadrilateral.
Solution. Let ABCD be the quadrilateral figure. Then, denoting by A, B, C and D
the internal angles of the figure, we have
( )
π−B π−C
ra cot + cot = BC;
(
2 2 )
B C
∴ ra tan + tan = BC;
2 2
Again in like manner we have
( )
A D
r cot + cot = DA,
( 2 2)
C B
∴ r tan + tan = DA,
2 2
for A + C = π, and B + D = π, by Euclid iii. 22.
ra BC
Hence = .
r DA
In the same manner we can show that
rb CD rc AD rd AB
= , = , and = .
r AB r BC r DC
ra rc rb rd
∴ = 1, and 2 = 1;
r2 r
∴ ra rb rc rd = r4 .
CHAPTER XVII
Use of Subsidiary Angles in solving Equations and in
adapting Formulae to Logarithmic Computation

Problem 1. Solve x 3 − 6x + 4 = 0 .
Solution. Here r = −4, q = 6; therefore
( ) 23 ( ) 23 ( )1
3 1 16 1 2 2 1
cos 3α = −16 = −16 =− = − √ = −√ .
24 8 8 8 2 2 2
3π π
Therefore 3α = . Hence α = . Therefore the roots are
4 4
( ) 21 ( ) 12 ( )
6 π 6 2π π
2 cos , and 2 cos ± .
3 4 3 3 4
( ) 12 √ 1
6 π
Now 2 cos = 2 2 √ = 2;
3 4 2
( ) 12 ( ) √ √ √
6 2π π 11π π
2 cos + = 2 2 cos = −2 2 cos = −( 3 + 1);
3 3 4 12 12
( ) 12 ( ) √ √
6 2π π 5π
2 cos − = 2 2 cos = 3 − 1.
3 3 4 12

Problem 2. Show that the roots of the equation x 3 − 3x − 1 = 0 are 2 cos 20 ◦ , −2 sin 10 ◦ ,
−2 cos 40 ◦ .
( ) 32 ( ) 32
3 1 1
Solution. Here r = 1, q = 3; therefore cos 3α = 4 =4 = ;
12 4 2
therefore 3α = 60◦ ; therefore α = 20◦ .
Therefore the roots are 2 cos 20◦ , and 2 cos(120◦ ± 20◦ ).
Also 2 cos(120◦ + 20◦ ) = 2 cos 140◦ = −2 cos 40◦ .
And 2 cos(120◦ − 20◦ ) = 2 cos 100◦ = −2 sin 10◦ .

Problem 3. Show that the roots of the equation x 5 − px 3 + qx − r = 0 are


( ) 12 ( ) 12 ( ) ( ) 12 ( )
p p 2π p 4π
2 cos α, 2 cos ± α , and 2 cos ±α ,
5 5 5 5 5
( )5
r 5 2
where cos 5α = ,
2 p
( )2 ( )5
r p
provided p2 = 5q and be not greater than .
2 5
Solution. Take the equation x5 − px3 + qx − r = 0.
Put x = ny; thus n5 y 5 − pn3 y 3 + qny − r = 0;
p q r
therefore y5 − 2 y3 + 4 y = 5 .
n n n
Now by Chapter viii : P roblem 59, cos 5α = 16 cos5 α − 20 cos3 α + 5 cos α.
5 5 cos 5α
Thus cos5 α − cos3 α + cos α = .
4 16 16
5 p 5 q r cos 5α
Assume y = cos α, = 2, = 4 , then 5 = .
( )2 4
2
n 16 n n 16
5 p 5 q 16p2 16q
Here = 4 , and = 4 ; so that we have n4 = , and n4 = .
4 n 16 n 25 5
2
Thus the process will not be admissible unless p = 5q; and this condition is satisfied
by hypothesis.
( ) 21
16r 4p
Then α must be found from cos 5α = 5 ; put for n its value : thus cos 5α =
n 5
CHAPTER XVII : Subsidiary Angles and Logarithmic Computation 158

( ) 25 ( ) 52
5 r 5
16r × = . Thus process then will not be admissible if this expression
4p 2 p
( )2 ( )5
r 5
is numerically greater than unity. Hence must not be greater than unity;
2 p
( )2 ( )
r p 5
that is must not be greater than .
2 5
Suppose this condition also to hold; then one root is n cos α,
( ) 21
p
that is 2 cos α.
5 ( ) ( )
2π 4π
Moreover we might also suppose y = cos ± α or y = cos ± α , and we
5 5
shall still arrive at the(same value
) for cos 5α, since ( )
2π 4π
cos 5 ± α = cos α and cos 5 ± α = cos 5α.
5 5
Hence we see that the other roots of the equation are
( ) 21 ( ) ( ) 12 ( )
p 2π p 4π
2 cos ± α and 2 cos ±α .
5 5 5 5

Problem 4. Find the roots of the equation


x5 − 10x3 + 20x − 8 = 0.
Solution. Proceed as in P roblem 3.
Here r = 8, q = 20, p = 10.
( ) 25
1 1
cos 5α = 4 = √ ; therefore 5α = 45◦ .
2 2
Hence the roots are √ √ √
2 2 cos 9◦ , 2 2 cos(72◦ ± 9◦ ), 2 2 cos(144◦ ± 9◦ ),
that is √ √ √ √ √
2 2 cos 9◦ , 2 2 cos 63◦ , 2 2 cos 81◦ , 2 2 cos 153◦ , 2 2 cos 135◦ ;
the last is equal to −2.

Problem 5. A person wishes to ascertain the side BC of a triangular field ABC , but is
only able to make measurement of lines within the boundary of a circle which passes
through A and touches BC : show how after measuring four straight lines he may
determine BC .
Solution. Let D denote the point of contact of the circle with BC. Let AC intersect
the circumference of the circle at E, and let AB intersect the circumference at F. Then
the four straight lines lines AE, ED, DF, F A can be measured. Then, by Art. 254
(page 346), the diagonal AD can be determined.
Then all the angles of the triangles ADE and ADF can be found; and thus the
angles of the triangles ADC and ADB are known. Thus DC and BD can be found.
See Euclid iii. 32.

Problem 6. Two men standing at the same point C observe the horizontal angle
subtended by two objects A and B; they then both move away, one in the direction
AC , the other in the direction BC , until each observes the horizontal angle to be half
what it was before. The distance each walked being given and the horizontal angle
at C , determine the distance AB.
Solution. Let D be the point on AC produced through C such that the angle ADB
is half the angle ACB; then CD = CB. Thus CB is known. Again, let E be the point
on BC produced through C such that the angle AEB is half the angle ACB; then
CE = CA. Thus CA is known. Then in the triangle ACB we know AC, and CB, and
the angle ACB; thus AB can be found by Art. 215 (page 336).

Problem 7. The altitude of a balloon is observed at three places A, B, C simultane-


ously to be 45 ◦ , 45 ◦ and 60 ◦ respectively; A and B are respectively West and North
of C : form an equation for determining the height of the balloon.
CHAPTER XVII : Subsidiary Angles and Logarithmic Computation 159

Solution. Let x denote the height of the balloon, and a, b, c the sides of the triangle
ABC. Let O be the plane of ABC which is vertically under the balloon. Then
x
AO = x cot 45◦ = x, BO = x cot 45◦ = x, CO = x cot 60◦ = √ . Therefore
3
x 2 x 2
b2 + − x2 3b2 − 2x2 a2 + − x2 3a2 − 2x2
cos ACO = 3 = √ , cos BCO = 3 = √ .
2bx 2bx 3 2ax 2ax 3
√ √
3 3
But ACB is a right angle, and therefore cos BCO = sin ACO; thus
( )2 ( )2
3b2 − 2x2 3a2 − 2x2
√ + √ = 1;
2bx 3 2ax 3
( )2 ( )2
therefore a2 3b2 − 2x2 + b2 3a2 − 2x2 = 12a2 b2 x2 ;
( 2 2) ( 2 2)
therefore 4x 4
a +b − 36a b x + 9a b
2 2 2 2 2
a +b = 0;
therefore 4c x − 36a b x + 9a b c = 0.
2 4 2 2 2 2 2 2

Problem 8. The distances b and c of a station A from two other stations B and C are
known, and the angle BAC is required. It not being practicable to observe the angle
BAC , the angle BOC (α) and the angle AOC (β) are observed at a position O situated
in the plane ABC , at a small known distance n from A, such that the triangle ABC is
entirely within the triangle OBC . Show that if θ be the circular measure of the angle
(BAC − BOC ) then approximately
{ }
sin(α − β) sin β
θ=n + .
b c
Solution. Here the angle BAC− the angle BOC = the sum of the angles ABO and
ACO.
sin ACO AO n sin β
Now = ; therefore sin ACO = , and since ACO is very small the
sin AOC AC c
n sin β
circular measure of it is nearly equal to the sine, so that it is nearly equal to .
c
sin ABO AO n sin(α − β)
Again = ; therefore sin ABO = , therefore the circular
sin AOB AB b
n sin(α − β)
measure of ABO is nearly equal to .
b { }
sin(α − β) sin β
Thus the circular measure of BAC − BOC is nearly n + .
b c

Problem 9. At a distance of 50 feet from the foot of a tower the elevation of its top
is 45 ◦ : if the elevation and the distance be correctly measured within 1 ′ and 1 inch
respectively, find approximately the greatest error in the height.
π π
Solution. If the distance is 50 feet and the elevation is , the height in feet is 50 tan ,
4 4
that is 50.
π
But suppose the distance to be 50 + h, and the elevation to be + α. Then the
( ) 4
π
height is (50 + h) tan + α . If α is very small this is very nearly equal to (50 +
( ) 4
π π
h) tan + α sec2 , by Art. 188 (page 334), that is (50 + h)(1 + 2α).
4 4
1
If h is also very small this is very nearly 50 + h + 100α. Now suppose h = and
12
π 1 π
α= ; then we obtain 50 + + . Thus the difference between this and the
180 × 60 12 108
1 π 1 1
former value is + , that is about + , that is, 1 13 inches.
12 108 12 36

Problem 10. A person standing at a distance a from a tower surmounted by a spire,


observes the tower and the spire to subtend the same angle : if b be the known height
CHAPTER XVII : Subsidiary Angles and Logarithmic Computation 160

of the tower, express the height of the spire (c) in terms of b and a.
If γ be the error in the height of the spire corresponding to a small error β in the
height of the tower, show that { }
γ β 4b2 a 2
= 1+ 4 .
c b a − b4
Solution. Suppose that the tower and the spire each subtend the angle α.
b b+c
Then tan α = , and tan 2α = .
a a
2b
b+c a 2ab
Therefore = = 2 ;
a b2 a − b2
1− 2
a
2a2 b 2a2 b (a2 + b2 )b
therefore b + c = 2 ; therefore c = −b= .
a −b 2 a −b
2 2 a2 − b2
If however the height of the tower is b + β, and the height of the spire is c + γ, we
have
a2 (b + β) + (b + β)3
c+γ = .
a2 − (b + β)2
Hence, by subtraction,
a2 (b + β) + (b + β)3 (a2 + b2 )b
γ= − .
a − (b + β)
2 2 a2 − b2
Now (b + β)2 = b2 + 2bβ + β 2 ,
and if β is very small this is very nearly b2 + 2bβ.
And (b + β)3 = b3 + 3b2 β + 3bβ 2 + β 3 ,
and if β is very small this is very nearly b3 + 3b2 β.
a2 b + b3 + (a2 + 3b2 )β a2 b + b3
Thus γ= − 2
a − b − 2bβ
2 2 a − b2
(a + 3b )(a − b ) + 2(a + b2 )b2
2 2 2 2 2
= β
(a2 − b2 − 2bβ)(a2 − b2 )
a + 4a b − b
4 2 2 4
= 2 β.
(a − b2 )(a2 − b2 − 2bβ)
γ (a4 + 4a2 b2 − b4 )β (a2 + b2 )b
Therefore = 2 ÷
c (a − b2 )(a2 − b2 − 2bβ) a2 − b2
β a4 + 4a2 b2 − b4
= · 2 .
b (a + b2 )(a2 − b2 − 2bβ)
But when β is very small we may put a − b for a − b2 − 2bβ; and thus
2 2 2

γ β a4 + 4a2 b2 − b4
= · .
c b (a4 − b4 )

Problem 11. The side a of a triangle and the opposite angle A remain constant : show
that the small variations of the other sides γ and β are connected by the relation
γ sec C + β sec B = 0 .
Solution. We have a2 = b2 + c2 − 2bc cos A;
suppose that b is changed to b + β, and c to c + γ; thus
a2 = (b + β)2 + (c + γ)2 − 2(b + β)(c + γ) cos A.
Therefore, by subtraction,
2bβ + β 2 + 2cγ + γ 2 − 2(bγ + cβ + βγ) cos A = 0.
If β and γ are very small this becomes very nearly
2bβ + 2cγ − 2(bγ + cβ) cos A = 0;
therefore β(b − c cos A) + γ(c − b cos A) = 0;
therefore βa cos C + γa cos B = 0, by Art. 216 (page 337).
β γ
Therefore + = 0,
cos B cos C
therefore β sec B + γ sec C = 0.
CHAPTER XVII : Subsidiary Angles and Logarithmic Computation 161

Problem 12. The angular altitude and breadth of a cylindrical tower on a level plane
are observed to be α and β respectively; and at a point a feet nearer the tower they
are observed to be α′ and β ′ : find the height and the radius of the tower. Find also
the relation existing between α, α′ , β, β ′ .
Solution. Suppose h the height of the tower, r the radius, x the distance of the first
place of observation from the centre. Then
x β x−a β′
= cosec , = cosec ;
r 2 r 2

h = x tan α, h = (x − a) tan α .
a β β′
Hence = cosec − cosec .
r 2 2
This finds r.
h tan α′
Also h = x tan α′ − a tan α′ = − a tan α′ ;
tan α
a tan α tan α ′
therefore h= .
tan α′ − tan α
This finds h.
Again, from the first and second equations,
β′
x−a cosec
= 2 .
x β
cosec
2
And from the third and fourth equations,
x−a cot α ′
= .
x cot α
β′
cosec ′
Therefore 2 = cot α .
β cot α
cosec
2

Problem 13. In the preceding Problem if the observed angular breadth be subject
to an error δ, and if ρ be the greatest consequent error in the calculated radius (r),
show that ρ will be given by the equation
2ρ 1( ′ ){ β β′ β β′
}
= cot β −β cosec cosec − cot cot δ.
r 4 2 2 2 2
◦ ′ ◦ ′
If β = 60 , β = 120 , δ = the circular measure of 6 , find approximately the ratio of
the greatest error in the calculated radius to the radius.
Solution.
a β β′
We have = cosec − cosec (20)
r 2 2
If we suppose an error δ of the same sign to be made in β and β ′ these errors will
tend to compensate each other; the greatest possible error in r will be determined
by supposing that errors of opposite signs are made in β and β ′ . Suppose then that
instead of β we ought to have β − δ, and instead of β ′ we ought to have β ′ + δ. Then
we have
a β−δ β′ + δ
= cosec − cosec .
r−ρ 2 2
a a aρ
Hence, by subtraction, − , that is
r−ρ r r(r − ρ)
{ }
β−δ β β′ + δ β′
= cosec − cosec − cosec − cosec .
2 2 2 2
Therefore, if δ and ρ be very small, we obtain
 
 cos β β′ 
aρ δ cos
= 2 + 2 ; see Art. 194 (page 335).
r2 2 2β 2 β
sin sin
2 2
CHAPTER XVII : Subsidiary Angles and Logarithmic Computation 162

( ) ( )
β β′ β′ β
cos 1 − cos2 1 − cos2
+ cos
aρ δ 2 2 2 2
Thus =
r2 2 2 β 2 β′
sin sin
( ) (2 2 )
β β′ β β′
cos + cos 1 − cos cos
δ 2 2 2 2
= ′
2 β β
sin2 sin2

2 (2 )
β+β β′ − β β β′
δ cos cos 1 − cos cos
4 4 2 2
= (21)
β β′
sin2 sin2
2 2
Now (20) may be put in the form
β′ β β′ − β β′ + β
a sin − sin 2 sin cos
= 2 2 = 4 4 (22)
r β′ β β′ β
sin sin sin sin
2 2 2 2
Divide (21) by (22) ; then
β β′
ρ δ 1 ′ 1 − cos cos
= cot (β − β) · 2 2
r 2 4 β′ β
sin sin
{ 2 2 }
δ 1 β′ β β′ β
= cot (β ′ − β) cosec cosec − cot cot .
2 4 2 2 2 2

If β = 60◦ and β ′ = 120◦ , we obtain for the value
r
cot 15 {cosec 30 cosec 60 − cot 30◦ cot 60◦ }δ,
◦ ◦ ◦

√ ( 4 ) √
5+2 3
that is (2 + 3) √ − 1 δ, that is √ δ.
3 3
π
Put for δ the circular measure of 6′ , that is .
√ 1800 √
2ρ 5+2 3 π ρ 5+2 3 π
Hence = √ × ; therefore = √ × .
r 3 1800 r 3 3600

Problem 14. P, Q, R are three known positions in a straight line, and PQ, QR are
observed to subtend equal angles at a certain point S : find the error in the calculated
distance of S from Q in consequence of a small error α in the observed angles.
Solution. Let β denote the angle P SQ, and the equal angle QSR; and let ϕ denote
the angle SQR.
PQ sin P SQ sin β
Then = = .
SQ sin SP Q sin(ϕ − β)
QR sin QSR sin β
and = = ,
SQ sin SRQ sin(ϕ + β)
PQ sin(ϕ + β)
therefore = .
QR sin(ϕ − β)
Let P Q = a, and QR = b; thus
a sin(ϕ − β) = b sin(ϕ + β),
therefore a(sin ϕ cos β − cos ϕ sin β) = b(sin ϕ cos β + cos ϕ sin β);
(a + b) sin β a+b
therefore tan ϕ = = tan β.
(a − b) cos β a−b
1 sin β 1 sin β
Also = , and = ;
SQ a sin(ϕ − β) SQ b sin(ϕ + β)
1 2
sin β 2
sin β
∴ = =
SQ2 ab sin(ϕ − β) sin(ϕ + β) ab(sin2 ϕ − sin2 β)
CHAPTER XVII : Subsidiary Angles and Logarithmic Computation 163

tan2 β
=
ab{sin2 ϕ(1 + tan2 β) − tan2 β}
(a + b)2 tan2 β
But sin2 ϕ = ,
(a − b)2 + (a + b)2 tan2 β
1 (a − b)2 + (a + b)2 tan2 β (a − b)2 + (a + b)2 tan2 β
thus = = .
SQ2 ab{(a + b)2 − (a − b)2 } 4a2 b2
Suppose that instead of β we ought to have β + α, and instead of SQ we ought to
have SQ + c, where α and c are very small. Then
1 (a − b)2 (a + b)2
= + 2 tan2 (β + α).
(SQ + c)2 4a2 b2 4a + b2
Hence, by subtraction,
1 1 (a + b)2
− = {tan2 (β + α) − tan2 β};
(SQ + c)2 SQ2 4a2 b2
SQ2 − (SQ + c)2 (a + b)2
therefore 2 2
= {(tan β + a sec2 β)2 − tan2 β},
SQ (SQ + c) 4a2 b2
approximately, by Art. 188 (page 334)
2c (a + b)2
Thus − 3
= 2 tan β sec2 βα, nearly;
SQ 4a2 b2
c (a + b)2 sin β
∴ =− 2 2 α, nearly.
SQ3 4a b cos3 β
CHAPTER XVIII
Inverse Trigonometrical Functions

3 1
Problem 1. Show that tan−1 = 2 tan−1 .
4 3
1 1
Solution. Let tan−1 = θ, then tan θ = ; therefore
3 3
2
3 6 3
tan 2θ = = = ;
1 8 4
1−
9
3 3 1
therefore 2θ = tan−1 . Therefore tan−1 = 2 tan−1 .
4 4 3

( )
1 1
Problem 2. Find the value of sin sin−1 + cos−1 .
2 2
Solution.
1 1
Let sin−1 = θ, and cos−1 = ϕ;
2 2 √
1 3
therefore sin θ = , and cos θ = ,
2 √ 2
1 3
and cos ϕ = , and sin ϕ = .
2 2
1 3
Therefore sin(θ + ϕ) = sin θ cos ϕ + cos θ sin ϕ = + = 1.
4 4

77 3 8
Problem 3. Show that sin−1 = sin−1 + sin−1 .
85 5 17
3 8
Solution. Let sin−1 = α, and sin−1 = β;
5 17 √( )
3 9 4
then sin α = , cos α = 1− = ;
5 25 5
√( )
8 64 15
and sin β = , cos β = 1− = ;
17 289 17

therefore sin(α + β) = sin α cos β + cos α sin β


3 × 15 4×8 45 + 32 77
= + = = ;
5 × 17 5 × 17 85 85
77
therefore α + β = sin−1 .
85

( )
Problem 4. Find the value of tan tan−1 x + cot−1 x .
Solution. Let α = tan−1 x, and β = cot−1 x;
1
then tan α = x, and cot β = x; therefore tan β =
,
x
1 1
tan α + tan β x+ x+
and tan(α + β) = = x = x.
1 − tan α tan β 1−1 0
Thus tan(α + β) is infinite.

1 1 1 1 π
Problem 5. Show that tan−1 + tan−1 + tan−1 + tan−1 = .
3 5 7 8 4
CHAPTER XVIII : Inverse Trigonometrical Functions 166

1 1 1 1
Solution. Let tan−1 = α, tan−1 = β, tan−1 = γ, tan−1 = δ.
3 5 7 8
1 1
Thus tan α = , and tan β = ;
3 5
1 1
tan α + tan β + 8 4
therefore tan(α + β) = = 3 5 = = .
1 − tan α tan β 1 1 14 7
1− ×
3 5
1 1
tan γ + tan δ + 15 3
And tan(γ + δ) = = 7 8 = = .
1 − tan γ tan δ 1 1 55 11
1− ×
7 8
Then
4 3
tan(α + β) + tan(γ + δ) + 65
tan(α + β + γ + δ) = = 7 11 = = 1;
1 − tan(α + β) tan(γ + δ) 4 3 65
1− ×
7 11
π
therefore α+β+γ+δ = .
4

a−b b−c
Problem 6. Show that tan−1 a = tan−1 + tan−1 + tan−1 c.
1 + ab 1 + bc
Solution.
Let tan−1 a = θ, and tan−1 b = ϕ;
then a = tan θ, and b = tan ϕ;
tan θ − tan ϕ a−b
and tan(θ − ϕ) = = .
1 + tan θ tan ϕ 1 + ab
a−b
Thus tan−1 = tan−1 a − tan−1 b.
1 + ab
b−c
Similarly tan−1 = tan−1 b − tan−1 c.
1 + bc
a−b b−c
Therefore tan−1 + tan−1 = tan−1 a − tan−1 c,
1 + ab 1 + bc
a−b b−c
and tan−1 + tan−1 + tan−1 c = tan−1 a.
1 + ab 1 + bc

1 1 1 π
Problem 7. Find the tangent of 3 tan−1 + tan−1 + tan−1 − .
7 3 26 4
Solution.
1 1 1
Let α = tan−1 , β = tan−1 , γ = tan−1 .
7 3 26
3 1
3 tan α − tan3 α − 3 146 73
tan 3α = = 7 7 = = .
1 − 3 tan2 α 3 322 161
1− 2
7
1 1
tan β + tan γ + 29
tan(β + γ) = = 3 26 = .
1 − tan β tan γ 1 77
1−
3 × 26
73 29
tan 3α + tan(β + γ) +
tan(3α + β + γ) = = 161 77
1 − tan 3α tan(β + γ) 73 29
1− ×
161 77
10290 1029
= = .
10280 1028
1029
( ) −1
π tan(3α + β + γ) − 1 1028 1
tan 3α + β + γ − = = = .
4 1 + tan(3α + β + γ) 1029 2057
1+
1028
CHAPTER XVIII : Inverse Trigonometrical Functions 167

Problem 8. Show{(√that ) } {(√ ) }


tan−1 2 + 1 tan α − tan−1 2 − 1 tan α = tan−1 (sin 2 α) .
Solution. √We see as in the solution√ of P roblem 6 that
tan−1 {( 2 + 1) tan α} − tan−1 {( 2 − 1) tan α}
√ √
( 2 + 1) tan α − ( 2 − 1) tan α
= tan−1 √ √
1 + ( 2 + 1)( 2 − 1) tan2 α
2 tan α
= tan−1 = tan−1 (sin 2α).
1 + tan2 α

Problem 9. If tan(θ − α) tan(θ − β) = tan2 θ; then


1 2 sin α sin β
θ = tan−1 .
2 sin(α + β)
Solution. tan(θ − α) tan(θ − β) = tan θ;
2

sin(θ − α) sin(θ − β) 1 − cos 2θ


therefore = ;
cos(θ − α) cos(θ − β) 1 + cos 2θ
cos(α − β) − cos(2θ − α − β) 1 − cos 2θ
therefore = ;
cos(α − β) + cos(2θ − α − β) 1 + cos 2θ
therefore cos(α − β) cos 2θ = cos(2θ − α − β)
= cos 2θ cos(α + β) + sin 2θ sin(α + β);
therefore tan 2θ sin(α + β) = cos(α − β) − cos(α + β) = 2 sin α sin β;
2 sin α sin β
therefore tan 2θ = ;
sin(α + β)
2 sin α sin β
therefore 2θ = tan−1 .
sin(α + β)


9 41 π
Problem 10. Show that cos−1 √ + cosec −1 = .
82 4√ 4
9 (41)
Solution. Let α = cos−1 √ , and β = cosec −1 ;
(82) 4
9 1
then cos α = √ , and sin α = √ ;
(82) (82)
4 5
sin β = √ , cos β = √ .
(41) (41)
Therefore cos(α + β) = cos α cos β − sin α sin β
45 − 4 41 1
= √ √ = √ = √ .
(82) × (41) 41 2 2
π
Therefore α+β = .
4

4 5 16 π
Problem 11. Show that sin−1 + sin−1 + sin−1 = .
5 13 65 2
4 5 16
Solution. Let α = sin−1 , β = sin−1 , γ = sin−1 ;
5 13 65
4 5 16
then sin α = , sin β = , sin γ = ,
5 13 65
3 12 63
and cos α = , cos β = , cos γ = .
5 13 65
48 + 15 63
Then sin(α + β) = sin α cos β + cos α + cos α sin β = = ;
65 65
π
thus sin(α + β) = cos γ, so that α + β + γ = .
2
CHAPTER XVIII : Inverse Trigonometrical Functions 168

1 1 π 1
Problem 12. Show that 3 tan−1 + tan−1 = − tan−1 .
4 20 4 1985
1 1
Solution. Let α = tan−1 , and β = tan−1 .
4 20
3 1
3 tan α − tan3 α − 3
tan 3α = = 4 4 = 47 .
1 − 3 tan2 α 3 52
1− 2
4
47 1
tan 3α + tan β + 992
tan(3α + β) = = 52 20 = .
1 − tan 3α tan β 47 993
1−
52 × 20
1
Again, let γ = tan−1 ; then
1985
( ) 1− 1
π 1985 = 1984 = 992 .
tan −γ =
4 1 1986 993
1+
1985
π
Therefore 3α + β = − γ.
4

2a − b 2b − a π
Problem 13. Show that tan−1 √ + tan−1 √ = .
b 3 a 3 3
2a − b 2b − a
Solution. Let θ = tan−1 √ , ϕ = tan−1 √ ;
b 3 a 3
2a − b 2b − a
√ + √
b 3 a 3 a(2a − b) + b(2b − a) √
then tan(θ + ϕ) = = 3
(2a − b)(2b − a) 3ab − (2a − b)(2b − a)
1−
3ab
2(a2 + b2 ) − 2ab √ √
= 3 = 3.
2(a2 + b2 ) − 2ab
π
Thus θ+ϕ= .
3

( ) ( )
Problem 14. Show that tan 2 tan−1 a = 2 tan tan−1 a + tan−1 a 3 .
Solution. Let tan−1 a = θ, and tan−1 a3 = ϕ.
2a
Then tan θ = a, and tan 2θ = .
1 − a2
tan θ + tan ϕ a+a 3 a
Also tan(θ + ϕ) = = = .
( ) (1 − tan θ tan ϕ 1 −
) a4 1 − a2
Therefore tan 2 tan−1 a = 2 tan tan−1 a + tan−1 a3 .

Problem 15. Show that( ) ( )


1
tan−1 tan 2A + tan−1 (cot A) + tan−1 cot3 A = 0 .
( 2 )
1 1
Solution. Let tan−1 tan 2A = α, then tan α = tan 2A;
2 2
let tan−1 (cot A) = β, then tan β = cot A;
let tan−1 (cot3 A) = γ, then tan γ = cot3 A.
tan β + tan γ cot A + cot3 A cot A
Thus tan(β + γ) = = =
1 − tan β tan γ 1 − cot4 A 1 − cot2 A
1
tan A tan A 1
= = = − tan 2A = − tan α.
1 tan2 A − 1 2
1−
tan2 A
tan α + tan(β + γ)
Therefore tan(α + β + γ) = = 0.
1 − tan α tan(β + γ)
CHAPTER XVIII : Inverse Trigonometrical Functions 169

Thus α + β + γ = 0.

Problem 16. Show that ( ) ( )


2b π 1 a π 1 a
= tan + cos−1 + tan − cos−1 .
a 4 2 b 4 2 b
−1 a a
Solution. Let cos = θ, then cos θ = .
b b
( ) ( ) 1 − tan θ 1 + tan θ
π θ π θ 2 + 2.
And tan − + tan + =
4 2 4 2 θ θ
1 + tan 1 − tan
2 2
( ) ( )
θ 2 θ 2 θ
1 − tan + 1 + tan 1 + tan2
2 2 2 1
= =2 =2
θ θ θ θ
1 − tan 2 1 − tan 2 cos2 − sin2
2 2 2 2
2 2b
= = .
cos θ a

Problem 17. Show(that ) ( )


a3 1 a b3 1 b ( )
cosec 2 tan−1 + sec2 tan−1 = (a + b) a 2 + b2 .
2 2 b 2 2 a
a a
Solution. Let tan−1 = θ; then tan θ = ;
b b √
2 θ 1 2 2 2 (a2 + b2 )
cosec = = = = √
2 θ 1 − cos θ b (a2 + b2 ) − b
sin2 1− √
2 (a2 + b2 )
√ √ √
2 a2 + b2 (a2 + b2 ) + b 2(a2 + b2 ) + 2b (a2 + b2 )
= √ ×√ = ;
(a2 + b2 ) − b (a2 + b2 ) + b a2

a3 θ √
therefore cosec 2 = a(a2 + b2 ) + ab (a2 + b2 ).
2 2
b b
Let tan−1 = ϕ; then tan ϕ = ;
a a √
2 ϕ 1 2 2 2 (a2 + b2 )
sec = = = a = √
2
cos2
ϕ 1 + cos ϕ 1+ √ (a2 + b2 ) + a
2 2 2
(a + b )
√ √ √
2 a2 + b2 (a2 + b2 ) − a 2(a2 + b2 ) − 2a (a2 + b2 )
= √ ×√ = ;
(a2 + b2 ) + a (a2 + b2 ) − a b2

b3 ϕ √
therefore sec2 = b(a2 + b2 ) − ab (a2 + b2 ).
2 2
a3 θ b3 ϕ
Therefore cosec 2 + sec2 = (a + b)(a2 + b2 ).
2 2 2 2

Solve the following seven equations in x :

x π
Problem 18. sin−1 x + sin−1 = .
2 4
x π
Solution. sin−1 x + sin−1 = ;
2 4
x π
therefore sin−1 = − sin−1 x.
2 4
Take the sines of both(sides; thus )
x π π √ π
= sin − sin−1 x = sin · (1 − x2 ) − cos · x
2 4 4 4
CHAPTER XVIII : Inverse Trigonometrical Functions 170


(1 − x2 ) − x
= √ ;
2
( ) √
1
therefore x √ +1 = (1 − x2 );
2
( )2
1
therefore x2 √ + 1 = 1 − x2 ;
(2 )
5 2
therefore x2 +√ = 1;
2 2

therefore x2 (5 + 2 2) = 2;

2 2 5−2 2 2 √
therefore x2 = √ = √ · √ = (5 − 2 2).
5+2 2 5+2 2 5−2 2 17

2a 2b
Problem 19. sin−1 + sin−1 = 2 tan−1 x.
1 + a2 1 + b2
2a
Solution. We shall first show that sin−1 = 2 tan−1 a.
1 + a2
2 tan θ 2a
Let tan−1 a = θ; then tan θ = a; and sin 2θ = = ;
1 + tan2 θ 1 + a2
2a
therefore sin−1 = 2θ = 2 tan−1 a.
1 + a2
2b
Similarly sin−1 = 2 tan−1 b.
1 + b2
Hence the equation may be written
2 tan−1 a + 2 tan−1 b = 2 tan−1 x;
therefore tan−1 x = tan−1 a + tan−1 b.
Take the tangents of both sides; thus
( ) a+b
x = tan tan−1 a + tan−1 b = .
1 − ab

Problem 20. tan−1 (x − 1 ) + tan−1 x + tan−1 (x + 1 ) = tan−1 3x.


Solution. Let tan−1 (x − 1) = α, tan−1 x = β, tan−1 (x + 1) = γ.
Thus tan−1 3x = α + β + γ.
Take the tangents of both sides; thus
tan α + tan β + tan γ − tan α tan β tan γ
3x = tan(α + β + γ) =
1 − tan β tan γ − tan γ tan α − tan α tan β
3x − x(x2 − 1) 4x − x3
= = .
1 − x(x + 1) − (x + 1)(x − 1) − x(x − 1) 2 − 3x2
Therefore either x = 0, or 3(2 − 3x ) = 4 − x ; the latter gives 8x = 2;
2 2 2
1 1
therefore x2 = ; therefore x = ± .
4 2

Problem 21. sin−1 2x − sin−1√ x 3 = sin−1 x.
Solution. sin−1 2x − sin−1 x 3 = sin−1 x.
Take the sines of both sides;
√ thus √

2x (1 − 3x2 ) − x 3 × (1 − 4x2 ) = x.
√ √ √
Thus either x = 0, or 2 (1 − 3x2 ) − 3 × (1 − 4x2 ) = 1.
√ √ √
Transpose, thus 2 (1 − 3x2 ) = 1 + 3 × (1 − 4x2 ).
√ √
Square, 4(1 − 3x2 ) = 1 + 2 3 × (1 − 4x2 ) + 3(1 − 4x2 );
√ √
therefore 2 3 × (1 − 4x2 ) = 0;
1
therefore 1 − 4x2 = 0, therefore x = ± .
2
CHAPTER XVIII : Inverse Trigonometrical Functions 171

1 1 1 1 π
Problem 22. tan−1 + 2 tan−1 + tan−1 + tan−1 = .
4 5 6 x 4
1 1 1 1 π
Solution. tan−1 + 2 tan−1 + tan−1 + tan−1 = .
4 5 6 x 4
1 1 1
Let tan−1 = α, tan−1 = β, tan−1 = γ.
4 5 6
Thus the equation may be written
1 π
tan−1 = − (α + 2β + γ);
x 4
1 1 − tan(α + 2β + γ)
therefore = .
x 1 + tan(α + 2β + γ)
1 1
+ 9
Now tan(α + β) = 4 5 = ,
1 1 19
1− ×
4 5
1 1
+ 11
tan(β + γ) = 5 6 = ;
1 1 29
1− ×
5 6
9 11
+ 470
therefore tan(α + β + β + γ) = 19 29 = .
9 11 452
1− ×
19 29
470
1 1−
Hence = 452 = − 18 = − 9 .
x 470 922 461
1+
452

Problem 23. sin 2 cos−1 cot 2 tan−1 x = 0 .


1 1 − x2
Solution. Let tan−1 x = θ, then tan θ = x; cot 2θ = = .
tan 2θ 2x
Thus the equation may be written
1 − x2
sin 2 cos−1 = 0.
2x
1 − x2
Now since 2 cos−1 has zero for its sine, the angle must be of the form nπ,
2x
where n is zero or some integer.
1 − x2 1 − x2 nπ
Thus 2 cos−1 = nπ; therefore cos−1 = ;
2x 2x 2
1−x 2 nπ
therefore = cos .
2x 2

Since n is zero or an integer we have cos = 0, or 1, or −1.
2
1 − x2
If = 0, then x = ±1.
2x
1−x 2 √
If = 1, then x2 + 2x = 1; and from this we deduce x = −1 ± 2.
2x
1 − x2 √
If = −1, then x2 − 2x = 1; and from this we deduce x = 1 ± 2.
2x

1 1 1
Problem 24. tan−1 = tan−1 + tan−1 2 .
a−1 x a −x +1
Solution.
1 1 1
tan−1 = tan−1 + tan−1 2 ;
a−1 x a −x+1
1 1 1
therefore tan−1 − tan−1 = tan−1 2 .
a−1 x a −x+1
CHAPTER XVIII : Inverse Trigonometrical Functions 172

Take the tangents of both sides; thus


1 1

a−1 x 1
= 2 ;
1 a −x+1
1+
(a − 1)x
x−a+1 1
therefore = 2 ;
ax − x + 1 a −x+1
therefore (x − a + 1)(a − x + 1) = ax − x + 1;
2

therefore −x2 + x(a2 + a) − a3 + a2 − a + 1 = ax − x + 1;


therefore x2 − x(a2 + 1) + a3 − a2 + a = 0.
By solving this quadratic in the ordinary way we obtain x = a or a2 − a + 1.

4 1 3
Problem 25. If sec θ − cosec θ = , show that θ = sin−1 .
3 2 4
4
Solution. sec θ − cosec θ = ;
3
1 1 4
therefore − = ;
cos θ sin θ 3
4 2
therefore sin θ − cos θ = sin θ cos θ = sin 2θ.
3 3
4
Square, thus 1 − sin 2θ = sin2 2θ.
9
3
By solving this quadratic in the usual way we obtain sin 2θ = , or −3; the former
4
3
value is alone applicable. Thus sin 2θ = ;
4
3 1 3
therefore 2θ = sin−1 ; therefore θ = sin−1 .
4 2 4

1 3
Problem 26. If sin(π cos θ) = cos(π sin θ), show that θ = ± sin−1 .
2 4
Solution. sin(π cos θ) = cos(π sin θ);
( )
π
therefore cos − π cos θ = cos(π sin θ).
2
Hence, by Art. 67 (page 322) the solutions are comprised in
π
− π cos θ = 2nπ ± π sin θ;
2
1
therefore cos θ ± sin θ = − 2n.
( 2 )2
1
Square, thus 1 ± sin 2θ = − 2n .
2
If we give to n any integral value, positive or negative, the value of sin 2θ is greater
1 3
than unity. Thus we must have n zero. Then 1 ± sin 2θ = ; and therefore sin 2θ = ± ;
4 4
3 1 3
thus 2θ = ± sin−1 , and θ = ± sin−1 .
4 2 4

1 π
Problem 27. Show that if sin2 θ + sin2 ϕ = , then (2n + 1 ) is one of the values of
2 2
ψ which satisfy the equation
ψ = sin−1 (sin θ + sin ϕ) + sin−1 (sin θ − sin ϕ).
Solution. Let ψ = sin−1 (sin θ + sin ϕ) + sin−1 (sin θ − sin ϕ).
Take the cosines of both sides; thus
√ √
cos ψ = 1 − (sin θ + sin ϕ)2 1 − (sin θ − sin ϕ)2 − (sin θ + sin ϕ)(sin θ − sin ϕ)
√ √
1 1
= 1− − 2 sin θ sin ϕ 1− + sin θ sin ϕ − (sin2 θ − sin2 ϕ)
2 2
CHAPTER XVIII : Inverse Trigonometrical Functions 173


1
= − 4 sin2 θ sin2 ϕ − (sin2 θ − sin2 ϕ).
4
1 1
Now = sin2 θ + sin2 ϕ, therefore = (sin2 θ + sin2 ϕ)2 ;
2 4
1
therefore − 4 sin2 θ sin2 ϕ = (sin2 θ − sin2 ϕ)2 .
4
Thus cos ψ = ±(sin2 θ − sin2 ϕ) − (sin2 θ − sin2 ϕ).
π
Taking the upper sign we have cos ψ = 0, and therefore ψ = (2n + 1) , where n is
2
any integer.

Problem 28. Find x from the following equation,


1 1 1
3 tan−1 √ − tan−1 = tan−1 .
2+ 3 x 3
Solution.
1 1 1
3 tan−1 √ − tan−1 = tan−1 .
2+ 3 x 3
1 1
Now let tan−1 √ = θ, then tan θ = √ ;
2+ 3 2+ 3
3 1
√ − √ √
2+ 3 (2 + 3)2 3(2 + 3)2 − 1
therefore tan 3θ = = √ √
3 (2 + 3)3 − 3(2 + 3)
1− √
(2 + 3)2

20 + 12 3
= √ = 1; therefore 3θ = tan−1 1.
20 + 12 3
This might also have been inferred from the fact that
1 π π
√ = tan 15◦ = tan , so that θ = .
2+ 3 12 12
The equation may now be written
1 1
tan−1 1 − tan−1 = tan−1 .
3 x
Take the tangents of both sides; thus
1
1−
3 = 1;
1 x
1+
3
1 1
therefore = , therefore x = 2.
x 2

Problem 29. Show that one of the expressions


√( )
−1 2b + a − c a+b
sin ± 2 sin−1
a+c a+c
π
is an odd multiple of .
√2( ) √( )
a+b a+b
Solution. Let sin−1 = θ, then sin θ =
;
a+c a+c
2(a + b) c − a − 2b
then cos 2θ = 1 − 2 sin2 θ = 1 − =
a+c a+c
−1 c − a − 2b
and 2θ = cos .
a+c
Thus the proposed expression is
2b + a − c c − a − 2b
sin−1 ± cos−1 ;
a+c a+c
−1 −1
that is sin p ± cos (−p);
2b + a − c
when p is put for .
a+c
CHAPTER XVIII : Inverse Trigonometrical Functions 174

{ } √ √
Now cos sin−1 p ± cos−1 (−p) = −p (1 − p2 ) ∓ p (1 − p2 );
thus zero is one of the values of the cosine, and the corresponding angle is an odd
π
multiple of .
2

Problem 30. Find all the positive integral solutions of


tan−1 x + cot−1 y = tan−1 3 .
Solution.
tan−1 x + cot−1 y = tan−1 3,
1
therefore tan−1 x + tan−1 = tan−1 3.
y
Take the tangents of both sides; thus
1
x+
y
x = 3;
1−
y
therefore 3(y − x) = yx + 1;
3y − 1 3y + 9 − 10 10
therefore x= = =3− .
y+3 y+3 y+3
Thus if x and y are to be positive integers y + 3 must be a divisor of 10. Try in
succession the various cases, namely y + 3 = 1 or 2 or 5 or 10. It will be found that
the only admissible cases are y + 3 = 5, and y + 3 = 10. These give y = 2 or 7, and the
corresponding values of x are 1 and 2.

Problem 31. Show that if c be a positive integer, the equation


tan−1 x + tan−1 y = tan−1 c
has no positive integral solutions; while the equation
cot−1 x + cot−1 y = cot−1 c
has as many as there are different divisors of 1 + c 2 .
Solution. tan−1 x + tan−1 y = tan−1 c.
x+y
Take the tangents of both sides; thus = c;
1 − xy
c−y
therefore x + y = c(1 − xy); therefore x = .
1 + cy
It is obvious that if c and y are positive integers x is either a positive or negative
proper fraction, and cannot be a positive integer.
Next take cot−1 x + cot−1 y = cot−1 c.
xy − 1
Take the cotangents of both sides; thus = c;
x+y
therefore xy − 1 = c(x + y),
cy + 1 cy − c2 + c2 + 1
therefore x= =
y−c y−c
c2 + 1
=c+ .
y−c
Thus if α denote any divisor of c + 1 we may put y − c = α, so that y = c + α; and
2

c2 + 1
then x = c + .
α
Hence we see that there are as many solutions in positive integers as there are
divisors of c2 + 1.

Problem 32. Show that


x c1 x − y c2 − c1 c3 − c2
tan−1 = tan−1 + tan−1 + tan−1 + ......
y c1 y + x c2 c1 + 1 c3 c2 + 1
cn − cn−1 1
+ tan−1 + tan−1 ,
cn cn−1 + 1 cn
CHAPTER XVIII : Inverse Trigonometrical Functions 175

where c1 , c2 , . . . . . . cn are any quantities whatever.


x 1

c 1 x − y y c −1 x − tan−1 1 ,
Solution. tan−1 = tan−1 1
x = tan
c1 y + x 1+ y c1
c1 y
as in the solution of P roblem 6.

c2 − c1 1 1
Similarly tan−1 = tan−1 − tan−1 ,
c2 c1 + 1 c1 c2
c3 − c2 1 1
tan−1 = tan−1 − tan−1 ,
c3 c2 + 1 c2 c3
and so on.
Thus the sum of the terms on the right-hand side of the proposed expression is
x
tan−1 .
y

Problem 33. Show that we can express the sum of any number of angles of the
2ab 2a ′ b′ 2mn
form sin−1 , sin−1 ′2 , . . . in the form sin−1 2 , where m and n are
2
a +b 2 a + b′2 m + n2
′ ′
rational functions of a, b, a , b , . . .
Solution.
2ab 2a′ b′
Let sin−1 2 = θ, and sin−1 ′2 = ϕ;
a + b2 a + b′2
2ab 2a′ b′
then sin θ = 2 , and sin ϕ = ′2 ;
a + b2 a + b′2
a −b
2 2 ′2
a −b ′2
therefore cos θ = 2 , and cos ϕ = ′2 ;
a + b2 a + b′2
′2 ′2 ′
2ab(a − b ) + 2a b (a − b2 ) ′ 2
therefore sin(θ + ϕ) =
(a2 + b2 )(a′2 + b′2 )
2ab(a′2 − b′2 ) + 2a′ b′ (a2 − b2 ) 2(ab′ + a′ b)(aa′ − bb′ )
= =
(ab′ + a′ b)2 + (aa′ − bb′ )2 (ab′ + a′ b)2 + (aa′ − bb′ )2
2pq
therefore θ + ϕ = sin−1 2 ,
p + q2
where p and q are rational expressions.
2a′′ b′′
Then if there be another angle sin−1 ′′2 , we may denote if by ψ; then sin{(θ +
a + b′′2
2rs
ϕ) + ψ} will take the form 2 where r and s are rational. And so on.
r + s2

(−1 )m
Problem 34. Write down the general value of sin−1 , where m is an integer.
2
(−1) m π
Solution. We may take for the simplest value of sin−1 the angle (−1)m ; as is
2 6
evident by supposing m first even and then odd. This will be the α of Art. 66 (page 321)
π
and the general solution is nπ + (−1)n α, that is nπ + (−1)m+n .
6
π
Or we may take the form (m + n)π + (−1)n .
6
For the sine of this angle
{ } { }
π π
= sin mπ cos nπ + (−1)n
+ cos mπ sin nπ + (−1)n
{ 6} 6
nπ 1 1
= cos mπ sin nπ + (−1) = cos mπ × = (−1) × .
m
6 2 2

(−1 )m
Problem 35. Write down the general value of cos−1 , where m is an integer.
2
CHAPTER XVIII : Inverse Trigonometrical Functions 176

1 π
Solution. If m be even the value is cos−1 , that is 2nπ ± .
( ) 2 ( )
3
1 π
If m be odd the value is cos−1 − , that is 2nπ ± π + .
2 3
π
Both forms may be comprised in (2p + m)π ± , where p is any integer.
3 ( )
π π π
For 2nπ ± consists of an even multiple of π augmented by ± ; and 2nπ ± π +
3 3 3
π
consists of an odd multiple of π augmented by ± .
3

Problem 36. Write down the general value of tan−1 (−1 )m , where m is an integer.
π
Solution. If m be even the value is tan−1 1, that is nπ + .
4
π
If m be odd the value is tan−1 (−1), that is nπ − .
4
π
Both forms may be comprised in nπ + (−1)m .
4
CHAPTER XIX
De Moivre’s Theorem


Problem 1. Extract the square root of cos 4A + −1 sin 4A.
{ √ } 12 { √ }
Solution. cos 4A + (−1) sin 4A =± cos 2A + (−1) sin 2A by Art. 267 (page 347).

1
Problem 2. Find the values of√(−1 ) 3 .
Solution. −1 = cos π = cos π + (−1) sin π;
π √ 1 π
therefore one value of (−1) 3 = cos + (−1) sin ,
3 3
1
so we may put −1 = cos 3π, or cos 5π, and thus we obtain two other values for (−1) 3 ,
namely,
3π √ 3π
cos + (−1) sin , that is − 1,
3 3
5π √ 5π
and cos + (−1) sin .
3 3

1
Problem 3. Obtain the six values of (−1 ) 6 .
Solution. We may put −1 = cos π, or cos 3π, or cos 5π, or cos 7π, or cos 9π, or cos 11π;
and thus √
−1 = cos θ + (−1) sin θ
where θ = π, or 3π, or 5π, or 7π, or 9π, or 11π.
1
Hence the six values of (−1) 6 are contained in
θ √ θ
cos + (−1) sin ,
6 6
where θ has any of the six values just specified.

{ √ } 13
Problem 4. Find the three values of 1+ −1 .
Solution. { √ }
√ √ 1 (−1)
1+ (−1) = 2 √ + √
2 2
√ √
= 2{cos θ +
(−1) sin θ},
π
where for θ we may put + 2nπ, where n is any integer.
4
√ { }
1 1 θ √ θ
Therefore {1 + (−1)} 3 = 2 6 cos + (−1) sin ;
3 3
π π
and the three values will be obtained by putting for θ in succession , 2π + , and
4 4
π
4π + .
4

sin θ 2165
Problem 5. Given = , show that θ is nearly the circular measure of 3 ◦ .
θ 2166
sin θ
Solution. Since is given nearly equal to unity, we may infer that θ is a small
θ
angle. Hence we have approximately, by Art. 274 (page 348),
θ3
sin θ = θ − ;
6
θ 2 2165
thus 1− = ;
6 2166
CHAPTER XIX : De Moivre’s Theorem 178

θ2 1
therefore = ,
6 2166
1
therefore θ2 = ,
361
1
therefore θ= .
19
1
This is the circular measure of the angle; therefore the number of degrees =
19
180 1
of = of 57◦ .29 . . . = 3◦ approximately.
π 19
( )
π
Problem 6. Given sin + θ = ·51 , find approximately the value of θ, neglecting
6
( the) second.
powers of θ above
π
Solution. sin + θ = .51.
6
π
As .51 is very nearly equal to sin we may infer that θ is very small.
6
We have
π π
sin cos θ + cos sin θ = .51,
( 6) √ 6
1 θ2 3
therefore 1− + θ = .51 approximately.
2 2 2

3 1 1
Hence, neglecting θ2 , we have θ= , and therefore θ = √ .
2 100 50 3
2
Then if we retain the term in θ we have
1 θ2
θ= √ + √ ;
50 3 2 3
and putting for θ2 its approximate value, we have for a closer approximation
( )2
1 1 1
θ= √ + √ √
50 3 2 3 50 3
1 1
= √ + √ .
50 3 15000 3
1
The same result will be obtained if we solve the quadratic equation θ = √ +
50 3
θ2
√ in the usual way, select the least root, and take its approximate value. See
2 3
Algebra, Art. 526, Example (3) (page 364).

a3 x 3 a5 x 5
Problem 7. If tan x = a1 x + + + ......
3 5
show that
(2n + 1)2n (2n + 1)2n(2n − 1)(2n − 2)
a2n+1 = a2n−1 − a2n−3 + . . .
1·2 4
+ . . . + (−1)n+1 (2n + 1)a1 + (−1)n .
Solution.
a3 x3 a5 x5
Suppose tan x = a1 x + + + ...;
3 5
{ }
a3 x3 a5 x5
then sin x = cos x a1 x +
+ + ...; .
3 5
Substitute for sin x and cos x by Art. 274 (page 348); thus
x3 x5 x7
x− + − + ...
3 5 7
{ }{ }
x2 x4 x6 a3 x3 a5 x5
= 1− + − + ... a1 x + + + ... .
2 4 6 3 5
CHAPTER XIX : De Moivre’s Theorem 179

Then, according to the known principles of Algebra, we may equate the coefficient
of any poser of x on the left-hand side to the coefficient of the same power obtained
by working out the product on the right-hand side. Take, for instance, the coefficient
x2n+1 ; thus we obtain
(−1)n a2n+1 a2n−1 a2n−3 a1
= − + . . . + (−1)n .
2n + 1 2n + 1 2 2n − 1 4 2n − 3 2n
Multiply by 2n + 1 and transpose; thus we get
(2n + 1)2n (2n + 1)2n(2n − 1)(2n − 2)
a2n+1 = a2n−1 − a2n−3
2 4
+ . . . + (2n + 1)(−1)n+1 a1 + (−1)n .

Problem 8. If θ cot θ = a0 + a2 θ2 + a4 θ4 + . . .
show that
a2n−2 a2n−4 (−1)n−1 a0 (−1)n
a2n = − + ... + + ;
3 5 2n + 1 2n
hence find θ cot θ to four terms.
Solution. Let θ cot θ = a0 + a2 θ2 + a4{ θ4 + . . . ; }
then θ cos θ = sin θ a0 + a2 θ2 + a4 θ4 + . . . .
Substitute for(cos θ and sin θ by Art. 274)(page 348); thus
θ2 θ4 θ6
θ 1− + − + ...
2 4 6
{ }
θ3 θ5 θ7 { }
= θ−+ − + ... a0 + a2 θ2 + a4 θ4 + . . .
3 5 7
Equate the coefficients of θ2n+1 ; thus
(−1)n a2n−2 a2n−4 (−1)n a0
= a2n − + − ... + .
2n 3 5 2n + 1
Transpose; thus we get
a2n−2 a2n−4 (−1)n−1 a0 (−1)n
a2n = − + ... + + .
3 5 2n + 1 2n
To find the first four terms of θ cot θ we have the following equations:
1 = a0 ,
1 a0
− = a2 − ,
2 3
1 a2 a0
= a4 − + ,
4 3 5
1 a4 a2 a0
− = a6 − + − .
6 3 5 7
Hence we obtain
1 1 1 1 1 1 1 1 1
a0 = 1, a2 = − = − =− , a4 = − − =− ;
3 2 6 2 3 4 33 5 45
1 1 1 1 2
a6 = − − + + =− .
6 45 3 35 7 945

Problem 9. If sec θ = a0 + a2 θ2 + a4 θ4 + . . . + a2n θ2n + . . .


show that
a2n−2 a2n−4 (−1)n+1 a0
a2n = − + ... + .
2 4 2n
Solution. Let sec θ = a0(+ a2 θ2 + a4 θ4 + . . . ; )
then 1 = cos θ a0 + a2 θ2 + a4 θ4 + . . .
( )
θ2 θ4 θ6 ( )
= 1− + − + ... a0 + a2 θ2 + a4 θ4 + . . . .
2 4 6
CHAPTER XIX : De Moivre’s Theorem 180

Then equating to zero the coefficient of θ2n in the expression on the right-hand
side we get
a2n−2 a2n−4 a2n−6 (−1)n
0 = a2n − + − + ... + a0 .
2 4 6 2n
a2n−2 a2n−4 (−1)n+1 a0
Transpose; then we obtain a2n = − + ... + .
2 4 2n


Problem 10. If cos 2 α + −1 sin 2 α be substituted for a in the expression
bc
, and similar quantities for b and c, and the result reduced to the form
(a + b)(a
√ + c)
A + B −1 , find the values of A and B in terms of α, β, γ.
Solution. √
a + b = cos 2α + cos 2β + (−1) {sin 2α + sin 2β}

= 2 cos(α + β) cos(α − β) + 2 (−1) sin(α + β) cos(α − β)
{ √ }
= 2 cos(α − β) cos(α + β) + (−1) sin(α + β) .

b cos 2β + (−1) sin 2β
Thus = { √ } ; multiply both numera-
a+b
2 cos(α − β) cos(α + β) + (−1) sin(α + β)

tor and denominator by cos(α + β) − (−1) sin(α + β);

cos(β − α) + (−1) sin(β − α)
thus we get .
2 cos(α −√β)
c cos(γ − α) + (−1) sin(γ − α)
Similarly =
a+c 2 cos(α − γ) √
bc cos(β + γ − 2α) + (−1) sin(β + γ − 2α)
Therefore = .
(a + b)(a + c) 4 cos(α − β) cos(α − γ)

Problem 11. Show that{ √ }n


cos θ + cos ϕ + −1(sin θ + sin ϕ)
{ √ }n
+ cos θ + cos ϕ − −1(sin θ + sin ϕ)
( )
θ−ϕ n n(θ + ϕ)
= 2n+1 cos cos .
2 2
Solution. { √ } n
cos θ + cos ϕ + (−1)(sin θ + sin ϕ)
{ √ }
θ+ϕ θ−ϕ θ+ϕ θ−ϕ n
= 2 cos cos +2 (−1) sincos
( 2 ) {2 2 }2
θ−ϕ n θ+ϕ √ θ+ϕ n
= 2n cos cos + (−1) sin
( 2 ) { 2

2 }
θ−ϕ n
n n
= 2n cos cos (θ + ϕ) + (−1) sin (θ + ϕ) .
2 2 2

Similarly {cos θ + cos ϕ − (−1)(sin θ + sin ϕ)}n
( ) { √ }
θ−ϕ n n n
= 2n cos cos (θ + ϕ) − (−1) sin (θ + ϕ) .
2 ( 2 ) 2
n+1 θ−ϕ n n
Hence by addition we get 2 cos cos (θ + ϕ).
2 2


−1 ,

Problem 12. Show that if x = e θ and 1 − c( 2 = nc − 1 ,
)
c n
1 + c cos θ = (1 + nx) 1 + .
2n x
CHAPTER XIX : De Moivre’s Theorem 181

Solution. ( ) { ( )}
c n c 1
(1 + nx) 1 + = 1 + n2 + n x +
2n x 2n x
c c(1 + n2 )
= {1 + n2 + 2n cos θ} = + c cos θ.
√ 2n 2n
Now (1 − c ) = nc − 1; therefore 1 − c = (nc − 1) , therefore −c2 = n2 c2 − 2nc,
2 2 2

therefore 2n = (n2 + 1)c.


c(1 + n2 )
Thus = 1, and the expression becomes 1 + c cos θ.
2n

Problem 13. Prove the following rule for finding the length of a small circular arc
: from eight times the chord of half the arc subtract the chord of the whole arc, and
one-third of the remainder will give the length of the arc nearly.
Solution. Let r denote the radius, and θ the circular measure of the angle; then the
length of the arc is rθ.
θ θ
The chord of the arc is 2r sin , and the chord of half the arc is 2r sin .
2 4
Now let it be required to determine two numbers l and m, such that approximately
θ θ
l × 2r sin + m × 2r sin = rθ.
2 4
θ θ
Expand sin and sin by Art. 274 (page 348). Thus
4 { 2 } { }
( )3 ( )3
θ 1 θ θ 1 θ
2l − + . . . + 2m − + . . . = 0.
2 3 2 4 3 4
Neglect all powers of θ above θ3 ; then to make this formula hold we must put
m l m
l+ = 1, + = 0.
2 (2)3 (4)3
Therefore m = −8l; therefore −3l = 1.
1 8
Thus l = − and m = .
3 3
This establishes the rule.

Problem 14. Show that the following rule for finding the length of a small circular
arc is more accurate than that in the preceding problem : to 256 times the chord of
one-fourth of the arc add the chord of the arc; subtract 40 times the chord of half the
arc, and divide the remainder by 45 .
Solution. Proceed as in P roblem 13.
θ
The chord of one-fourth of the arc is 2r sin .
8
Let it be required to determine the numbers l, m, n such that approximately
θ θ θ
l × 2r sin + m × 2r sin + n × 2r sin = rθ.
2 4 8
In this case we can make the approximation closer than in P roblem 13; for we shall
retain θ5 and
{ neglect only the higher}powers.{ Thus }
( )3 ( )5 ( )3 ( )5
θ 1 θ 1 θ θ 1 θ 1 θ
2l − + + 2m − +
2 3 2 5 2 4 3 4 5 4
{ ( )3 ( )5 }
θ 1 θ 1 θ
+ 2n − + = θ.
8 3 8 5 8
Hence we must put
m n l m n l m n
l+ + = 1, + + = 0, + + = 0.
2 4 (2)3 (4)3 (8)3 (2)5 (4)5 (8)5
The values of l, m, n given by these equations are
1 40 256
l= , m=− , n= .
45 45 45
CHAPTER XIX : De Moivre’s Theorem 182

Problem 15. From the identical equation


(x − b)(x − c) (x − c)(x − a) (x − a)(x − b)
+ + = 1,
(a − b)(a − c) (b − c)(b − a) (c − a)(c − b)
deduce the following by assuming √
x = cos 2 θ + −1 sin 2 θ,
and corresponding assumptions for a, b, and c :
sin(θ − β) sin(θ − γ)
sin 2(θ − α)
sin(α − β) sin(α − γ)
sin(θ − γ) sin(θ − α)
+ sin 2(θ − β)
sin(β − γ) sin(β − α)
sin(θ − α) sin(θ − β)
+ sin 2(θ − γ) = 0.
sin(γ − α) sin(γ − β)
Solution. √ √
x − b = cos 2θ + (−1) sin 2θ − cos 2β − (−1) sin 2β
{ √ }
= 2 sin(β − θ) sin(β + θ) − (−1) cos(β + θ)
{ √ }
2 sin(β − θ)
= √ cos(β + θ) + (−1) sin(β + θ) .
(−1)
In like manner { }
2 sin(β − α) √
a−b= √ cos(β + α) + (−1) sin(β + α) .
(−1)

x−b sin(β − θ) cos(β + θ) + (−1) sin(β + θ)
Therefore = · √ , multiply both numer-
a−b sin(β − α) cos(β + α) + (−1) sin(β + α)

ator and denominator by cos(β + α) − (−1) sin(β + α);
{ √ }
sin(θ − β)
thus we get cos(θ − α) + (−1) sin(θ − α) .
sin(α − β)
x−c
Similarly we transform ; and thus we obtain
a−c
{ √ }
(x − b)(x − c) sin(θ − β) sin(θ − γ)
= cos 2(θ − α) + (−1) sin 2(θ − α) .
(a − b)(a − c) sin(α − β) sin(α − γ)
(x − c)(x − a) (x − a)(x − b)
In like manner we transform and . Then by equating to
(b − c)(b − a) (c − a)(c − b)
zero the coefficient of the imaginary part we obtain

sin(θ − β) sin(θ − γ)
sin 2(θ − α)
sin(α − β) sin(α − γ)
sin(θ − γ) sin(θ − α)
+ sin 2(θ − β)
sin(β − γ) sin(β − α)
sin(θ − α) sin(θ − β)
+ sin 2(θ − γ) = 0.
sin(γ − α) sin(γ − β)
And then by equating the real parts we have

sin(θ − β) sin(θ − γ) sin(θ − γ) sin(θ − α)


cos 2(θ − α) + cos 2(θ − β)
sin(α − β) sin(α − γ) sin(β − γ) sin(β − α)
sin(θ − α) sin(θ − β)
+ cos 2(θ − γ) = 1.
sin(γ − α) sin(γ − β)
CHAPTER XX
Expansions of Some Trigonometrical Functions

Problem 1. Expand (sin θ)4n+2 in terms of cosines of multiples of θ.


Solution. Proceed as in Art. 282 (page 350). Thus we obtain
−24n+1 (sin θ)4n+2 = cos(4n + 2)θ − (4n + 2) cos 4nθ
(4n + 2)(4n + 1)
+ cos(4n − 2)θ − . . .
2
(4n + 2)(4n + 1) . . . (2n + 2)
− .
2 2n + 1

Problem 2. Expand (sin θ)4n+1 in terms of sines of multiples of θ.


Solution. Proceed as in Art. 283 (page 350). Thus we obtain
24n (sin θ)4n+1 = sin(4n + 1)θ − (4n + 1) sin(4n − 1)θ
(4n + 1)4n
+ sin(4n − 3)θ − . . .
2
(4n + 1)4n(4n − 1) . . . (2n + 2)
+ sin θ.
2n

Problem 3. Expand (cos θ)2n in terms of cosines of multiples of θ.


Solution. Proceed as in Art. 280 (page 349). Thus we obtain
2n(2n − 1)
22n−1 cos2n θ = cos 2nθ + 2n cos(2n − 2)θ + cos(2n − 4)θ
2
2n(2n − 1) . . . (n + 1)
+... .
2n

Problem 4. Prove that in any triangle


a2 cos 12 (B − C) b2 cos 21 (C − A) c2 cos 12 (A − B)
+ +
cos 12 (B + C) cos 12 (C + A) cos 21 (A + B)
= 2(ab + bc + ca).
Solution. We have
1 1 1 1
a2 cos (B − C) a2 cos (B − C) a2 cos A cos (B − C)
2 = 2 = 2 2
1 1 1 1
cos (B + C) sin A cos A sin A
2 2 2 2
2a2 1 1 a2
= sin (B + C) cos (B − C) = (sin B + sin C)
sin A 2 2 sin A
2
a sin B 2
a sin C 2
a b 2
a c
= + = + = ab + ac.
sin A sin A a a
1
b2 cos (C − A)
Similarly 2 = ba + bc,
1
cos (C + A)
2
1
c cos (A − B)
2

and 2 = ca + cb.
1
cos (A + B)
2
Then by addition we obtain the required result.

Problem 5. From the angles of a triangle ABC , perpendiculars AD, BE, CF are let
CHAPTER XX : Expansions of Some Trigonometrical Functions 184

fall on the opposite sides : prove that


a sin(BAD − CAD) + b sin(CBE − ABE) + c sin(ACF − BCF) = 0 .
Solution. Suppose the triangle to have all its angles acute.
Then a sin(BAD − CAD) = a(sin BAD cos CAD − cos BAD sin CAD)
= a(cos B sin C − sin B cos C).
Similarly b sin(CBE − ABE) = b(cos C sin A − sin C cos A),
and c sin(ACF − BCF ) = c(cos A sin B − sin A cos B).
The sum of the three expressions
= cos A(c sin B − b sin C) + cos B(a sin C − c sin A)
+ cos C(b sin A − a sin B) = 0, by Art. 214 (page 336).
If the triangle has an obtuse angle, let it be C; then it will be found that instead of
cos C we have cos(180◦ − C) in the preceding expressions; and the result is still zero.

Problem 6. From A and B two acute angles of a triangle draw AD and BD at right
angles respectively to AC and BC . If ρ be the radius of the circle inscribed in ABD,
then
AB = ρ (sec A + sec B + tan A + tan B) .
{ }
1 1
Solution. We have ρ cot DAB + cot DBA = AB.
2 2 ( )
1 π
( ) cos −A
1 1 π 2 2
Now cot DAB = cot −A = ( )
2 2 2 1 π
sin −A
( 2) 2 ( )
1 π π
2 cos2 −A 1 + cos −A
2 2
= ( ) ( )= ( 2 )
1 π 1 π π
2 sin − A cos −A sin −A
2 2 2 2 2
1 + sin A
= = sec A + tan A.
cos A
1
Similarly cot DBA = sec B + tan B.
2
Therefore ρ {sec A + tan A + sec B + tan B} = AB.

Problem 7. Three equal circles of radius a touch each other : show that the area of
the space between them is (√ )
π
3− a2 .
2
Solution. By joining the centers of the circles we form √ an equilateral triangle of
(2a)2 3 √
which each side is 2a; and therefore the area is , that is a2 3. The area of
4
a2 2π
each of the three sectors which are formed by the radii and arc of a circle is × ,
2 6
πa 2 πa 2
that is ; therefore the area of the three sectors is . Hence the area of the space
6
√ 2 (√ ) 2
πa π
between the circles = a2 3 − = a2 3− .
2 2

Problem 8. Let area of a regular polygon inscribed in a circle is a geometric mean


between the areas of an inscribed and of a circumscribed regular polygon of half the
number of sides.
Solution. Let R be the radius of the circle. The inscribed polygon of n sides consists
of n triangles; and therefore the area of the polygon is
R2 2π
n sin .
2 n
CHAPTER XX : Expansions of Some Trigonometrical Functions 185

The area of an inscribed figure of half the number of sides is


n R2 4π
sin .
2 2 n
n n 2π
The area of a circumscribed polygon of sides is R2 tan .
2 2 n
We have to show that
( )2
nR2 2π n 4π n 2π
sin = R2 sin × R2 tan ,
2 n 4 n 2 n
2π 1 4π 2π
or that sin2 = sin tan ;
n 2 n n
4π 2π 2π
and this is obvious since sin = 2 sin cos .
n n n

Problem 9. The area of a regular polygon circumscribed about a circle is an har-


monic mean between the areas of an inscribed regular polygon of the same number
of sides, and of a circumscribed regular polygon of half that number.
Solution. Let R be the radius of the circle. The area of the circumscribed polygon
π nR2 2π
of n sides is nR2 tan . The area of the inscribed polygon of n sides is sin . The
n 2 n
n n 2 2π
area of the circumscribed polygon of sides is R tan . We have to show that
2 2 n
π nR 2 2π nR 2 2π π
nR2 tan is an harmonic mean between sin and tan ; or that 2 tan is
n 2 n 2 n n
2π 2π
an harmonic mean between sin and tan .
n n
2π 2π
Now the harmonic mean between sin and tan is
n n
2π 2π 2π
2 sin tan 2 sin
n n , that is n , that is 2 tan π .
2π 2π 2π n
sin + tan 1 + cos
n n n

Problem
√ 10. If the side of a pentagon inscribed in a circle be c, the radius is

c 5+ 5
√ .
10
2π π
Solution. Let r be the radius of the circle; then c = 2r sin = 2r sin ; therefore
10 5
c 2c 2c
r = π = √ √ = √ √ . Multiply both numerator and denom-
2 sin (10 − 2 5) √
5 2 (5 − 5)
√ √ √ √
√ √ 2c (5 + 5) 2c (5 + 5)
inator by (5 + 5). Thus we obtain √ √ , that is √ , that is
2 × (25 − 5) (40)
√ √
c (5 + 5)
√ .
(10)

Problem 11. Three circles whose radii are a, b, c touch each other externally :
prove that the tangents at the points of contact meet at a point whose distance from
any one of them is
( )1
abc 2
.
a+b+c
Solution. Let A, B, C denote the centers of the three circles. Let tangents to the
arc of the first circle meet at T ; then the distance of T from the point of contact is
A
a tan .
2
(a + c)2 + (a + b)2 − (b + c)2 a2 + a(b + c) − bc
Now cos A = = ;
2(a + c)(a + b) (a + c)(a + b)
CHAPTER XX : Expansions of Some Trigonometrical Functions 186

1 − cos A bc
therefore = ;
1 + cos A a(a + b + c)
√ √
A bc A abc
therefore tan = ; therefore a tan = .
2 a(a + b + c) 2 a+b+c
We shall obtain the same symmetrical expression for the distance of the point of
intersection of any two tangents from the points of contact; and thus it follows that
the three tangents meet at a common point.

Problem 12. The sides taken in order of a quadrilateral whose opposite angles are
supplementary are 3 , 3 , 4 , 4 : find the area and the radii of the inscribed and cir-
cumscribed circles.
Solution. Use Article
√ 254 (page 346). Here s = 7, s−a = 4, s−b = 4, s−c = 3, s−d = 3.
Thus the area = 4 × 4 × 3 × 3 = 12.
Since the sum of a pair of opposite sides is equal to the sum of the other pair, a
circle may be inscribed in the quadrilateral. Let ρ denote the radius of this inscribed
ρ
circle; then (a + b + c + d) = the area of the quadrilateral. Thus ρs = the area.
2
12
In the present case ρ=
7
Also ab + cd = 25, ac + bd = 24, ad + bc = 24.

Hence the radius of the circumscribed circle
1 25 × 24 × 24 5 × 24 5
= = = .
4 3×3×4×4 4 × 12 2

Problem 13. The area of a regular polygon inscribed in a circle is to that of the
circumscribed polygon of the same number of sides as 3 is to 4 : find the number of
sides.
Solution. Let n be the number of the sides, R the radius of the circle.
n 2 2π π
Then R sin is to nR2 tan as 3 is to 4.
2 n n

sin
Thus n = 3;
π 4
2 tan
n √
π 3 π 3
therefore cos2 = ; therefore cos = .
n 4√ n 2
π 3
But cos = ; therefore n = 6.
6 2

Problem 14. If the radii of three circles which touch each other be a, b, c, and α, β, γ
be the chords of the arcs between
( the points
) ( of contact
)( in each,
) show that
8 1 1 1 1 1 1
= + + + .
αβγ a b b c c a
Solution. Let A, B, C denote the centers of the three circles.
A B C
Then α = 2a sin , β = 2b sin , γ = 2c sin .
2 2 2
Now from the solution of P roblem 11 we see that
a2 + a(b + c) − bc 2bc
cos A = ; therefore 1 − cos A = ;
(a + b)(a + c) (a + b)(a + c)

A bc
therefore sin = ;
2 (a + b)(a + c)

bc
therefore α = 2a .
(a + b)(a + c)
CHAPTER XX : Expansions of Some Trigonometrical Functions 187

Similar expressions hold for β and γ. Thus


{ } 21
1 1 (a + b)(a + c) (b + a)(b + c) (c + a)(c + b)
= × ×
αβγ 8abc bc ac bc
(a + b)(b + c)(c + a)
= ;
8a2 b2 c2
( )( )( )
8 1 1 1 1 1 1
therefore = + + + .
αβγ b a c b a c

( )3
tan θ θ2
Problem 15. Show that the limit of , when θ is indefinitely diminished, is
θ
e.
( )3
tan θ θ2 3 tan θ
Solution. Let u = ; then log u = 2 log .
θ θ θ
θ 3
Now tan θ = θ + + terms in θ5 and higher powers of θ; see Chapter xix : P roblem 7.
3
tan θ θ2
Therefore =1+ + ...
( ) θ 3
θ 2 θ 2
Then log 1 + + ... = + terms in θ4 and higher powers of θ.
3 3
3 tan θ
Therefore 2 log = 1+ terms in θ2 and higher powers of θ.
θ θ
3 tan θ
Therefore when θ is indefinitely diminished 2 log = 1, and therefore
θ θ
( ) 32
tan θ θ
= e.
θ

Problem 16. The two diagonals of a quadrilateral figure whose opposite angles are
supplementary cannot be equal unless some one of the sides be equal to the opposite
one.
Solution. Equate the expressions for AC 2 and BD2 given in Art 254 (page 346). Thus
(ac + bd)(ad + bc) (ac + bd)(ab + cd)
= .
ab + cd ad + bc
2 2
Therefore (ad + bc) = (ab + cd) ;
therefore ad + bc = ab + cd;
therefore (a − c)(d − b) = 0.
Therefore either a = c or b = d.

Problem 17. Two circles whose radii are a and b cut one another at an angle γ :
show that the length of the common chord is
2ab sin γ
√ .
a 2 + 2ab cos γ + b2
Solution. Let A and B be the centers of the two circles, and C a point of intersection.
The angle between the tangents at C is therefore γ.
π π
The angleACB = + − γ = π − γ.
2 2
Then AB 2 = a2 + b2 − 2ab cos(π − γ) = a2 + b2 + 2ab cos γ.
Let x denote the length of the common chord; then the area of the triangle ABC =
1 x 1
× × AB; and this area also = AC · CB sin ACB.
2 2 2
2AC · CB sin ACB
Thus x=
AB
2ab sin γ
= √ .
(a2 + b2 + 2ab cos γ)
CHAPTER XX : Expansions of Some Trigonometrical Functions 188

Problem 18. The radius of the circle inscribed in a triangle can never be greater
than half the radius of the circle described about the triangle.
r S abc 4S 2 A B C
Solution. We have = ÷ = = 4 sin sin sin .
R s 4S sabc 2 2 2
Now we have shown in the solution of Chapter xiii : P roblem 40, that the expression
A B C 1 1
4 sin sin sin can never be greater than . Hence r cannot be greater than R.
2 2 2 2 2
CHAPTER XXI
Exponential Values of the Cosine and Sine

Problem 1. Apply the exponential values of the sine and cosine to show that
sin 2 θ
= cot θ.
1 − cos 2 θ
Solution.
sin 2θ e2θι − e−2θι e2θι − e−2θι
= ( )=
1 − cos 2θ e2θι + e−2θι ι(2 − e2θι − e−2θι )
2ι 1 −
2
( ) ( )( )
ι e2θι − e−2θι ι eθι + e−θι eθι − e−θι
=
e2θι + e−2θι − 2
= ( )2
eθι − e−θι
( −θι
)
ι eθι + e eθι + e−θι eθι − e−θι cos θ
= = ÷ = .
eθι − e−θι 2 2ι sin θ

Problem 2. If the sides of a right-angled triangle be 49 and 51 , show that the angles
opposite to them are 43 ◦ 51 ′ 15 ′′ and 46 ◦ 8 ′ 45 ′′ nearly.
π
Solution. Let the angle opposite to the smaller side be − θ, and the angle opposite
4
π
to the larger side + θ. Thus
4 ( )
π
sin −θ
49
(4 )= ;
π 51
sin +θ
( ) (
4 )
π π
sin + θ − sin −θ
51 − 49 1
therefore (4 ) (4 )= = ;
π π 51 + 49 50
sin + θ + sin −θ
4 4
π
2 sin θ cos 1
therefore 4
π = 50 ;
2 cos θ sin
4
1
therefore tan θ = .
50
But by Art. 293 (page 351) we have
1
θ = tan θ − tan3 θ + . . . ,
3
1 1
thus θ = .02 − (.02)3 + (.02)5 − . . .
3 5
If we stop at the first term we have θ = .02.
Then the number of degrees in the angle = .02 × 57.29577951... = 1.14591559;
and this = 10 8′ 45′′ .

Problem 3. If the angle C of a triangle be given, and the other two adjacent sides
a, b be nearly equal, show that the
{ other angles are nearly equal}to
( )3
C 180◦ a − b C 1 a−b C
90◦ − ± cot − cot .
2 π a+b 2 3 a+b 2
Solution. We have, as in Art. 229 (page 338),
A−B a−b C
tan = cot .
2 a+b 2
CHAPTER XXI : Exponential Values of the Cosine and Sine 190

A−B
Hence by Art. 293 (page 351), the circular measure of
2
k3 k5
=k− + − ...
3 5
a−b C
where k stands for cot .
a+b 2
A−B
Therefore the number of degrees in
{ 2 }
180 k3 k5
= k− + − ... .
π 3 5
A+B C
Also = 90◦ − . Thus A is found by taking the upper sign, and B by taking
2 2
the lower sign in { }
C 180◦ k3 k5
90◦ − ± k− + − ... .
2 π 3 5

Problem 4. In any triangle, if A − B be small compared with C , show that the


circular measure of A − B is equal to
( )
a−b a−b 2
2 sin B + sin 2B nearly.
c c
sin A a sin B b
Solution. = , = ;
sin C c sin C c
sin A − sin B a−b
therefore = ;
sin C c
A−B A+B
sin cos a−b
therefore 2 2 = ;
C C c
sin cos
2 2
A−B a−b C
therefore sin = cos
2 c 2
a−b A+B
= sin
c ( 2 )
a−b A−B
= sin +B .
c 2
A−B
Hence by Art. 208 (page 335) the circular measure of
2
n2 n3
= n sin B + sin 2B + sin 3B + . . . ,
2 3
a−b
where n stands for . Therefore the circular measure of A − B
c
= 2n sin B + n2 sin 2B nearly.

Problem 5. If a and b be the sides of a plane triangle, A and B the opposite angles,
then will log b − log a
1 1
= cos 2A − cos 2B + (cos 4A − cos 4B) + (cos 6A − cos 6B) + . . .
2 ( 3 )
b sin B eBι − e−Bι eBι 1 − e−2Bι
Solution. = = Aι = Aι .
a sin A e − e−Aι e (1 − e−2Aι )
Take the logarithms : thus ( ) ( )
log b − log a = Bι − Aι + log 1 − e−2Bι − log 1 − e−2Aι
{ }
1 1
= (B − A)ι − e−2Bι + e−4Bι + e−6Bι + . . .
2 3
1 1
+ e−2Aι + e−4Aι + e−6Aι + . . .
2 3
Now e−2Bι = cos 2B − ι sin 2B, e−2Aι = cos 2A − ι sin 2A,
CHAPTER XXI : Exponential Values of the Cosine and Sine 191

and so on. Then, as the real and imaginary parts of the expression must be sepa-
rately equal, we have
1
log b − log a = cos 2A − cos 2B + (cos 4A − cos 4B)
2
1
+ (cos 6A − cos 6B) + . . .
3

π 1 1 1
Problem 6. Show that = + + + . . .‘
8 1 ·3 5 ·7 9 · 11
Solution. By Art. 294 (page 351)
π 1 1 1 1 1
=1− + − + − + ...
4 3 5 7 9 11
2 2 2
= + + + ...;
1·3 5·7 9 · 11
π 1 1 1
therefore = + + + ...
8 1·3 5·7 9 · 11

Problem 7. If A + Bι = log(m + nι), show that


n ( )
tan B = , and 2A = log n 2 + m 2 .
m
Solution.
Let A + Bι = log(m + nι);
therefore eA+Bι = m + nι;
therefore m + nι = eA eBι = eA (cos B + ι sin B);
therefore m = eA cos B,
and n = eA sin B.
n
By division = tan B.
m
By squaring and adding m + n2 = e2A ;
2
( )
therefore 2A = log m2 + n2 .

Problem 8. Reduce cos(θ + ϕι) to the form α + βι.


Solution.
cos(θ + ϕι) = cos θ cos ϕι − sin θ sin ϕι
e−ϕ + eϕ e−ϕ − eϕ
= cos θ − sin θ
2 2ι
e−ϕ + eϕ e−ϕ − eϕ
= cos θ + ι sin θ ;
2 2
this is of the form α + βι where
e−ϕ + eϕ e−ϕ − eϕ
α = cos θ , and β = sin θ .
2 2

Problem 9. Reduce sin(θ + ϕι) to the form α + βι.


Solution.
sin(θ + ϕι) = sin θ cos ϕι + cos θ sin ϕι
e−ϕ + eϕ e−ϕ − eϕ
= sin θ + cos θ
2 2ι
e−ϕ + eϕ e−ϕ − eϕ
= sin θ − ι cos θ ;
2 2
this is of the form α + βι where
e−ϕ + eϕ e−ϕ − eϕ
α = sin θ , and β = − cos θ .
2 2

Problem 10. If u = (a + bι)p+qι , express log u in the form α + βι.


CHAPTER XXI : Exponential Values of the Cosine and Sine 192

Solution.
log u = (p + qι) log(a + bι)
{ }
√ a bι
= (p + qι) log (a2 + b2 ) √ + √
(a2 + b2 ) (a2 + b2 )

= (p + qι) log (a2 + b2 ){cos γ + ι sin γ},

a b
where cos γ = √ , and sin γ = √ ,
(a2 + b2 ) (a2 + b2 )

= (p + qι) log eγι (a2 + b2 )
{ √ }
γι
= (p + qι) log e + log (a2 + b2 )
{ √ }
= (p + qι) γι + log (a2 + b2 )
√ { √ }
= p log (a2 + b2 ) − qγ + pγ + q log (a2 + b2 ) ι.
This is of the form α + βι, where
√ √
α = p log (a2 + b2 ) − qγ, and β = pγ + q log (a2 + b2 ).

Problem 11. Reduce (a + bι)p+qι to the form α + βι.


Solution. By P roblem 10 we can express log(a + bι)p+qι in the form α + βι;
therefore
(a + bι)p+qι = eα+βι = eα eβι = eα (cos β + ι sin β);
and this is of the form λ + ιµ, where λ = eα cos β, and µ = eα sin β.

{ Prove that±αι
Problem 12. }n { }
sin(α − θ) + e sin θ = sinn−1 α sin(α − nθ) + e ±αι sin nθ .
Solution. {sin(α − θ) + eαι sin θ}n = {sin(α − θ) + (cos α + ι sin α) sin θ}n
= (sin α cos θ + ι sin α sin θ)n = sinn α(cos θ + ι sin θ)n
= sinn α(cos nθ + ι sin nθ).

Again sinn−1 α{sin(α − nθ) + eαι sin nθ}


= sinn−1 α{sin(α − nθ) + (cos α + ι sin α) sin nθ}
= sinn−1 α{sin α cos nθ + ι sin α sin nθ}
= sinn α(cos nθ + ι sin nθ) :
thus the two expressions agree.
In a similar way we may proceed when we take the lower sign in the expressions.
CHAPTER XXII
Summation of Trigonometrical series

Problem 1. Find the sum of n terms of the series


sin2 α + sin2 (α + β) + sin2 (α + 2 β) + . . .
Solution.
1
sin2 α = (1 − cos 2α),
2
1
sin2 (α + β) = {1 − cos 2(α + β)},
2
1
sin2 (α + 2β) = {1 − cos 2(α + 2β)},
2
and so on.
Hence the sum of n terms
n 1
= − {cos 2α + cos 2(α + β) + cos 2(α + 2β) + . . .}
2 2
n cos{2α + (n − 1)β} sin nβ
= − .
2 2 sin β

Problem 2. Find the sum of n terms of the series


sin3 α + sin3 (α + β) + sin3 (α + 2 β) + . . .
Solution.
1
sin3 α = (3 sin α − sin 3α),
4
1
sin3 (α + β) = {3 sin(α + β) − sin 3(α + β)},
4
1
sin3 (α + 2β) = {3 sin(α + 2β) − sin 3(α + 2β)},
4
and so on
Hence the sum of n terms
3
= {sin α + sin(α + β) + sin(α + 2β) + . . .}
4
1
− {sin 3α + sin 3(α + β) + sin 3(α + 2β) + . . .}
4 ( ) ( )
n−1 nβ n−1 3nβ
sin α + β sin sin 3α + 3β sin
3 2 2 1 2 2
= − .
4 1 4 3
sin β sin β
2 2

Problem 3. Find the sum of n terms of the series


cos4 α + cos4 (α + β) + cos4 (α + 2 β) + . . .
1
Solution. We have cos2 θ = (1 + cos 2θ);
2
1 1
∴ cos θ = (1 + cos 2θ)2 = (1 + 2 cos 2θ + cos2 2θ)
4
4( 4 )
1 1 + cos 4θ 3 1 1
= 1 + 2 cos 2θ + = + cos 2θ + cos 4θ.
4 2 8 2 8
Apply this transformation to every term of the proposed series; thus the sum of n
terms
3n 1
= + {cos 2α + cos 2(α + β) + cos 2(α + 2β) + . . .}
8 2
1
+ {cos 4α + cos 4(α + β) + cos 4(α + 2β) + . . .}
8
3n cos{2α + (n − 1)β} sin nβ cos{4α + (n − 1)2β} sin 2nβ
= + + .
8 2 sin β 8 sin 2β
CHAPTER XXII : Summation of Trigonometrical series 194

sin θ + sin 3 θ + sin 5 θ + . . . to n terms


Problem 4. Show that tan nθ = .
cos θ + cos 3 θ + cos 5 θ + . . . to n terms
Solution. sin θ + sin 3θ + sin 5θ + . . . to n terms
sin{θ + (n − 1)θ} sin nθ sin2 nθ
= =
sin θ sin θ
cos θ + cos 3θ + cos 5θ + . . . to n terms
cos{θ + (n − 1)θ} sin nθ sin nθ cos nθ
= = .
sin θ sin θ
Divide the former result by the latter; thus we obtain tan nθ.

Problem 5. Sum to n terms the series


cos θ cos(θ + α) + cos(θ + α) cos(θ + 2 α) + cos(θ + 2 α) cos(θ + 3 α) + . . .
1 1
Solution. cos A cos B = cos(A − B) + cos(A + B).
2 2
Apply this transformation to every term of the proposed series; thus the sum of n
terms
n 1
= cos α + {cos(2θ + α) + cos(2θ + 3α) + cos(2θ + 5α) + . . .}
2 2
n cos{2θ + α + (n − 1)α} sin nα
= cos α +
2 2 sin α
n cos(2θ + nα) sin nα
= cos α + .
2 2 sin α

Problem 6. Show that


sin θ − sin 2 θ + sin 3 θ − . . . to n terms n+1
= tan (π + θ).
cos θ − cos 2 θ + cos 3 θ − . . . to n terms 2
Solution. By Art. 307 (page 353) we have
sin θ − sin 2θ + sin 3θ − . . . to n terms
{ }
(n − 1)(θ + π) n(θ + π)
sin θ+ sin
2 2
= .
θ+π
sin
2
And cos θ − cos 2θ + cos 3θ − . . . to n terms
{ }
(n − 1)(θ + π) n(θ + π)
cos θ+ sin
2 2
=
θ+π
sin
2
Divide the former by the latter : the result
{ } { }
(n − 1)(θ + π) (n − 1)(θ + π)
sin θ + sin θ + +π
2 2
= { }= { }
(n − 1)(θ + π) (n − 1)(θ + π)
cos θ + cos θ + +π
2 2
n+1
sin (θ + π) n+1
= 2 = tan (θ + π).
n+1 2
cos (θ + π)
2

Problem 7. Sum to n terms the series


sin(p + 1 )θ cos θ + sin(p + 2 )θ cos 2 θ + . . .
1 1
Solution. sin A cos B = sin(A + B) + sin(A − B).
2 2
Apply this transformation to every term of the proposed series; thus the sum of n
terms
n sin pθ 1
= + {sin(p + 2)θ + sin(p + 4)θ + sin(p + 6)θ + . . .}
2 2
n sin pθ sin(p + 1 + n)θ sin nθ
= + .
2 2 sin θ
CHAPTER XXII : Summation of Trigonometrical series 195

Problem 8. Sum to n terms the series


sin α sin 2 α + sin 2 α sin 3 α + sin 3 α sin 4 α + . . .
1 1
Solution. sin A sin B = cos(A − B) − cos(A + B).
2 2
Apply this transformation to every term of the proposed series; thus the sum of n
terms
n 1
= cos α − {cos 3α + cos 5α + cos 7α + . . .}
2 2
n cos{3α + (n − 1)α} sin nα n cos(n + 2)α sin nα
= cos α − = cos α − .
2 2 sin α 2 2 sin α

Problem 9. Deduce from the result of Problem 8 the sum to n terms of the series
1 · 2 + 2 · 3 + 3 · 4 + ...
Solution. Suppose that in the preceding result we put for the sines of the angles
their values from Art. 274 (page 348); the proposed series become an expansion in
powers of α, and it is obvious that the coefficient of α2 is
1 · 2 + 2 · 3 + 3 · 4 . . . + n(n + 1).
We must therefore find the coefficient of α2 in the expansion of the expression found
(
for the sum of the Trigonometrical ) and equate it to the above.
Series,
n n α2 α4
Now cos α = 1− + − . . . , so that the coefficient of α2 in this term is
2 2 2 4
n
− ;
4
{ }{ }
(n + 2)2 2 n 3 α3
1− α + ... nα − + ...
cos(n + 2)α sin nα 2 6
and = ( )
2 sin α α3
2 α− + ...
6
{ }{ }
(n + 2)2 2 n2 α2
1− α + ... 1− + ...
2 6
=n ( )
α2
2 1− + ...
6
{ }{ }{ }−1
n (n + 2)2 2 n2 α2 α2
= 1− α + ... 1− + ... 1− + ...
2 2 6 6
{ }{ }{ }
n (n + 2)2 2 n2 α2 α2
= 1− α + ... 1− + ... 1+ + ... .
2 2 6 6
Multiply out and it will be found 2
{ that the coefficient
} of α is
n −(n + 2)2 n2 1
− + .
2 2 6 6
Hence the required sum { }
n n (n + 2)2 n2 1
=− − − − +
4 2 2 6 6
n n(n + 2)2 n3 n
=− + + −
4 4 12 12
n
= {3(n + 2)2 + n2 − 1 − 3}
12
n n
= {4n2 + 12n + 8} = (n2 + 3n + 2)
12 3
n(n + 1)(n + 2)
= .
3

Problem 10. Sum to n terms the series


sin 3 θ sin θ + sin 6 θ sin 2 θ + sin 12 θ sin 4 θ + . . .
CHAPTER XXII : Summation of Trigonometrical series 196

1 1
Solution. sin A sin B = cos(A − B) − cos(A + B).
2 2
Apply this transformation to every term of the proposed series; hence the sum of
n terms
1 1 1
= (cos 2θ − cos 4θ) + (cos 4θ − cos 8θ) + (cos 8θ − cos 16θ) + . . .
2 2 2
1( )
= cos 2θ − cos 2n+1 θ
2

Sum to infinity the following series contained in the Problems from 11 to


16 inclusive:

cos θ cos2 θ cos3 θ


Problem 11. cos θ + cos 2 θ + cos 3 θ + cos 4 θ + . . .
1 1 ·2 3
Solution. Omitting the first term, we can find the sum of the rest of the series by
Art. 311 (page 355); we must put cos θ for c, and put α = β = θ. Hence the sum of the
whole series
2 2
= cos θ + ecos θ cos(θ + sin θ cos θ) − cos θ = ecos θ cos(θ + sin θ cos θ).

sin 2 θ sin 3 θ
Problem 12. sin θ − + − ...
1 ·2 3
Solution. In the first series of Art. 311 (page 355) put −1 for c, and 0 for α, and θ for
β, and change the sign. Thus we obtain for the required sum
−e− cos θ sin(− sin θ), that is e− cos θ sin(sin θ).

cos 2 θ cos 4 θ
Problem 13. 1 − + − ...
1 ·2 4
Solution. For cos 2θ, cos 4θ, . . . put the exponential values; thus denoting eιθ by z, the
proposed series becomes{ }
1 1 ( 2 ) 1 ( 4 )
2− z + z −2 + z + z −4 − . . .
2 2 4
1 −1
= {cos z + cos z }
2
1
= {cos θ + ι sin θ) + cos(cos θ − ι sin θ)}
2
1
= cos(cos θ) cos(ι sin θ) = cos(cos θ)(e− sin θ + esin θ ).
2

3 4 5
Problem 14. 2 cos θ + cos2 θ + cos3 θ + cos4 θ + . . .
2 3 4
Solution. The proposed series
= cos θ + cos2 θ + cos3 θ + cos4 θ + . . .
1 1 1
+ cos θ + cos2 θ + cos3 θ + cos4 θ + . . .
2 3 4
cos θ
= − log(1 − cos θ).
1 − cos θ

sin 2 θ cos2 θ sin 3 θ cos3 θ


Problem 15. sin θ cos θ + + + ...
1 ·2 3
Solution. In the first series of Art. 311 (page 355) put 0 for α and cos θ for c, and θ for
2
β. Thus the sum = ecos θ sin(sin θ cos θ).

sin θ sin2 θ
Problem 16. cos θ + cos 2 θ + cos 3 θ + . . .
1 1 ·2
CHAPTER XXII : Summation of Trigonometrical series 197

Solution. In the second series of Art. 311 (page 355) put θ for α, and θ for β, and sin θ
for c. Thus
sin2 θ sin3 θ
sin θ cos 2θ + cos 3θ + cos 4θ + . . .
2 3
( )
= esin θ cos θ cos θ + sin2 θ − cos θ.
sin2 θ sin3 θ
Therefore cos θ + sin θ cos 2θ + cos 3θ + cos 4θ + . . .
2 3
( )
= esin θ cos θ cos θ + sin2 θ .

( )
1 1 θ
Problem 17. Show that cos θ − cos 2 θ + cos 3 θ − . . . = log 2 cos .
2 3 2
Solution. Put the exponential values for cos θ, cos 2θ, cos 3θ, . . . Thus denoting eιθ by
z, the proposed
{ series becomes } { }
1 1 2 1 1 1 1 1 1
z − z + z3 − z4 + . . . + z −1 − z −2 + z −3 − z −4 + . . . ;
2 2 3 4 2 2 3 4
1 1 ( ) 1
that is log(1 + z) + log 1 + z −1 , that is log(1 + z)(1 + z −1 ),
2 2 2
1 ( −1
) 1
that is log 2 + z + z , that is log(2 + 2 cos θ),
2 ( ) 2 ( )
1 θ θ
that is log 4 cos2 , that is log 2 cos .
2 2 2

1 1 1
Problem 18. Show that cos 2 θ + cos 6 θ + cos 10 θ + . . . = log(cot θ).
3 5 2
Solution. Proceed
{ as in the solution of}P roblem
{ 17. Thus the proposed series} becomes
1 2 1 6 1 10 1 −2 1 −6 1 −10
z + z + z + ... + z + z + z + ...
2 3 5 2 3 5
( )
1 1 + z2 1 1 + z −2 1 1 + z2 1 + z −2
= log + log = log ×
4 1−z 2 4 1 − z −2 4 1−z 2 1 − z −2
1 2 + z 2 + z −2 1 2 + 2 cos 2θ 1 1 + cos 2θ
= log = log = log
4 2 − z 2 − z −2 4 2 − 2 cos 2θ 4 1 − cos 2θ
1 1
= log cot2 θ = log cot θ.
4 2

Problem 19. Show that ( )


x 2 sin 2 θ x 3 sin 3 θ cosec θ
x sin θ − + − . . . = cot−1 + cot θ .
2 3 x
Solution. Put the exponential values for sin θ, sin 2θ, sin 3θ, . . . Thus, denoting eιθ by
z, the proposed series{ becomes }
1 1 1 1
xz − x2 z 2 + x3 z 3 − x4 z 4 + . . .
2ι { 2 3 4 }
1 −1 1 2 −2 1 3 −3 1
− xz − x z + x z − x4 z −4 + . . .
2ι 2 3 4
1 1 ( −1
)
This = log(1 + xs) − log 1 + xz
2ι 2
1 1 + xz 1 1 + x(cos θ + ι sin θ)
= log = log .
2ι 1 + xz −1 2 1 + x(cos θ − ι sin θ)
x sin θ
Assume tan ϕ = ; thus the sum of the proposed series
1 + x cos θ
1 1 + ι tan ϕ 1 cos ϕ + ι sin ϕ
= log = log
2ι 1 − ι tan ϕ 2 cos ϕ − ι sin ϕ
1 eιϕ 1 1 + x cos θ
= log −ιϕ = log e2ιϕ = ϕ = cot−1
2ι (e 2ι ) x sin θ
cosec θ
= cot−1 + cot θ .
x
CHAPTER XXII : Summation of Trigonometrical series 198

Problem 20. Show that ( )


θ θ sin 2 θ
log cos θ + log cos+ log cos 2 + . . . = log .
2 2 2θ
θ θ θ sin θ
Solution. By Art. 129 (page 331) the limit of cos cos cos . . . is ;
2 4 8 θ
θ θ θ cos θ sin θ sin 2θ
therefore the limit of cos θ cos cos cos . . . is , that is . Then take the
2 4 8 θ 2θ
logarithms of both sides.

Sum the following series to n terms contained in the Problems from 21 to 35 in-
clusive :

( )2 ( )2 ( )2
θ θ θ θ θ
Problem 21. sin θ sin + 2 sin sin + 4 sin sin + ...
2 2 4 4 8
Solution. ( )2
θ 1 1 1
sin θ sin sin θ(1 − cos θ) = sin θ − sin 2θ,
=
2 2 2 4
( ) ( )
θ θ 2 θ θ θ 1
2 sin sin = sin 1 − cos = sin − sin θ.
2 4 2 2 2 2
( ) ( )
θ θ 2 θ θ θ θ
4 sin sin = 2 sin 1 − cos = 2 sin − sin ,
4 8 4 4 4 2
( ) ( )
θ θ 2 θ θ θ θ
8 sin sin = 4 sin 1 − cos = 4 sin − 2 sin .
8 16 8 8 8 4
Proceeding in this way, and adding the terms, we see that all cancel on the right-
hand side except two, namely
θ 1
2n−2 sin n−1 − sin 2θ.
2 4

θ θ θ θ θ
Problem 22. tan sec θ + tan sec + tan sec + . . .
2 4 2 8 4
Solution. ( )
θ θ θ θ
sin sin θ − sin θ cos − cos θ sin
θ 2 2 2 2
tan sec θ = = =
2 θ θ θ
cos cos θ cos cos θ cos cos θ
2 2 2
θ
= tan θ − tan ;
2
θ θ θ θ
therefore tan sec = tan − tan ,
4 2 2 4
θ θ θ θ
tan sec = tan − tan ,
8 4 4 8
and so on.
Then adding the terms, we see that all cancel on the right-hand side except two,
namely
θ
tan θ − tan n .
2

Problem 23. cot θ cosec θ + 2 cot 2 θ cosec 2 θ + 2 2 cot 2 2 θ cosec 2 2 θ + . . .


Solution.
θ
cos θ 2 cos2 − 1
cot θ cosec θ = = 2
sin2 θ sin2 θ
2 θ
2 cos 1 1 1
= 2 − = − ;
2
θ 2 θ sin2 θ 2 θ sin2 θ
4 cos sin 2 sin
2 2 2
CHAPTER XXII : Summation of Trigonometrical series 199

1 2
therefore 2 cot 2θ cosec 2θ = − ,
sin2 θ sin2 2θ
2 4
4 cot 4θ cosec 4θ = − .
sin2 2θ sin2 4θ
Proceeding in this way, and adding the terms, we see that all cancel on the right-
hand side except two, namely
1 2n−1
− 2 2n−1 θ
.
θ sin
2 sin2
2

1 1 1
Problem 24. + + + ...
sin θ sin 2 θ sin 2 θ sin 3 θ sin 3 θ sin 4 θ
Solution.
1 1 sin(2θ − θ) 1 sin 2θ cos θ − cos 2θ sin θ
= · = ·
sin θ sin 2θ sin θ sin θ sin 2θ sin θ sin θ sin 2θ
1
= (cot θ − cot 2θ).
sin θ
1 1 sin(3θ − 2θ)
Similarly =
sin 2θ sin 3θ sin θ sin 2θ sin 3θ
1
= (cot 2θ − cot 3θ);
sin θ
1 1
= (cot 3θ − cot 4θ).
sin 3θ sin 4θ sin θ
Proceeding in this way, and adding the terms, we see that all cancel on the right-
hand side except two, namely
1
{cot θ − cot(n + 1)θ}.
sin θ

1 1 1
Problem 25. − + − ...
sin θ cos 2 θ cos 2 θ sin 3 θ sin 3 θ cos 4 θ
π
Solution. Let ϕ = θ + ; thus the proposed series becomes
2
1 1 1
+ + + ...
cos ϕ cos 2ϕ cos 2ϕ cos 3ϕ cos 3ϕ cos 4ϕ
1 1 sin(2ϕ − ϕ) 1
Now = = (tan 2ϕ − tan ϕ),
cos ϕ cos 2ϕ sin ϕ cos ϕ cos 2ϕ sin ϕ
1 1 sin(3ϕ − 2ϕ) 1
= = (tan 3ϕ − tan 2ϕ),
cos 2ϕ cos 3ϕ sin ϕ cos 2ϕ cos 3ϕ sin ϕ
1 1 sin(4ϕ − 3ϕ) 1
= = (tan 4ϕ − tan 3ϕ).
cos 3ϕ cos 4ϕ sin ϕ cos 3ϕ cos 4ϕ sin ϕ
Proceeding in this way, and adding the terms, we see that all cancel on the right-
hand side except two, namely
1
{tan(n + 1)ϕ − tan ϕ}.
sin ϕ

1 1 1
Problem 26. tan−1 + tan−1 + tan−1 + ...
1 + 1 + 12 1 + 2 + 22 1 + 3 + 32
1 1 1
Solution. tan−1 = tan−1 − tan−1 ;
1 + m + m2 m 1+m
1 1
this is obvious, for by taking the tangent of tan−1 − tan−1 , we obtain
m 1+m
1 1

m m+1 1
, that is 2 .
1 m +m+1
1+
m(m + 1)
CHAPTER XXII : Summation of Trigonometrical series 200

Apply this transformation to every term of the proposed series; thus we obtain
1 1 1 1 1 1
tan−1 − tan−1 + tan−1 − tan−1 + tan−1 − tan−1 + . . . ,
1 2 2 3 3 4
1 π 1
that is tan−1 1 − tan−1 , that is − tan−1 .
n+1 4 n+1

x x
Problem 27. tan−1 x + tan−1 + tan−1 + ...
1 + 1 · 2 · x2 1 + 2 · 3 · x2
x
Solution. tan−1 = tan−1 (m + 1)x − tan−1 mx;
1 + m(m + 1)x2
this is obvious, for by taking the tangent of tan−1 (m + 1)x − tan−1 mx, we obtain
(m + 1)x − mx x
, that is .
1 + m(m + 1)x2 1 + m(m + 1)x2
Apply this transformation to every term of the proposed series after the first; thus
we obtain
tan−1 x + tan−1 2x − tan−1 x + tan−1 3x − tan−1 2x + . . . ,
that is tan−1 nx.

α 3α α 3α
Problem 28. sin α sin 3 α + sin sin + sin 2 sin 2 + . . .
2 2 2 2
1 1
Solution. sin A sin B = cos(A − B) − cos(A + B).
2 2
Apply this transformation to every term of the proposed
( series;)thus we obtain
1 1 1 α
(cos 2α − cos 4α) + (cos α − cos 2α) + cos − cos α + . . .
2 2( 2) 2
1 α
that is cos n−2 − cos 4α .
2 2

1 1 1
Problem 29. + + + ...
cos θ + cos 3 θ cos θ + cos 5 θ cos θ + cos 7 θ
Solution.
1 1 1 sin(2θ − θ)
= = ·
cos θ + cos 3θ 2 cos θ cos 2θ 2 sin θ cos θ cos 2θ
1
= (tan 2θ − tan θ),
2 sin θ
1 1 1 sin(3θ − 2θ)
= = ·
cos + cos 5θ 2 cos 2θ cos 3θ 2 sin θ cos 2θ cos 3θ
1
= (tan 3θ − tan 2θ),
2 sin θ
1 1 1 sin(4θ − 3θ)
= = ·
cos θ + cos 7θ 2 cos 3θ cos 4θ 2 sin θ cos 3θ cos 4θ
1
= (tan 4θ − tan 3θ).
2 sin θ
Proceeding in this way, and adding the terms, we see that all cancel on the right-
hand side except two, namely
1
{tan(n + 1)θ − tan θ}.
2 sin θ

sin θ sin 2 θ sin 3 θ


Problem 30. + + + ...
cos 2 θ + cos θ cos 4 θ + cos θ cos 6 θ + cos θ
Solution.
sin θ sin θ
=
cos 2θ + cos θ θ 3θ
2 cos cos
 2 
2
 
1 1 1
= − ;
4 sin
θ  cos 3θ cos
θ 
2 2 2
CHAPTER XXII : Summation of Trigonometrical series 201

 
 
sin 2θ sin 2θ 1 1 1
= = − .
cos 4θ + cos θ
2 cos

cos

4 sin
θ  cos 5θ cos
3θ 
2 2 2  2 2 
 
sin 3θ sin 3θ 1 1 1
= = − .
cos 6θ + cos θ 5θ 7θ θ  7θ 5θ 
2 cos cos 4 sin cos cos
2 2 2 2 2
Proceeding in this way, and adding the terms, we see that all cancel on the right
hand except two, namely  
 
1 1 1
− .
θ 
cos 
(2n + 1)θ θ
4 sin cos
2 2 2

sin θ 3 sin 3 θ 3 2 sin 3 2 θ


Problem 31. + + + ...
1 + 2 cos θ 1 + 2 cos 3 θ 1 + 2 cos 3 2 θ
Solution.
sin θ sin θ sin θ
= ( )=
1 + 2 cos θ θ θ
1 + 2 1 − 2 sin2 3 − 4 sin2
2 2
θ θ θ 3θ
sin θ sin sin θ sin cos − cos
= ( 2 )= 2 = 2 2
θ θ 3θ 3θ
sin 3 − 4 sin2 sin 2 sin
2 2 2 2
θ 3θ θ θ θ 3θ
2 cos − 2 cos cos (1 + 2 cos θ) + cos − 2 cos θ cos − 2 cos
= 2 2 = 2 2 2 2
3θ 3θ
4 sin 4 sin
2 2
θ θ θ 3θ
cos cos − 2 cos θ cos − 2 cos
= 2 + 2 2 2
θ 3θ
4 sin 4 sin
2 2
θ 3θ
cos 3 cos
= 2 − 2 = 1 cot θ − 3 cot 3θ .
θ 3θ 4 2 4 2
4 sin 4 sin
2 2
3 sin 3θ 3 3θ 9 9θ
Similarly = cot − cot ,
1 + 2 cos 3θ 4 2 4 2
32 sin 32 θ 9 9θ 27 27θ
= cot − cot .
1 + 2 cos 32 θ 4 2 4 2
Proceeding in this way, and adding the terms, we see that all cancel on the right
hand except two, namely
1 θ 3n 3n θ
cot − cot .
4 2 4 2

( ) ( ) ( )
Problem 32. cot−1 2a−1 + a + cot−1 2a−1 + 3a + cot−1 2a−1 + 6a
−1
( )
+ cot 2a−1 + 10a + . . .
{ }
m(m + 1) m m+1
Solution. cot−1 2a−1 + a = cot−1 a − cot−1 a.
2 2 2
m m+1
For if we take the cotangent of cot−1 a − cot−1 a,
2 2
m m+1
a· a+1
we obtain 2 2 ,
m+1 m
a− a
2 2
m(m + 1)
that is a + 2a−1 .
2
CHAPTER XXII : Summation of Trigonometrical series 202

Apply this transformation to every term of the proposed series; thus we obtain
a 2a 2a 3a 3a 4a
cot−1 − cot−1 + cot−1 − cot−1 + cot−1 − cot−1 + ...;
2 2 2 2 2 2
a n + 1
that is cot−1 − cot−1 a.
2 2

1 1 1
Problem 33. sec θ + 2 sec θ sec 2 θ + 3 sec θ sec 2 θ sec 2 2 θ + . . .
2 2 2
Solution.
1 1 sin θ sin θ sin(2θ − θ)
sec θ = = = =
2 2 cos θ 2 sin θ cos θ sin 2θ sin 2θ
= cos θ − cot 2θ sin θ = sin θ(cot θ − cot 2θ);

1 1
sec θ sec 2θ = sec θ sin 2θ(cot 2θ − cot 4θ)
22 2
= sin θ(cot 2θ − cot 4θ);
1 1
sec θ sec 2θ sec 4θ = sec θ sin 2θ(cot 4θ − cot 8θ)
23 2
= sin θ(cot 4θ − cot 8θ).
Proceeding in this way, and adding the terms, we see that all cancel on the right-
hand side except two, namely
sin θ(cot 4θ − cot 2n θ).

1 1 1
Problem 34. log tan 2 θ + 2 log tan 2 2 θ + 3 log tan 2 3 θ + . . .
2 2 2
sin 2θ sin2 2θ 2 sin2 2θ 4 sin2 2θ
Solution. tan 2θ = = = = ;
cos 2θ sin 2θ cos 2θ sin 4θ 2 sin 4θ
1 1
therefore log tan 2θ = log 2 sin 2θ − log 2 sin 4θ,
2 2
1 1 1
log tan 2 2
θ = log 2 sin 4θ − log 2 sin 8θ,
22 2 22
1 1 1
log tan 23 θ = 2 log 2 sin 8θ − 3 log 2 sin 16θ.
23 2 2
Proceeding in this way, and adding the terms, we see that all cancel on the right-
hand side except two, namely
1
log 2 sin 2θ − n log 2 sin 2n+1 θ.
2

θ θ θ θ θ θ
Problem 35. cos + 2 cos cos 2 + 2 2 cos cos 2 cos 3 + . . .
2 2 2 2 2 2
Solution.
θ θ 2 θ θ
θ 2 sin cos
2 2 sin θ sin θ 2 cos 4 − cos 2
cos = = =
2 θ θ 2 θ
2 sin 2 sin sin
 2 2  2
 2 cos2 θ  { }
sin θ 4 θ sin θ θ θ
= − cos = cot − cot ;
2  θ θ 2 2 4 2
2 sin cos
4 4
θ
{ }
θ θ θ sin 2 θ θ
2 cos cos 2 = 2 cos cot − cot
2 2 { 2
2 8 } 4
sin θ θ θ
= cot − cot ;
2 8 4
θ
{ }
θ θ θ θ sin 2 θ θ
22 cos cos 2 cos 3 = 2 cos · cot − cot
2 2 2 { 2
2 16} 8
sin θ θ θ
= cot − cot .
2 16 8
CHAPTER XXII : Summation of Trigonometrical series 203

Proceeding in this way, and adding the terms, we see that all cancel on the right-
hand side except two, namely
{ }
sin θ θ θ
cot n+1 − cot .
2 2 2

Problem 36. An equilateral polygon is inscribed in a circle and from any point in the
circumference chords are drawn to the angular points : find the sum of the squares
of the chords and the sum of the fourth powers of the chords.
Solution. Let R denote the radius of the circle, n the number of sides of the polygon.
π
Put β for . Let 2α denote the angular distance of a fixed point in the circumference
n
from one of the angular points; then the angular distances from the other angular
points in succession will be
2α + 2β, 2α + 4β, 2α + 6β, . . . 2α + 2(n − 1)β.
The lengths of the succession chords will be
2R sin α, 2R sin(α + β), 2R sin(α + 2β), . . . 2R sin{α + (n − 1)β}.
To find the sum of the squares of the chords, we have
1
sin2 θ = (1 − cos 2θ);
2
and applying this transformation to every term of the proposed series, we obtain
2nR2 − 2R2 {cos 2α + cos(2α + 2β) + cos(2α + 4β) + . . .}.
The sum of the series of cosines is zero, as in Art. 304 (page 353) and thus the
result is 2nR2 .
Next, to find the sum of the fourth powers of the chords. We have
3 1 1
sin4 θ = − cos 2θ + cos 4θ;
8 2 8
and applying this transformation to every term of the proposed series, we obtain
6nR − 8R {cos 2α + cos(2α + 2β) + cos(2α + 4β) + . . .}
4 4

+ 2R4 {cos 4α + cos(4α + 4β) + cos(4α + 8β) + . . .},


that is 6nR4 .

Problem 37. Circles are inscribed in triangles, whose bases are the sides of a
regular polygon of n sides, and whose vertices ( lie in one of)the angular points : show
π
that the sum of the radii of the circles is 2r 1 − n sin2 , where r is the radius of
2n
the circle circumscribing the polygon.
Solution. Let A be the common vertex; let B, C, . . . be the successive angular points.
π
Put β for .
n
Let P Q be one of the sides of the polygon, such that the arc ABP contains m of the
sides; then the angle AQP = mβ, the angle P AQ = β, and the angle AP Q = π−(m+1)β.
Let P Q = c, and let rm denote
{ the radius of the circle } inscribed in AP Q. Then
1 1
rm cot AP Q + cot AQP = c,
{ 2 2 }
π − (m + 1)β mβ
therefore rm cot + cot = c,
{ 2 }
2
m+1 mβ
therefore rm tan β + cot = c,
2 2
β m+1 mβ
therefore rm cos = c cos β sin
2 { 2 2 }
c 2m + 1 β
= sin β − sin .
2 2 2
Now there are n − 2 circles in all which can be drawn; so that we have to sum up
the values of { }
c β 2m + 1 β
sec sin β − sin
2 2 2 2
CHAPTER XXII : Summation of Trigonometrical series 204

for values of m from 1 to n{− 2 inclusive. The


} sum then is
 
 sin 3β + (n − 3) β sin n − 2 β 
c β 2 2 2 β
sec − (n − 2) sin ,
2 2 β 2
sin
 2 
 
c β cos β β
that is sec − (n − 2) sin ,
2 2  sin β 2
 2 
 
c β 1 β
that is sec − n sin .
2 2  sin β 2
2
But c = 2r sin β; thus we get
β
r sin β rn sin β sin
− 2 , that is 2r − 2rn sin2 β ,
β β β 2
cos sin cos
2 2 ( 2 )
π
that is 2r 1 − n sin2 .
2n

Problem 38. Circles are inscribed in triangles whose bases are the sides of a regular
polygon of n sides and whose vertices lie in one of the angular points ; r is the radius
of the circle circumscribing the polygon : show that the sum of the areas of the circles
is { }
π n π n−4
16 πr 2 sin2 sin2 + .
2n 4 2n 8
Solution. Use the notation of the preceding solution. The area of the mth circle
{ }
πc2 β 2m + 1 β 2
= sec2 sin β − sin
4 2 2 2
{ }
πc2 2 β 2 2m + 1 2m + 1 β β
= sec sin β − 2 sin β sin + sin2
4 2 2 2 2 2
{ }
πc2 2 β 2m + 1 β β
= sec 1 − cos(2m + 1)β − 4 sin β sin + 2 sin2 .
8 2 2 2 2
Then as before we have to sum this expression for the values of m from 1 to n − 2
inclusive. Thus we obtain

 ( )
πc2 β β cos{3β + (n − 3)β} sin(n − 2)β
sec2 (n − 2) 1 + 2 sin2 −
8 2  2 sin β
{ } 
3β n−3 n−2
sin + β sin β
β 2 2 2
− 4 sin ;
2
sin
β 
2
{ ( ) }
πc2 β β
and this = sec2 (n − 2) 1 + 2 sin2 − 2 cos β
8 2 2
{ }
πc2 2 β β
= sec n − 4 + 2n sin2
8 2 2
β 4r2 sin2 β β
But c = 2r sin β; therefore c2 sec2 = = 16r2 sin2 ;
2 2
β 2
cos
{ 2 }
π n π n−4
so that the result = 16πr 2 sin2 sin2 + .
2n 4 2n 8

Problem 39. Show that if n be a positive integer


n sin θ + (n − 1) sin 2θ + (n − 2) sin 3θ + . . . + sin nθ
CHAPTER XXII : Summation of Trigonometrical series 205

n+1 θ sin(n + 1)θ


= cot − .
2 2 θ
4 sin2
2
Solution. Let Sn denote the sum of the series; so that
Sn = n sin θ + (n − 1) sin 2θ + (n − 2) sin 3θ + . . . + sin nθ.
In like manner let Sn−1 denote the sum of the series formed by changing n into
n − 1, so that
Sn−1 = (n − 1) sin θ + (n − 2) sin 2θ + . . . + sin(n − 1)θ;

∴ Sn − Sn−1 = sin θ + sin 2θ + sin 3θ + . . . + sin nθ


n+1 nθ
sin θ sin { }
= 2 2 = 1 θ
cos − cos
2n + 1
θ .
θ θ 2 2
sin sin
2 2
Similarly we have
{ }
1 θ 2n − 1
Sn−1 − Sn−2 = cos − cos θ ;
θ 2 2
2 sin
2 { }
1 θ 2n − 3
Sn−2 − Sn−3 = cos − cos θ
θ 2 2
2 sin
2
.............................................
{ }
1 θ 5θ
S2 − S1 = cos − cos .
θ 2 2
2 sin
2 { }
1 θ 3θ
S1 = cos − cos .
θ 2 2
2 sin
2
Hence by addition from this series of equations we obtain
{ }
1 θ 3θ 5θ 2n + 1
Sn = n cos − cos − cos . . . − cos θ
θ 2 2 2 2
2 sin
2  { } 
3θ θ nθ
 cos + (n − 1) sin 
1 θ 2 2 2
= n cos −
2 sin  
θ 2 θ
sin
2 2
(n + 2) nθ
cos θ sin
n
= cot −
θ 2 2 = n cot θ − sin(n + 1)θ − sin θ
2 2 2 θ 2 2 θ
2 sin 4 sin2
2 2
n+1 θ sin(n + 1)θ
= cot − .
2 2 θ
4 sin2
2

Problem 40. Show that if n be a positive integer


(n + 1)n sin θ + n(n − 1) sin 2θ + (n − 1)(n − 2) sin 3θ + . . . + 2 · 1 sin nθ
{ }
n(n + 3) θ 1 θ 3θ 2n + 3
= cot − cosec 3 cos − cos θ .
2 2 4 2 2 2
Solution. Let Sn denote the required sum, and Sn−1 the sum of the series when n is
changed to n − 1. Thus
Sn = (n + 1)n sin θ + n(n − 1) sin 2θ + . . . + 2 · 1 sin nθ,
Sn−1 = n(n − 1) sin θ + (n − 1)(n − 2) sin 2θ + . . . + 2 · 1 sin(n − 1)θ;
∴ Sn − Sn−1 = 2{n sin θ + (n − 1) sin θ + . . . + sin nθ};
that is, by P roblem 39,
θ sin(n + 1)θ
Sn − Sn−1 = (n + 1) cot − .
2 θ
2 sin2
2
CHAPTER XXII : Summation of Trigonometrical series 206

θ sin nθ
Similarly Sn−1 − Sn−2 = n cot − ;
2 θ
2 sin2
2
.................................
θ sin 3θ
S2 − S1 = 3 cot − ;
2 θ
2 sin2
2
θ sin 2θ
S1 = 2 cot − .
2 θ
2 sin2
2
Hence by addition from this series of equations we obtain
n(n + 3) θ 1
Sn = cot − {sin 2θ + sin 3θ + . . . + sin(n + 1)θ}
2 2 θ
2 sin2
{ 2 }
θ nθ
sin 2θ + (n − 1) sin
n(n + 3) θ 2 2
= cot − .
2 2 2 θ θ
2 sin sin
2 2
3θ 2n + 3
n(n + 3) θ cos − cos θ
= cot − 2 2 .
2 2 θ
4 sin3
2
CHAPTER XXIII
Resolution of Trigonometrical Expressions Into Factors

Problem 1. Sum the infinite series


1 1 1 1
+ 2 + 2 + 2 + ...
12 2 3 4
Solution. By Art. 274 (page 348) we have
sin θ θ2 θ4 θ6
=1− + − + ...;
θ 3 5 7
and by Art. 320 (page 359) (
we have) ( )( )
sin θ θ2 θ2 θ2
= 1− 2 1− 2 2 1− 2 2 ...
θ π 2 π 3 π
Take the logarithms
{ of the equivalent expressions;
} thus
θ2 θ4 θ6
log 1− + − + ...
3 5 7
( ) ( ) ( )
θ2 θ2 θ2
= log 2
+ log 1 − 2 2 + log 1 − 2 2 + . . .
1−
π 2 π 3 π
Expand the logarithms; then both sides become series arranged according to pow-
ers of θ; and by equating the coefficients of θ2 we obtain
( )
θ2 1 1 1
− = −θ2 2
+ 2 2 + 2 2 + ... ;
3 π 2 π 3 π
1 1 1 π2
therefore 2
+ 2 + 2 + ... = .
1 2 3 6

Problem 2. Sum the infinite series


1 1 1 1
+ 4 + 4 + 4 + ...
14 2 3 4
Solution. Equate the coefficients of θ4 in the two equivalent series of the preceding
solution; thus since
{ } { } { }2
θ2 θ4 θ2 θ4 1 θ2 θ4
log 1− + ... =− − + ... − − + ... − ...
3 5 3 5 2 3 5
( )2 ( )
1 1 1 1 1 1 1
we have − =− + 4 + 4 + ... ;
5 2 3 2π 4 14 2 3
( )
1 1 1 1 1
therefore 4
+ 4 + 4 + . . . = 2π 4 −
1 2 3
( 72 ) 120
π4 1 1 π4
= − = .
12 3 5 90

Problem 3. Sum the infinite series


1 1 1 1
+ 2 + 2 + 2 + ...
12 3 5 7
Solution.
1 1 1 1
Let S = n + n + n + n + ...;
1 2 3 4
1 1 1 1
and let Σ = n + n + n + n + ....
1 3 5 7
1 1 1 1
Then S = n + n + n + n + ...
1 3 5 7
1 1 1 1
+ n + n + n + n + ...
2 4 6 8
CHAPTER XXIII : Trigonometrical Expressions Into Factors 208

1 1 1 1
= + n + n + n + ...
1n 3{ 5 7 }
1 1 1 1 1
+ n + n + n + + ...
2 1n 2 3 4n
1
= Σ + n S.
2
2n − 1
Therefore Σ= S.
2n
Hence Σ can be found when S is known.
π2 3 π2 π2
If n = 2 we have S = by P roblem 1; and then Σ = · = .
6 4 6 8

Problem 4. Sum the infinite series


1 1 1 1
+ 4 + 4 + 4 + ...
14 3 5 7
π4
Solution. In the preceding solution suppose n = 4; then we have S = by P roblem 2,
90
15 π 4 π4
and therefore Σ = · = .
16 90 96

π
Problem 5. α = , show that
4n
1
sin α sin 5 α sin 9 α . . . . . . sin(4n − 3 )α = 2 −n+ 2 .
Solution. By Art. 313 (page 355) we have
sin nϕ = 2n−1 sin ϕ sin(2β + ϕ) sin(4β + ϕ) . . . sin(2nβ − 2β + ϕ)
π
where β = .
2n
1 π 1
Let α = β, and let ϕ = α; then sin nϕ = sin = √ ; thus
2 4 2
1
√ = 2n−1 sin α sin 5α sin 9α . . . sin(4n − 3)α;
2
1
therefore sin α sin 5α sin 9α . . . sin(4n − 3)α = 2−n+ 2 .

Problem 6. A polygon of n sides inscribed in a circle is such that its sides subtend
angles α, 2 α, 3 α, . . . nα at the centre : show that the ratio of the area of this polygon

to the area of the regular inscribed polygon of n sides is equal to that of sin to
2
α
n sin .
2
Solution. Let r be the radius of the circle. The polygon can be resolved into n
triangles; and thus the area of the polygon
r2
= {sin α + sin 2α + sin 3α + . . . + sin α}
2
n+1 nα
r2 sin 2 α sin 2
= · α .
2 sin
2
But α + 2α + 3α + . . . + nα = 2π;
n(n + 1)α
that is = 2π,
2
n+1 2π
so that α= .
2 n
Now the area of the regular polygon of n sides
nr2 2π nr2 n+1
= sin = sin α.
2 n 2 2

sin
Hence the ratio of the former area to the latter = 2 .
α
n sin
2
CHAPTER XXIII : Trigonometrical Expressions Into Factors 209

Problem 7. The product of all the straight lines that can be drawn from one of the
angles of a regular polygon of n sides inscribed in a circle whose radius is a to all the
other angular points is na n−1 .
Solution. Let A, B, C, . . . be the angles of the polygon. From A draw straight lines to
the other angles. Let AP be the mth straight line, so that AP subtends at the centre
2π π
of the circle the angle m . Then AP = 2a sin mβ where β = .
n n
Thus the product of all the straight lines
= (2a) n−1
sin β sin 2β sin 3β . . . sin(n − 1)β
= nan−1 ;
for by Art. 318 (page 358) we have
n = 2n−1 sin β sin 2β sin 3β . . . sin(n − 1)β.

Problem 8. If p1 , p2 , . . . p2n−1 , p2n be the perpendiculars drawn from any point


in the circumference of a circle of radius a on the sides of a regular circumscribing
polygon of 2n sides, show that
an
p1 p3 p5 . . . p2n−1 + p2 p4 . . . p2n = n−2 .
2
Solution. Let A, B, C, . . . be the points of contact of the circle with the circumscribed
polygon taken in order. Let O be the fixed point, and suppose the arc OA to subtend
an angle 2ϕ at the centre of the circle. Then the angle between OA and the tangent
at A is ϕ; and the length of the perpendicular from O on this tangent is OA sin ϕ, that
is 2a sin2 ϕ. Thus we have
p1 = 2a sin2 ϕ.
π
Let β = , then we obtain in a similar way
2n
p2 = 2a sin2 (ϕ + β),
p3 = 2a sin2 (ϕ + 2β),
p4 = 2a sin2 (ϕ + 3β),
and so on.

Thus p1 p3 p5 . . . p2n−1 = (2a)n sin2 ϕ sin2 (ϕ + 2β) . . . sin2 {ϕ + (2n − 2)β}


{ }
sin nϕ 2
= (2a)n n−1
, by Art. 318(page 358),
2
an
= n−2 sin2 nϕ.
2
In the same way we have
p2 p4 . . . p2n = (2a)n sin2 (ϕ + β) sin2 (ϕ + 3β) . . . sin2 {ϕ + (2n − 1)β}
an
= n−2 cos2 nϕ.
2
Hence by addition we obtain
an ( 2 ) an
n−2
sin nϕ + cos2 nϕ , that is n−2 .
2 2

Problem 9. A polygon is described about a circle touching it at the angular points


of an inscribed polygon; the product of the perpendiculars drawn to the several sides
of the inscribed polygon from any point in the circumference of the circle is equal to
the product of the perpendiculars drawn from the same point to the several sides of
the circumscribed polygon.
Solution. Let A, B, C, D, . . . be the angular points of the inscribed polygon. Let O be
the fixed point from which the perpendiculars are drawn. Let the arc OA subtend an
angle 2α at the centre of the circle, let the arc OB subtend an angle 2β, the arc OC
an angle 2γ, and so on.
Let p1 , p2 , p3 , . . . denote the perpendiculars from O on the sides of the circum-
scribed polygon which the circle at A, B, C . . . respectively. Then
p1 = OA sin α, p2 = OB sin β, p3 = OC sin γ, . . .
CHAPTER XXIII : Trigonometrical Expressions Into Factors 210

Again let q1 , q2 , q3 . . . denote the perpendiculars from O on the sides of the in-
scribed polygon AB, BC, CD, . . . respectively. Then
q1 = OA sin OAB = OA sin(π − β) = OA sin β;
similarly q2 = OB sin γ, q3 = OC sin δ, . . .
Thus p1 p2 p3 . . . and q1 q2 q3 . . . are equal, for each is equal to the product of the same
series of lengths into the same series of sines.

Problem 10. Show that


16 cos A cos (72 ◦ − A) cos (72 ◦ + A) cos (144 ◦ − A) cos (144 ◦ + A) = cos 5A.
Solution. By Art. 313 (page 355) we have
cos 5A = 16 sin(A + 18◦ ) sin(A + 54◦ ) sin(A + 90◦ ) sin(A + 126◦ ) sin(A + 162◦ )
= 16 cos(72◦ − A) cos(36◦ − A) cos A cos(A + 36◦ ) cos(A + 72◦ );
and cos(36◦ − A) = − cos(144◦ + A), cos(A + 36◦ ) = − cos(144◦ − A),

therefore
cos 5A = 16 cos(72◦ − A) cos(72◦ + A) cos A cos(144◦ − A) cos(144◦ + A).

Problem 11. From the expression for sin θ in factors show that
36 144 324 576
π=3· · · · ......
35 143 323 575
π
Solution. Put for θ in the expression for sin θ in Art. 320 (page 359); thus
6 ( )( )( )
1 π 1 1 1
= 1− 2 1− 2 2 1 − 2 2 ...;
2 6 6 2 6 3 6
35 143 323 575
therefore 3=π · · · ...;
36 144 324 576
36 144 324 576
therefore π =3· · · · ...
35 143 323 575

Problem 12. Show that ( )( )( )


4z 2 4z 2
4z 2
e z + e −z = 2 1+ 1+ ... 1+
π2 3 2 π2
5 2 π2
π
Solution. In the general result of Art. 321 (page 360) put for θ, thus
( )( )( )2( )
z −z 4z 2 4z 2 4z 2 4z 2
e +e =2 1+ 2 1+ 2 2 1+ 2 2 1 + 2 2 ...
π 3 π 5 π 7 π

Problem 13. Show that ( )( )( )


−x x2 x2 x2
e −ex
= 2x 1 + 2 1+ 2 2 1+ 2 2 ...
π 2 π 3 π
Solution. It is shown in Art. 321((page 360)
) { that }{ }
θ z2 z2 z2
ez − 2 cos θ + e−z = 4 sin2 1+ 2 1+ 1+ ...
2 θ (2π + θ)2 (2π − θ)2
The product of the first two factors on the right-hand side
 θ 2
θ sin
= 4 sin2 + z 2  2 ,
2 θ
2
and this is equal to z 2 when θ vanishes.
Thus, by supposing θ = 0, we obtain
( )2 ( )2 ( )2
−z z2 z2 z2
e −2+e
z
=z 2
1+ 2 2 1+ 2 2 1+ 2 2 ...
2 π 4 π 6 π
CHAPTER XXIII : Trigonometrical Expressions Into Factors 211

Extract the square root and(put 2x for


) (z; thus ) ( )
x 2 x2 x2
−x
e −e
x
= 2x 1 + 2 1+ 2 2 1 + 2 2 ...
π 2 π 3 π

Problem 14. Find the sum of the series formed by multiplying together every two
1 1 1 1
of the terms of the series , , , , . . ..
1 22 32 42
Solution. Let s denote the series of which we require the sum,
( )2
1 1 1 1 1 1 1
then 2
+ 2 + 2 + 2 + ... = 2s + 4 + 4 + 4 + . . .
1 2 3 4 1 2 3
Hence by P roblems 1 and 2 we have
( )2
π2 π4
= 2s + ;
6 90
( ) ( )
π4 1 1 π4 1 1 π4
therefore s= − = − = .
2 36 90 36 2 5 120

Problem 15. If n be ( even )show(that ) ( )


π 2π n−1 n
tan ϕ tan ϕ + tan ϕ + . . . tan ϕ + π = (−1 ) 2 .
n n n
Solution. By Art. 318 (page 358) ( we have) ( ) ( )
π 2π n−1
sin nϕ = 2n−1 sin ϕ sin ϕ + sin ϕ + . . . sin ϕ + π .
n n n
π
Change ϕ into ϕ + ; then since n is even we have
2 ( ) ( )
π nπ nπ
sin n ϕ + = sin nϕ + = sin nϕ cos ;
2 2 2
thus ( ) ( ) ( )
nπ π 2π n−1
sin nϕ cos = 2n−1 cos ϕ cos ϕ + cos ϕ + . . . cos ϕ + π .
2 n n n
Divide the former result by(this; then ) we ( obtain) ( )
nπ π 2π n−1
sec = tan ϕ tan ϕ + tan ϕ + . . . tan ϕ + π .
2 n n n
nπ 1 1 n
And sec = nπ = n = (−1) 2 .
2 cos (−1) 2
2

Problem 16. Show that sin 5A − cos 5A


= 16 cos(A − 27 ◦ ) cos(A + 9 ◦ ) sin(A + 27 ◦ ) sin(A − 9 ◦ )(cos A − sin A).
√ √
Solution. sin 5A − cos 5A = 2 sin(5A − 45◦ ) = 2 sin 5(A − 9◦ ).
And by Art. 318 (page 358) we have sin 5(A − 9◦ )
= 24 sin B sin(B + 36◦ ) sin(B + 72◦ ) sin(B + 108◦ ) sin(B + 144◦ ),
where B = A − 9◦ ,
= 24 sin(A − 9◦ ) sin(A + 27◦ ) cos(27◦ − A) cos(A + 9◦ ) sin(A + 135◦ )
1
= 24 sin(A − 9◦ ) sin(A + 27◦ ) cos(27◦ − A) cos(A + 9◦ )(cos A − sin A) √ .
2
Therefore sin 5A − cos 5A
= 24 sin(A − 9◦ ) sin(A + 27◦ ) cos(A − 27◦ ) cos(A + 9◦ )(cos A − sin A).

Problem 17. Show that


π 2 · 2 · 4 · 4 · 6 · 6 · 8 · 8 ...
=
2 1 · 3 · 3 · 5 · 5 · 7 · 7 · 9 ...
Solution. By Art. 320 (page(359) we) have
( )( )
sin θ θ2 θ2 θ2
= 1− 1− 1− ...
θ π2 2 π2
2 3 π2
2
CHAPTER XXIII : Trigonometrical Expressions Into Factors 212

π
Put for θ : thus
2 ( )( )( )
2 1 1 1
= 1− 1− 2 1 − 2 ...;
π 22 4 6
π 2 · 2 · 4 · 4 · 6 · 6 · 8 · 8...
therefore = .
2 1 · 3 · 3 · 5 · 5 · 7 · 7 · 9...

sin 2 θ
Problem 18. By aid of the formula cos θ = deduce the expression for cos θ
2 sin θ
obtained in Art. 320 (page 359) from that for sin θ.
Solution. We have ( )( )( )( )
4θ2 4θ2 4θ2 4θ2
sin 2θ = 2θ 1 − 2 1− 2 2 1− 2 2 1 − 2 2 ...
π 2 π 3 π 4 π
( )( )( )( )
θ2 θ2 π2 θ2
sin θ = θ 2
1−
1− 2 2 1− 1− .
π 2 π 32 π 2 4 π2
2

Divide the first by the second


( : thus
)( )( )
sin 2θ 4θ2 4θ2 4θ2
=2 1− 1− 1− ...;
sin θ π2 32 π 2 52 π 2
( )( )( )
4θ2 4θ2 4θ2
therefore cos θ = 1− 1− 1− ...
π2 32 π 2 52 π 2

Problem 19. Show that



4 · 36 · 100 · 196 · 324 . . .
2 = .
3 · 35 · 99 · 195 · 323 . . .
π
Solution. In the formula for cos θ in Art. 320 (page 359) put for θ; thus
( )( )( )( 4 )
1 1 1 1 1
√ = 1− 2 1− 2 2 1− 2 2 1 − 2 2 ...
2 2 2 ·3 2 ·5 2 ·7
3 · 35 · 99 · 195 . . .
= ;
4 · 36 · 100 · 196 . . .
√ 4 · 36 · 100 · 196 . . .
therefore 2= .
3 · 35 · 99 · 195 . . .

Problem 20. Show that √


3 8 · 80 · 224 · 440 . . .
= .
2 9 · 81 · 225 · 441 . . .
π
Solution. In the formula for cos θ in Art. 320 (page 359) put for θ; thus
√ ( )( )( )( 6 )
3 1 1 1 1
= 1− 2 1− 2 2 1− 2 2 1 − 2 2 ...
2 3 3 ·3 3 ·5 3 ·7
8 · 80 · 224 · 440 . . .
= .
9 · 81 · 225 · 441 . . .

y
Problem 21. Show that cos x + tan sin x =
( )( 2 )()( )( )
2x 2x 2x 2x 2x
1+ 1− 1+ 1− 1+ ...
π−y π+y 3π − y 3 π + y( )5 π − y
y y y
cos x cos + sin x sin cos x −
y 2 2 2
Solution. cos x + tan sin x = y = y .
2 cos cos
2 2
Now by Art. 320 (page{ 359) }{ }{ }
( )
y (2x − y)2 (2x − y)2 (2x − y)2
cos x − = 1− 1− 1− ...
2 π2 32 π 2 52 π 2
CHAPTER XXIII : Trigonometrical Expressions Into Factors 213

( )( )( )
y y2 y2 y2
= 1− 2
cos 1− 2 2 1− ...
2 π 3 π 5 π2
2

Divide the former by the latter. Then

(2x − y)2
1− π 2 − (2x − y)2 π 2 − y 2 − 4x2 + 4xy
π2 = =
y 2 π −y
2 2 π2 − y2
1− 2
π
4x2 4xy
=1− 2 + 2
( π − y 2
)( − y )
π 2
2x 2x
= 1+ 1− .
π−y π+y
(2x − y)2
1− ( )( )
32 π 2 2x 2x
Similarly = 1+ 1− .
y 2 3π − y 3π + y
1− 2 2
3 π
And so on. Thus the required result is obtained.

y
Problem 22. Show that cos x − cot sin x =
( )( 2 )( )( )( )
2x 2x 2x 2x 2x
1− 1+ 1− 1+ 1− ...
y 2π − y 2π + y 4 π − y( )4 π + y
y y y
cos x sin − sin x cos sin −x
y 2 2 2
Solution. cos x − cot sin x = y = y .
2 sin sin
2 2
Now by Art. 320 (page359){ }{ }{ }
( ) ( )
y y (y − 2x)2 (y − 2x)2 (y − 2x)2
sin −x = −x 1− 1− 1− ...
2 2 4 · π2 4 · 22 π 2 4 · 32 π 2
( )( )( )
y y y2 y2 y2
sin
= 1− 1− 1− ...
2 2 4 · π2 4 · 22 π 2 4 · 32 π 2
Divide the former by the latter. Then
y
−x 2x
2 =1− .
y y
2
(y − 2x)2
1− 4π 2 − (y − 2x)2 4π 2 − y 2 − 4x2 + 4xy
4 · π2 = =
y2 4π 2 − y 2 4π 2 − y 2
1−
4 · π2 ( )( )
4x2 4xy 2x 2x
=1− + = 1 + 1 − .
4π 2 − y 2 4π 2 − y 2 2π − y 2π + y
(y − 2x)2
1− ( )( )
Similarly 4 · 22 π 2 = 1 + 2x 1−
2x
.
y 2 4π − y 4π + y
1−
4 · 22 π 2
And so on. Thus the required result is obtained.

Problem 23.
Show that ( ){ }{ }
cos x − cos y x2 x2 x2
= 1− 2 1− 1 −
1 − cos y y (2π − y)2 (2π + y)2
{ }{ }
x2 x2
1− 1− ....
(4π − y)2 (4π + y)2
CHAPTER XXIII : Trigonometrical Expressions Into Factors 214

1 1
cos x − cos y 2 sin (y − x) sin (y + x)
Solution. = 2 2
1 − cos y y
2 sin2
2
Now by Art. 320 (page 359) { }{ }
1 1 (y − x)2 (y − x)2
sin (y − x) = (y − x) 1− 1− ...
2 2 4π 2 4 · 22 π 2
{ }{ }
1 1 (y + x)2 (y + x)2
sin (y + x) = (y + x) 1− 1− ...
2 2 4π 2 4 · 22 π 2
( )( )
1 1 y2 y2
y = y 1−
sin 1− ...
2 2 4π 2 4 · 22 π 2
Divide the first by the third, and divide the second by the third, and multiply the
results together.
1 1
(y − x) ( )( )
2 x 2 (x + y) x x x x2
Then =1− ; =1+ ; 1− 1+ =1− 2.
1 y 1 y y y y
y y
2 2
And as in the solution of P roblem 22,
(y − x)2
1− ( )( )
4π 2 x x
= 1+ 1− ;
y 2 2π − y 2π + y
1−
4π 2
(y + x)2
1− ( )( )
4π 2 x x
= 1− 1+ ;
y 2 2π − y 2π + y
1−
( )
4π(2 )( )( )
x x x x
1+ 1− 1− 1+
2π − y{ 2π + y } { 2π − y } 2π + y
x2 x2
= 1− 1− .
(2π − y)2 (2π + y 2 )
Similarly

1−
(y − x)2
1−
(y + x)2 { }{ }
4 · 22 π 2 · 4 · 22 π 2 = 1 − x2 x2
1 − .
y2 y2 (4π − y)2 (4π + y)2
1− 1 −
4 · 22 π 2 4 · 22 π 2
And so on. Thus the required result is obtained.

cos x + cos y
Problem 24. Show that =
{ } { 1 + cos y } { }{ }
x2 x2 x2 x2
1− 1 − 1 − 1− . . ..
(π − y)2 (π + y) 2 (3 π − y) 2 (3 π + y)2
1 1
cos x + cos y 2 cos (y − x) cos (y + x)
Solution. = 2 2
1 + cos y y
2 cos2
2
Now by Art. 320 (page 359)
{ }{ }{ }
1 (y − x)2 (y − x)2 (y − x)2
cos (y − x) = 1− 1− 1− ...
2 π2 32 π 2 52 π 2
{ }{ }{ }
1 (y + x)2 (y + x)2 (y + x)2
cos (y + x) = 1− 1− 1− ...
2 π2 32 π 2 52 π 2
( )( )( )
y y2 y2 y2
= 1− 2
cos 1− 2 2 1 − 2 2 ...
2 π 3 π 5 π
Divide the first by the third, and divide the second by the third, and multiply the
CHAPTER XXIII : Trigonometrical Expressions Into Factors 215

two results together; the reductions will be similar to those in the preceding solution.
1−
(y − x)2
1−
(y + x)2 { }{ }
π 2 π2 x2 x2
Thus · = 1 − 1 − ;
y2 y2 (π − y)2 (π + y)2
1− 2 1− 2
π π
1−
(y − x)2
1 −
(y + x)2 { }{ }
32 π 2 · 32 π 2 = 1 − x2 x2
1− .
y2 y2 (3π − y)2 (3π + y 2 )
1− 2 2 1− 2 2
3 π 3 π
And so on. Thus the required result is obtained.
Or we may obtain the result in P roblem 24 by changing y into π − y in the result of
P roblem 23.

sin x + sin y
Problem 25. Show that =
( )( ) (y
sin )( )( )
x x x x x
1+ 1+ 1− 1+ 1− ...
y π−y π+y 2π + y 2π − y
1 1
sin x + sin y 2 sin (x + y) cos (x − y)
Solution. = 2 2
sin y y y
2 sin cos
2 2
Now in the course of the solution of P roblem 23 we see that
1
sin (x + y) ( )( )( )
2 x x x
= 1+ 1− 1+
1 y 2π − y 2π + y
sin y
2 ( )( )
x x
1− 1+ ...
4π − y 4π + y
And by changing y into π − y we see that
1
cos (x − y) ( )( )( )( )
2 x x x x
= 1+ 1− 1+ 1− ...
1 π−y π+y 3π − y 3π + y
cos y
2
Hence by multiplication the required result is obtained.

Problem 26. In Problem 21 by expanding both sides in powers of x and equating the
coefficients of x, show that
y 2 2 2 2 2 2
tan = − + − + − + ...
2 π−y π+y 3π − y 3π + y 5π − y 5π + y
y
Solution. We have cos x + tan sin x
2 ( )
x2 x4 y x3 x5
=1− + − . . . + tan x− + − ... ;
2 4 2 3 5
y
thus the coefficient of x is tan .
2
Now conceive the factors on the right-hand side of the formula of P roblem 21 mul-
tiplied together, and the product arranged according to powers of x. The first term
will be unity; the second term will involve x, and the coefficient will be
2 2 2 2
− + − + ...
π−y π+y 3π − y 3π + y
Hence by equating the coefficients we obtain the required result.

Problem 27. Show in like manner from Problem 22 that


y 2 2 2 2 2
cot = − + − + − ...
2 y 2π − y 2π + y 4π − y 4π + y
Solution. Proceed as in the solution of P roblem 36. Then on the left-hand side the
CHAPTER XXIII : Trigonometrical Expressions Into Factors 216

y
coefficient of x will be − cot, and on the right-hand side
2
2 2 2 2 2
− + − + − + ...
y 2π − y 2π + y 4π − y 4π + y
Equate the coefficients, and then change the signs of both sides; thus we obtain
the required result.

Problem 28. Show that


π 1 1 1 1 1 1
√ =1− + − + − + − ...
3 3 2 4 5 7 8 10
π
Solution. In the formula of P roblem 26 put for y; then
3
y π 1
tan = tan = √ ;
2 6
{ 3 }
1 1 6 6 6 6 6
thus √ = − + − + − ... ;
3 {π 2 4 8 10 14
}
1 6 1 1 1 1 1 1
that is √ = − + − + − + ... ;
3 π 2 4 8 10 14 16
π 1 1 1 1 1
therefore √ = 1 − + − + − + ...
3 3 2 4 5 7 8

Problem 29. Show that


π 1 1 1 1 1 1
√ =1− + − + − + − ...
2 3 5 7 11 13 17 19
π
Solution. In the formula of P roblem 27 put for y; then
3 √
y π
cot = cot = 3;
√ { 2 6 }
6 1 1 1 1 1 1
thus 3= 1− + − + − + − ... ;
π 5 7 11 13 17 19
π 1 1 1 1 1 1
therefore √ =1− + − + − + − ...
2 3 5 7 11 13 17 19

1
Problem 30. Show that =
sin y
1 1 1 1 1 1 1 1
+ − − + + − − + ...
y π−y 2π − y π+y 2π + y 3π − y 4π − y 3π + y
Solution. Add together the results given in P roblems 26 and 27; then
y y
y y sin cos 1 2
tan + cot = 2 2
2 2 y + y = y y = sin y .
cos sin sin cos
2 2 2 2
Thus
2 2 2 2 2 2
= + − − +
sin y y π−y 2π − y π+y 2π + y
2 2 2
+ − − + ...
3π − y 4π − y 3π + y
Then divide both sides by 2.
Or we may equate the coefficients of x in P roblem 25.
CHAPTER XXIV
Miscellaneous Propositions

θ θ π−θ π+θ
Problem 1. Prove that sin θ cos = 8 sin sin2 sin2 .
2 2 ( 4 ) 4
θ θ θ θ π θ
Solution. sin θ cos = 2 sin cos2 = sin sin2 −
( )
2 ( 2 )2 2 (2 2 ) ( )
θ π θ π θ θ π θ π θ
= sin sin2 − cos2 − = 8 sin sin2 − sin2 + .
2 4 4 4 4 2 4 4 4 4

( Prove that
Problem 2. ) ( )
θ θ θ θ θ θ 2θ
cosec 2 − sec2 tan = tan2 cosec 2 − sec2 cot .
6 2 3  2 6 2 3
( )
θ θ θ 1 1  θ
Solution. cosec 2 − sec2 tan = − tan
6 2 3 θ θ 3
sin2 cos2
(6 ) 2( )
θ θ θ θ θ θ
cos2 − sin2 cos − cos +
2 6 θ 2 6 2 6 θ
= tan = tan
θ θ 3 θ θ 3
sin2 cos2 sin2 cos2
6 2 6 2
θ 2θ θ θ 2θ
cos cos tan sin cos
= 3 3 3 = 3 3 .
θ θ θ θ
sin2 cos2 sin2 cos2
6 2 6 2
Again
( )
θ θ θ 2θ
tan2 cosec 2 − sec2 cot
2 6 2 3 θ

sin2 1  2θ
= 2 − cot
θ 2 θ θ 3
cos2 sin cos 2
2 6 2
θ θ
sin2 − sin2 2θ
= 2 6 cot
θ θ 3
cos2 sin2
( 2 )6 ( )
θ θ θ θ
sin − sin +
2 6 2 6 2θ
= cot
θ θ 3
cos2 sin2
2 2
θ 2θ 2θ θ 2θ
sin sin cot sin cos
= 3 3 3 = 3 3 .
θ θ θ θ
cos2 sin2 cos2 sin2
2 6 2 6

Problem 3. Prove that


tan 3 θ − tan 2 θ − tan θ = tan 3 θ tan 2 θ tan θ.
Solution. tan 3θ − tan 2θ − tan θ
sin 3θ sin 2θ
= − − tan θ
cos 3θ cos 2θ
sin 3θ cos 2θ − sin 2θ cos 3θ
= − tan θ
cos 3θ cos 2θ
sin θ sin θ
= −
cos 3θ cos 2θ cos θ
CHAPTER XXIV : Miscellaneous Propositions 218

sin θ
= {cos θ − cos 3θ cos 2θ}
cos θ cos 2θ cos 3θ
sin θ
= {cos(3θ − 2θ) − cos 3θ cos 2θ}
cos θ cos 2θ cos 3θ
sin θ sin 2θ sin 3θ
= = tan θ tan 2θ tan 3θ.
cos θ cos 2θ cos 3θ

Problem 4. Find x from the equation


tan3 x + cot3 x = m 3 − 3m.
Solution. tan3 x + cot3 x = m3 − 3m;
therefore (tan x + cot x)3 − 3(tan2 x cot x + cot2 x tan x) = m3 − 3m;
therefore (tan x + cot x)3 − 3(tan x + cot x) = m3 − 3m;
therefore (tan x + cot x)3 − m3 = 3(tan x + cot x) − 3m.
Put y for tan x + cot x; thus
y 3 − m3 = 3(y − m);
therefore (y − m)(y 2 + ym + m2 ) = 3(y − m).
Therefore either y − m = 0, or y 2 + ym + m2 = 3.
Take y − m = 0; thus tan x + cot x = m,
sin x cos x
therefore + = m;
cos x sin x
1
therefore = m;
sin x cos x
2
therefore sin 2x = .
m
2 2
Again, take y + ym + m = 3. By solving this quadratic in the usual way we obtain

−m ± (12 − 3m2 )
y=
2
and thus we obtain two other values for sin 2x.

Problem 5. The circumference of a circle is divided into 2n equal parts at the


points A, P, Q, . . . . Tangents are drawn at the points A, P, Q, . . . and perpendiculars
OA, OB, OC , . . . are let fall on them from O the extremity of the diameter OA. Show
that
OA2 + OB 2 + OC 2 + . . . = 3n(radius)2 .
Solution. Let T denote the mth point, counting from A as the first, so that the angle
π
T OA = mβ, where β = . Then, since the angle OT A is a right angle, we have
2n
OT = OA cos T OA = 2r cos mβ, where r is the radius of the circle. The angle between
π
OT and the tangent at T is equal to the angle T AO, and is therefore − mβ. Thus
( ) 2
π
the perpendicular from O on the tangent at T = OT sin − mβ = OT cos mβ =
2
2
2r cos mβ. The square of the perpendicular is 4r cos mβ. 2 4

Thus the sum of the squares of the perpendiculars.


= 4r2 {1 + cos4 β + cos4 2β + . . . + cos4 (2n − 1)β}.
3 1 1
Now cos4 θ = + cos 2θ + cos 4θ.
8 2 8
Apply this transformation to every term in the series, observing that 1 may be
considered as cos 0.
Thus the sum of the squares of the perpendiculars
3
= 4r2 · · 2n + 2r2 {cos 0 + cos 2β + cos 4β + . . . + cos 2(2n − 1)β}
8
r2
+ {cos 0 + cos 4β + cos 8β + . . . + cos 4(2n − 1)β}.
2
Each series of cosines will vanish as in Art. 305 (page 353); thus we find that the
sum of the squares of the perpendiculars = 3nr2 .
CHAPTER XXIV : Miscellaneous Propositions 219

Problem 6. ABC is a quadrant; AP, AQ, AR are three arcs in ascending order of
magnitude, each being less than AB, and their sum equal to twice AB; radii CP, CQ, CR
are produced to meet the tangent at A at p, q, r and a triangle is formed with Ap, Aq, Ar.
Find the condition that this may be possible, and the inferior limit of Aq and the su-
perior limit of Ap. Prove also that in all such triangles the radii of the inscribed and
circumscribed circles are inversely proportional.
Solution. Let ACP = θ, ACQ = ϕ, ACR = ψ; let AC = ρ. Then θ + ϕ + ψ = π. Now,
in order that a triangle may exist with Ap, Aq, and Ar as sides we must have Ap + Aq
greater than Ar; thus tan θ + tan ϕ must be greater than tan ψ.
But by Art. 114 (page 328) we have
tan θ + tan ϕ + tan ψ = tan θ tan ϕ tan ψ.
Hence tan θ tan ϕ tan ψ must be greater than 2 tan ψ, and therefore tan θ tan ϕ greater
than 2. Therefore a fortiori tan2 ϕ must be greater than √ 2.
Thus the inferior limit of Aq is when tan QCA = 2.
Again, since θ + ϕ + ψ = π, the superior limit of θ is the value of θ when θ, ϕ and ψ
π
are all equal; that is, when θ = .
3
When the triangle is formed the radius of the inscribed circle, by Art. 248 (page 342)
2S
is ; and the radius of the circumscribed circle by Art. 252 (page 344)
ρ(tan θ + tan ϕ + tan ψ)
3
ρ tan θ tan ϕ tan ψ ρ2 tan θ tan ϕ tan ψ ρ2
is . The product of these radii is · , that is
4S 2 tan θ + tan ϕ + tan ψ 2
by Art. 114 (page 328).
Hence the product of the radii is constant, whatever θ, ϕ and ψ may be; and there-
fore one radius varies inversely as the other.

Problem 7. ABC is a right-angled triangle, C being the right angle, E is the point
at which the inscribed circle touches BC , and F the point at which the circle drawn
to touch AB and the sides CA, CB produced meets CA; show that if EF be joined the
triangle FEC is half the triangle ABC .
Solution. As in Art. 250 (page 344) we see that CF = s; also CE = r; hence
1
the area of CEF = rs.
2
And, as in Art. 248 (page 342) the area of ABC = rs; therefore CEF is half ABC.

Problem 8. Through the angular points of a triangle straight lines are drawn bi-
secting the exterior angles. If S be the area of the original triangle and S ′ that of the
new triangle, show that
1 A B C
S ′ = S cosec cosec cosec .
2 2 2 2
Solution. The straight lines bisecting the external angles are the same as those
which join the centres of the escribed circles.
abc
Thus, by Chapter xvi : P roblem 34, we have S ′ = ;
2r
S′ abc sabc abc
therefore = = =
S 2rS 2S 2 2(s − a)(s − b)(s − c)
1 A B C
= cosec cosec cosec .
2 2 2 2

Problem 9. ABCD is a horizontal straight line. From a point immediately above D


the known distances AB and BC are observed to subtend the same angle α. If AB = a
and BC = b, show that the height of the observer’s position above D is
2ab(a + b) tan α
.
(a − b)2 + (a + b)2 tan2 α
Solution. Let P denote the point above D. Then P D × AC = twice the area of the
triangle AP C = AP · P C sin α; therefore
AP · P C · sin 2α (a + b)AP · P C · sin 2α
PD = =
AC AC 2
CHAPTER XXIV : Miscellaneous Propositions 220

(a + b)AP · P C · sin 2α
=
AP 2 + P C 2 − 2AP · P C cos 2α
(a + b)ab sin 2α AP a
= 2 , for = by Euclid vi. 3,
a + b2 − 2ab cos 2α PC b
2(a + b)ab sin α cos α
= 2
(a + b2 )(sin2 α + cos2 α) − 2ab(cos2 α − sin2 α)
2(a + b)ab tan α
=
(a − b)2 + (a + b)2 tan2 α

Problem 10. If in any arc not greater than a quadrant a point be taken, and from
this point two straight lines be drawn, one to the extremity of the arc, the other
perpendicular to its chord and terminated by it, prove that the sum of these two
straight lines is less than the chord of the arc.
Solution. Let AB be the arc, and C the centre of the circle; in AB take any point P,
join P A, and draw P M perpendicular to AB.
Let BCA = 2γ, P CA = 2θ, and AC = r.
Then AB = 2r sin γ, AP = 2r sin θ, P M = AP sin P AB = AP sin(γ − θ).
We have then to show that 2r sin θ{1 + sin ∗(γ − θ)} is less than 2r sin γ, or that
sin(γ − θ) sin θ is less than sin γ − sin θ, or that
1 1 1 1
2 sin (γ − θ) cos (γ − θ) is less than 2 sin (γ − θ) cos (γ + θ),
2 2 2 2
1 1
or that cos (γ − θ) sin θ is less than cos (γ + θ).
2 2
1
Now this is the case, for cos (γ − θ) sin θ is less than sin θ, and therefore less than
2
1 1
sin γ and cos (γ + θ) is greater than cos (γ + γ), that is greater than cos γ; and cos γ is
2 2
π
greater than sin γ, since γ is less than .
4

Problem 11. Suppose α the angle of elevation of a cloud, β the angle of depression
of the image of the cloud seen by reflection from a lake, h the height of the observer’s
eye above the lake, then the height of the cloud is
h sin(β + α)
.
sin(β − α)
Solution. Let A be the position of the observer’s eye, C the cloud, B the image of
the cloud formed by the lake. Draw the horizontal straight line AH. Then HAC = α,
and HAB = β.
The straight lines CB and AB are equally inclined to the surface of the water by
the Laws of Optics, and thus the angle between CB and AH is equal to β.
CB sin CAB sin(β + α)
Now = = ;
AB sin ACB sin(β − α)
AB sin(β + α)
therefore CB = .
sin(β − α)
sin(β + α)
The height of the cloud = CB sin β = AB sin β
sin(β − α)
sin(β + α)
=h .
sin(β − α)

Problem 12. At noon a person standing on a cliff h feet above the level of the sea,
observes the altitude of a cloud in the plane of the meridian to be α and the angle of
depression of its shadow on the surface of the water to be β; the sun being behind
the observer when he is looking at the cloud. Show that, if γ be the sun’s altitude at
the time of observation, the height of the cloud above the surface of the water will be
h sin γ sin(α + β)
.
sin β sin(γ + α)
CHAPTER XXIV : Miscellaneous Propositions 221

Solution. Let A be the position of the observer’s eye, C the cloud, B the shadow of
the cloud on the sea. Then the position of the sun is on BC produced through C. Draw
from A a horizontal straight line meeting BC at H. Then HAC = α, and HAB = β. On
account of the enormous distance of the sun, the straight line HC may be considered
as parallel to the straight line drawn from A to the sun; so that CHA = γ.
BC sin BAC sin(α + β) sin(α + β)
Now = = = .
BA sin BCA sin(π − α − γ) sin(α + γ)
The height of the cloud above the surface of the sea
sin γ sin γ sin(α + β)
= BC sin γ = BC sin β = BA sin β
sin β sin β sin(α + γ)
sin γ sin(α + β)
=h · .
sin β sin(α + γ)

Problem 13. Show that the formula of Art. 280 (page 349) may be verified by
induction.
Solution. Assume that
n(n − 1)
2n−1 cosn θ = cos nθ + n cos(n − 2)θ + cos(n − 4)θ + . . .
1·2
n(n − 1) . . . (n − r + 1)
+ cos(n − 2r)θ + . . .
r
Multiply both sides by 2 cos θ. Thus
2n cosn+1 θ = 2 cos nθ cos θ + 2n cos(n − 2)θ cos θ
n(n − 1)
+2 cos(n − 4)θ cos θ + . . .
1·2
Now use the formula 2 cos A cos B = cos(A + B) + cos(A − B) for the terms on the
right-hand side. Thus
2n cosn+1 θ = cos(n + 1)θ + cos(n − 1)θ
+ n{cos(n − 1)θ + cos(n − 3)θ}
n(n − 1)
+ {cos(n − 3)θ + cos(n − 5)θ}
1·2
n(n − 1) . . . (n − r + 1)
+ {cos(n − 2r + 1)θ + cos(n − 2r − 1)θ}
r
+ ...
Then re-arrange the terms on the right-hand side, and we obtain a series like that
with which we started except that we have n + 1 instead of n.
For instance, the term involving cos(n − 3)θ is
{ }
n(n − 1)
n+ cos(n − 3)θ;
1·2
(n + 1)n
that is cos(n + 1 − 4)θ.
1·2
And generally the term involving cos(n + 1 − 2r)θ is
{ }
n(n − 1) . . . (n − r + 2) n(n − 1) . . . (n − r + 1)
+ cos(n + 1 − 2r)θ;
r−1 r
( )
n(n − 1) . . . (n − r + 2) n−r+1
that is 1+ cos(n + 1 − 2r)θ;
r−1 r
(n + 1)n . . . (n + 1 − r + 1)
that is cos(n + 1 − 2r)θ.
r
This shows that if the formula holds for an assigned value of n it holds also when
n is changed into n is changed into n + 1. Moreover the formula evidently holds when
n = 1.
We have not paid special attention to the last term in the expansion, but it is easy
to do this if required.

Problem 14. Show that the formula of Arts. 282 (page 350) and 283 (page 350) may
π
be obtained from that of Art. 280 (page 349) by changing θ into − θ.
2
CHAPTER XXIV : Miscellaneous Propositions 222

Solution. Take the formula of Art. 280 (page 349), and suppose n even; thus
n(n − 1)
2n−1 cosn θ = cos nθ + n cos(n − 2)θ + cos(n − 4)θ + . . .
1·2 ( )
1
n(n − 1) . . . n+1
n(n − 1)(n − r + 1) 2
+ cos(n − 2r)θ . . . +
r 1
2 n
2
π
Change θ into − θ; thus
2 ( )
π
cosn θ becomes cosn − θ , that is sinn θ;
(2 )
π π n
cos nθ becomes cos n − θ , that is cos n cos nθ, that is (−1) 2 cos nθ;
2 ( ) 2
π n−2
cos(n − 2)θ becomes cos(n − 2) − θ , that is (−1) 2 cos(n − 2)θ;
2
and so on.
n n−2
Thus 2n−1 sinn θ = (−1) 2 cos nθ + n(−1) 2 cos(n − 2)θ
n(n − 1) n−4
+ (−1) 2 cos(n − 4)θ + . . .
n
1 · 2
Multiply both sides by (−1) 2 ; thus we obtain the formula of Art. 282 (page 350) .
Next suppose n odd. Then in the same manner we deduce the formula of Art. 283
(page 350) from that of Art.
( 280)(page 349); we observen−1
now that
π π
cos n − θ = sin n sin nθ = (−1) 2 sin nθ,
2 ( )2
π n−3
cos(n − 2) − θ = (−1) 2 sin(n − 2)θ,
2
and so on.

( )
Problem 15. Express cos 6 tan−1 x in terms of x.
Solution. Let tan−1 x = θ, so that tan θ = x; then
1 x
cos θ = √ and sin θ = √ .
(1 + x2 ) (1 + x2 )
And by Art. 270 (page 348)
cos 6θ = cos6 θ − 15 cos4 θ sin2 θ + 15 cos2 θ sin4 θ − sin6 θ
1 { }
= 1 − 15x2 + 15x4 − x6 .
(1 + x2 )3

Problem 16. If a quadrilateral can be inscribed in a circle and can also have a
circle described about it, the area of the quadrilateral is equal to the square root of
the product of the four sides.
Solution. Since
√ the quadrilateral can be inscribed in a circle, the area by Art. 254
(page 346) is {(s − a)(s − b)(s − c)(s − d)}. But when a quadrilateral can be described
about a circle it may be shown by geometry that the sum of two opposite sides is equal
to the sum of the other two. Thus in the present case a + c = b + d.
1
Now s = (a + b + c + d) = a + c = b + d;
2
therefore s − a = c, s − b = d, s − c = a, s − d = b;
√ √
therefore {(s − a)(s − b)(s − c)(s − d)} = abcd.

Thus the area = abcd.

Problem 17. The sides of a quadrilateral figure are a, b, c, d; and the sum of two
opposite angles is θ. If S denote the area of the figure, and s half the sum of the sides,
show that
θ
S 2 = (s − a)(s − b)(s − c)(s − d) − abcd cos2 .
2
CHAPTER XXIV : Miscellaneous Propositions 223

Solution. Let a, b, c, d denote the sides taken in succession; let B denote the angle
between the first two, and D the angle between the last two. Thus B + D = θ.
Then dividing the quadrilateral into two triangles, as in Art. 254 (page 346), we
have
1 1
S = ab sin B + cd sin D (23)
2 2
And from two values which can be obtained for the square of the diagonal opposite
B and D we have
a2 + b2 − 2ab cos B = c2 + d2 − 2cd cos D,
a2 + b2 − c2 − d2 1 1
therefore = ab cos B − cd cos D (24)
4 2 2
Square and add (23) and (24): thus
( )2
a2 + b2 − c2 − d2 1 1
S2 + = (a2 b2 + c2 d2 ) − abcd cos(B + D)
4 4 2
( )
1 2 2 1 1 2 2 1 θ
= (a b + c d ) − abcd cos θ = (a b + c d ) − abcd 2 cos2 − 1 ;
2 2 2 2
4 2 4 2 2
( )2
1 a2 + b2 − c2 − d2 θ
therefore S2 = (ab + cd)2 − − abcd cos2 .
4 4 2
Now we know that if θ = π the expression for S 2 must reduce to
(s − a)(s − b)(s − c)(s − d).
( )2
1 a2 + b2 − c2 − d2
Hence we are sure that (ab + cd)2 − must take the form just
4 4
given; and {
this is easily verified. For this expression
}{ }
1 a2 + b2 − c2 − d2 1 a2 + b2 − c2 − d2
= (ab + cd) + (ab + cd) −
2 4 2 4
1
= {(a + b)2 − (c − d)2 }{(c + d)2 − (a − b)2 }
16
1
= (a + b + c − d)(a + b − c + d)(c + d + a − b)(c + d − a + b).
16
θ
Thus S 2 = (s − a)(s − b)(s − c)(s − d) − abcd cos2 .
2

Problem 18. Show that


n(n + 1 ) n(n + 1 )(n + 2 )(n + 3 )
cosn θ cos nθ = 1 − tan2 θ + tan4 θ − . . .
2 4
Solution. Let t stand for{tan θ. By Art. 270 (page 348) we have }
n(n − 1)n(n − 1)(n − 2)(n − 3) 4
cos nθ = cosn θ 1− t2 + t − ... .
2 4

Put ι for −1; then we may write the formula thus
2 cos nθ = cosn θ{(1 + ιt)n + (1 − ιt)n }.
Therefore 2 cosn θ cos nθ = cos2n θ{(1 + ιt)n + (1 − ιt)n }
1 1
= {(1 + ιt)n + (1 − ιt)n } = {(1 + ιt)n + (1 − ιt)n }
(1 + t2 )n (1 + ιt)n (1 − ιt)n
= (1 − ιt)−n + (1 + ιt)−n .
Expand the two terms on the right-hand side by the Binomial Theorem, and the
required result is obtained.

Problem 19. Show that


n(n + 1 )(n + 2 )
cosn θ sin nθ = n tan θ − tan3 θ + . . .
3
Solution. Proceed as in the solution of P roblem 18. Thus we obtain
cosn θ
2 sin nθ = {(1 + ιt)n − (1 − ιt)n };
ι
CHAPTER XXIV : Miscellaneous Propositions 224

cosn θ
therefore 2 cosn θ sin nθ = {(1 + ιt)n − (1 − ιt)n }
ι
1 1
= {(1 + ιt)n − (1 − ιt)n } = {(1 + ιt)n − (1 − ιt)n }
ι(1 + t2 )n ι(1 + ιt)n (1 − ιt)n
1
= {(1 − ιt)−n − (1 + ιt)−n }.
ι
Expand the two terms on the right-hand side by the Binomial Theorem, and the
required result is obtained.

π θ
Problem 20. If θ is a positive angle less than show that continually increases
2 sin θ
with θ.
π
Solution. Let θ have any value between 0 and ; let h be a small positive quantity.
2
θ+h θ
We have then to show that is greater than , that is we must show that
sin(θ + h) sin θ
θ+h θ
− is positive.
sin(θ + h) sin θ
Now the sign of the last expression is the same as the sign of
(θ + h) sin θ − θ sin(θ + h),
and is therefore the same as the sign of
θ sin θ(1 − cos h) + h sin θ − θ cos θ sin h,
or as the sign of
( )
h θ
θ sin θ(1 − cos h) + sin θ sin h − .
sin h tan θ
h θ
Now 1 − cos h is positive; and is greater than unity while is less than
sin h tan θ
unity, by Art. 118 (page 330); thus the expression is positive.

π θ
Problem 21. If θ is a positive angle less than show that continually decreases
2 tan θ
as θ increases.
π
Solution. Let θ have any value between 0 and ; let h be a small positive quantity.
2
θ θ+h
We have then to show that is greater than , that is we must show that
tan θ tan(θ + h)
θ θ+h
− is positive.
tan θ tan(θ + h)
Now the sign of the last expression is the same as the sign of
θ cos θ sin(θ + h) − (θ + h) cos(θ + h) sin θ,
that is of θ sin h − h cos(θ + h) sin θ,
θ h
that is of − cos(θ + h).
sin θ sin h
θ
But as we may suppose h less than θ, we know by P roblem 20 that is greater
sin θ
h θ h
than , and therefore is greater than cos(θ + h).
sin h sin θ sin h

Problem 22. In the diagram of Art. 332 (page 362) if PO be joined, show that it
bisects DF, and is bisected by DF.
Solution. Let P O intersect F D at K; then by similar triangles, since P A = 2P F, we
have OA = 2F K; but DF = OA; therefore DF = 2F K; therefore DF is bisected at K.
Also since P A is bisected at F, it follows by similar triangles that P O is bisected at
K.

Problem 23. Show also that PO divides DA into parts which are in the ratio of 1 to
2.
CHAPTER XXIV : Miscellaneous Propositions 225

Solution. Let P O intersect AD at L. Then the triangles ALP and DLO are similar;
LD LA LD OD FA 1
therefore = ; therefore = = = .
OD PA LA PA PA 2

Problem 24. Show that the following four points connected with any triangle are in
a straight line : the centre of the circumscribing circle, the centre of the nine points
circle, the point of intersection of the perpendiculars from the angles on the opposite
sides, and the point of intersection of the straight lines drawn from the angles to the
middle points of the opposite sides.
Solution. The point K in the solution of P roblem 22 being the middle point of DF is
the centre of the nine points circle. Thus P, K, and O are in one straight line. Also
LD 1
this straight line cuts AD at a point L, such that = ; and OP is divided at L so
LA 2
OL 1
that = .
LP 2
In like manner the straight line from B to the middle point of AC cuts OP at the
same point as AD does; and so also does the straight line from O to the middle point
of AB. Hence the point L is the intersection of the three straight lines from the angles
of ABC to the middle points of the opposite sides.

Problem 25. Show that the length of the perpendicular from the centre of the nine
1
points circle on BC is R cos(C − B).
2
Solution. The centre of the nine points circle is the middle point of OP, hence the
1
perpendicular from it on BC = (OD + P G). Now OD = R cos A, and
2
BG cos C BG cos C c cos B cos C
P G = BP sin P BG = BP cos C = = =
cos P BG sin C sin C
= 2R cos B cos C.
Hence the perpendicular required
1 R
= (R cos A + 2R cos B cos C) = {2 cos B cos C − cos(B + C)}
2 2
R R
= (cos B cos C − sin C sin B) = cos(C − B).
2 2
Or thus : the required perpendicular
R R R
= sin HDG = cos OAG = cos(BAG − OAB)
2 2 2
R R
= cos {90◦ − B − (90◦ − C)} = cos(C − B).
2 2

Problem 26. Show that the length of the perpendicular from the centre of the nine
1
points circle on AG in the diagram of Art. 332 (page 362) is R sin(C − B).
2
Solution. The perpendicular from the
( centre of)the nine points circle on 4G
1 1 1 a 1
= DG = (CD − CG) = − b cos C = (R sin A − 2R sin B cos C)
2 2 2 2 2
R R
= {sin(B + C) − 2 sin B cos C} = sin(C − B).
2 2
Or thus : the required perpendicular
R R R
= cos HDG = sin OAG = sin(C − B).
2 2 2

Problem 27. In the diagram of Art. 332 (page 362) show that
OP 2 = R2 (1 − 8 cos A cos B cos C ).
Solution. We have AP = 2AF = 2OD = 2R cos A; and as shown in the solution of
P roblem 25, the angle OAP = C − B.
Then, from the triangle OAP,
OP 2 = OA2 + P A2 − 2OA · P A cos(C − B)
= R2 {1 + 4 cos2 A − 4 cos A cos(C − B)}
CHAPTER XXIV : Miscellaneous Propositions 226

= R2 + 4R2 cos A{cos A − cos(C − B)}


= R2 + 4R2 cos A{− cos(B + C) − cos(C − B)}
= R2 − 8R2 cos A cos B cos C.

Problem 28. Show that the distance of the centre of the nine points circle from the
R√
angular point A is 1 + 8 cos A sin B sin C .
2
Solution. Donate the centre of the nine points circle by K; then K is the middle point
of OP.
Now OA2 = OK 2 + KA2 − 2OK · KA cos OKA,
P A2 = P K 2 + KA2 − 2P K · KA cos P KA;
therefore, by addition,
OA2 + P A2 = 2OK 2 + 2KA2 ;
1
thus 2KA2 = R2 + 4R2 cos2 A − 2OK 2 = R2 + 4R2 cos2 A − P O2 .
2
Therefore, by the aid of P roblem 27,
4KA2 = 2R2 + 8R2 cos2 A − R2 (1 − 8 cos A cos B cos C)
= R2 + 8R2 cos A(cos A + cos B cos C)
= R2 + 8R2 cos A{− cos(B + C) + cos B cos C}
= R2 + 8R2 cos A sin B sin C;
R√
therefore KA = (1 + 8 cos A sin B sin C).
2

Problem 29. The centre of the nine points circle cannot coincide with the centre of
the circumscribed circle unless the triangle is equilateral.
Solution. Take the diagram of Art. 332 (page 362). The centre of the nine points
circle is on a straight line which bisects DG at right angles; and so it cannot be at
O unless D and G coincide, that is unless the perpendicular AG bisects BC, that is
unless B = C. Similarly we see, by considering the side AC instead of BC, that it will
be necessary to have A = C. Thus the triangle must be equilateral.

Problem 30. The centre of the nine points circle cannot coincide with the centre of
the inscribed circle unless the triangle is equilateral.
Solution. Take the diagram of Art. 332 (page 362). The centre of inscribed circle is on
the straight line AE, which bisects the angle A; and the centre of the nine points circle
is on DF : hence when the two coincide it must be at the point H. Thus, by P roblem 25;,
we must have H at the middle point of OP. Then from the similar triangle OHE and
AHP we find that OE must be equal to AP, that is to twice AF. Thus R = 2R cos A;
therefore A = 60◦ . Similarly we see by considering the side AC instead of BC, that
B = 60◦ . Hence the triangle must be equilateral.
Or we may use P roblem 25; thus we must have
R R R
r= cos(B − C) = cos(C − A) = cos(A − B);
2 2 2
so that cos(B − C) = cos(C − A) = cos(A − B) :
these lead to A = B = C.
CHAPTER XXV
Miscellaneous Examples

Problem 1. If an angle of 3 ◦ be represented by .15 , find how many degrees are


contained in the unit of that measure. Find also what number will represent a right
angle in the same measure.
Solution. Let x denote the number of degrees in the unit. Then 3 : x :: .15 : 1. Hence
3 90
x= = 20. The measure of a right angle will be , that is 4 21 .
.15 20

Problem 2. The difference of two angles is 1 ◦ ; the circular measure of their sum is
1 . Find the circular measure of each angle.
Solution. Let x denote the circular measure of the larger angle, y that of the smaller
π π
angle. Then, since the circular measure of 1◦ is , we have x − y = , x + y = 1.
( ) ( ) 180 180
1 π 1 π
Hence x = 1+ , y= 1− .
2 180 2 180

Problem 3. Find tan x from the equation tan x + ab cot x = a + b.


ab
Solution. Here tan x + = a + b; therefore tan2 x − (a + b) tan x + ab = 0. By solving
tan x
this quadratic equation we obtain tan x = a, or tan x = b.


Problem 4. If sin 3 θ = sin θ cos 2 θ then θ = where n is zero or an integer.
2
Solution. Here sin(2θ + θ) = sin θ cos 2θ, that is
sin 2θ cos θ + cos 2θ sin θ = sin θ cos 2θ;
therefore sin 2θ cos θ = 0, that is 2 sin θ cos2 θ = 0.
π
If cos θ = 0 we have θ an odd multiple of ; and if sin θ = 0 we have θ an even
2
π π
multiple of : hence all the solutions are comprised in θ = n , where n is zero or an
2 2
integer.

Problem 5. If an angle be divided into two equal and also into two unequal parts,
the product of the sines of the unequal parts together with the square of the sine of
the angle between the dividing straight lines is equal to the square of the sine of half
the angle.
Solution. Let 2A denote the whole angle, and A + x one of the two unequal parts;
then A − x denotes the other. Hence we have to show that
sin(A + x) sin(A − x) + sin2 x = sin2 A;
and this is obvious by Art. 83 (page 324).

Problem 6. Show that


(sec θ sec ϕ + tan θ tan ϕ)2 − (tan θ sec ϕ + sec θ tan ϕ)2
2 (1 + tan2 θ tan2 ϕ) − sec2 θ sec2 ϕ
sec 2θ sec 2ϕ
= .
sec2 θ sec2 ϕ
Solution. (sec θ sec ϕ + tan θ tan ϕ)2 − (tan θ sec ϕ + sec θ tan ϕ)2
= sec2 θ sec2 ϕ + tan2 θ tan2 ϕ − tan2 θ sec2 ϕ − sec2 θ tan2 ϕ
= sec2 ϕ(sec2 θ − tan2 θ) − tan2 ϕ(sec2 θ − tan2 θ)
= sec2 ϕ − tan2 ϕ = 1.
CHAPTER XXV : Miscellaneous Examples 228

2(cos2 θ cos2 ϕ + sin2 ϕ sin2 θ) − 1


2(1 + tan2 θ tan2 ϕ) − sec2 θ sec2 ϕ =
cos2 θ cos2 ϕ
(1 + cos 2θ) cos2 ϕ + (1 − cos 2θ) sin2 ϕ − 1
=
cos2 θ cos2 ϕ
cos 2θ(cos2 ϕ − sin2 ϕ) cos 2θ cos 2ϕ
= = .
cos2 θ cos2 ϕ cos2 θ cos2 ϕ
cos 2θ cos 2ϕ 2 2
cos θ cos ϕ sec 2θ sec 2ϕ
And 1÷ = = .
cos2 θ cos2 ϕ cos 2θ cos 2ϕ sec2 θ sec2 ϕ

Problem 7. If A + B + C = 360 ◦ , show that


1 − cos2 A − cos2 B − cos2 C + 2 cos A cos B cos C = 0 .
Solution. Since A + B + C = 360◦ , we have cos C = cos(A + B).
Thus 1 − cos2 A − cos2 B − cos2 C + 2 cos A cos B cos C
= 1 − cos2 A − cos2 B + cos C(2 cos A cos B − cos C)
= 1 − cos2 A − cos2 B + cos(A + B)(cos A cos B + sin A sin B)
= 1 − cos2 A − cos2 B + (cos A cos B − sin A sin B)(cos A cos B + sin A sin B)
= 1 − cos2 A − cos2 B + cos2 A cos2 B − sin2 A sin2 B
= 1 − cos2 A − cos2 B + cos2 A cos2 B − (1 − cos2 A)(1 − cos2 B)
= 0.

3 12 7
Problem 8. If sin A = , sin B = , and sin C = , where A, B, and C are positive
5 13 25
angles less than 90 ◦ , find sin(A + B + C ).

3 4
Solution. ; therefore cos A = .
sin A =
5 5
12 5
sin B = ; therefore cos B = .
13 13
7 24
sin C = ; therefore cos C = .
25 25
63 16
Hence we obtain sin(A + B) = , cos(A + B) = − ;
65 65
63 × 24 − 7 × 16 1400 56
then sin(A + B + C) = = = .
25 × 65 25 × 65 65

1 1
Problem 9. If x = r sin (θ − α) and y = r sin (θ + α), show that
2 2
x 2 − 2xy cos α + y 2 = r 2 sin2 α.
( ) ( )
θ α θ α θ α θ α
Solution. x = r sin cos − cos sin , y = r sin cos + cos sin .
2 2 2 2 2 2 2 2
θ x+y θ y−x
From these we obtain sin = α , cos 2 = α
2 2r cos 2r sin
{ 2 } 2
1 (x + y)2 (y − x)2
Square and add; thus 1 = α + α ;
4r2 cos 2 sin2
2 2
α α α α
therefore 4r2 sin2 cos2 = (x + y)2 sin2 + (y − x)2 cos2 ;
2 2 ( 2 ) 2
α α
that is r2 sin2 α = x2 + y 2 − 2xy cos2 − sin2 = x2 + y 2 − 2xy cos α.
2 2

Problem 10. Eliminate θ from the equations


θ ϕ
(a − b) sin(θ + ϕ) = (a + b) sin(θ − ϕ), a tan − b tan = c.
2 2
CHAPTER XXV : Miscellaneous Examples 229

sin(θ + ϕ) a+b sin(θ + ϕ) + sin(θ − ϕ) a


Solution. Here = , therefore = ,
sin(θ − ϕ) a−b sin(θ + ϕ) − sin(θ − ϕ) b
sin θ cos ϕ a
that is = ; so that a tan ϕ = b tan θ.
cos θ sin ϕ b
ϕ θ
a tan b tan
Hence 2 = 2 ;
ϕ θ
1 − tan2 1 − tan2
2 2 ( )
ϕ θ θ ϕ θ ϕ
∴ a tan − b tan = tan tan a tan − b tan
2 2 2 2 2 2
θ ϕ
= c tan tan ;
( ) 2 2
ϕ θ ϕ
∴ a tan = tan b + c tan .
2 2 2
θ
Substitute for tan from the second of the given equations, and we obtain
2 ( )( )
ϕ ϕ ϕ
a2 tan = b + c tan c + b tan .
2 2 2

Problem 11. The number of dgrees in one of the acute angles of a right-angled
triangle is three-tenths of the number of grades in the other : determine the angles
in degrees.
Solution. Let x denote the number of degrees in one angle; then 90 − x denotes the
10
number of degrees in the other angle, and consequently (90 − x) the number of
9
3 10 1
grades. Hence x = × (90 − x) = (90 − x). Therefore 4x = 90, and x = 22 12 .
10 9 3


Problem 12. Show that if the circular measure of an angle is , where n is any
20
integer, the angle can be expressed by an integer both in degrees and in grades.

Solution. Let the circular measure of an angle be ; then the number of degrees
20
nπ 180 nπ 200
in it is · , that is 9n; and the number of grades is · , that is 10n.
20 π 20 π

Problem 13. If sin(α + β) cos γ = sin(α + γ) cos β, show that β − γ is a multiple of π,


π
or α an odd multiple of .
2
Solution. Here (sin α cos β + cos α sin β) cos γ = (sin α cos γ + cos α sin γ) cos β;
therefore cos α(sin β cos γ − sin γ cos β) = 0,
that is cos α sin(β − γ) = 0.
π
Either then cos α = 0, so that α is an odd multiple of ; or sin(β − γ) = 0, so that
2
β − γ is a multiple of π.

Problem 14. Show that


sin 4A tan4 A + 4 tan3 A + 2 sin 4A tan2 A − 4 tan A + sin 4A = 0 .
Solution. sin 4A(tan4 A + 2 tan2 A + 1) = sin 4A(tan2 A + 1)2
sin 4A 2 sin 2A cos 2A 4 sin A cos 2A
= = =
cos4 A cos4 A cos3 A
4 sin A(cos2 A − sin2 A)
=
cos3 A
And 4 tan3 A − 4 tan A = 4 tan A(tan2 A − 1)
4 sin A(sin2 A − cos2 A)
= .
cos3 A
Therefore sin A(tan A + 2 tan A + 1) + 4 tan A − 4 tan A = 0.
4 2 3
CHAPTER XXV : Miscellaneous Examples 230


5 −1
Problem 15. Show that sin2 24 ◦ − sin2 6 ◦ = .
8
Solution.
By Art. 83 (page 324), sin2 24◦ − sin2 6◦ = sin(24◦ + 6◦ ) sin(24◦ − 6◦ )
= sin 30◦ sin 18◦ .

1 5−1
Also sin 30◦ = , and sin 18◦ = .
2 4

Problem 16. If A + B + C = 360 ◦ , show that


2(cos A sin B sin C + cos B sin C sin A + cos C sin A sin B)
+ sin2 A + sin2 B + sin2 C = 0.
Solution. The given expression is
sin A(sin A + cos B sin C + cos C sin B)
+ sin B(sin B + cos C sin A + cos A sin C)
+ sin C(sin C + cos A sin B + cos B sin A),
that is sin A{sin A + sin(B + C)} + sin B{sin B + sin(C + A)}
+ sin C{sin C + sin(A + B)}.
Now since A + B + C = 360◦ , we have
sin(B + C) = − sin A, sin(C + A) = − sin B, sin(A + B) = − sin C :
thus the whole expression vanishes.

Problem 17. If α and β are the two values of θ in the equation


cos θ cos γ sin θ sin γ 1
+ = ,
a b c
Show that
(b2 + c2 − a2 ) cos α cos β + (a2 + c2 − b2 ) sin α sin β = a2 + b2 − c2 .
Solution. We have
cos α cos γ sin α sin γ 1 cos β cos γ sin β sin γ 1
+ = , and + = .
a b c a b c
From these equations we find cos γ and sin γ. We get
1 1
a(sin β − sin α) 2a sin (β − α) cos (β + α)
cos γ = = 2 2
c(cos α sin β − cos β sin α) c sin(β − α)
1
a cos (β + α)
= 2 ;
1
c cos (β − α)
2
1 1
b(cos α − cos β) 2b sin (β + α) sin (β − α)
sin γ = = 2 2
c(cos α sin β − cos β sin α) c sin(β − α)
1
b sin (β + α)
= 2
1
c cos (β − α)
2
Square, and add; thus
1 1
a2 cos2 (β + α) + b2 sin2 (β + α)
1= 2 2
1
2
c cos 2 (β − α)
2
∴ c2 {1 + cos(β − α)} = a2 {1 + cos(β + α)} + b2 {1 − cos(β + α)};
∴ (b2 + c2 − a2 ) cos α cos β + (a2 + c2 − b2 ) sin α sin β = a2 + b2 − c2 .

1 1
Problem 18. If sin A = , and sin B = , where A and B are positive angle less than
3 2
90 ◦ , find sin 2 (A + B).
CHAPTER XXV : Miscellaneous Examples 231

1 2 7
Solution. sin A = ; therefore cos 2A = 1 − = ,
3 √( ) √
9 9
49 32
and sin 2A = 1− = .
81 9
1 1 1
sin B = ; therefore cos 2B = 1 − = ,
2 √( ) √
2 2
1 3
and sin 2B = 1− = .
4 2
√ √ √ √
32 + 7 3 4 2+7 3
Hence sin(2A + 2B) = = .
18 18

Problem 19. Solve the equation cos 4x + cos 2x + cos x = 0 .

Solution. cos 4x + cos 2x + cos x = 0;


therefore 2 cos 3x cos x + cos x = 0;
therefore either cos x = 0 or 2 cos 3x + 1 = 0.
π
If cos x = 0, then x = (2n + 1) .
2
1 2π
If cos 3x = − then 3x = 2nπ ± .
2 3

Problem 20. If A + B + C + D = 360 ◦ . show that


A+B B+C C +A
cos A + cos B + cos C + cos D = 4 cos cos cos .
2 2 2
Solution.
A+B A−B
cos A + cos B = 2 cos cos ;
2 2
C+D C−D
cos C + cos D = 2 cos cos
2 2
A+B C−D
= −2 cos cos , by Art. 48 (page 317).
2 2
Hence, by addition, { }
A+B A−B C−D
cos A + cos B + cos C + cos D = 2 cos cos − cos
2 2 2
A+B A+C−B−D C+B−A−D
= 4 cos sin sin .
2 4 4
Also
A+C−B−D 2A + 2C − 360◦
sin = sin
4 ( 4 )
A+C A+C
= sin − 90◦ = − cos ;
2 2
C+B−A−D B+C
and in like manner sin = − cos .
4 2
A+B B+C C+A
Thus we obtain finally 4 cos cos cos .
2 2 2

Problem 21. With two units of angular measurement differing by 10 ◦ the measures
of an angle are as 3 is to 2 : determine those units.
Solution. Suppose that the smaller unit contains x degrees, and therefore the larger
unit x + 10 degrees. Let n denote the number of degrees in the angle measured; then
n n 2 3
is to as 3 is to 2. Therefore = ; whence x = 20.
x x + 10 x x + 10

Problem 22. If sin x + sin2 x = 1 , find sin x; and show that


cos2 x + cos4 x = 1 .
CHAPTER XXV : Miscellaneous Examples 232

Solution.
√ sin2 x+sin x = 1. Solving this quadratic in the ordinary way we obtain sin x =
−1 ± 5
; the upper sign must be taken, as the lower would make sin x numerically
2
greater than unity.

6−2 5
Thus sin2 x = ;
√ 4 √ √
6−2 5 −2 + 2 5 −1 + 5
therefore cos2 x = 1 − = = ;
4 √ 4 √ 2
6−2 5 3− 5
therefore cos4 x = = ;
4 2
2 4
therefore cos x + cos x = 1.
Or thus : sin x = 1 − sin2 x = cos2 x; square;
therefore sin x = cos4 x, that is 1 − cos2 x = cos4 x;
2

therefore 1 = cos2 x + cos4 x.

Problem 23. Solve the equation tan2 x + cot2 x = 2 .


1
Solution. Here tan2 x + = 2;
tan2 x
therefore tan x − 2 tan2 x + 1 = 0;
4

hence tan2 x = 1; therefore tan x = ±1;


π
therefore x = nπ ± .
4

Problem 24. If a sin θ + b cos θ = c = a cosec θ + b sec θ, show that


2ab
sin 2 θ = 2 .
c − a 2 − b2
a cos θ + b sin θ
Solution. We have a sin θ + b cos θ = c, = c;
sin θ cos θ
2
hence (a sin θ + b cos θ)(a cos θ + b sin θ) = c sin θ cos θ;
therefore (a2 + b2 ) sin θ cos θ + ab = c2 sin θ cos θ;
therefore sin 2θ(c2 − a2 − b2 ) = 2ab.

Problem 25. Simplify cos2 (A + B) + cos2 (A − B) − cos 2A cos 2B.


Solution.
1 + cos(2A + 2B) 1 + cos(2A − 2B)
cos2 (A + B) + cos2 (A − B) = +
2 2
= 1 + cos 2A cos 2B;
therefore cos2 (A + B) + cos2 (A − B) − cos 2A cos 2B = 1.

Problem 26. If 2 tan A = 3 tan B, then


tan B sin 2B
tan(A − B) = = .
2 + 3 tan2 B 5 − cos 2B
Solution.
3
tan A − tan B tan B − tan B
tan(A − B) = = 2
1 + tan A tan B 3
1 + tan2 B
2
tan B sin B cos B
= =
2 + 3 tan2 B 2 cos2 B + 3 sin2 B
sin 2B
=
2(1 + cos 2B) + 3(1 − cos 2B)
sin 2B
= .
5 − cos 2B
CHAPTER XXV : Miscellaneous Examples 233

Problem 27. Solve the equation


n+1 n−1
sin θ + sin θ = sin θ.
2 2
n+1 n−1
Solution. sin θ + sin θ = sin θ,
2 2
nθ θ θ θ
therefore 2 sin cos = 2 sin cos .
2 2 2 2
θ nθ θ
Thus either cos = 0, or sin = sin .
2 2 2
θ π
From the former we have = (2m + 1) . All the solutions of the latter are com-
2 2
nθ θ
prised in = mπ + (−1)m , where m is zero or an integer.
2 2

Problem 28. If
tan(2 α − 3 β) = cot(3 α − 2 β), and tan(2 α + 3 β) = cot(3 α + 2 β),
π
show that both α and β are multiples of .
10
Solution. ( )
π
Here tan(2α − 3β) = tan − 3α + 2β ,
(2 )
π
and tan(2α + 3β) = tan − 3α − 2β .
2
Hence all possible solutions are comprised in
π π
2α − 3β = mπ + − 3α + 2β, and 2α + 3β = nπ + − 3α − 2β,
2 2
where m and n are zero or integers.
π π
From these we obtain α = (m + n + 1) , β = (n − m) ,
10 10
π
so that α and β are multiples of .
10

Problem 29. Solve the equation


1 − 2 cos 2 α
tan(α + x) tan(α − x) = .
1 + 2 cos 2 α
sin(α + x) sin(α − x) 1 − 2 cos 2α
Solution. Here = ;
cos(α + x) cos(α − x) 1 + 2 cos 2α
sin α − sin x
2 2 1 − 2 cos 2α
therefore, by Art. 83 (page 324), = ;
cos2 α − sin2 x 1 + 2 cos 2α
therefore 4 cos 2α sin2 x = sin2 α(1 + 2 cos 2α) − cos2 α(1 − 2 cos 2α)
= − cos 2α + 2 cos 2α = cos 2α;
1 1 π
therefore sin x = ; therefore sin x = ± ; therefore x = nπ ± .
2
4 2 6

Problem 30. If A + B + C + D = 360 ◦ , show that


A+B B+C C +A
sin A + sin B + sin C + sin D = 4 sin sin sin .
2 2 2
Solution.
A+B A−B
sin A + sin B = 2 sin cos ;
2 2
C+D C−D
sin C + sin D = 2 sin cos
2 2
A+B C−D
= 2 sin cos , by Art. 48 (page 317).
2 2
Hence, by addition,
{ }
A+B A−B C−D
sin A + sin B + sin C + sin D = 2 sin cos + cos
2 2 2
A+B A+C−B−D A+D−B−C
= 4 sin cos cos .
2 4 4
CHAPTER XXV : Miscellaneous Examples 234

Then, as in P roblem 20, we can show that


A+C−B−D A+C A+D−B−C B+C
cos = sin , and cos = sin .
4 2 4 2

Problem 31. One angle of a quadrilateral contains 60 degrees, another contains 50



grades, the circular measure of another is . Express all the four angles in degrees.
4
9
Solution. The first angle contains 60 degrees; the second angle contains × 50
10
3π 180
degrees, that is 45 degrees; the third angle contains × degrees, that is 135
4 π
degrees. Therefore the fourth angle must contain 360 − 60 − 45 − 135 degrees, that is
120 degrees.

40 60
Problem 32. If cos A = and cos B = , where A and B are angles less than a
41 61
A − B 1
right angle, show that sin2 = .
√ ( )2 √41 × 61 √
40 2 (41 − 40)(41 + 40) 81 9
Solution. sin A = 1− = = = ;
41 41 41 41
√ ( ) √ √
60 2 (61 − 60)(61 + 60) 121 11
sin B = 1− = = = .
61 61 61 61
40 × 60 + 9 × 11 2499
cos(A − B) = cos A cos B + sin A sin B = = ;
41 × 61 2501
1 2499
thus 1 − 2 sin2 (A − B) = ;
2 2501
1 2
therefore 2 sin2 (A − B) = ;
2 41 × 61
1 1
therefore sin2 (A − B) = .
2 41 × 61

Problem 33. Solve the equation sin 3 θ = 8 sin3 θ.


Solution. Here 3 sin θ − 4 sin3 θ = 8 sin3 θ;
therefore 3 sin θ = 12 sin3 θ;
1
therefore either sin θ = 0, or sin2 θ = :
4
π
the former gives θ = nπ; the latter gives θ = nπ ± .
6

Problem 34. Eliminate θ and ϕ from the equations


a 2 cos2 θ − b2 cos2 ϕ = c 2 , a cos θ + b cos ϕ = r, a tan θ = b tan ϕ.
Solution. Divide the first by the second; thus we get
c2
a cos θ − b cos ϕ = ;
( ) r ( )
1 c 2 1 c2
therefore cos θ = r+ , cos ϕ = r− .
2a r 2b r
a2 sin2 θ b2 sin2 ϕ
Now 2
= ; therefore a2 (sec2 θ − 1) = b2 (sec2 ϕ − 1);
cos θ cos{ϕ } { }
4r2 a2 4r2 b2
therefore a2 = b 2
− 1 .
(r2 + c2 )2 − 1 (r2 − c2 )2

Problem 35. If tan A, tan B, tan C are in Arithmetical Progression, and


tan A, tan B, tan D in Harmonical Progression, then
tan C 8 sin2 (A − B)
=1− .
tan D sin 2A sin 2B
CHAPTER XXV : Miscellaneous Examples 235

1 1 2
Solution. Here tan A + tan C = 2 tan B, and + = ;
( tan A ) tan D ( tan B )
tan C 2 1 tan B tan A
therefore = (2 tan B − tan A) − =5−2 +
tan D tan B
( ) tan A 2 tan A tan B
sin B cos A sin A cos B sin B cos2 A + sin2 A cos2 B
=5−2 + =5−2
cos B sin A cos A sin B sin A cos A sin B cos B
2(sin A cos B − cos A sin B)2 8 sin2 (A − B)
=1− =1− .
sin A cos A sin B cos B sin 2A sin 2B

Problem 36. Find cos x from the equation cos x sin2 x = sin α cos2 α, having given as
one solution cos x = sin α.
Solution. Here cos x(1 − cos2 x) = sin α(1 − sin2 α); therefore
cos x − sin α = cos3 x − sin3 α = (cos x − sin α)(cos2 x + cos x sin α + sin2 α);
therefore either cos x − sin α = 0, or 1 = cos2 x + cos x sin α + sin2 a.
The latter gives cos2 x + cos x sin α = cos2 α; by solving this quadratic equation
we obtain √
− sin α ± sin2 α + 4 cos2 α
cos x = ;
2
it will be found that only one of these values is numerically less than unity, namely,
the numerically less of the two.

Problem 37. Show that ( ) ( )


(2 cos θ − 1) (2 cos 2θ − 1) 2 cos 22 θ − 1 . . . 2 cos 2n−1 θ − 1
2 cos 2n θ + 1
= .
2 cos θ + 1
Solution.
4 cos2 θ − 1 2 cos 2θ + 1
We have 2 cos θ − 1 = = ,
2 cos θ + 1 2 cos θ + 1
4 cos 2θ − 1
2 2 cos 4θ + 1
2 cos 2θ − 1 = = ,
2 cos 2θ + 1 2 cos 2θ + 1
and so on, which we use down to
4 cos2 2n−1 θ − 1 2 cos 2n θ + 1
2 cos 2n−1 θ − 1 = n−1
= .
2 cos 2 θ+1 2 cos 2n−1 θ + 1
Multiply these expressions together, then by cancelling we obtain the required
result.

Problem 38. If tan(π cot θ) = cot(π tan θ), show that



2n + 1 4n 2 + 4n − 15
tan θ = ± ,
4 4
(
where n is a positive or negative integer. )
π
Solution. Here tan(π cot θ) = tan − π tan θ ;
2
hence all possible solutions are comprised in the formula
π
π cot θ = nπ + − π tan θ;
( )2
1
thus tan2 θ − n + tan θ + 1 = 0;
2
by solving this quadratic equation we obtain the value of tan θ.

Problem 39. Express in factors


cos2 A + cos2 B + cos2 C − 2 cos A cos B cos C − 1 .
Solution. cos A + cos2 B + cos2 C − 2 cos A cos B cos C − 1
2

= (cos A − cos B cos C)2 + cos2 B + cos2 C − 1 − cos2 B cos2 C


= (cos A − cos B cos C)2 − (1 − cos2 B)(1 − cos2 C)
= (cos A − cos B cos C)2 − sin2 B sin2 C
= (cos A − cos B cos C + sin B sin C)(cos A − cos B cos C − sin B sin C)
CHAPTER XXV : Miscellaneous Examples 236

= {cos A − cos(B + C)}{cos A − cos(B − C)}


A+B+C B+C−A A+B−C B−C−A
= 4 sin sin sin sin .
2 2 2 2
A+B+C B+C−A A+C−B A+B−C
= −4 sin sin sin sin .
2 2 2 2

Problem 40. If A + B + C + D = 360 ◦ , show that


A+B B+C C +A
sin A − sin B + sin C − sin D = 4 cos cos sin .
2 2 2
Solution.
A−B A+B
sin A − sin B = 2 sin cos ;
2 2
C−D C+D
sin C − sin D = 2 sin cos
2 2
C−D A+B
= −2 sin cos , by Art. 48 (page 317).
2 2
Hence, by addition,
{ }
A+B A−B C−D
sin A − sin B + sin C − sin D = 2 cos sin − sin
2 2 2
A+B A+D−B−C A+C−B−D
= 4 cos sin cos .
2 4 4
Then, as in P roblem 20, we can show that
A+D−B−C B+C A+C−B−D A+C
sin = cos , and cos = sin .
4 2 4 2

Problem 41. Express in each system of angular measurement the angle described
by the minute hand of a watch in 25 minutes.
25
Solution. The angle described is of four right angles; the number of degrees
60
5 5
= × 360 = 150; the number of grades = × 400 = 166 23 ; the circular measure
12 12
5 5π
= × 2π = .
12 6

Problem 42. Show that ( )


π
(cos θ + sin θ) (cos 2 θ + sin 2 θ) = cos θ + cos 3 θ − .
√ ( ) √ ( ) 2
cos θ sin θ π
Solution. cos θ + sin θ = 2 √ + √ = 2 · cos θ − ;
2 2
√ ( )4
π
similarly cos 2θ + sin 2θ = 2 cos 2θ − :
( ) ( ) 4( )
π π π
the product = 2 cos θ − cos 2θ − = cos θ + cos 3θ − .
4 4 2

Problem 43. Show that


cosec A cosec 2A + cosec 2A cosec 3A = cosec A (cot A − cot 3A) .
1 sin A + sin 3A
Solution. cosec 2A(cosec A + cosec 3A) = ·
sin 2A sin A sin 3A
1 2 sin 2A cos A 2 cos A
= · = ;
sin 2A sin A sin 3A (sin A sin 3A )
1 cos A cos 3A
and cosec A(cot A − cot 3A) = −
sin A sin A sin 3A
1 sin 3A cos A − cos 3A sin A 1 sin(3A − A)
= · = ·
sin A sin A sin 3A sin A sin A sin 3A
sin 2A 2 sin A cos A 2 cos A
= = = :
sin2 A sin 3A sin2 A sin 3A sin A sin 3A
thus the proposed expressions are equal.
CHAPTER XXV : Miscellaneous Examples 237

Problem 44. Show that


1 3
1 cot2 A − cot2 A
2
sec A sec A 2 2 = 8.
2 3
1 + cot2 A
2
cot 2 1 A − cot2 3 A
1 2 2
Solution. sec2 A sec A
2 2
3
1 + cot A
2
1 3 2 3 2 1
1 cos 2 A sin 2 A − cos 2 A sin 2 A
2 2
1
=
1 cos A
( )
1 3A 3A
cos2 A sin2 A cos2 + sin2
( 2 2 )
2( 2 )
1 3 3 1 1 3 3 1
cos A sin A − cos A sin A cos A sin A + cos A sin A
2 2 2 2 2 2 2 2
=
2 1 2
1
sin A cos A cos A
( ) ( 2) 2
3A A 3A A
sin − sin +
2 2 2 2 sin A sin 2A
= =
1 1 1 1
sin2 A cos2 A cos A sin2 A cos2 A cos A
2 2 ( )2
2 2
1 1
2 2 sin A cos A
2 sin2 A 2 2
= = = 8.
1 1 1 1
sin2 A cos2 A sin2 A cos2 A
2 2 2 2

Problem 45. Show that { }{ }


1 1
{sec A + cosec A(1 + sec A)} 1 − tan2 A 1 − tan2
A
( 2 ) 4
1 1 1
= sec A + cosec A sec2 A.
2( 2) 4
1 1 1
Solution. sec A + cosec (1 + sec A) = + 1+
cos A sin A cos A
1 1 1
1 + cos A + sin A 2 cos2 A + 2 sin A cos A
= = 2 2 2
cos A sin A 1 1
2 sin A cos A cos A
2 2
1 1 1
cos A + sin A cos A ( 1 1
)
= 2 2 = 2 sec A + cosec A ;
1 cos A 2 2
sin A cos A
2
1 1
cos2 A − sin2 A
2 1 2 2 cos A
1 − tan A= = ;
2 2
1 1
cos A cos2 A
2 2
cos2 1 A − sin2 1 A cos
1
A
1 4 4 = 2 .
1 − tan2 A =
4 1 1
cos2 A cos2 A
4 4
Hence by multiplication we obtain the required result.

Problem 46. If
√ √
a 2 b2 a 2 b2
(a − b) sec θ = a4 + 2 , and (a + b) sec ϕ = a4 + ,
a −1 a2 − 1
1 b
then tan (θ − ϕ) = √ .
2 a a2 − 1
CHAPTER XXV : Miscellaneous Examples 238

a2 b2
Solution. Put c2 for a4 + : thus
a2 − 1
a−b a+b
cos θ = , and cos ϕ = :
c c
from these we obtain
a(a2 − 1) + b a(a2 − 1) − b
sin θ = √ , and sin ϕ = √ .
c − 1)(a2 c (a2 − 1)
a −a −b
4 2 2
Next we obtain cos(θ − ϕ) = 4 ;
a − a2 + b2
1 1 − cos(θ − ϕ) b2
and thus tan2 (θ − ϕ) = = 4 .
2 1 + cos(θ − ϕ) a − a2

Problem 47. Eliminate θ and ϕ from the equations


a sin(ϕ − θ) c b sin θ
= , = cos(ϕ − θ), = .
b sin(ϕ + θ) x x sin ϕ
Solution.
a sin(ϕ − θ)
= (25)
b sin(ϕ + θ)
c
= cos(ϕ − θ) (26)
x
b sin θ
= (27)
x sin ϕ
b+a sin ϕ cos θ x cos θ
From (25) we get = = , by (27).
b−a sin θ cos ϕ b cos ϕ
c
By (26) we have = cos ϕ cos θ + sin ϕ sin θ
x
b b+a b
= · cos2 ϕ + sin2 ϕ;
x b−a x
(c − b)(b − a)
hence we get cos2 ϕ = ;
2ab
(c − b)(a + b) b22
therefore cos2 θ = .
2ab(b − a) x2
b 2
But from (27) we have 2 (1 − cos2 ϕ) = 1 − cos2 θ,
x { }
b2 b2 (c − b)(b − a) (c − b)(a + b)2
so that −1= 2 − ,
x2 x 2ab 2ab(b − a)
b2 (c − b)4ab 2b2 (c − b)
or x2 − b2 = = ;
2ab(b − a) b−a
therefore x (b − a) = b (2c − a − b).
2 2

x
Problem 48. Find cos from the equation
2 ( )
π β x
cos x cos − = sin β cos .
4 2 2
x
Solution. Put 2 cos2 − 1 for cos x; thus we obtain the quadratic equation
2 ( ) ( )
2 x π β x π β
2 cos cos − − sin β cos = cos − .
2 4 2 2 4 2
By solving this we have
√{ ( )}
π β
sin β ± sin2 β + 8 cos2 −
x 4 2
cos = ( )
2 π β
4 cos −
( ) 4 2{ ( )}
π β π
and sin2 β + 8 cos2 − = sin2 β + 4 1 + cos −β
4 2 2
CHAPTER XXV : Miscellaneous Examples 239

= sin2 β + 4 + 4 sin β.
x sin β ± (2 + sin β)
Hence cos = ( )
2 π β
4 cos −
4 2
x 1 + sin β
Take the upper sign; then cos = ( )
2 π β
2 cos −
( ) (4 2 )
π π β
1 + cos −β 2 cos2 − ( )
2 4 2 π β
= ( ) = ( ) = cos − .
π β π β 4 2
2 cos − 2 cos −
4 2 4 2
1
Take the lower sign; then cos x = − ( ).
π β
2 cos −
4 2

Problem 49. If
cos(θ + 3ϕ) = sin(2θ + 2ϕ), and sin(ϕ + 3θ) = cos(2θ + 2ϕ),
π π π π
show that θ = (3m − 5n) + and ϕ = (3n − 5m) + ,
8 16 8 16
π
or else ϕ − θ = 2mπ − ; where m and n are integers.
2
Solution. Write the first equation
( in the form )
π
cos − 2θ − 2ϕ = cos(θ + 3ϕ);
2
hence all possible solutions are comprised in
π
− 2θ − 2ϕ = 2mπ ± (θ + 3ϕ).
2
If we take the upper sign we have
π
3θ + 5ϕ = − 2mπ (28)
2
If we take the lower sign we have
π
ϕ − θ = 2mπ − (29)
2
Again, write the second equation
( in the
) form
π
cos − ϕ − 3θ = cos(2θ + 2ϕ);
2
hence all possible solutions are comprised in
π
− ϕ − 3θ = 2nπ ± (2θ + 2ϕ).
2
If we take upper sign we have
π
5θ + 3ϕ = − 2nπ (30)
2
If we take the lower sign we have
π
ϕ − θ = 2nπ − ;
2
this agrees with (29).
Thus either (29) holds, or both (28) and (30) hold. From (28) and (30) we obtain
16θ = (3m − 5n)2π + π, 16ϕ = (3n − 5m)2π + π.

Problem 50. Show that


(1 + sec 2θ) (1 + sec 4θ) (1 + sec 8θ) . . . (1 + sec 2n θ)
tan 2n θ
= .
tan θ
1 + cos 2θ sin 2θ 1 + cos 2θ
Solution. We have 1 + sec 2θ = = ·
cos 2θ cos 2θ sin 2θ
tan 2θ
= tan 2θ · cot θ = .
tan θ
tan 4θ
Similarly 1 + sec 4θ = ,
tan 2θ
CHAPTER XXV : Miscellaneous Examples 240

and so on, which we use down to


tan 2n θ
1 + sec 2n θ = .
tan 2n−1 θ
Multiply these expressions together, then by cancelling we obtain the required
result.

Problem 51. The circular measure of a certain angle is equal to the ratio of the
number of degrees in it to the number of grades. Find the magnitude of the angle in
degrees.
9
Solution. Here the circular measure of an angle is given equal to ; hence the
10
9 180 162
number of degrees in it is · , that is .
10 π π

Problem 52. Show that


{sin(A − B) + sin(A + 3B)} sec 2B = (cos 2B − cos 2A) cosec (A − B).
Solution.
sin(A − B) + sin(A + 3B) = 2 sin(A + B) cos 2B;
therefore {sin(A − B) + sin(A + 3B)} sec 2B = 2 sin(A + B).
cos 2B − cos 2A = 2 sin(A − B) sin(A + B);
therefore (cos 2B − cos 2A) cosec (A − B) = 2 sin(A + B) :
thus the proposed expressions are equal.

tan θ 1 + cos2 θ
Problem 53. If = , show that sin(3 θ + α) = 7 sin(θ − α).
tan α 1 + sin2 θ
sin θ sin α
Solution. Here (1 + sin2 θ) = (1 + cos2 θ);
cos θ cos α
therefore cos α(sin θ + sin3 θ) = sin α(cos θ + cos3 θ);
3 sin θ − sin 3θ 3 cos θ + cos 3θ
therefore cos α sin θ + cos α = sin α cos θ + sin α ;
4 4
therefore 7(sin θ cos α − cos θ sin α) = sin 3θ cos α + cos 3θ sin α;
that is 7 sin(θ − α) = sin(3θ + α).

Problem 54. Solve the equation √


cos 3 θ + cos 5 θ + 2 (cos θ + sin θ) cos θ = 0 .

Solution. Here 2 cos 4θ cos θ + 2(cos θ + sin θ) cos θ = 0;
1
therefore either cos θ = 0 or cos 4θ = − √ (cos θ + sin θ).
2
π
Take the former; then θ = (2n + 1) .
(2 )

Take the latter; thus cos 4θ = cos +θ ;
4 ( )

thereofore 4θ = 2nπ ± +θ .
4

Problem 55. Eliminate ϕ from the equations


n sin θ − m cos θ = 2m sin ϕ, n sin 2 θ − m cos 2 ϕ = n.
n sin θ − m cos θ
Solution. We have sin ϕ = ;
2m
and n sin 2θ − m(1 − 2 sin2 ϕ) = n;
( )
n sin θ − m cos θ 2
therefore n sin 2θ + 2m = m + n;
2m
therefore 2mn sin 2θ + (n sin θ − m cos θ) = 2m(m + n);
2

therefore (n sin θ + m cos θ)2 = 2m(m + n).


CHAPTER XXV : Miscellaneous Examples 241

Problem 56. ( Solve the


) equation ( ) ( ) √
π π π
8 sin θ − cos3 θ + 8 cos θ − sin3 θ − 6 sin 2 θ − = 3.
3 3 3
3 cos θ + cos 3θ 3 sin θ − sin 3θ
Solution. Substitute for cos3 θ and for sin3 θ thus the
4 4
equation becomes ( ) ( )
π π
2 sin θ − {3 cos θ + cos 3θ} + 2 cos θ − {3 sin θ − sin 3θ}
3 ( ) √ 3
π
−6 sin 2θ − = 3,
( ) 3( ) √
π π
that is 2 sin θ − cos 3θ − 2 cos θ − sin 3θ = 3,
3 3√
( )
π 3
that is − sin 3θ − θ + = ;
3 2
( ) √
π 3
thus sin 2θ + =− ;
3 2
π 4π
therefore 2θ + = nπ + (−1)n .
3 3

Problem 57. Among all values of θ between 0 and π find that which makes sin θ cos(β − θ)
π
greatest; β being a given angle between 0 and .
2
1
Solution. sin θ cos(β − θ) = {sin β + sin(2θ − β)}.
2
π β π
The greatest value of sin(2θ − β) is obviously when 2θ − β = , so that θ = + .
2 2 4

Problem 58. Show that


cos 55◦ + cos 65◦ + cos 175◦ = 0,
3
cos 55◦ cos 65◦ + cos 65◦ cos 175◦ + cos 55◦ cos 175◦ = − ,
4 √
1+ 3
cos 55◦ cos 65◦ cos 175◦ = − √ .
8 2
Solution. cos 55◦ + cos 65◦ = 2 cos 60◦ cos 5◦ = cos 5◦ = − cos 175◦ ;
therefore cos 55◦ + cos 65◦ + cos 175◦ = 0.
1
cos 55◦ cos 65◦ = (cos 10◦ + cos 120◦ ),
2
1
cos 65◦ cos 175◦ = (cos 110◦ + cos 240◦ ),
2
1
cos 55◦ cos 175◦ = (cos 120◦ + cos 230◦ );
2
3 1
hence by addition we obtain − + (cos 10◦ + cos 110◦ + cos 230◦ ),
4 2
3 1
that is − + (cos 10◦ + cos 110◦ − cos 50◦ ),
4 2
3 1 3
that is − + (2 cos 60◦ cos 50◦ − cos 50◦ ), that is − .
4 2 4
1
cos 55◦ cos 65◦ cos 175◦ = (cos 10◦ + cos 120◦ ) cos 175◦
2
1 1
= (cos 165◦ + cos 185◦ ) + (cos 55◦ + cos 295◦ )
4 4
1
= (− cos 15◦ + cos 175◦ + cos 55◦ + cos 65◦ )
4
1
= − cos 15◦ , by what has been already shown,
4 √ √
1 3+1 3+1
=− · √ =− √ .
4 2 2 8 2
CHAPTER XXV : Miscellaneous Examples 242

Problem 59. If x cos(α + β) + cos(α − β)


= x cos(β + γ) + cos(β − γ) = x cos(γ + α) + cos(γ − α)
tan α tan β tan γ
then = = .
1 1 1
tan (β + γ) tan (γ + α) tan α + β)
2 2 2
Solution. From x cos(α + β) + cos(α − β) = x cos(β + γ) + cos(β − γ)
( )
α+γ
sin −β
cos(β − γ) − cos(α − β)
we obtain x= =− ( 2 ).
cos(α + β) − cos(β + γ) α+γ
sin +β
2
Two other expressions for the value of x may be obtained; and thus we have
( ) ( ) ( )
α+γ β+α γ+β
sin −β sin −γ sin −α
( 2 )= ( 2 )= ( 2 ) = −x;
α+γ β+α γ+β
sin +β sin +γ sin +α
2 2 2
α+γ α+γ
sin cos β 1−x tan 1−x
hence 2 = , that is 2 = .
α+γ 1 + x tan β 1+x
cos sin β
2
β+α γ+β
tan tan 1−x
Similarly 2 and 2 are also equal to .
tan γ tan α 1+x

Problem 60. If A + B + C + D = 360 ◦ , show that


A+B B+C C +A
cos A − cos B + cos C − cos D = 4 sin sin cos .
2 2 2
Solution.
A+B B−A
cos A − cos B = 2 sin sin ;
2 2
C+D D−C
cos C − cos D = 2 sin sin
2 2
A+B D−C
= 2 sin sin , by Art. 48 (page 317).
2 2
Hence by addition,
{ }
A+B B−A D−C
cos A − cos B + cos C − cos D = 2 sin sin + sin
2 2 2
A+B B+D−A−C B+C−A−D
= 4 sin sin cos .
2 4 4
Then, as in P roblem 20, we can show that
B+D−A−C A+C B+C−A−D B+C
sin = cos , and cos = sin .
4 2 4 2

Problem 61. The number of degrees in an angle of one regular polygon is to the
number of grades in an angle of another as 3 is to 5 . Find the number of sides in each
polygon, showing that there are only three solutions.
Solution. Let x denote the number of sides in the first regular polygon, and y the
number of sides in the second. All the angles of the first polygon are equal to 2x − 4
2x − 4
right angles; therefore each angle is equal to right angles, and therefore con-
x
2x − 4
tains 90 degrees. In the same way each angle of the second polygon contains
x
2y − 4 2x − 4 2y − 4
100 grades. Then, by supposition, we have 90 : 100 :: 3 : 5;
y x y
2x − 4 2y − 4
therefore 5 90 = 3 100;
x y
3(x − 2) 2(y − 2)
therefore = ;
x y
CHAPTER XXV : Miscellaneous Examples 243

therefore 3y(x − 2) = 2x(y − 2); therefore y(6 − x) = 4x. This formula shows that x
cannot be greater than 5; for if x = 6 we should have y × 0 = 24, which is absurd; and
if x is greater than 6 we should have a negative value for y, which is also absurd. And
x cannot be less than 3. Thus the only possible solutions are x = 3, x = 4, and x = 5;
which give respectively y = 4, y = 8, and y = 20.

x x
Problem 62. Solve the equation sec2 + cosec 2 = 16 cot x.
2 2
1 1 16 cos x
Solution. Here x + x = sin x ;
cos2 sin2
2 2
x x
x x 16 cos x cos2 sin2
therefore sin2 + cos2 = 2 2 = 8 cos x cos x sin x ;
2 2 sin x 2 2
therefore 1 = 4 cos x sin x = 2 sin 2x;
1 π
therefore sin 2x = ; therefore 2x = nπ + (−1)n .
2 6

Problem 63. Eliminate θ from the equations


m sin 2 θ = n sin θ, p cos 2 θ = q cos θ.
Solution. Here m sin 2θ = n sin θ, p cos 2θ = q cos θ;
n
from the first equation 2m sin θ cos θ = n sin θ; therefore cos θ = .
2m
Substitute in the second equation, that is in p(2 cos2 θ − 1) = q cos θ;
{ ( )2 }
n qn
thus p 2 −1 = ; therefore p(n2 − 2m2 ) = qmn.
2m 2m

Problem 64. Find θ from the equation


cos θ − sin θ = cos α − sin α.
Solution. We may write the equation thus :
1 1 1 1
√ cos θ − √ sin θ = √ cos α − √ sin α;
2 ( 2 ) 2 ( ) 2
π π
therefore cos θ + = cos α + ;
4 ( )
4
π π
therefore θ + = 2nπ ± α + .
4 4

Problem 65. Show that if sin(A + B + C + D) = 0 , then


sin(A + C ) sin(A + D) = sin(B + C ) sin(B + D).
Solution. sin(A + C) sin(A + D) − sin(B + C) sin(B + D)
1 1
= {cos(C − D) − cos(2A + C + D)} − {cos(C − D) − cos(2B + C + D)}
2 2
1
= {cos(2B + C + D) − cos(2A + C + D)}
2
= sin(A − B) sin(A + B + C + D).
Thus, if sin(A + B + C + D) vanishes, the difference between the two proposed ex-
pressions vanishes; and therefore the two expressions are equal.

Problem 66. Show that all the values of θ which satisfy the equations
sin θ + sin ϕ = p, cos θ + cos ϕ = q,
are contained in the expression nπ − α + (−1 )n β, where α and β are angles deter-
mined by the equations
q 1√ 2
tan α = , sin β = p + q2 .
p 2
Solution. We have sin ϕ = p − sin θ, cos ϕ = q − cos θ; square and add,
thus 1 = p2 + q 2 − 2p sin θ − 2q cos θ + 1;
CHAPTER XXV : Miscellaneous Examples 244

therefore 2p sin θ + 2q cos θ = p2 + q 2 .


q
Now assume that tan α = , so that
p
q p
sin α = √ and cos α = √ ;
(p2 + q 2 ) (p2 + q 2 )

thus 2 (p2 + q 2 ){sin θ cos α + cos θ sin α} = p2 + p2 ,

(p2 + q 2 )
therefore sin(θ + α) = = sin β say ;
2
therefore θ + α = nπ + (−1)n β.

Problem 67. Show that ( )7


π 2π 3π 4π 5π 6π 7π 1
cos cos cos cos cos cos cos = .
15 15 15 15 15 15 15 2
Solution.
( ) ( √ ) √
π 4π 1 π π 1 1 5+1 3+ 5
cos cos = cos + cos = + = ;
15 15 2 3 5 2 2 4 8
( ) ( √ ) √
2π 7π 1 π 3π 1 1 5−1 3− 5
cos cos = cos + cos = − = ;
15 15 2 3 5 2 2 4 8
( ) (√ √ )
3π 6π 1 π 3π 1 5+1 5−1 1
cos cos = cos + cos = − = ;
15 15 2 5 5 2 4 4 4
5π 1
cos = ;
15 2
π 2π 3π 4π 5π 6π 7π
therefore cos cos cos cos cos cos cos
15 15 √ 15 √ 15 15 15 15
3+ 5 3− 5 1 4 1
= · · = 3 = 7.
8 8 8 8 2

Problem 68. Show that whatever θ may be,


a sin2 θ + b sin θ cos θ + c cos2 θ
lies in value between
1 1√ 2 1 1√ 2
(a + c) + b + (a − c)2 and (a + c) − b + (a − c)2 .
2 2 2 2
2 2
Solution. a sin θ + b sin θ cos θ + c cos θ
1
= {a(1 − cos 2θ) + b sin 2θ + c(1 + cos 2θ)}
2
1
= {a + c + b sin 2θ − (a − c) cos 2θ}.
2
a−c b
Now let α be an angle such that tan α = , so that cos α = √ , and
b b2 + (a − c)2
a−c
sin α = √ . Then the above expression
b2 + (a − c)2
1 1√ 2
= (a + c) + b + (a − c)2 {sin 2θ cos α − cos 2θ sin α}
2 2
1 1 √
= (a + c) + b2 + (a − c)2 sin(2θ − α).
2 2
Then as sin(2θ − α) must lie between −1 and +1, we obtain the required result.

Problem 69. Show that ( ) ( )


2π 2π
cos α + cos + α + cos − α = 0,
( ) (3 ) (3 ) ( )
2π 2π 2π 2π 3
cos α cos + α + cos α cos − α + cos + α cos −α =− ,
3 ( 3 ) ( )
3 3 4
2π 2π cos 3α
cos α cos + α cos −α = .
3 3 4
CHAPTER XXV : Miscellaneous Examples 245

Solution. ( ) ( )
2π 2π 2π
cos + α + cos − α = 2 cos cos α = − cos α;
3 ( 3 ) ( 3 )
2π 2π
therefore cos α + cos + α + cos − α = 0.
( )3 { 3 ( )}
2π 1 2π 2π
cos α cos +α = cos + cos + 2α
(3 ) 2{ 3 (3 )}
2π 1 2π 2π
cos α cos −α = cos + cos − 2α
( 3) ( 2 ) 3 { 3 }
2π 2π 1 4π
cos + α cos −α = cos + cos 2α .
( 3 ) (3 )2 3
2π 2π
Now cos 2α + cos + 2α + cos − 2α is zero, in the manner already shown;
3 3
2π 4π 1 3
and cos and cos are each − : thus the sum is − .
3 3 ( ) 2 ( ) 4
2π 2π
cos α cos + α cos −α
3( 3 )

= cos α cos2 α − sin2 , by Art. 83 (page 324),
( ) 3
3 1 cos 3α
= cos α cos2 α − = cos α(4 cos2 α − 3) = .
4 4 4

Problem 70. (Find an expression


) ( for the product ) ( )
α β α β α β
cos + cos cos 2 + cos 2 . . . cos n + cos n .
2 2 2 2 2 2
Solution.
α β α β
α β cos2 n − cos2 n 1 cos 2n−1 − cos 2n−1
cos n + cos n = 2 2 = ;
2 2 α β 2 α β
cos n − cos n cos n − cos n
2 2 2 2
α β
α β 1 cos − cos
similarly cos n−1 + cos n−1 = 2n−2 2n−2 ;
2 2 2 α β
cos n−1 − cos n−1
2 2
and we use a series of these transformation down to
α β 1 cos α − cos β
cos + cos = .
2 2 2 1 1
cos α − cos β
2 2
Then by multiplication we obtain for the product
1 cos α − cos β
.
2n α β
cos n − cos n
2 2

Problem 71. An angle is the excess of a ◦ b′ above pg q ′ . Find the ratio of this angle
to a right angle.
60a + b 100p + q
Solution. The angle a◦ b′ is of a right angle; the angle pg q ′ is of a
60 × 90 100 × 100
right angle. { }
60a + b 100p + q
Hence the excess of the former above the latter is − of a right
60 × 90 100 × 100
angle.

Problem 72. Solve the equation 2 sin2 x + sin2 2x = 2 .


Solution.
Here 1 − cos 2x + sin2 2x = 2;
therefore 1 − cos 2x + 1 − cos2 2x = 2;
therefore cos 2x(1 + cos 2x) = 0.
Therefore either cos 2x = 0, or 1 + cos 2x = 0.
CHAPTER XXV : Miscellaneous Examples 246

π
If cos 2x = 0, we have 2x = 2nπ ± , which may be written more simply as 2x =
2
π
(2m + 1) .
2
If 1 + cos 2x = 0 we have cos 2x = −1, and therefore 2x = 2nπ ± π which may be
written more simply as 2x = (2m + 1)π.

Problem 73. Show that


tan A + 2 tan 2A + 4 tan 4A + 8 cot 8A = cot A.
Solution.
sin A cos A sin2 A − cos2 A 2(sin2 A − cos2 A)
tan A − cot A = − = =
cos A sin A sin A cos A 2 sin A cos A
2 cos 2A
=− = −2 cot 2A.
sin 2A
Similarly 2 tan 2A − 2 cot 2A = −4 cot 4A,
and 4 tan 4A − 4 cot 4A = −8 cot 8A.
Therefore by addition and cancelling
tan A − cot A + 2 tan 2A + 4 tan 4A = −8 cot 8A;
therefore tan A + 2 tan 2A + 4 tan 4A + 8 cot 8A = cot A.

Problem 74. Solve the equation cos 2x − cos 4x = sin x.


Solution. Here 2 sin 3x sin x = sin x; therefore either sin x = 0 or 2 sin 3x = 1.
1 π
If sin x = 0, then x = nπ. If sin 3x = , then 3x = nπ + (−1)n .
2 6

Problem 75. If the sum of the angles A, B, C , D be four right angles, and their
tangents in geometrical progression, show that the ratio = −1 ; or else that tan A tan D
= tan B tan C = 1 .
Solution. Let r denote the common ratio of the Geometrical Progression, so that
tan B = r tan A, tan C = r tan B, tan D = r tan C;
therefore tan A tan D = tan B tan C.
Now since A + D = 360◦ − B − C, we have tan(A + D) = − tan(B + C);
tan A + tan D tan B + tan C
therefore =− .
1 − tan A tan D 1 − tan B tan C
Thus we must have either tan A tan D = tan B tan C = 1,
or else tan A + tan D = −(tan B + tan C).
The latter gives (1 + r3 ) tan A = −(r + r2 ) tan A,
so that 1 + r + r + r2 = 0, that is (1 + r)(1 + r2 ) = 0.
3

the only possible solution is 1 + r = 0, so that r = −1.

Problem 76. The angles A, B, C of a triangle are in Arithmetical Progression; and


cosec 2A, cosec 2B, cosec 2C are in Arithmetical
√ Progression. Show that the cosine of
3
the common difference of the angles is .
8
Solution. Let A, B, C denote the angles; then A + B + C = 180◦ ; and since the angles
are in Arithmetical Progression A + C = 2B; thus 3B = 180◦ ; therefore B = 60◦ .
1 1 2 4
Again we have + = = √ . Let x denote the common difference
sin 2A sin 2C sin 2B 3
of the angles; so that A = 60◦ − x, and C = 60◦ + x. Then
sin 2A + sin 2C 4 2 sin(A + C) cos(A − C) 4
= √ ; therefore = √ ;
sin 2A sin 2C 3 sin(120◦ − 2x) sin(120◦ + 2x) 3

3 cos 2x 4
therefore = √ ; therefore
sin2 120◦ − sin2 2x 3 ( )
4 ◦ 4 3 1 4
cos 2x = (sin 120 − sin 2x) =
2 2
− 1 + cos2 2x = − + cos2 2x.
3 3 4 3 3
CHAPTER XXV : Miscellaneous Examples 247

1
By solving this quadratic we obtain cos 2x = 1, or − . The latter must be taken :
( ) 4
1 1 3
then cos2 x = 1− = .
2 4 8

3A
cos 2A sin
Problem 77. Show that cos A + cos 2A + cos 3A = 2 .
A
sin
2
Solution.
cos A + cos 2A + cos 3A = 2 cos 2A cos A + cos 2A
( )
1
= cos 2A(2 cos A + 1) = cos 2A 2 − 4 sin2 A + 1
( ) ( 2 )
1 cos 2A 1 1
= cos 2A 3 − 4 sin2 A = 3 sin A − 4 sin3 A
2 1 2 2
sin A
2
cos 2A 3A
= sin .
1 2
sin A
2

Problem 78.( Show that )( )( )


2π 4π 6π
x − 2 cos x − 2 cos x − 2 cos = x 3 + x 2 − 2x − 1 .
7 7 7
Solution. Multiply the given expression
( out. The coefficient ) of x2 is
2π 4π 6π
−2 cos + cos + cos ;
7 7 7
4π 3π
2 cos sin ( )
by P roblem 77 this = − 7 7 =− 1 sin

− sin
π
= 1.
π π 7 7
sin sin
7 7
The coefficient of(x is )
2π 4π 2π 6π 4π 6π
4 cos cos + cos cos + cos cos ;
7 7 7 7 7 7
( )
2π 6π 4π 8π 2π 10π
this = 2 cos + cos + cos + cos + cos + cos
( 7 7 7 ) (7 7 7 )
2π 6π 4π 8π 2π 10π
= 2 cos + cos + cos + 2 cos + cos + cos
( 7 7 7 ) ( 7 7 7 )
2π 6π 4π 8π 16π 24π
= 2 cos + cos + cos + 2 cos + cos + cos .
7 7 7 7 7 7
The former expression = −1, as we have already shown. And by P roblem 77 the
latter expression
16π 12π
2 cos sin ( )
7 7 = 1 4π
= sin 4π − sin = −1.
4π 4π 7
sin sin
7 7
Hence the entire coefficient is −2.
2π 4π 6π
The term independent of x is −8 cos cos cos ; this
( 7 ) 7 7
6π 2π 6π
= −4 cos cos + cos
( 7 7 7 )
8π 4π 12π
= −2 cos + cos + cos +1
7 7 7
8π 6π
2 cos sin ( )
=− 7 7 −2=− 1 sin 2π − sin

−2
2π 2π 7
sin sin
7 7
= 1 − 2 = −1.
CHAPTER XXV : Miscellaneous Examples 248

Problem 79. If A + B + C = 180 ◦ , show that


A B C 3A 3B 3C
sin3 A + sin3 B + sin3 C = 3 cos cos cos + cos cos cos .
2 2 2 2 2 2
Solution. sin3 A + sin3 B + sin3 C
1
= (3 sin A + 3 sin B + 3 sin C − sin 3A − sin 3B − sin 3C).
4
Then by P roblem 32 of Chap. viii we have
A B C
sin A + sin B + sin C = 4 cos cos cos ,
2 2 2
3A 3B 3C
and sin 3A + sin 3B + sin 3C = −4 cos cos cos .
2 2 2

Problem 80. Investigate the conditions which must hold in order that the equation
sin2 x + 2b sin x + c = 0 may give two admissible values for sin x, when b is positive.
Solution. By solving the quadratic equation we obtain

sin x = −b ± (b2 − c).

Hence b2 −c must not be negative, and b+ (b2 − c) must not be greater than unity,
in order that both values may be admissible.

Problem 81. The number of the sides of one regular polygon is to the number of
the sides of another as m is to n; and the number of degrees in an angle of the first is
to the number of grades in an angle of the second as p is to q. Determine the number
of sides in each polygon.
Solution. Let mx denote the number of sides in the first regular polygon, and nx
the number of sides in the second. Then, proceeding as in P roblem 61, we find that
2mx − 4
the number of degrees in an angle of the first polygon is 90, and the number
nx
2nd − 4 2mx − 4
of grades in an angle of the second polygon is 100. Therefore 90 :
nx mx
2nx − 4
100 :: p : q.
nx
mx − 2 nx − 2 2(9qn − 10pm)
Therefore 9q = 10p ; therefore x = .
m n mn(9q − 10p)
Hence mx and nx are known.

Problem 82. Solve the equation cos x + cos 7x = cos 4x.


Solution. Here 2 cos 3x cos 4x = cos 4x. Therefore either cos 4x = 0 or 2 cos 3x = 1.
π
If cos 4x = 0 then 4x = (2n + 1) .
2
1 π
If cos 3x = then 3x = 2nπ ± .
2 3

Problem 83. Eliminate α from the equations


x tan(α − β) = y tan(α + β), (x − y) cos 2 α + (x + y) cos 2 β = z.
Solution. From the first equation we have
x sin(α − β) cos(α + β) = y sin(α + β) cos(α − β),
therefore x(sin 2α − sin 2β) = y(sin 2α + sin 2β),
therefore (x − y) sin 2α = (x + y) sin 2β.
(x + y) sin 2β
Thus sin 2α = ,
x−y
z − (x + y) cos 2β
and cos 2α = .
x−y
Square and add; thus
(x + y)2 sin2 2β {z − (x + y) cos 2β}2
1= + .
(x − y) 2 (x − y)2
Therefore (x − y)2 =(x + y)2 sin2 2β + {z − (x + y) cos 2β}2
=(x + y)2 + z 2 − 2z(x + y) cos 2β;
CHAPTER XXV : Miscellaneous Examples 249

therefore z 2 + 4xy = 2z(x + y) cos 2β.

Problem 84. If x and y vary so that their sum is constant, find between what limits
sin x sin y ranges, and its greatest value.
A A
Solution. Let A denote the sum of x and y. Suppose x = + z, then y = − z; and
( ) ( ) 2 2
A A A
sin x sin y = sin − z sin + z = sin2 − sin2 z. Now sin2 z ranges between the
2 2 2
A A
values 0 and 1; hence sin x sin y ranges between the values sin2 and − cos2 : the
2 2
former is always the greatest value algebraically.

Problem 85. (If A + B)+ C =(180 ◦ , show


) that ( )
B C A
sin A + + sin B + + sin C + +1
2 2 2
A−B B−C C−A
= 4 cos cos cos .
( ) ( 4 )
4 4
B A−C A+B+C A−C
Solution. We have sin A + = sin + = cos ;
( )2 2 ( 2) 2
C B−A A C−B
similarly sin B + = cos , sin C + = cos .
2 2 2 2
A−C B−A B−C 2A − B − C
Then cos + cos = 2 cos cos ,
2 2 4 4
C−B B − C
and 1 + cos = 2 cos2 ;
2 4
A−C B−A C−B
therefore cos + cos + cos +1
2 { 2 2 }
B−C B−C 2A − B − C
= 2 cos cos + cos
4 4 4
B−C A−C A−B
= 4 cos cos cos
4 4 4
A−B B−C C−A
= 4 cos cos cos .
4 4 4

Problem 86. If cos2 A + cos2 B + cos2 C = 1 , cos2 α + cos2 β + cos2 γ = 1 ,

and cos A cos α + cos B cos β + cos C cos γ = 0 ;


Show that
sin α sin 2 α sin β sin 2 β sin γ sin 2 γ 2 cos α cos β cos γ
+ + + = 0.
cos A cos B cos C cos A cos B cos C
Solution. Bring the proposed expression to a common denominator; then the numer-
ator
= 2 cos α(1 − cos2 α) cos B cos C + 2 cos β(1 − cos2 β) cos C cos A
+ 2 cos γ(1 − cos2 γ) cos A cos B + 2 cos α cos β cos γ
= 2 cos α(cos2 β + cos2 γ) cos B cos C + 2 cos β(cos2 γ + cos2 α) cos C cos A
+ 2 cos γ(cos2 α + cos2 β) cos A cos B + 2 cos α cos β cos γ
= 2 cos α cos β(cos α cos A + cos B cos β) cos C
+ 2 cos β cos γ(cos β cos B + cos γ cos C) cos A
+ 2 cos γ cos α(cos γ cos C + cos α cos A) cos B + 2 cos α cos β cos γ
= −2 cos α cos β cos γ cos2 C − 2 cos α cos β cos γ cos2 A
− 2 cos α cos β cos γ cos2 B + 2 cos α cos β cos γ
= 2 cos α cos β cos γ(1 − cos2 C − cos2 A − cos2 B) = 0.

π 1 √ √ √ √
Problem 87. Show that sin = (1 + 2 − 3 ) 2 − 2 .
24 4
CHAPTER XXV : Miscellaneous Examples 250

{ √ }
1◦ 1 1 1 3+1
Solution. sin2 7 = (1 − cos 15◦ ) = {1 − cos(45◦ − 30◦ )} = 1− √
2 2 2 2 2 2
√ √ √ √
2 2− 3−1 8−2 6−2 2
= √ = .
4 2 16
Now it will
√ be found
√ that√ √ √ √ √ √ √
8 − 2 6 − 2 2 = (2 − 2)(6 + 2 2 − 2 3 − 2 6) = (2 − 2)(1 + 2 − 3)2 ;
√ √ √ √
1◦ 1+ 2− 3
therefore sin 7 = 2 − 2.
2 4

1 + 2c 2
Problem 88. If tan ϕ = tan θ,
1 −(c 2 ) ( )
π ϕ 1 +c π θ
and tan + = tan + ,
4 2 1 −c 4 2
2
then either sin θ = or else cos θ = 0 .
c
ϕ θ
1 + tan 1 + c 1 + tan 2
Solution. The second equation gives 2 =
ϕ 1−c θ
1 − tan 1 − tan
2 2
θ
ϕ c + tan
therefore tan = 2 .
2 θ
1 + c tan
2
ϕ θ
2 tan 1 + 2c2 2 tan 2
The first equation gives 2 = ;
ϕ 1 − c2 θ
1 − tan2 1 − tan2
( )( 2 ) 2
θ θ θ
c + tan 1 + c tan tan
2 2 1 + 2c2 2
therefore ( ) ( ) = 1 − c2 .
θ 2 θ 2 θ
1 + c tan − c + tan 1 − tan2
2 2 2
2 θ 2 θ 2 θ
c + (1 + c ) tan + c tan (1 + 2c ) tan
therefore ( 2 ) 2 = ( 2 );
θ θ
(1 − c2 ) 1 − tan2 (1 − c2 ) 1 − tan2
2 2
θ θ θ
therefore either 1 − tan2 = 0, or c tan2 − c2 tan + c = 0.
2 2 2
θ θ
The former gives cos θ = 0; the latter gives 1 − c sin cos = 0, therefore 2 − c sin θ =
2 2
0.

Problem 89. Find cos x from the equation cos 2x + b cos x + c = 0 . Investigate the
conditions which must hold in order that there may be at least one admissible value
of cos x, supposing b positive.
Solution. Put 2 cos2 x − 1 for cos 2x; then 2 cos2 x + b cos x + c − 1 = 0. By solving this
quadratic equation we obtain

−b ± b2 − 8(c − 1)
cos x = .
4 √
Hence b2 + 8 − 8c must not be negative, and (b2 + 8 − 8c) − b must not be numer-
ically greater than 4.

π
Problem 90. If γ is not greater than show that sin θ {1 + sin(γ − θ)} continually
4
increases as θ increases from 0 to γ.
Solution. Suppose θ2 greater than θ1 , and each between 0 and γ. Now
1
sin θ{1 + sin(γ − θ)} = sin θ + sin γ sin 2θ − cos γ sin2 θ.
2
CHAPTER XXV : Miscellaneous Examples 251

Put θ1 and θ2 in succession for θ, and subtract the second value of the expression
from the first. Thus we get
{ }
1 sin 2θ2 − sin 2θ1
(sin θ2 − sin θ1 ) 1 + sin γ − (sin θ2 + sin θ1 ) cos γ .
2 sin θ2 − sin θ1
Now (sin θ2 +sin θ1 ) cos γ is less than 2 sin γ cos γ, that is less than sin 2γ, and therefore
less than 1. Hence the preceding expression is necessarily positive, and this is what
was to be proved.

Problem 91. Show that there are eleven, and only eleven, pairs of regular polygons
which are such that the number of degrees in an angle of one of them is equal to the
number of grades in an angle of the other; and that there are only four pairs when
these angles are expressed as integers.
Solution. Let x be the number of sides in one regular polygon, and y the number of
sides in another. Then, as in P roblem 61, the number of degrees in an angle of the first
2x − 4
polygon is 90, and the number of grades in an angle of the second polygon is
x
2y − 4 2x − 4 2y − 4
100. Hence we must have 90 = 100; therefore 9y(x − 2) = 10x(y − 2);
y x y
therefore
x(20 − y) = 18y.
We must then try in succession values of y from 3 to 19 inclusive, and ascertain in
how many cases we obtain an integral value of x. The admissible values will be found
to be these :
y 5 8 10 11 12 14 15 16 17 18 19
x 6 12 18 22 27 42 54 72 102 162 342.
The cases in which the angles are expressed by integers are when
y = 5, 8, 10 or 16.

Problem 92. If sec α sec β + tan α tan β = tan γ, show that cos 2 γ cannot be positive.

1 + sin α sin β
Solution. We have tan γ = ,
cos α cos β
1 − tan γ
2 cos α cos β − (1 + sin α sin β)2
2 2
and cos 2γ = 2
= .
1 + tan γ cos2 α cos2 β + (1 + sin α sin β)2
The numerator
= (cos α cos β + 1 + sin α sin β)(cos α cos β − 1 − sin α sin β)
= −{1 + cos(α − β)}{1 − cos(α + β)};
and this cannot be positive, for 1 + cos(α − β) and 1 − cos(α + β) cannot be negative.

Problem 93. Find the general value of an angle such that its cosine is to its tangent
as 3 is to 2 .
cos θ 3 3
Solution. Denote the angle by θ; then = ; therefore cos2 θ = sin θ, therefore
tan θ 2 2
3 1
1 − sin2 θ = sin θ. By solving this quadratic equation we get sin θ = , or −2; the
2 2
former is the only admissible value, and hence
π
θ = nπ + (−1)n .
6

Problem 94. If x and y vary so that their sum is constant, find between what limits
sin x + sin y ranges, and its greatest value.
Solution. Let A denote the sum of x and y. Then
x+y x−y A x−y
sin x + sin y = 2 sin cos = 2 sin cos ;
2 2 2 2
x−y
and as cos ranges between −1 and +1 the value of sin x + sin y ranges between
2
A A
−2 sin and 2 sin : and the positive value out of these two is algebraically and
2 2
CHAPTER XXV : Miscellaneous Examples 252

arithmetically the greatest value of sin x + sin y.


Problem 95. Solve the equation cos θ − sin θ =( 2 . )
cos θ sin θ π
Solution. Here √ − √ = 1; therefore cos θ + = 1; therefore
2 2 4
π
θ + = 2nπ.
4

Problem 96. Express in four factors


sin2 A + sin2 B + sin2 C − 2 sin A sin B sin C − 1 .

Solution. sin2 A + sin2 B + sin2 C − 2 sin A sin B sin C − 1


= (sin A − sin B sin C)2 + sin2 B + sin2 C − 1 − sin2 B sin2 C
= (sin A − sin B sin C)2 − (1 − sin2 B)(1 − sin2 C).
= (sin A − sin B sin C)2 − cos2 B cos2 C
= (sin A − sin B sin C − cos B cos C)(sin A − sin B sin C + cos B cos C)
= {sin A − cos(B − C)}{sin A + cos(B + C)}
{ ( ) }{ ( ) }
π π
= cos − A − cos(B − C) cos − A + cos(B + C)
2 ( )
2 ( )
B−C−A π B−C+A π
= the product of 4 sin + sin −
( 2 ) (4 2 ) 4
B+C−A π B+C+A π
into cos + cos − .
( 2 )4 ( 2 )4
B−C−A π B−C+A π
Instead of sin + sin − we may put
( 2 4 ) ( 2 4 )
A+C−B π A+B−C π
− cos + cos + .
2 4 2 4
Thus the expression becomes ( the product )of ( )
A+B+C π B+C−A π
−4 cos − cos +
( 2 4 ) ( 2 4 )
A+C−B π A+B−C π
into cos + cos + .
2 4 2 4

π 1 √ √ √ √
Problem 97. Show that cos = (−1 + 2 + 3 ) 2 + 2 .
24 4 { √ }
1◦ 1 ◦ 1 ◦ ◦ 1 3+1
2
Solution. cos 7 = (1 + cos 15 ) = {1 + cos(45 − 30 )} = 1+ √
2 2 2 2 2 2
√ √ √ √
2 2+ 3+1 8+2 6+2 2
= √ = .
4 2 16
Now it √
will be√found that
√ √ √ √ √ √ √
8 + 2 6 + 2 2 = (2 + 2)(6 − 2 2 − 2 3 + 2 6) = (2 + 2)(−1 + 2 + 3)2 ;
√ √ √
1◦ −1 + 2 + 3 √
therefore cos 7 = 2 + 2.
2 4

Problem 98. Eliminate(θ between


) ( )
π π c
a sin θ + + b sin θ − = √ ,
4 4
( ) ( ) 2 ( )
π π π
and a cos θ − + b cos θ + = c sin 2θ + .
4 4 4
Solution.
By addition 2a(sin θ + cos θ) = c(1 + sin 2θ + cos 2θ)
= 2c cos θ(sin θ + cos θ);
therefore a = c cos θ (31)
CHAPTER XXV : Miscellaneous Examples 253

Again, by subtraction, 2b(sin θ − cos θ) = c(1 − sin 2θ − cos 2θ)


= 2c sin θ(sin θ − cos θ);
therefore b = c sin θ (32)
Square and add (31) and (32); thus a2 + b2 = c2 . This assumes that tan θ is neither
equal to 1 nor to −1.

Problem 99. If A, B, C be any quantities, and α, β, γ angles such that


A cot α + B cot β + C cot γ = (A + B + C) cot α cot β cot γ,
and
(B + C) cot β cot γ + (C + A) cot γ cot α + (A + B) cot α cot β = 0;
Show that A sin 2α + B sin 2β + C sin 2γ = 0.
Solution. We have
A cot α(1 − cot β cot γ) + B cot β(1 − cot α cot γ) + C cot γ(1 − cot β cot α) = 0,
and
A cot α(cot β + cot γ) + B cot β(cot γ + cot α) + C cot γ(cot α + cot β) = 0.
These may be written
A cos α cos(β + γ) + B cos β cos(γ + α) + C cos γ cos(α + β) = 0,
A cos α sin(β + γ) + B cos β sin(γ + α) + C cos γ sin(α + β) = 0,
Hence by Algebra, Art. 385 (page 363), we have
A = k cos β cos γ{cos(γ + α) sin(α + β) − cos(α + β) sin(γ + α)},
B = k cos γ cos α{cos(α + β) sin(β + γ) − cos(β + γ) sin(α + β)},
C = k cos α cos β{cos(β + γ) sin(γ + α) − cos(γ + α) sin(β + γ)}.
where k is some constant.
Thus A = k cos β cos γ sin(β − γ),
B = k cos γ cos α sin(γ − α),
C = k cos α cos β sin(α − β).
Therefore A sin 2α + B sin 2β + C sin 2γ
= 2k cos α cos β cos γ{sin α sin(β − γ) + sin β sin(γ − α) + sin γ sin(α − β)}.
The term within the brackets will be seen to vanish, since
1
sin α sin(β − γ) = {cos(γ + α − β) − cos(α + β − γ)},
2
1
sin β sin(γ − α) = {cos(α + β − γ) − cos(β + γ − α)},
2
1
and sin γ sin(α − β) = {cos(β + γ − α) − cos(γ + α − β)}.
2
Or we might proceed thus : let π stand for α + β + γ; then the two given relations
may be written
A cos α cos(σ − α) + B cos β cos(σ − β) + C cos γ cos(σ − γ) = 0,
A cos α sin(σ − α) + B cos β sin(σ − β) + C cos γ sin(σ − γ) = 0;
therefore (A cos2 α + B cos2 β + C cos2 γ) cos σ
= −(A sin α cos α + B sin β cos β + C sin γ cos γ) sin σ (33)
And (A cos2 α + B cos2 β + C cos2 γ) sin σ
= (A sin α cos α + B sin β cos β + C sin γ cos γ) cos σ (34)
Multiply (33) by sin σ and (34) by cos σ and subtract : thus
A sin α cos α + B sin β cos β + C sin γ cos γ = 0,
which is the required result.
Again, multiply (33) by cos σ and (34) by sin σ and add; then we obtain the additional
result A cos2 α + B cos2 β + C cos2 γ = 0.

Problem 100. √
If cos θ = cos α cos β − sin α sin β 1 − c2 sin2 θ,

and cos ϕ = cos α cos β + sin α sin β 1 − c2 sin2 ϕ;
CHAPTER XXV : Miscellaneous Examples 254

Show that
2 cos α cos β
cos θ + cos ϕ = ,
1 − c2 sin2 α sin2 β
and
cos2 α + cos2 β
1 + cos θ cos ϕ = .
1 − c2 sin2 α sin2 β
θ ϕ
Hence find sin θ sin ϕ and tan tan .
2 2
Solution. From the first equation
(cos θ − cos α cos β)2 = sin2 α sin2 β(1 − c2 sin2 θ);
substitute 1 − cos2 θ for sin2 θ; thus
cos2 θ(1 − c2 sin2 α sin2 β) − 2 cos θ cos α cos β
+ cos2 α cos2 β − (1 − c2 ) sin2 α sin2 β = 0.
The second equation leads to the same quadratic for finding cos ϕ. Hence we infer
that cos θ is one root of the quadratic and cos ϕ the other. Hence by the theory of
quadratic equations, Algebra, Chapter xxii,
2 cos α cos β
cos θ + cos ϕ = ,
1 − c2 sin2 α sin2 β
cos α cos β − (1 − c2 ) sin2 α sin2 β
2 2
cos θ cos ϕ = ;
1 − c2 sin2 α sin2 β
1 − sin α sin β + cos α cos β
2 2 2 2
therefore 1 + cos θ cos ϕ =
1 − c2 sin2 α sin2 β
cos α + cos2 β
2
= .
1 − c2 sin2 α sin2 β
Then sin θ sin ϕ = (1 − cos θ)(1 − cos2 ϕ)
2 2 2

= (1 + cos θ cos ϕ)2 − (cos θ + cos ϕ)2


(cos2 α − cos2 β)2
= ;
(1 − c2 sin2 α sin2 β)2
cos2 α − cos2 β
therefore sin θ sin ϕ = ± .
1 − c2 sin2 α sin2 β
θ ϕ 1 − cos θ 1 − cos ϕ
And tan tan = ·
2 2 sin θ sin ϕ
1 − (cos θ + cos ϕ) + cos θ cos ϕ
=
sin θ sin ϕ
(cos α − cos β)2 cos α − cos β
= =± .
±(cos2 α − cos2 β) cos α + cos β

Problem 101. Show that ( ) ( )


π π
cos 11A + 3 cos 9A + 3 cos 7A + cos 5A = 16 cos3 A cos 4A + cos 4A − .
4 4
Solution.
cos 11A + cos 5A = 2 cos 8A cos 3A,
3 cos 9A + 3 cos 7A = 6 cos 8A cos A;
hence by addition we find that the proposed expression
= 2 cos 8A(cos 3A + 3 cos A) = 8 cos 8A cos3 A
( )
1
= 8 cos3 A(2 cos2 4A − 1) = 16 cos3 A cos2 4A −
( ) (2 ) ( )
2 π π π
= 16 cos A cos 4A − sin
3 2 3
= 16 cos A cos 4A + cos 4A − ;
4 4 4
see Art. 83 (page 324).

Problem 102. Find approximately the distance at which a coin an inch in diameter
must be held from the eye looking towards the moon so as just to conceal it, the
apparent angular diameter of the moon being half a degree.
CHAPTER XXV : Miscellaneous Examples 255

1
2
Solution. Let the distance be denoted by x inches; then we must have = the
x
tangent of a quarter of a degree. As the tangent of a small angle is approximately
1 1 π
equal to its circular measure we have approximately = · ; therefore x =
2x 4 180
2 × 180
= 114.6 nearly.
π

3
Problem 103. Solve the equation cos3 θ sin 3 θ + sin3 θ cos 3 θ = .
4
Solution. Let P roblem 27 of Chapter vi. this becomes sin 4θ = 1; therefore
π
4θ = (4n + 1) .
2

Problem 104. Eliminate θ and ϕ from


c sin θ = a sin(θ + ϕ), a sin ϕ = b sin θ,
cos θ − cos ϕ = 2m.
Solution. Here c sin θ = a sin θ cos ϕ + a cos θ sin ϕ;
substitute for sin ϕ and cos ϕ; thus
b
c sin θ = a(cos θ − 2m) sin θ + a cos θ × sin θ;
a
therefore c = a(cos θ − 2m) + b cos θ;
c + 2am
therefore cos θ = .
a+b
c + 2am c − 2bm
Therefore cos ϕ = − 2m = .
a+b a+b
But a2 sin2 ϕ = b2 sin2 θ; therefore
a2 (c − 2bm)2 − b2 (c + 2am)2
a2 − b2 = a2 cos2 ϕ − b2 cos2 θ =
(a + b)2
(a − b)c2 − 4abcm
= .
a+b

Problem 105. In any triangle


a sin(B − C ) + b sin(C − A) + c sin(A − B) = 0 .
Solution. We have, by Art. 252 (page 344)
a = 2R sin A, b = 2R sin B, c = 2R sin C;
hence the proposed expression
= 2R{sin A sin(B − C) + sin B sin(C − A) + sin C sin(A − B)}
= 2R{sin(B + C) sin(B − C) + sin(C + A) sin(C − A) + sin(A + B) sin(A − B)}
= 2R{sin2 B − sin2 C + sin2 C − sin2 A + sin2 A − sin2 B}
= 0.

Problem 106. Show that


1
cos2 18 ◦ sin2 36 ◦ − cos 36 ◦ sin 18 ◦ =
.
16
Solution. By Art. 108 (page 328) the proposed expression
√ √ √ √
10 + 2 5 10 − 2 5 5+1 5−1 5 1 1
= · − · = − = .
16 16 4 4 16 4 16

Problem 107. The perpendiculars from the angles of a triangle on the opposite
sides meet at O; and OA = x, OB = y, OC = z; show that
a b c abc
+ + = .
x y z xyz
Solution. From the triangle OAB we have
OA sin OBA cos A
= = ;
AB sin AOB sin C
CHAPTER XXV : Miscellaneous Examples 256

c cos A a cos A
therefore x= = .
sin C sin A
b cos B c cos C
Similarly y= , and z = .
sin B sin C
Hence we have only to show that
tan A + tan B + tan C = tan A tan B tan C;
and this is known to be true by Art. 114 (page 328).

Problem 108. The top of a pole placed against a wall at an angle A with the hori-
zontal plane just touches the coping; and when its foot is moved a yards further from
the wall and its angle of inclination is B, it rests on the sill of a window. Show that
the perpendicular distance between the coping and the sill is
A+B
a cot .
2
Solution. Let l denote the length of the pole. The distance of the coping from the
ground is l sin A, and the distance of the sill from the ground is l sin B; hence the
distance from the coping to the sill = l(sin A − sin B).
The distance of the foot of the ladder from the wall is l cos A at first, and l cos B
afterwards; therefore a = l(cos B − cos A).
Substitute for l in the former expression, and we obtain
1
a(sin A − sin B) a cos (A + B) 1
, that is 2 , that is a cot (A + B).
cos B − cos A 1 2
sin (A + B)
2

Problem 109. AB is the diameter of a circle, C its centre; a straight line AP is


drawn dividing the semicircle into two equal parts; θ is the circular measure of the
complement of the angle PCB. Show that cos θ = θ.
( )
r2 π
Solution. Let r denote the radius. Then the area of the sector P CB = −θ ,
2 2
by Art. 258 (page 347);
2 ( )
r π
and the area of the triangle ACP = sin + θ , by Art. 247 (page 342).
2 2
The sum of these two areas by supposition is equal to half the area of the semicir-
cle; thus
( )
r2 π r2 πr 2
−θ + cos θ = ;
2 2 2 4
then by simplifying we obtain cos θ = θ.

Problem 110. The sides of a regular pentagon and of a regular decagon inscribed
in a circle of radius R are a and a ′ , and the radii of the circles inscribed in them are
r and r ′ respectively. Show that
a a′ 2R
a 2 − a ′2 = R2 , and + ′ = ′ .
r r r
Solution. By Art. 255 (page 347) ◦ ′ ◦
{ we√have a√= 2R sin}36 , and a = 2R sin 18 ;
10 − 2 5 ( 5 − 1) 2 4R × 4
2
therefore a2 − a′2 = 4R2 − = = R2 .
16 16 16
a a′
Also = 2 tan 36◦ , and ′ = 2 tan 18◦ ;
r ( r )
a a′ sin 36◦ sin 18◦ 2 sin(36◦ + 18◦ )
therefore + ′ =2 ◦
+ ◦
= .
r r cos 36 cos 18 cos 36◦ cos 18◦
2 sin 54 ◦ 2 2R
= = = ′ .
cos 36◦ cos 18◦ cos 18◦ r

Problem 111. If A + B + C = (2n + 1 )π, show that


( )
A B C A B B C C A
cos4 + cos4 + cos4 − 2 cos2 cos2 + cos2 cos2 + cos2 cos2
2 2 2 2 2 2 2 2 2
CHAPTER XXV : Miscellaneous Examples 257

A B C
+ 4 cos2 cos2 cos2 = 0.
2 2 2
Solution.
A 1
cos2 = (1 + cos A).
2 2
A 1 1
cos4 = (1 + cos A)2 = (1 + 2 cos A + cos2 A)
2 4{ 4 }
1 1 3 1 1
= 1 + 2 cos A + (1 + cos 2A) = + cos A + cos 2A.
4 2 8 2 8
In this way the proposed expression becomes
9 1 1
+ (cos A + cos B + cos C) + (cos 2A + cos 2B + cos 2C)
8 2 8
1 1 1
− (1 + cos A)(1 + cos B) − (1 + cos B)(1 + cos C) − (1 + cos C)(1 + cos A)
2 2 2
1
+ (1 + cos A)(1 + cos B)(1 + cos C)
2
1 1 1
= + (cos 2A + cos 2B + cos 2C) + cos A cos B cos C
8 8 2
1 1
= − = 0; see Example viii. 18.
8 8

Problem 112. If tan2 θ is less than unity, show that


1 1 1 1
tan2 θ − tan4 θ + tan6 θ . . . . . . = sin2 θ + sin4 θ + sin6 θ + . . . . . .
2 3 2 3
1 1
Solution. We have 1 + tan2 θ = = ; take the logarithms of both sides;
cos2 θ 1 − sin2 θ
thus
( ) 1 ( )
log 1 + tan2 θ = log = − log 1 − sin2 θ ;
1 − sin2 θ
therefore, by Art. 146 (page 333),
1 1 1 1
tan2 θ − tan4 θ + tan6 θ − . . . = sin2 θ + sin4 θ + sin6 θ + . . .
2 3 2 3
The series are convergent, since tan2 θ is supposed to be less than unity.

Problem 113. Solve the equation cos θ − cos 2 θ = sin 3 θ.


Solution.
3θ θ 3θ 3θ
Here 2 sin sin = 2 sin cos ;
2 2 2 2
3θ θ 3θ
therefore either sin = 0, or sin = cos .
2 2 2
3θ 3θ
If sin = 0, then = nπ.
2 2 ( )
θ 3θ π θ 3θ π θ 3θ
If sin = cos , or cos − = cos , then − = 2nπ ± .
2 2 2 2 2 2 2 2

Problem 114. In any triangle


a 2 sin(B − C ) b2 sin(C − A) c 2 sin(A − B)
+ + = 0.
sin A sin B sin C
Solution. We have
a = 2R sin A, b = 2R sin B, c = 2R sin C;
hence the proposed expression
= 4R2 {sin A sin(B − C) + sin B sin(C − A) + sin C sin(A − B)},
and this is zero, as in the solution of P roblem 105.

Problem 115. Solve the equation cos 3 θ + sin 3 θ = cos θ + sin θ.


cos 3θ sin 3θ cos θ sin θ
Solution. We have √ + √ = √ + √ ;
2 2( 2) 2( )
π π
therefore cos 3θ − = cos θ − ;
4 4
CHAPTER XXV : Miscellaneous Examples 258

( )
π π
therefore 3θ − = 2nπ ± θ − .
4 4


Problem 116. If tan2 x = tan(α + x) tan(α − x) then sin 2x = 2 · sin α.
sin(α + x) sin(α − x) sin α − sin2 x
2
Solution. We have tan2 x = = ,
cos(α + x) cos(α − x) cos2 x − sin2 α
therefore sin x(cos x − sin α) = cos x(sin α − sin x),
2 2 2 2 2 2

therefore 2 sin2 x cos2 x = sin2 α(sin2 x + cos2 x) = sin2 α,


therefore 4 sin2 x cos2 x = 2 sin2 α,

therefore 2 sin x cos x = 2. sin α,

therefore sin 2x = 2. sin α,

Problem 117. If
tan2 A tan A′ = tan2 B tan B ′ = tan2 C tan C ′ = tan A tan B tan C ,
and cosec 2A + cosec 2B + cosec 2C = 0 ,
( ) ( ) ( )
show that tan A − A′ = tan B − B ′ = tan C − C ′ .
Solution. Put x for tan A, y for tan B, z for tan C, x′ for tan A′ , y ′ for tan B ′ , and z ′ for
tan C ′ , for the sake of abbreviation. Then we have given that
x2 x′ = y 2 y ′ = z 2 z ′ = xyz (35)
1 + x2 1 + y2 1 + z2
and + + =0 (36)
2x 2y 2z
yz
( ) x − x′ x−
x , by (35), = x − yz .
2
Now tan A − A′ = ′
=
1 + xx 1 + yz x(1 + yz)
But from (36) we have
1 1 1
x + + y + + z + = 0,
x y z
therefore xyz(x + y + z) + xy + yz + zx = 0,
∴ x2 − yz = x2 + xyz(x + y + z) + xy + zx
= x(x + y + z) + xyz(x + y + z) = x(1 + yz)(x + y + z);
x2 − yz
therefore =x+y+z
x(1 + yz)
( )
Thus tan A − A′ = tan A + tan B + tan C.
Similarly tan(B − B ′ ) and tan(C − C ′ ) may be shown to be equal to the same ex-
pression.

Problem 118. If
cos 60◦ = sin 36◦ cos A, cos 36◦ = sin 60◦ cos B,
and cos C = cos A cos B,
then one value of A + B + C is 90 ◦ .
Solution.
cos 60◦
Here cos A = ,
√ sin 36◦
sin2 36◦ − cos2 60◦
therefore sin A = ,
sin 36◦ (√ )2
√ √
10 − 2 5 1 6−2 5 5−1
and sin2 36◦ − cos2 60◦ = − = = ;
16 4 16 4

5−1
therefore sin A = ◦
.
√ 4 sin 36√
5−1 5−1
Hence tan A = = .
4 cos 60◦ 2
cos 36◦
Again, cos B = ;
sin 60◦
CHAPTER XXV : Miscellaneous Examples 259

√ √ √
sin2 60◦ − cos2 36◦ sin2 36◦ − cos2 60◦ 5−1
∴ sin B = = = .
sin 60◦ sin 60◦ 4 sin 60◦
Hence √ √ √ √ √
5−1 5−1 ( 5 − 1)2 6−2 5 3− 5
tan B = = √ = √ √ = = .
4 cos 36◦ 5+1 ( 5 + 1)( 5 − 1) 4 2
√ √
5−1 3− 5
Therefore tan A + tan B = + = 1;
2 2
sin A sin B
therefore + = 1, therefore sin(A + B) = cos A cos B = cos C; therefore A +
cos A cos B
B = 90◦ − C is one solution.

Problem 119. Two rows of houses of equal height stand at an angle to each other.
On a sunshiny day the distance of the corner of the shadow from the corner where
the rows meet is observed twice and found to be a and b respectively. Show that if h
be the height of the houses and α the difference between the altitudes of the sun on
the two occasions
h 2 + h(b − a) cot α + ab = 0 .
Solution. Let θ be the sun’s altitude at the first observation, and θ + α that at the
second observation; then
h = a tan θ, and h = b tan(θ + α).
hb
b(tan θ + tan α) + b tan α
Thus h= = a ,
1 − tan θ tan α h
1 − tan α
( ) a
h hb
therefore h 1 − tan α = + b tan α,
a a
therefore h tan α + h(b − a) + ab tan α = 0,
2

therefore h2 tan α + h(b − a) cot α + ab = 0.

Problem 120. P is any point in the arc of a semicircle APB; two circles are described
touching the semicircle and also touching AP, BP at their middle points. Show that
the rectangle contained by the radii of these circles is one eighth of the square de-
scribed on the radius of the circle which is inscribed in the triangle APB.
Solution.
Let AP = b, BP = a, AB = c.
The diameter of the circle which touches the semicircle and also touches AP at its
c c c ca c−a
middle point is − sin P AB, that is − , that is ; therefore the radius of
2 2 2 2c 2
c−a
this circle is .
4
Similarly the radius of the circle which touches the semicircle and also touches
c−b
BP at its middle point is . We have then to show that
4
(c − a)(c − b) r2 (c − a)(c − b)
= , that is = r2 .
16 8 2
S ab ab(a + b − c) a+b−c
But r = = = = , since c2 = a2 + b2 ; therefore
s a+b+c (a + b)2 − c2 2
(a + b − c)2 2c2 + 2ab − 2c(a + b) (c − a)(c − b)
r2 = = = .
4 4 2

Problem 121. Show that


sin α sin(β − γ) cos(β + γ − α) + sin β sin(γ − α) cos(γ + α − β)
+ sin γ sin(α − β) cos(α + β − γ) = 0.
Solution.
sin α sin(β − γ) cos(β + γ − α)
1
= {cos(α − β + γ) − cos(α + β − γ)} cos(β + γ − α)
2
CHAPTER XXV : Miscellaneous Examples 260

1
= {cos 2γ + cos 2(α − β) − cos 2β − cos 2(α − γ)}.
4
The other two terms may be transformed in a similar manner, and then it will be
obvious that the sum is zero.

Problem 122. Show that


1 1
log cot θ = cos 2 θ + (cos 2 θ)3 + (cos 2 θ)5 + . . . . . .
3 5
cos θ 2 cos2 θ 1 + cos 2θ
Solution. cot θ = = =
sin θ √
2 sin θ cos θ sin 2θ
1 + cos 2θ 1 + cos 2θ
= √ = .
(1 − cos2 2θ) 1 − cos 2θ
Hence, taking logarithms we have
1 1 + cos 2θ 1 1
log cot θ = log = cos 2θ + (cos 2θ)3 + (cos 2θ)5 + . . .
2 1 − cos 2θ 3 5

Problem 123. If A + B + C = (2n + 1 )π, show that


cot A + cot B + cot C − 2 (cot 2A + cot 2B + cot 2C)
( )
A B C
= cot + cot + cot (sec A − 1)(sec B − 1)(sec C − 1).
2 2 2
Solution. We have shown in the solution of Example viii. 15 that
A B C A B C
cot + cot + cot = cot cot cot ;
2 2 2 2 2 2
hence the expression on the right hand-side
2 A 2 B 2 C
A B C 8 sin 2 sin 2 sin 2
= cot cot cot ·
2 2 2 cos A cos B cos C
A A B B C C
8 sin cos sin cos sin cos
= 2 2 2 2 2 2 = tan A tan B tan C.
cos A cos B cos C
2(cos2 A − sin2 A)
Again cot A − 2 cot 2A = cot A − = tan A;
2 cos A sin A
similarly cot B − 2 cot 2B = tan B, and cot C − 2 cot 2C = tan C;
thus the expression on the left-hand side
= tan A + tan B + tan C = tan A tan B tan C, by Art. 114 (page 328).
Thus the two expressions are equivalent.

Problem 124. Show that


sin A sin B sin(A − B) + sin B sin C sin(B − C) + sin C sin A sin(C − A)
1
= {sin(2A − 2B) + sin(2B − 2C) + sin(2C − 2A)} .
4
1
Solution. sin A sin B sin(A − B) = {cos(A − B) − cos(A + B)} sin(A − B)
2
1 1
= sin(2A − 2B) − (sin 2A − sin 2B).
4 4
Transform the second and third terms in the same way; then by addition we obtain
the required result.

Problem 125. In any triangle


1 A 1 B 1 C (a + b + c)2
cos2 + cos2 + cos2 = .
a 2 b 2 c 2 4abc
1 A 1 B 1 C s(s − a) + s(s − b) + s(s − c)
Solution. cos2 + cos2 + cos2 =
a 2 b 2 c 2 abc
3s2 − s(a + b + c) 3s2 − 2s2 s2 (a + b + c)2
= = = = .
abc abc abc 4abc
CHAPTER XXV : Miscellaneous Examples 261

Problem 126. Find x from ( the equation


) ( )
π π √
sec + x + sec − x = 2 2.
4 4
1 1 √
Solution. Here ( )+ ( ) = 2 2,
π π
cos +x cos −x
4 √ 4 √
2 2 √
therefore + = 2 2,
cos x − sin x cos x + sin x
therefore cos x = cos2 x − sin2 x = cos 2x,
therefore 2x = 2nπ ± x.

Problem 127. Show that


sin(θ − α) sin(θ − β) sin(θ − γ)
+ + = 0.
sin(α − β) sin(α − γ) sin(β − α) sin(β − γ) sin(γ − α) sin(γ − β)
Solution. Express the fractions with the common denominator
sin(α − β) sin(β − γ) sin(γ − α) :
then the numerator becomes
−{sin(β − γ) sin(θ − α) + sin(γ − α) sin(θ − β) + sin(α − β) sin(θ − γ)}.
1 1
Now sin(β − γ) sin(θ − α) = cos(θ − α − β + γ) − cos(θ − α + β − γ),
2 2
1 1
sin(γ − α) sin(θ − β) = cos(θ − β + α − γ) − cos(θ − β + γ − α),
2 2
1 1
sin(α − β) sin(θ − γ) = cos(θ − γ + β − α) − cos(θ − γ + α − β);
2 2
thus the sum of the expressions is zero.

Problem 128. From each of two stations on a horizontal plane, at a distance c from
each other, a pillar on a distant hill, in the vertical plane passing through the stations,
is seen under the same visual angle, and the angles of elevation of the top of the pillar
at the stations are α and β. Show that the length of the pillar is equal to
c cos(β + α)
.
sin(β − α)
Solution. Let x denote the length of the pillar, h the height of the foot of the pillar
above the horizontal plane, b the horizontal distance of the pillar from the first station.
Let θ be the angle subtended by the pillar. Then
h h h+x h+x
= tan(α − θ), = tan(β − θ), = tan α, = tan β.
b b+c b b+c
And from the fact that a circle would pass through the two stations and the top
π
and the foot of the pillar we have α + β − θ = . Thus
2
h h+x
= cot β, = tan α; therefore
b b
x cos(α + β)
= tan α − cot β = − .
b cos α sin β
x cos(α + β)
Similarly = tan β − cot α = − .
b+c cos β sin α
c cos α sin β − cos β sin α sin(β − α)
Therefore = = ;
x cos(α + β) cos(β + α)
c cos(β + α)
therefore x= .
sin(β − α)

Problem 129. If in a right-angled triangle twice the distance between the centres
of the inscribed and circumscribed circles is a mean proportional between the hy-
potenuse and the excess of the sum of the sides over the hypotenuse, shew that the
radius of the inscribed circle is equal to one sixth of the hypotenuse.
1
Solution. In every right-angled triangle r = (a+b−c); see the solution of P roblem 120.
2
CHAPTER XXV : Miscellaneous Examples 262

( )
√ √ c c2
In the present case 2 R2 − 2Rr = c(a + b − c); and R = : thus 4 − cr = c(a +
2 4
b−c); therefore c−4r = a+b−c; therefore 4r = 2c−a−b; therefore 2(a+b−c) = 2c−a−b;
4c c
therefore a + b = ; and therefore r = .
3 6

Problem 130. In any triangle show that


1 1 1 r
cos2 A + cos2 B + cos2 C = 2 + .
2 2 2 2R
1 1 1 1
Solution. cos2 A + cos2 B + cos2 C = (3 + cos A + cos B + cos C)
2 ( 2 2 2)
1 A B C
= 4 + 4 sin sin sin , by Art. 114 (page 328).
2 2 2 2
2S 2 2rS r
=2+ =2+ =2+ .
sabc abc 2R


sin(x + A) sin 2A
Problem 131. If = , then tan2 x = tan A tan B.
sin(x + B) sin 2B
Solution.
sin(x + A) sin(x + B)
√ = √ ;
sin 2A sin 2B
sin x cos A + cos x sin A sin x cos B + cos x sin B
therefore √ = √ ,
sin 2A sin 2B
cos A(tan x + tan A) cos B(tan x + tan B)
therefore √ = ,
sin 2A sin 2B
tan x + tan A tan x + tan B
therefore √ = √ ,
√ √ tan A √ tan B √ √
∴ tan x( tan B − tan A) = tan A tan B( tan B − tan A),

therefore tan x = tan A tan B.

Problem 132. If A + B + C = (2n + 1 )π, show that


sin2 2A + sin2 2B + sin2 2C + 2 cos 2A cos 2B cos 2C = 2 .
Solution. sin2 2A + cos 2A cos 2B cos 2C = 1 − cos2 2A + cos 2A cos 2B cos 2C
= 1 + cos 2A{cos 2B cos 2C − cos 2A}
= 1 + cos 2A{cos 2B cos 2C − cos(2B + 2C)}
= 1 + cos 2A sin 2B sin 2C.
Similarly
sin2 2B + cos 2A cos 2B cos 2C = 1 + cos 2B sin 2A sin 2C.
Hence the proposed expression
= 2 + sin 2C{sin 2C + sin 2B cos 2A + sin 2A cos 2B}
= 2 + sin 2C{− sin(2A + 2B) + sin(2A + 2B)}
= 2.

Problem 133. Sum the infinite series


1 + 22 1 + 23
(1 + 2 ) log 2 + (log 2 )2 + (log 2 )3 + . . . .
2 3
Solution. The series may be separated into two, namely
1 1
log 2 + (log 2)2 + (log 2)3 + . . .
2 3
1 1
and 2 log 2 + (2 log 2)2 + (2 log 2)3 + . . .
2 3
and is therefore equal to elog 2 − 1 + e2 log 2 − 1, that is to elog 2 − 1 + elog 4 − 1, that is to
2 − 1 + 4 − 1, that is to 4.
CHAPTER XXV : Miscellaneous Examples 263

Problem 134. Show that


sin(A − B) cos(C − B) cos(A − C) + sin(B − C) cos(A − C) cos(B − A)
+ sin(C − A) cos(B − A) cos(C − B)
1
= {sin(2B − 2A) + sin(2C − 2B) + sin(2A − 2C)} .
4
Solution.
sin(A − B) cos(C − B) cos(A − C)
1
= sin(A − B){cos(A + B − 2C) + cos(A − B)}
2
1 1 1
= sin 2(A − C) + sin 2(C − B) + sin 2(A − B).
4 4 4
Transform the second and third terms in like manner, then by addition we obtain
the required result.

sin(A − B) a 2 − b2
Problem 135. In any triangle = .
sin(A + B) c2
sin(A − B) sin(A + B) sin(A − B) sin A − sin2 B
2
Solution. = 2
= , by Art. 83 (page 324),
sin(A + B) sin (A + B) sin2 C
( )2 ( )2 ( )2 ( )2
sin A sin B a b a2 − b2
= − = − = .
sin C sin C c c c2

Problem 136. The diagonals of a quadrilateral figure are in length h and k respec-
tively, and inclined at an angle A. Show that the area of the greatest rectangle which
can be drawn with its four sides passing through the four corners of this quadrilateral
is
1
hk(1 + sin A).
2
Solution. Suppose the diagonal h of the quadrilateral to make an angle θ with the
sides of the rectangle which pass through its extremities; then each of the other sides
is equal to h sin θ. It will be seen from a diagram that the diagonal k of the quadrilat-

eral will make an angle −(θ +A) with the sides of the rectangle which pass through
2 ( )

its extremities; then each of the other sides is equal to k sin − θ − A , that is to
2
hk
−k cos(θ + A). Hence the area of the rectangle = −hk sin θ cos(θ + A) = {− sin(2θ + A)+
2
3π hk
sin A}. The greatest value of this is when 2θ + A = , and is (1 + sin A).
2 2

Problem 137. A person standing at the door of a house observes that he can just
see the top of a church spire over the intervening wall at an angle α; he then ascends
to the root, whence he is just able to get a view of the entire building, and he observes
that the elevation of the spire top is β. Having given the height of the house, that of
the observer being neglected, determine the heights of the spire and the wall.
Solution. Let h denote the height of the house, x the height of the wall, y the height
of the church. Then x cot α is the distance of the wall from the house, and y cot α is the
h y
distance of the church from the house. By similar triangles = ;
x cot α y cot α − x cot α
therefore h(y − x) = xy.
y−h h h tan α
Also = tan β; therefore y = = .
y cot α 1 − cot α tan β tan α − tan β
hy h tan α
Then x= = .
h+y 2 tan α − tan β

Problem 138. In any triangle show that


1 1 1 S
a cos2 A + b cos2 B + c cos2 C = s + .
2 2 2 R
CHAPTER XXV : Miscellaneous Examples 264

1 1 1
Solution. a cos2 A + b cos2 B + c cos2 C.
2 2 2
1
= (a + b + c + a cos A + b cos B + c cos C)
2
1
= s + R(sin 2A + sin 2B + sin 2C)
2
= s + 2R sin A sin B sin C, by Art. 114 (page 328),
8S 3 4S 2 S
= s + 2R 2 2 2 = s + =s+ .
a b c abc R

Problem 139. A, B, C are three points on a plane inclined to the horizon, C being
the lowest; it is found that CA is inclined to the horizon at an angle α, and CB at an
angle β; and the angle ACB is γ. If θ be the inclination of the plane to the horizon,
show that
sin2 θ sin2 γ = sin2 α + sin2 β − 2 sin α sin β cos γ.
Solution. Through C draw a plane parallel to the horizon; from A draw AP perpen-
dicular to the intersection of this plane with that which contains A, B and C; from B
draw BQ perpendicular to the same intersection. Let ACP = ϕ, and BCQ = ψ; so that
ϕ + ψ + γ = π. Therefore
cos γ = sin ϕ sin ψ − cos ϕ cos ψ.
Now AP = AC sin ϕ; thus the perpendicular from A on the plane drawn through
C parallel to the horizon = AP sin θ = AC sin θ sin ϕ; but this perpendicular also =
AC sin α; therefore
sin α = sin θ sin ϕ.
Similarly sin β = sin θ sin ψ.

sin α sin β (sin2 θ − sin2 α)(sin2 θ − sin2 β)
Hence cos γ = − ,
sin2 θ sin2 θ
therefore (cos γ sin2 θ − sin α sin β)2 = (sin2 θ − sin2 α)(sin2 θ − sin2 β),
therefore cos2 γ sin2 θ − 2 cos γ sin α sin β = sin2 θ − sin2 α − sin2 β,
therefore sin2 θ sin2 γ = sin2 α + sin2 β − 2 sin α sin β cos γ.

Problem 140. If a ′ , b′ , c ′ are the sides of the triangle formed by joining the points
of contact of the inscribed circle with the sides of a triangle, show that
a ′ b′ c ′ r2
= .
abc 2R2
Solution. With the diagram of Art. 248 (page 342) we see that
1
a′ = 2r sin F OA = 2r cos A;
2
and we have similar values for b′ and c′
1 1 1 sS
Thus a′ b′ c′ = 8r3 cos A cos B cos C = 8r3 ;
2 2 2 abc
a′ b′ c′ 8r2 S 2 8r2 r2
therefore = 2 2 2 = = .
abc a b c (4R)2 2R2

Problem 141. Find tan θ from the equation tan 3 θ + tan 2 θ + tan θ = 0 .
3 tan θ − tan3 θ 2 tan θ
Solution. Here + + tan θ = 0.
1 − 3 tan2 θ 1 − tan2 θ
3 − tan θ
2 2
Therefore either tan θ = 0 or + + 1 = 0.
1 − 3 tan2 θ 1 − tan2 θ
The latter gives
(3 − tan2 θ)(1 − tan2 θ) + 2(1 − 3 tan2 θ) + (1 − tan2 θ)(1 − 3 tan2 θ) = 0;
therefore 4 tan4 θ − 14 tan2 θ + 6 = 0.
1
By solving this quadratic we obtain tan2 θ = 3 or .
2
CHAPTER XXV : Miscellaneous Examples 265

Problem 142. ( Show)that ( ) ( ) ( )


π 8 3π 8 5π 8 7π 8 17
cos + cos + cos + cos = .
8 8 8 8 16
Solution. We may obtain the result by taking the values of the four cosines and
raising them to the eighth power. Or we may proceed thus :

π 3π 5π 7π
cos8 + cos8 + cos8 + cos8
8 ( 8 8 ) (
8 )
π 3π π π
= 2 cos8 + cos8 = 2 cos8 + sin8
( 8 8 ) 8 8
1 π
= cos π + 28 cos + 35 , by Example ix. 13,
32 2
34 17
= = .
32 16

1 1 a+b
Problem 143. If a cos ϕ = b cos θ then cot (ϕ + θ) cot (ϕ − θ) = .
2 2 a−b
b cos ϕ a+b cos θ + cos ϕ
Solution. Here = ; therefore =
a cos θ a−b cos θ − cos ϕ
1 1
2 cos (ϕ + θ) cos (ϕ − θ) 1 1
= 2 2 = cot (ϕ + θ) cot (ϕ − θ).
1 1 2 2
2 sin (ϕ + θ) sin (ϕ − θ)
2 2

A B C
Problem 144. If A, B, C be the angles of a triangle, show that cos cos cos
√ 2 2 2
3 3
cannot be greater than .
8 ( ) ( ) ( )
A B C π A π B π C
Solution. cos cos cos = sin − sin − sin − ;
2 2 2 2 2 2 2 2 2
π A π B π C
and the sum of − , − , and − is a fixed quantity, namely π.
2 2 2 2 2 2
Hence proceeding as in Chapter xiii : P roblem 40, we see that the proposed product
is greatest when
π A π B π C
− = − = − ,
2 2 2 2 2 2
A B C π
that is when = = = ;
2( 2 ) 2 6
√ 3 √
3 3 3
and then the product − = .
2 8

Problem 145. In any triangle


A B C a+b+c A
cot + cot + cot = cot .
2√ 2 2 b+c−a 2
B s(s − b) s−b A
Solution. We have cot = = cot ;
2 (s − a)(s − c) s−a 2
C s−c A
similarly cot = cot .
2 s−a 2
( )
A B C s−b s−c A
Hence cot + cot + cot = 1+ + cot
2 2 2 s−a s−a 2
3s − a − b − c A s A
= cot = cot
s−a 2 s−a 2
a+b+c A
= cot .
b+c−a 2

Problem 146. The diagonals of a four-sided figure are h and k, and the area is C .
CHAPTER XXV : Miscellaneous Examples 266

Show that the area of the circumscribing square is


h 2 k 2 − 4C 2
.
h 2 + k 2 − 4C
1
Solution. Let A denote the angle between the diagonals; then C = hk sin A; and by
2
the solution of P roblem 136 the area of the circumscribed rectangle is
−hk sin θ cos(A + θ).
And since the rectangle is to be a square, we have by the solution of P roblem 136
h sin θ = −k cos(A + θ);
therefore h = −k(cos A cot θ − sin A);
k sin A − h
therefore cot θ = ;
k cos A
2
k cos A2 k2 − k2 sin2 A
therefore sin2 θ = = 2
(k sin A − h)2 + k2 cos2 A k − 2kh sin A + h2
4C 2
k2 − 2
= 2 h .
h + k2 − 4C
h k − 4C 2
2 2
And the area of the circumscribing square = h2 sin2 θ = 2 .
h + k2 − 4C

Problem 147. Show that x, y, z can be so taken that for all values of θ the following
expression shall have a given constant value,
x sin(θ − β) sin(θ − γ) + y sin(θ − γ) sin(θ − α) + z sin(θ − α) sin(θ − β).
Solution. We may put the proposed expression in the form
L sin2 θ + M sin θ cos θ + N cos2 θ,
where L, M, N involve the angles α, β, γ and also x, y, z : moreover x, y, z occur only in
the first power. Now if we put M = 0 and L = N = the given constant, the expression
is equal to the given constant whatever θ may be. So we have only to determine x, y
and z from the three simple equations M = 0, and L = N = the given constant.
As soon as we have thus shown that such values of x, y, z as we require must exist,
we can determine the values more simply. For let C denote the given constant; put α
for θ, then
x sin(α − β) sin(α − γ) = C.
This finds x. Similarly, by putting β for θ we find y, and by putting γ for θ we find z.

Problem 148. If from the extremities of a side of a regular pentagon inscribed in


a circle straight lines be drawn to the middle of the arc subtended by the adjacent
side, their difference is equal to the radius of the circle, their product is equal to the
square on the radius, and the sum of their squares is equal to three times the square
on the radius.
Solution. Let AB denote the side of the regular pentagon, P the middle point of the
arc subtended by the side adjacent to AB at B. Then the angle AP B is the angle sub-
tended at the circumference of the circle by the side of a regular pentagon inscribed
π π
in the circle, so that the angle = . Similarly the angle P AB = ; and therefore the
5 10

angle ABP =
10.
Let r denote the radius of the circle, so that
π π 7π 3π
Ab = 2r sin , P B = 2r sin , and P A = 2r sin = 2r sin .
5 {10√ √ } 10 10
5+1 5−1
Hence P A − P B = 2r − = r,
4 4
√ √
4r2 ( 5 + 1)( 5 − 1)
PA · PB = = r2 ,
{( √ 16
)2 ( √ )2 }
2 2 2 5+1 5−1
P A + P B = 4r + = 3r2 .
4 4
CHAPTER XXV : Miscellaneous Examples 267

Problem 149. If a flag-staff at the top of a tower of height a subtend a small angle θ
at the eye of an observer when at the distance b from the tower, show that the length
a 2 + b2
of the flag-staff is θ nearly.
b
Solution. Suppose the tower to subtend an angle ϕ at the eye of the observer; let x
be the length of the flag-staff : then
a a+x tan ϕ + tan θ a + b tan θ
= tan ϕ, = tan(ϕ + θ) = = ;
b b 1 − tan ϕ tan θ b − a tan θ
x a + b tan θ a 2 2
(b + a ) tan θ
therefore = − = ;
b b − a tan θ b b(b − a tan θ)
then if θ be very small we may put θ for tan θ, and neglect a tan θ in comparison with
b2 + a2
b, so that x = θ nearly.
b

Problem 150. In any triangle


(s − a)2 sin A + (s − b)2 sin B + (s − c)2 sin C
A B C
= 4r(2R − r) cos cos cos .
2 2 2
Solution. We have by Art. 249 (page 343)
(s − a)2 sin A + (s − b)2 sin B + (s − c)2 sin C
{ }
A B C
= r (s − a) sin A cot + (s − b) sin B cot + (s − c) sin C cot
{ 2 2 } 2
2 A 2 B 2 C
= 2r (s − a) cos + (s − b) cos + (s − c) cos ;
2 2 2
and by P roblems 130 and 138, this
{( ) ( )} ( )
r S S S
= 2r 2+ s− s+ = 2r s + −
( 2R
) ( R ) 2R R
S S S S(2R − r)
= 2r s − = 2r − = .
2R r 2R R
A B C Ss s
And cos cos cos = = ;
2 2 2 abc 4R
A B C s S(2R − r)
so that 4r(2R − r) cos cos cos = 4r(2R − r) = .
2 2 2 4R R
Thus the proposed expressions are equal.

Problem 151. Show that


2 sin 7A cos A + 16 sin A cos3 A = sin 6A + 4 sin 2A(1 + 2 cos3 2A).

Solution. 2 sin 7A cos A = sin 8A + sin 6A;


Therefore 2 sin 7A cos A + 16 sin A cos3 A = sin 6A + sin 8A + 16 sin A cos3 A
= sin 6A + 2 sin 4A cos 4A + 8 sin 2A cos2 A
= sin 6A + 4 sin 2A cos 2A cos 4A + 8 sin 2A cos2 A
( )
= sin 6A + 4 sin 2A 2 cos2 A + cos 2A cos 4A
= sin 6A + 4 sin 2A {1 + cos 2A(1 + cos 4A)}
( )
= sin 6A + 4 sin 2A 1 + 2 cos3 2A .

√ √
Problem 152. √ Find the logarithm of 32 to the base 3 4, and the logarithm of 81 3 3
3
to the base 9. √ √
Solution. Let x denote the logarithm of 32 to the base 3 4; then 32 = ( 3 1)x , that is
x 2x 2x 15
25 = 4 3 = 2 3 ; therefore 5 = ; therefore x =
3 √ 2 √ √ √
Let x denote the logarithm of 81 3 3 to the base 3 9; then 81 3 3 = ( 3 9)x , that is
1 x 2x 2x 13 13
34+ 3 = 9 3 = 3 3 ; therefore = 4 13 = ; therefore x = .
3 3 2
CHAPTER XXV : Miscellaneous Examples 268

Problem 153. If tan(A + B) = 3 tan A, show that


sin(2A + 2B) + sin 2A = 2 sin 2B.
sin(A + B) 3 sin A
Solution. Here = ;
cos(A + B) cos A
therefore sin(A + B) cos A − cos(A + B) sin A = 2 sin A cos(A + B);
therefore sin(A + B − A) = 2 sin A cos(A + B);
that is sin B = 2 sin A cos(A + B) = sin(2A + B) − sin B;
therefore 2 sin B = sin(2A + B);
therefore 2 sin B cos B = sin(2A + B) cos B;
1
therefore sin 2B = {sin(2A + 2B) + sin 2A}
2
therefore 2 sin 2B = sin(2A + 2B) + sin 2A.

Problem 154. In any triangle


1 1 1 1 1 1 2ab + 2bc + 2ca − a 2 − b2 − c 2
sin2 A + sin2 B + sin2 C = .
a 2 b 2 c 2 4abc
1 A 1 B 1 C
Solution. sin2 + sin2 + sin2
a 2 b 2 c 2
1
= {(s − b)(s − c) + (s − a)(s − c) + (s − a)(s − b)}
abc
1
= {3s2 − 2s(a + b + c) + ab + bc + ca}
abc
1
= {ab + bc + ca − s2 }
abc
1
= {4ab + 4bc + 4ca − (a + b + c)2 }
4abc
1
= {2ab + 2bc + 2ca − a2 − b2 − c2 }
4abc

Problem 155. The sides of a quadrilateral figure are divided in order in the ratio of
m to n, and a new quadrilateral is formed by joining the points of division. Show that
the area of this quadrilateral is to the area of the original quadrilateral as m 2 + n 2 is
to (m + n)2 .
Solution. Let ABCD denote the quadrilateral figure. Let P, Q, R, S be taken in
AB, BC, CD, DA respectively, such that
AP BQ CR DS m
= = = = .
PB QC RD SA n
PB n BQ m
Then = , = ;
AB m+n BC m+n
1 mn
and the area of the triangle P BQ = BP · BQ sin B = AB · BC sin B
2 2(m + n)2
mn
= area of the triangle ABC.
(m + n)2
mn
Similarly the area of the triangle RDS = area of the triangle ADC.
(m + n)2
mnH
Therefore the area of the triangle P BQ and RDS = , where H denotes
(m + n)2
the area of the quadrilateral figure ABCD.
mnH
In the same way we show that the area of the triangles QCR and SAP = .
(m + n)2
2mnH
Thus the area of the four triangles P BQ, QCR, RDS, and SAP = .
(m + n)2
Therefore the area of the quadrilateral figure P QRS
{ }
2mn H(m2 + n2 )
=H 1− 2
= .
(m + n) (m + n)2
CHAPTER XXV : Miscellaneous Examples 269

1
Problem 156. Show that cos θ = − is a solution of the equation
2
1
cos θ + cos 3θ = ;
2
and find the other values of cos θ.
1 1
Solution. cos θ + cos 3θ = ; therefore cos θ + 4 cos3 θ − 3 cos θ = ;
2 ( ) (2 )
1 1 1
therefore 4 cos3 θ − 2 cos θ − = 0; therefore 4 cos3 θ + − 2 cos θ + = 0;
2 8 2
( )( ) ( )
1 1 1 1
therefore 2 cos θ + cos2 θ − cos θ + − cos θ + = 0;
(2 )( 2 4 ) 2
1 1
thereofore cos θ + 2 cos2 θ − cos θ − = 0.
2 2
1 1
Thus either cos θ + = 0 or 2 cos2 θ − cos θ − = 0;
2 2 √
1 1± 5
the former gives cos θ = − ; the latter gives cos θ = .
2 4

Problem 157. Show that


cos β cos γ sin(γ − β) + cos γ cos α sin(α − γ) + cos α cos β sin(β − α)
= sin(α − β) sin(β − γ) sin(γ − α).
1
Solution. cos β cos γ sin(γ − β) = {cos(β − γ) + cos(β + γ)} sin(γ − β)
2
1
= {sin(2γ − 2β) + sin 2γ − sin 2β}.
4
Transform the other two terms in the same way; and thus we obtain finally as the
sum
1
{sin(2γ − 2β) + sin(2α − 2γ) + sin(2β − 2α)}.
4
Again, sin(α − β) sin(β − γ) sin(γ − α)
1
= {cos(α + γ − 2β) − cos(α − γ)} sin(γ − α)
2
1
= {sin(2γ − 2β) + sin(2β − 2α) + sin(2α − 2γ)}.
4
Thus the proposed expressions are equal.
Or thus : from Chapter viii : P roblem 12 we see that
sin β sin γ sin(γ − β) + sin γ sin α sin(α − γ) + sin α sin β sin(β − α)
= sin(α − β) sin(β − γ) sin(γ − α).
π π π
In this formula change α, β, γ into + α, + β, + γ respectively; and thus we
2 2 2
obtain the required result.

Problem 158. If A + B + C = 180 ◦ , show that


sin A sin(A − B) sin(A − C) + sin B sin(B − C) sin(B − A)
+ sin C sin(C − A) sin(C − B)
= sin A sin B sin C{1 − 8 cos A cos B cos C}.
1
Solution. sin A sin(A − B) sin(A − C) = sin A{cos(C − B) − cos(2A − B − C)}
2
1
= {sin(A + C − B) + sin(A + B − C) − sin(3A − B − C) − sin(B + C − A)}
4
1
= {sin 2B + sin 2C + sin 4A − sin 2A}.
4
In this way we see that the expression on the left-hand side in the proposed formula
1
= {sin 2A + sin 2B + sin 2C + sin 4A + sin 4B + sin 4C}.
4
Then by Chapter viii : P roblem 33 we have
sin 2A + sin 2B + sin 2C = 4 sin A sin B sin C,
sin 4A + sin 4B + sin 4C = −4 sin 2A sin 2B sin 2C
= −32 sin A sin B sin C cos A cos B cos C.
CHAPTER XXV : Miscellaneous Examples 270

Thus we obtain the required result.

Problem 159. A person with his eye on the level of the ground close to a pole,
observed that he saw the top of a distant windw over an intervening wall at an ele-
vation α. He then climbed up the pole c feet, when he saw the whole window, and
the elevations of the tops of the window and wall were then β and γ. Show that the
height of the lowest part of the window above the ground was
c(tan α − tan β + tan γ)
.
tan α − tan β
Solution. Let A denote the bottom of the pole, B the point on the pole to which the
man climbs, F the top of the window, E the bottom. Let AF and BE intersect at D,
which is therefore the top of the wall. Draw DC perpendicular to the ground, and
produce F E to meet the ground at H. Draw from B a horizontal straight line meeting
F H at G.
c cos α
Then from the triangle BAF we get BF = ;
sin(α − β)
c cos α cos β c
BG = BF cos β = = ,
sin(α − β) tan α − tan β
c tan γ
EG = BG tan γ = ,
tan α − tan β
c(tan α − tan β + tan γ)
EH = c + EG = .
tan α − tan β

Problem 160. ABC is a triangle; straight lines bisecting the angles A and B meet
the opposite sides at D and E respectively. Show that the area of the triangle CED is
A B
S sin sin
2 2 .
C −A C −B
cos cos
2 2
1
CE sin B
Solution. From the triangle CEB we have = ( 2 );
a 1
sin C + B
2
1
CD sin A
and from the triangle CDA we have = ( 2 ).
b 1
sin C + A
2
1
Thus the area of the triangle CED = CE · CD sin C
2
1 1 1 1
ab sin C sin A sin B S sin A sin B
= ( ) (
2 2 )= 2 2 .
1 1 C−A C−B
2 sin C + B sin C + A cos cos
2 2 2 2

Problem 161.
If p = 2 cos A − 5 cos3 A + 4 cos5 A,
and q = 2 sin A − 5 sin3 A + 4 sin5 A;
Show that p cos 3A + q sin 3A = cos 2A,
1
and p sin 3A − q cos 3A = sin 2A.
2
Solution. We have p = 2 cos A + cos3 A(−5 + 4 cos2 A)
1
= 2 cos A + (cos 3A + 3 cos A)(−5 + 4 cos2 A)
4
1
= 2 cos A + (cos 3A + 3 cos A)(−3 + 2 cos 2A)
4
3 9 1 3
= 2 cos A − cos 3A − cos A + cos 3A cos 2A + cos A cos 2A
4 4 2 2
CHAPTER XXV : Miscellaneous Examples 271

3 1 1 3
=− cos 3A − cos A + (cos 5A + cos A) + (cos 3A + cos A)
4 4 4 4
1
= (cos 5A + 3 cos A).
4
In the same way we find that
1
q= (sin 5A + 3 sin A).
4
Therefore
1 1
p cos 3A + q sin 3A = (cos 5A + 3 cos A) cos 3A + (sin 5A + 3 sin A) sin 3A
4 4
1 3
= (cos 5A cos 3A + sin 5A sin 3A) + (cos 3A cos A + sin 3A sin A)
4 4
1 3
= cos(5A − 3A) + cos(3A − A) = cos 2A.
4 4
And
1 1
p sin 3A − q cos 3A = (cos 5A + 3 cos A) sin 3A − (sin 5A + 3 sin A) cos 3A
4 4
1 3
= (cos 5A sin 3A − sin 5A cos 3A) + (sin 3A cos A − cos 3A sin A)
4 4
1 3 1
= − sin(5A − 3A) + sin(3A − A) = sin 2A.
4 4 2

( )cot2 β
α n when n is infinite.
Problem 162. Find the limit of cos
n
( )cot2 β
α n ; therefore
Solution. Let u = cos
n
β α β β α
log u = cot2 log cos = cos2 × cosec 2 log cos .
n n n n n
Now as in the solution of Chapter xii : P roblem 33 we can show that
β α α2
cosec 2 log cos = − 2 when n is infinite.
n n 2β
β
And cos2 = 1 when n is infinite.
n
2
α2 − α
Thus log u = − 2 ; and therefore u = e 2β 2 .

Problem 163. Sum the infinite series


1 +2 1 + 2 + 22 1 + 2 + 22 + 23
1+ + + + ...
2 3 4
Solution. If n be a positive integer, we have 1 + 2 + 22 + . . . + 2n = 2n+1 − 1,
Hence the infinite series
22 − 1 23 − 1 24 − 1
=2−1+ + + + ...
2 3 4
{ }
22 23 24 1 1 1
=2+ + + + ... − 1+ + + + ...
2 3 4 2 3 4
= e2 − 1 − {e − 1} = e2 − e.

Problem 164. Find cos(x − y) and cos(x + y) from the equations


sec α cos(x + y) = 1 + tan x tan y, sec β cos(x − y) = 1 − tan x tan y.

cos(x − y)
Solution. Here sec α cos(x + y) = ,
cos x cos y
cos(x + y)
and sec β cos(x − y) = ;
cos x cos y
CHAPTER XXV : Miscellaneous Examples 272

cos β cos(x + y) cos(x − y)


therefore, by division, · = ,
cos α cos x − y cos(x + y)

cos(x − y) cos β
so that = (37)
cos(x + y) cos α
cos(x − y)
And cos(x − y) + cos(x + y) = 2 cos x cos y = 2 cos α
cos(x + y)

=2 cos α cos β (38)
From (37) and (38) we have
{√ }
cos β √
cos(x + y) +1 =2 cos α cos β;
cos α

2 cos α cos β
therefore cos(x + y) = √ √ .
cos α + cos β

2 cos β cos α
Then by (37) we have cos(x − y) = √ √ .
cos α + cos β

Problem 165. In any triangle


1 1 1
a cos (B − C ) b cos (C − A) c cos (A − B) 2 (ab + bc + ca)
2 + 2 + 2 = .
1 1 1 abc
bc cos (B + C ) ca cos (C + A) ab cos (A + B)
2 2 2
Solution. It may be shown as in the solution of Example xx. 4 that
1
a cos (B − C) b+c a(b + c)
2 = = ;
1 bc abc
bc cos (B + C)
2
1 1
b cos (C − A) b(c + a) c cos (A − B) c(a + b)
similarly 2 = , and 2 = .
1 abc 1 abc
ca cos (C + A) ab cos (A + B)
2 2
Hence by addition we obtain the required result.

Problem 166. O is any point in the interior of a quadrilateral ABCD; OP, OQ,
OR, OS are perpendiculars on the sides AB, BC , CD, DA respectively. Show that the
area of PQRS is
1 1 ( )
ABCD − OA2 sin 2A + OB 2 sin 2B + OC 2 sin 2C + OD 2 sin 2D .
2 8
Solution. AP = OA cos OAP, AS = OA cos OAS;
1
therefore the area of the triangle AP S = OA2 cos OAP cos OAS sin A.
2
In the same way the area of the triangle OP S
1 1
= OA2 sin OAP sin OAS sin P OS = OA2 sin OAP sin OAS sin(180◦ − A)
2 2
1
= OA2 sin OAP sin OAS sin A.
2
Hence triangle AP S - triangle OP S
1
= OA2 sin A {cos OAP cos OAS − sin OAP sin OAS}
2
1 1 1
= OA2 sin A cos(OAP + OAS) = OA2 sin A cos A = OA2 sin 2A.
2 2 4
In the same way we obtain
1
triangle BQP − triangle OQP = OB 2 sin 2B,
4
1
triangle CRQ − triangle ORQ = OC 2 sin 2C,
4
CHAPTER XXV : Miscellaneous Examples 273

1
and triangle DSR − triangle OSR = OD2 sin 2D,
4
Hence by addition we have
triangle AP S+ triangle BQP + triangle CRQ+ triangle DSR− quadrilateral P QRS
1{ }
= OA2 sin 2A + OB 2 sin 2B + OC 2 sin 2C + OD2 sin 2D .
4
But the sum of the four triangles and the quadrilateral
= the quadrilateral ABCD.
Hence by subtraction we have
twice the quadrilateral P QRS = the quadrilateral ABCD
1
− {OA2 sin 2A + OB 2 sin 2B + OC 2 sin 2C + OD2 sin 2D}.
4

Problem 167. In any triangle


B−C A C −A B A−B C
a sin cosec + b sin cosec + c sin cosec = 0.
2 2 2 2 2 2
Solution. We have a = 2R sin A, b = 2R sin B, c = 2R sin C;
thus the proposed expression
{ }
B−C A C−A B A−B C
= 4R sin cos + sin cos + sin cos
{ 2 2 2 2 2 2 }
B−C B+C C−A C+A A−B A+B
= 4R sin sin + sin sin + sin sin
{ 2 2 2 2 2} 2
2 B 2 C 2 C 2 A 2 A 2 B
= 4R sin − sin + sin − sin + sin − sin
2 2 2 2 2 2
= 0.

Problem 168. Show by the aid of Trigonometry that if x + y + z = xyz, then


3x − x 3 3y − y 3 3z − z 3 (3x − x 3 )(3y − y 3 )(3z − z 3 )
+ + = .
1 − 3x 2 1 − 3y 2 1 − 3z 2 (1 − 3x 2 )(1 − 3y 2 )(1 − 3z 2 )
Solution. Assume x = tan A, y = tan B, z = tan c; then since x + y + z = xyz it
will follow in the manner of Art. 114 (page 328) that tan(A + B + C) is zero; therefore
A + B + C = nπ where n is zero or some integer. Therefore 3A + 3B + 3C = 3nπ; and
therefore in the manner of Art. 114 (page 328) we have
tan 3A + tan 3B + tan 3C = tan 3A tan 3B tan 3C.
3 tan A − tan3 A 3x − x3
But tan 3A = = ;
1 − 3 tan2 A 1 − 3x2
3y − y 2 3z − z 3
similarly tan 3B = , tan 3C = :
1 − 3y 2 1 − 3z 2
thus the required result follows.

Problem 169. If l, m, n be the perpendiculars from the centre of the circumscribed


circle on the sides of a triangle, show that
( )
a b c abc
4 + + = .
l m n lmn
Solution. We have l = R cos A, m = R cos B, n = R cos C; thus we have to show that
4a 4b 4c abc
+ + = 2 .
R cos A R cos B R cos C R cos A cos B cos C
Now a = 2R sin A, b = 2R sin B, c = 2R sin C; thus the proposed identity becomes
tan A + tan B + tan C = tan A tan B tan C;
and this is true by Art. 114 (page 328) .

Problem 170. If A, B and C are the angles of a triangle, show that:


A B C 3
sin2 + sin2 + sin2 cannot be less than .
2 2 2 4
A B C 3 1
Solution. sin2 + sin2 + sin = − (cos A + cos B + cos C)
2 2 2 2 2
CHAPTER XXV : Miscellaneous Examples 274

A B C
= 1 − 2 sin
sin sin , by Art. 114 (page 328).
2 2 2
A B C
Now we have seen in Chapter xiii : P roblem 40 that sin sin sin cannot be
2 2 2
1 A B C 1
greater than ; hence sin2 + sin2 + sin cannot be less than 1 − , that is than
8 2 2 2 4
3
.
4

Problem 171. Find the limit when θ is indefinitely diminished of


sin aθ vers aθ
and of .
sin bθ vers bθ
sin aθ a sin aθ bθ
Solution. = · ; and when θ is indefinitely diminished the limit of
sin bθ b aθ sin bθ
sin aθ bθ sin aθ a
is unity, and so also is the limit of : thus the limit of is .
aθ sin bθ sin bθ b
 2 
aθ  sin aθ 
vers aθ 1 − cos aθ sin2
Also = = 2 = 2 .
vers bθ 1 − cos bθ
sin 2 bθ  sin
bθ 
2 2

sin
Now the limit of 2 is a in the manner just shown; therefore the limit of vers aθ
bθ b vers bθ
sin
2
a2
is 2 .
b

Problem 172. Show that


√ { }
1 1 1 1
log 2 = + + + ... .
4 1 ·1 2 ·3 3 ·5
Solution. { }
1 1 1 1 1
+ + + + ...
4 {1 · 1 2·3 3·5 4·7 }
1 1 1 1 1
= + + + + ...
2 1·2 3·4 5·6 7·8
1 √
= log 2, by Art. 146( page 333), = log 2.
2

Problem 173. If A + B + C = 180 ◦ , show that


A A B C B B C A
tan + cos sec sec = tan + cos sec sec
2 2 2 2 2 2 2 2
C C A B
= tan + cos sec sec .
2 2 2 2
Solution.
A A
A A B C sin cos
tan + cos sec sec = 2 + 2
2 2 2 2 A B C
cos cos cos
2 2 2
2 A A B C
cos + sin cos cos
= 2 2 2 2
A B C
cos cos cos
2 2 2
A A B C
The numerator of this fraction = 1 − sin2 + sin cos cos
2 2 2 2
{ }
A B C B+C A B C
= 1 + sin cos cos − cos = 1 + sin sin sin .
2 2 2 2 2 2 2
A B C A B C
Thus the fraction = sec sec sec + tan tan tan .
2 2 2 2 2 2
CHAPTER XXV : Miscellaneous Examples 275

Similarly the other two proposed expressions may be reduced to this symmetrical
form; and thus the three expressions are equal.

Problem 174. If sin(π cot θ) = cos(π tan θ), show that either cosec 2 θ or cot 2 θ is of the
1
form m + , where m is an integer positive or negative.
4
Solution.
sin(π cot θ) = cos(π tan θ);
( )
π
therefore cos(π tan θ) = cos − π cot ϕ ;
2
therefore all the solutions are comprised in( )
π
π tan θ = 2nπ ± − π cot θ ,
2
where n is zero, or some integer, positive or negative.
1 1 1 1
Take the upper sign; thus 2n + = tan θ + cot θ = , so that n + = .
2 sin θ cos θ 4 sin 2θ
1 1
Take the lower sign; thus − 2n = cot θ − tan θ = 2 cot 2θ, so that cot 2θ = − n.
2 4
Thus either cosec 2θ or cot 2θ takes the prescribed form.

Problem 175. In any triangle


( )
sin A + sin B + sin C 2 a cos A + b cos B + c cos C
= .
a+b+c 2abc
Solution. We have a = 2R sin A, b = 2R sin B, c = 2R sin C.
( )2
1
Thus the left-hand member = . And,
2R
sin 2A + sin 2B + sin 2C 1
the right-hand member = = , by Art. 114 (page 328).
16R2 sin A sin B sin C 4R2

Problem 176. A circle of radius r is inscribed in a sector of a circle of radius a, and


2c is the chord of the sector. Show that
1 1 1
= + .
r a c
Solution. Let θ be the angle of the sector; then we see from a diagram that
r θ
= sin .
a−r 2
θ
But 2c = 2a sin .
2
r c
∴ = ;
a−r a
a−r a
∴ = ;
r c
1 1 1
∴ = + .
r a c

Problem 177. If tan x + tan 2x√+ tan 3x + tan 4x = 0 , show that either 5x = nπ, or
2x = (2m + 1 )π, or 8 cos 2x = 1 ± 17 .

sin x sin 4x sin 2x sin 3x


Solution. + + + = 0;
cos x cos 4x cos 2x cos 3x
sin x cos 4x + cos x sin 4x sin 2x cos 3x + cos 2x sin 3x
therefore + = 0;
cos x cos 4x cos 2x cos 3x
sin 5x sin 5x
therefore + = 0;
cos x cos 4x cos 2x cos 3x
1 1
therefore either sin 5x = 0 or + = 0;
cos x cos 4x cos 2x cos 3x
If we take the former, then 5x = nπ.
CHAPTER XXV : Miscellaneous Examples 276

If we take the latter, then cos 2x cos 3x + cos x cos 4x = 0;


therefore cos 2x(4 cos3 x − 3 cos x) + cos x cos 4x = 0;
therefore either cos x = 0 or cos 2x(4 cos2 x − 3) + cos 4x = 0.
π
If we take the former, then x = (2m + 1) .
2
If we take the latter, then cos 2x(2 + 2 cos 2x − 3) + 2 cos2 2x − 1 = 0;
therefore 4 cos 2x − cos 2x − 1 = 0;
2

1 ± 17
and by solving this quadratic we obtain cos 2x = .
8

Problem 178. Show that


sin(θ − β) sin(θ − γ) sin(θ − γ) sin(θ − α) sin(θ − α) sin(θ − β)
+ + = 1.
sin(α − β) sin(α − γ) sin(β − γ) sin(β − α) sin(γ − α) sin(γ − β)
Solution. We may proceed as in the solution of P roblem 147, and seek for the values
of x, y, and z, which make
x sin(θ − β) sin(θ − γ) + y sin(θ − γ) sin(θ − α) + z sin(θ − α) sin(θ − β)
1
always equal to 1. Then we shall find that x = , and so on.
sin(α − β) sin(α − γ)
Or we may verify the formula by direct work. For reduce the three fractions to the
common denominator sin(α − β) sin(β − γ) sin(γ − α). Then the numerator will become
L sin2 θ + M sin θ cos θ + N cos2 θ, where
L = cos β cos γ sin(γ − β) + cos γ cos α sin(α − γ) + cos α cos β sin(β − α),
M = − sin(γ + β) sin(γ − β) − sin(α + γ) sin(α − γ) − sin(β + α) sin(β − α),
N = sin β sin γ sin(γ − β) + sin γ sin α sin(α − γ) + sin α sin β sin(β − α).
It is obvious by Art. 83 (page 324) that M = 0; and we have seen in the solution
of P roblem 157 that L and N are each equal to the common denominator; so that
L sin2 θ + N cos2 θ is also equal to this denominator, and the expression is equal to
unity.

Problem 179. ABC is a triangle; straight lines are drawn bisecting the angles
A, B, C and meeting the opposite sides at D, E, F respectively. Show that the area of
the triangle DEF
A B C
2S sin sin sin
= 2 2 2 .
B−C C −A A−B
cos cos cos
2 2 2
ac ab
Solution. By Euclid vi. 2 we find that BD = , and CD = .
b+c b+c
Similar expressions hold for the segments of the other sides of ABC.
Therefore the area of the triangle DCE
1 ab ab Sab
= · sin C = .
2b+c a+c (a + c)(b + c)
Similar expressions hold for the areas of EF A and F DB.
Therefore{the area of DEF }
ab bc ca
=S 1− − −
(a + c)(b + c) (b + a)(c + a) (c + b)(a + b)
S
= {(a + b)(b + c)(c + a) − ab(a + b) − bc(b + c) − ca(c + a)}
(a + b)(b + c)(c + a)
2abcS a b c
= = 2S · · · ·.
(a + b)(b + c)(c + a) b+c c+a a+b
A A A
a sin A sin cos sin
Now = = 2 2 = 2 .
b+c sin B + sin C B+C B−C B−C
sin cos cos
2 2 2
CHAPTER XXV : Miscellaneous Examples 277

B C
b sin c sin
Similarly = 2 , and = 2 .
c+a C−A a+b A−B
cos cos
2 2
Thus the required result is obtained.

Problem 180. From the top of a mountain the angles of depression of two stations in
the plane at its foot are observed to be α and β, and the difference of their bearings is
observed to be γ. Show that if c be the distance between the two stations, the height
of the mountain will be
c sin α sin β sin 2 α sin 2 β γ
, where sin2 ϕ = cos2 .
sin(α + β) cos ϕ sin2 (α + β) 2
Solution. Let x denote the height of the mountain; then the distances of the two
stations from the point in the horizontal plane which is vertically under the top of the
mountain are x cot α and x cot β respectively.
Thus c2 = x2 cot2 α + x2 cot2 β − 2x2 cot α cot β cos γ; (Art. 215 : page 336)
c2
∴ x2 =
cot2α+ cot2β · 2 cot α cot β cos γ
c2 sin2 α sin2 β
=
sin β cos α + sin α cos2 β − 2 sin α cos α sin β cos β cos γ
2 2 2

The denominator of this fraction may be put in the form


γ
(sin β cos α + cos β sin α)2 − sin 2α sin 2β cos2 ,
2
so that with the specified value of ϕ it becomes sin2 (α + β) cos2 ϕ;
c sin α sin β
and therefore x= .
sin(α + β) cos ϕ

Problem 181. Find approximately the angle subtended by a target two feet wide at
the distance of 450 yards.
θ 1
Solution. Let θ denote the angle; then tan = ; therefore approximately
2 3 × 450
θ 1 1 180 1
= ; therefore θ = . Hence the number of degrees in the angle is × ,
2 1350 675 π 675
180 60 4 180 4
and the number of minutes is × , that is × , that is × 57.29 . . . that is
π 675 45 π 45
about 5.

Problem 182. Sum the following infinite series :


1 4 9 16
+ + + + ...
2 3 4 5
n2
Solution. The general term of the series is ; for we obtain all the terms by
n+1
putting successively 1, 2, 3, . . . for n in this expression.
n2 n(n + 1) − (n + 1) + 1 1 1 1
Now = = − + .
n+1 n+1 n−1 n n+1
If then we split up each term into three in this manner, beginning with the second
term, we obtain
1 1 1 1
+1+ + + + ...
2 2 3 4
{ }
1 1 1 1
− + + + + ...
2 3 4 5
1 1 1
++ + + ...;
3 4 5
1 1
that is + e − 1 − (e − 2) + e − 2 − , that is e − 1.
2 2
CHAPTER XXV : Miscellaneous Examples 278

Problem 183. If 2 sec θ = sec(θ + 2 α) + sec(θ − 2 α), show that


cos2 θ = 2 cos2 α.
Solution.
2 1 1 2 cos θ cos 2α
Here = + =
cos θ cos(θ + 2α) cos(θ − 2α) cos(θ + 2α) cos(θ − 2α)
2 cos θ cos 2α
= ;
cos2 θ − sin2 2α
therefore cos θ − sin 2α = cos θ cos 2α,
2 2 2

therefore cos2 θ(1 − cos 2α) = sin2 2α = 4 sin2 α cos2 α;


therefore cos2 θ = 2 cos2 α.

Problem 184. Solve the equations


2 (sin 2 θ + sin 2 ϕ) = 1 = 2 sin(θ + ϕ).
Solution. Here 4 sin(θ + ϕ) cos(θ − ϕ) = 1, and 2 sin(θ + ϕ) = 1;
1 1
therefore sin(θ + ϕ) = , and cos(θ − ϕ) = ;
2 2
π π
therefore θ + ϕ = nπ + (−1)n , and θ − ϕ = 2mπ ± .
6 3

Problem 185. In any triangle


( )
A B C A B C
(sin A + sin B + sin C ) tan + tan + tan = 4 + 4 sin sin sin .
2 2 2 2 2 2
A B C
Solution. sin A + sin B + sin C = 4 cos cos cos , by Chapter viii : P roblem 16.
2 2 2
A+B C
A B sin cos
And tan + tan = 2 = 2 ;
2 2 A B A B
cos cos cos cos
2 2 2 2
C C
A B C cos sin
therefore tan + tan + tan = 2 + 2
2 2 2 A B C
cos cos cos
2 2 2
C C A B
cos2 + sin cos cos
= 2 2 2 2 ;
A B C
cos cos cos
2 2 2
2 C C A B
the numerator = 1 − sin + sin cos cos
2{ 2 2 2 }
C A B A+B A B C
= 1 + sin cos cos − cos = 1 + sin sin sin ;
2 2 2 2 2 2 2
A B C
1 + sin sin sin
and thus the fraction = 2 2 2
A B C
cos cos cos
2 2 2
Hence by multiplication we obtain the required result.

Problem 186. A, B, C , D, E, . . . are the angular points of a polygon inscribed in


a circle; from the centre of the circle perpendiculars are drawn to the sides, and a
second polygon is formed by joining the feet of the perpendiculars. Show that if the
area of the first polygon is double that of the second,
sin 2A + sin 2B + sin 2C + sin 2D + sin 2E + . . . = 0 .
Solution. Proceed as in the solution of P roblem 166. Then we obtain the following
expression for the excess of the sum of all triangles at the corners above the second
polygon
r2
{sin 2A + sin 2B + sin 2C + sin 2D + . . .} ,
4
where r is the radius of the circle.
CHAPTER XXV : Miscellaneous Examples 279

Hence this vanishes if sin 2A + sin 2B + . . . = 0, and then the sum of the triangles at
the corners is equal to the second polygon, and therefore the first polygon is double
the second.

Problem 187. In any triangle


B−C A C −A B A−B C
a sin sec + b sin sec + c sin sec = 0.
2 2 2 2 2 2
Solution. We have a = 2R sin A, b = 2R sin B, c = 2R sin C;
hence the proposed
{ expression }
B−C A C−A B A−B C
= 4R sin sin + sin sin + sin sin
{ 2 2 2 2 2 2
A+C−B A+B−C B+A−C B+C−A
= 2R cos − cos + cos − cos
2 2 2 } 2
B+C−A A+C−B
+ cos − cos = 0.
2 2

Problem 188. In any triangle


a 2 sin(B − C ) b2 sin(C − A) c 2 sin(A − B)
+ + = 0.
sin B + sin C sin C + sin A sin A + sin B
Solution. We have a = 2R sin A, b = 2R sin B, c = 2R sin C;
hence the proposed
{ expression }
2 sin2 A sin(B − C) sin2 B sin(C − A) sin2 C sin(A − B)
= 4R + + .
sin B + sin C sin C + sin A sin A + sin B
sin2 A sin(B − C) sin A sin(B + C) sin(B − C)
Now =
sin B + sin C sin B + sin C
sin A(sin2 B − sin2 C)
= = sin A(sin B − sin C).
sin B + sin C
In this way the proposed expression
= 4R2 {sin A(sin B − sin C) + sin B(sin C − sin A) + sin C(sin A − sin B)} = 0.

Problem 189. If a be the side of a regular polygon inscribed in a circle of radius r,


and b the side of another regular polygon of twice the number of sides inscribed in
the same circle, show that √ √
( ) ( )
a a
b= r r+− r r− .
2 2
Solution. If n be the number of sides in the first polygon we have
π π
a = 2r sin , b = 2r sin .
n 2n
π π
By Art. 100 (page 327), since lies between 0 and , we have
√n
( ) √ (
2
)
π π π
2 sin = 1 + sin − 1 − sin ;
2n n n
√( ) √( )
b a a
therefore = 1+ − 1− .
r 2r 2r
Multiply by r, and we obtain the required result.

Problem 190.
( If A +)B(+ C = 180)◦ , show that
( )( )
B C A C A B
1 − sin 1 − sin cos + 1 − sin 1 − sin cos
2 2 2 ( 2 )( 2) 2
A B C
+ 1 − sin 1 − sin cos
2 2 2
A B C
= cos cos cos .
2 2 2
CHAPTER XXV : Miscellaneous Examples 280

( )( )
B C A
Solution. 1 − sin 1 − sin cos
2 ( 2 2 )
A B C A B C A
= cos − sin + sin cos + sin sin cos .
2 2 2 2 2 2 2
Develop each of the other two terms in the same way; the aggregate
A B C A+B B+C C+A B C A
= cos + cos + cos − sin − sin − sin + sin sin cos +
2 2 2 2 2 2 2 2 2
C A B A B C
sin sin cos + sin sin cos .
2 2 2 2 2 2
A B+C B C+A C A+B
But cos = sin , cos = sin , cos = sin ;
2 2 2 2 2 2
thus the expression

B C A C A B A B C
= sin sin cos + sin sin cos + sin sin cos
2 2 2 2 ( 2 2 2 2 ) 2
B C A A C B B C
= sin sin cos + sin sin cos + sin cos
2 2 2 2 2 2 2 2
B C A A B+C
= sin sin cos + sin sin
2 { 2 2 2 } 2 { }
A A B C A B+C B C
= cos sin + sin sin = cos cos + sin sin
2 2 2 2 2 2 2 2
A B C
= cos cos cos .
2 2 2
Or instead of the last four lines we may use Art. 113 (page 328), observing that
here( )
A B C
cos + + = 0.
2 2 2

Problem 191. A person walks in a straight line towards a very distant object,
and observes that at three points A, B, C , the elevations of the objects are α, 2 α, 3 α
respectively. Show that AB = 3 · BC nearly.
Solution. Let D denote the top of the object.
AB
From the triangle ABD we have = 1, for the angles BAD and BDA are equal.
BD
BD sin 3α
From the triangle BDC we have = .
BC sin α
AB sin 3α
Therefore = = 3 − 4 sin2 α.
BC sin α
Since the object is very distant α is very small; therefore AB = 3BC nearly.

Problem 192. If x is less than unity so also is


1 1
+ .
x log(1 − x)
1 2 1 1
1 1 1 1 x + x3 + x4 + . . .
Solution. + = − = 2 3 4 .
x loge (1 − x) x 1 2 1 3 2
1 3 1 4
x + x + x + ... x + x + x + ...
2 3 2 3
Here every term in the numerator is less than the corresponding term in the de-
nominator, and thus the fraction is less than unity.

Problem 193. Having given


6 sin B 3 sin 2B 2 sin 3B
= = ,
cos(A + B) cos(A + 2B) cos(A + 3B)
show that it is impossible that any value can be assigned to B other than nπ.
6 sin B 6 sin B cos B
Solution. Here = ;
cos(A + B) cos(A + 2B)
thus either sin B = 0 or cos(A + 2B) = cos(A + B) cos B;
the latter gives cos(A + B) cos B − sin(A + B) sin B = cos(A + B) cos B,
so that either sin B = 0 or sin(A + B) = 0.
CHAPTER XXV : Miscellaneous Examples 281

Suppose that sin(A + B) = 0; then since


3 sin 2B 2 sin 3B
= ,
cos(A + B + B) cos(A + B + 2B)
3 sin 2B 2 sin 3B
we have = ,
cos(A + B) cos B cos(A + B) cos 2B
3 sin B sin 3B
so that = ;
cos(A + B) cos(A + B) cos 2B
therefore 3 sin B cos 2B = sin 3B,
therefore 3 sin B(1 − 2 sin2 B) = 3 sin B − 4 sin3 B,
therefore 6 sin3 B = 4 sin3 B,
therefore sin B = 0.

sin θ cos θ sin2 θ cos2 θ 6


Problem 194. If = , and + = 2 , find a general expres-
x y y2 x2 x + y2
sion for the value of θ.
x x2 y2
Solution. Here tan θ = ; therefore sin2 θ = 2 2
, and cos2 θ = 2 . Substitute
y x +y x + y2
in the second given
( equation; ) thus
x2 y2 1 6 x2 y2
2
+ 2 = 2 ; therefore 2 + 2 = 6.
y x x2
+y 2 x +y 2 y x
x2 x2 √ √
From this quadratic in 2 we find 2 = 3 ± 2 2 = ( 2 ± 1)2 ; therefore tan θ =
y y
√ √ 3π π
±( 2 + 1) or ±( 2 − 1). The former gives θ = nπ ± ; and the latter gives θ = nπ ± .
8 8

Problem 195. If A, B C are the angles of a triangle, and sin A, sin B, sin C are in
Harmonical Progression, then 1 − cos A, 1 − cos B, 1 − cos C are in Harmonical Pro-
gression.
Solution. Since the sines of the angles are in Harmonical Progression, so are the
sides of the opposite angles. Thus a, b, c are in Harmonical Progression, and we have
(s − b)(s − c) (s − c)(s − a) (s − a)(s − b)
to show that , , are so also. Multiply each term
bc ca ab
abc a b c
by ; thus we see it is sufficient to show that , , ; are
(s − a)(s − b)(s − c) s−a s−b s−c
s−a s−b s−c
in Harmonical Progression, or that , , are in Arithmetical Progression,
a b c
s s s
or that , , are in Arithmetical Progression; and this is the case since a, b, c are
a b c
in Harmonical Progression.

Problem 196. If R be the radius of a circle described about a regular pentagon


R 17
whose side is a, show that = nearly.
a 20
π
Solution. We have a = 2R sin ;
5 √ √ √ √
R 1 2 2 10 + 2 5 2 200 + 40 5
∴ = π = √ √ = √ = √
a 2 sin 10 − 2 5 80 80 × 20
√ 5 √ √
200 + 40 5 289.44 . . . 17
= = = nearly.
20 20 20

sin A m cos A p
Problem 197. In a triangle = , and = . Show that
sin B n cos B q
mp − nq
cos C = .
np − mq
CHAPTER XXV : Miscellaneous Examples 282

sin(B + C) m cos(B + C) p
Solution. Here = , =− ;
sin B n cos B q
m p
therefore cos C + cot B sin C = , tan B sin C − cos C = ;
( n )( ) q
m p
therefore 2
sin C = − cos C + cos C ;
n ( q )
mp m p
therefore 1= + cos C − ;
nq n q
mp − nq
therefore cos C = .
np − mq

Problem 198. If O be the centre of the circle inscribed in a triangle ABC , and D,
E, F the points of contact with BC , CA, AB respectively , show that
OA · OB · OC (AF + BD + CE) = 4R · AF · BD · CE.
r r r
Solution. We have OA = , OB = , OC = ;
A B C
sin sin sin
2 2 2
A B C
AF = r cot , BD = r cot , CE = r cot .
2 2 2
Hence we have to show{that }
r4 A B C A B C
cot + cot + cot = 4Rr3 cot cot cot .
A B C 2 2 2 2 2 2
sin sin sin
2 2 2
By Chapter viii : P roblem 15 the left-hand member
A B C
r4 cos cos cos
= 2 2 2 ;
A B C
sin2 sin2 sin2
2 2 2
thus we have to show that
1 1 1
4R sin A sin B sin C = r.
2 2 2
√ member √
The left-hand √
(s − b)(s − c) (s − a)(s − c) (s − a)(s − b) 4RS 2 S
= 4R = = = r.
bc ac ab sabc s

Problem 199. In the semi-circle ABC , whose centre is O, and radius ρ, the straight
line OB is drawn at an angle 2 α to OC . Circles are inscribed in the triangles OAB, OCB.
Show that the distance between their√centres is
ρ 2 − sin 2 α
√ .
(1 + sin α)(1 + cos α)
Solution. The radius of the circle inscribed in the triangle OBC
1 2
area of the triangle ρ sin 2α ρ sin 2α
= = 2 = .
semi-perimeter ρ(1 + sin α) 2(1 + sin α)
Let P denote the position of the centre; then
ρ sin 2α 1 ρ cos α
OP = × = .
2(1 + sin α) sin α 1 + sin α
Again, let Q denote the position of the centre of the circle inscribed in the triangle
OAB; then, as 2α is now to be changed to π − 2α, we have
( )
π
ρ cos −α
ρ sin α
OQ = (2 )= .
π 1 + cos α
1 + sin −α
2
And since P OQ is a right angle, P Q2 = OP 2 + OQ2
CHAPTER XXV : Miscellaneous Examples 283

{ }
cos2 α sin2 α
= ρ2 2
+
(1 + sin α) (1 + cos α)2
{ }
1 − sin α 1 − cos α ρ2 (2 − sin 2α)
= ρ2 + = ;
1 + sin α 1 + cos α (1 + sin α)(1 + cos α)

ρ 2 − sin 2α
therefore PQ = √ .
(1 + sin α)(1 + cos α)

Problem 200. A man walking along a straight road observes the directions with
respect to the road of two objects when the angle which they subtend is greatest, and
then measures the distance from the point of observation to the point whence they
appear in the same straight line. Find the distance between them.
Solution. Let A and B be the two objects. Suppose a circle to pass through A and B,
and to touch the straight line at P ; then P is the point at which the greatest angle is
subtended: see Appendix to Euclid, page 308. Produce AB to meet the straight line at
Q. Let the angle BP Q = α, and let β be the angle between AP and the straight line.
Then also P AB = α, and P BA = β, by Euclid iii. 32. Let P Q = c.
BP sin(β − α) AB sin(β + α)
Then = , = ;
PQ sin β BP sin α
AB sin(β + α) sin(β − α)
therefore = .
c sin α sin β

Problem 201. Show that r1 + r2 + r3 − r = 4R.


Solution. We have { }
1 1 1 1
r1 + r2 + r3 − r = S + + −
{s − a s − b s − c } s
2s − a − b c
=S +
{(s − a)(s − b) s(s − c) }
1 1
= cS +
(s − a)(s − b) s(s − c)
cS c
= 2 {s(s − c) + (s − a)(s − b)} = {2s2 − s(a + b + c) + ab}
S S
abc
= = 4R.
S

1 π
Problem 202. Show that sin−1 √ + cot−1 3 = .
5 4
1 1 1
Solution. sin−1 √ = tan−1 , cot−1 3 = tan−1 ;
5 2 3
1 1
1 1 +
tan−1 + tan−1 = tan−1 2 3 = tan−1 1 = π .
2 3 1 4
1−
6

Problem 203. ABC is a triangle; a second triangle is formed by the external bi-
sectors of the angles of ABC ; then a third triangle is in like manner formed from the
second, and so on. Determine the angles of the n th triangle.
Solution. The angle of the second triangle which is opposite to the angle C of the
π C
first triangle will be found to be − ; Similarly the corresponding angle of the third
( )2 2
π 1 π C π π C
triangle will be − − , that is − + . Proceeding in this way we find
2 2 2 2 2 4 4
that the corresponding angle of the nth triangle is
CHAPTER XXV : Miscellaneous Examples 284

{ }
1 1 1 (−1)n−1 (−1)n−1 C
π − + − ... − + ,
2 4 8 2n−1 2n−1
( )
1 n−1
1− −
π2 (−1)n−1 C
that is + ;
2 1 2n−1
1+
{ 2 }
π (−1)n−1 (−1)n−1 C
that is 1− + .
3 2n−1 2n−1
Similar expressions hold for the other angles.

( )
Problem 204. Find in terms of a the value of cos 4 tan−1 a .
Solution. Suppose θ = tan−1 a; then we require cos 4θ.
1 − tan2 θ 1 − a2
Now tan θ = a, cos 2θ = 2
= ,
1 + tan θ 1 + a2
2(1 − a )
2 2 1 − 6a2 + a4
cos 4θ = 2 cos2 2θ − 1 = 2 2
−1= .
(1 + a ) (1 + a2 )2

Problem 205. Find the general term in the expansion of e ax cos(bx + c) in powers of
x.
Solution. We have eax cos(bx + c) = eax (cos bx cos c − sin bx sin c);
then by Art. 300 (page 352) the required general term is
( ) n2
a2 + b2
(cos nθ cos c − sin nθ sin c),
n
( ) n2
a2 + b2
that is cos(nθ + c),
n
b
where θ is such that tan θ = .
a

Problem 206. A circle touches the sides AB and AC of a triangle produced, and
( that )
touches the side BC at D. Show
a s2 − AD 2 = 4s(s − b)(s − c).
Solution.
We have AD2 = AB 2 + BD2 − 2AB · BD cos B
by Art. 250 (page 344) :
= c2 + (s − c)2 − 2c(s − c) cos B,
therefore s − AD = s2 − c2 − (s − c)2 + 2c(s − c) cos B
2 2

= (s − c){s + c − (s − c) + 2c cos B}
B
= 2c(s − c)(1 + cos B) = 4c(s − c) cos2 ;
2
B
therefore a(s2 − AD2 ) = 4ac(s − c) cos2 = 4s(s − b)(s − c).
2
( √ ) √
Problem 207. If cos−1 α + β −1 = θ + ϕ −1 , where the letters denote real quan-
tities, show that
α2 β2 α2 β2 1
− = 1 , and ( )2 + ( )2 = 4 .
cos2 θ sin2 θ e ϕ + e −ϕ e ϕ − e −ϕ
Solution. We have√ ( √ ) √ √
α + β −1 = cos θ + ϕ −1 = cos θ cos ϕ −1 − sin θ sin ϕ −1
e−ϕ + eϕ e−ϕ − eϕ
= cos θ − sin θ √
2 2 −1
CHAPTER XXV : Miscellaneous Examples 285

eϕ + e−ϕ eϕ − e−ϕ √
= cos θ − sin θ −1.
2 2
Hence by equating the possible and the impossible parts we have
eϕ + e−ϕ eϕ − e−ϕ
α = cos θ , β = − sin θ .
(2 )2 ( 2 )2
α 2 β 2 ϕ
e +e −ϕ e −e
ϕ −ϕ
Therefore − = − = 1;
cos2 θ sin2 θ 2 2
α2 β2 cos2 θ + sin2 θ 1
and −ϕ
+ ϕ −ϕ
= = .
(eϕ +e ) 2 (e − e ) 2 4 4

Problem 208. Show { that log sec θ }


1 1 1
= 2 sin2 θ − sin2 2 θ + sin2 3 θ − sin2 4 θ + . . . .
2 3 4
Solution.
1 1 1 2
log sec θ = log = log
2 cos2 θ 2 1 + cos 2θ
1 4 1 4
= log = log
2 2 + e2ιθ + e−2ιθ 2 (1 + e2ιθ )(1 + e−2ιθ )
1{ ( ) ( )}
= 2 log 2 − log 1 + e2ιθ − log 1 + e−2ιθ ;
2( )
1 1 1
∴ 2 log sec θ = 2 1 − + − + . . .
( 2 3 4 )
1 1 1
− e2ιθ − e4ιθ + e6ιθ − e8ιθ + . . .
( 2 3 4 )
1 1 1
− e−2ιθ − e−4ιθ + e−6ιθ − e−8ιθ + . . . .
2 3 4
Now 2 − e2ιθ − e−2ιθ = −(eιθ − e−ιθ )2 = 4 sin2 θ;
1( ) 1 ( 2ιθ )2 4
−2 + e4ιθ + e−4ιθ = e − e−2ιθ = − sin2 2θ,
2 2 2
1 1 4
(2 − e6ιθ − e−6ιθ ) = − (e3ιθ − e−3ιθ )2 = sin2 3θ,
3 3 3
and so on; thus
{ }
1 1
2 log sec θ = 4 sin2 θ −sin2 2θ + sin2 3θ − . . . ;
{ 2 3 }
1 1
therefore log sec θ = 2 sin2 θ − sin2 2θ + sin2 3θ − . . . .
2 3

Problem 209. Show that the sum of n terms of the series


sec α sec(α + β) + sec(α + β) sec(α + 2β) + sec(α + 2β) sec(α + 3β) + . . .
= cosec β {tan(α + nβ) − tan α} .

1
Solution. sec α sec(α + β) = {tan(α + β) − tan α} ,
sin β
1
sec(α + β) sec(α + 2β) = {tan(α + 2β) − tan(α + β)} ,
sin β
and so on.
Then by addition we obtain the required result.

Problem 210. In a regular hexagon, one of whose sides is equal to a, a circle is


inscribed; and in this circle another regular hexagon, and so on until there are in all
n hexagons. Show that the sum of the areas of the hexagons is
√ { ( )n }
3
6 3 1− a2 .
4
Solution. The regular hexagon may √be divided into six equilateral triangles; and thus
6a2 3
the area of the first hexagon = .
4
CHAPTER XXV : Miscellaneous Examples 286


a a 3
By Art. 255 (page 347) the radius of the first circle = cot 300 = ; and the
2 2
side of the second hexagon is equal to this, so that the area of the second hexagon =
√ ( √ )2
6a2 3 3
. In this way we see that the areas of the hexagons form a geometrical
4 2
( )n
2
√ 1− 3
3 6a 3 4
progression of which the ratio is ; and the sum of the areas = =
4 4 3
1−
√ { ( )n } 4
3
6a2 3 1 − .
4

Problem 211. Adapt the expression a cos A + b cos B + c cos C to logarithmic compu-
tation, the letters denoting the sides and the angles of a triangle.
Solution. We have a = 2R sin A, b = 2R sin B, c = 2R sin C; thus the proposed
expression
= 2R(sin A cos A + sin B cos B + sin C cos C)
= R(sin 2A + sin 2B + sin 2C)
= 4R sin A sin B sin C, by Art. 114 (page 328),
= 2a sin B sin C.
The expression is now adapted to logarithms.

Problem 212. A, B, C are telegraph posts at equal intervals by the side of a rail-
road; t and t ′ are the tangents of the angles which AB and BC subtend at any point
P; T is the tangent of the angle which the road makes with PB. Show that
2 1 1
= ′ − .
T t t
Solution. Let θ denote the angle AP B, ϕ the angle BP C, and ψ the angle ABP.
AB sin θ BC sin ϕ
We have = , = ;
PB sin(ψ + θ) PB sin(ψ − ϕ)
but AB is supposed equal to BC, and thus
sin(ψ + θ) sin(ψ − ϕ)
= ;
sin θ sin ϕ
therefore sin ψ cot θ + cos ψ = sin ψ cot ϕ − cos ψ;
therefore 2 cot ψ = cot ϕ − cot θ;
2 1 1
therefore = ′ − .
T t t

Problem 213. Show that


π 1 1 1
= 4 tan−1 − 2 tan−1 + tan−1 .
4 5 408 1393
π 1 1
Solution. It is shown in Art. 296 (page 351) that = 4 tan−1 − tan−1 ;
4 5 239
hence we have only to show that
1 1 1
tan−1 = 2 tan−1 − tan−1 ,
239 408 1393
1 1 1
or that tan−1 + tan−1 = 2 tan−1 .
239 1393 408
1 1
+
−1 1 −1 1 −1 239 1393
Now tan + tan = tan
239 1393 1
1−
239 × 1393
1393 + 239 1632 816
= tan−1 = tan−1 = tan−1 ;
239 × 1393 − 1 332926 166463
CHAPTER XXV : Miscellaneous Examples 287

2
−1 1 2 × 408
and 2 tan = tan−1 408
( ) = tan−1 166463 .
408 1 2
1−
408
Thus the required result is established.

Problem 214. If P denote the point of intersection of the perpendiculars from the
angles of a triangle on the opposite sides, show that PA2 = 4R2 − a 2 .
Solution. By the diagram of Art. 332 (page ( 362) we)see that
π
sin −A
PA sin P BA 2 cos A
= = = ;
AB sin AP B sin(A + B) sin C
c cos A a cos A
therefore PA = = ;
( )C
sin sin A
a 1 − sin A
2 2
a2
therefore P A2 = = − a2 = 4R2 − a2 .
sin2 A sin2 A

Problem 215. If α =(15 ◦ , find the value ) (of )


√ √
cos α + −1 sin α cos 2 α + −1 sin 2 α
√ .
√ cos 3 α√− −1 sin 3 α √
(cos α + −1 sin α)(cos 2α + −1 sin 2α) cos 3α + −1 sin 3α
Solution. √ = √ ;
cos 3α − −1 sin 3α √ 3α − −1 sin 3α
cos
multiply both numerator and denominator
√ by cos 3α + −1 sin 3α; thus we obtain unity
the denominator, and cos 6α + −1 sin 6α in the numerator: and this numerator
in √
= −1 since α = 15◦ .

Problem 216. ABC is a triangle; a new triangle is formed by the external bisectors
A B
of the angles. Show that the sides of the new triangle are 4R cos , 4R cos , and
2 2
C
4R cos respectively.
2
Solution. The new triangle will have for its angular points the centres of the escribed
circles of the original triangle. Now from Art. 250 (page 344) we have
C
OC = CE sec OCE = (s − b) cosec ;
2
and in the same manner the distance from C of the centre of the circle which touches
C
BC and BA produced = (s − a) cosec . Hence the sum of these two = (2s − b −
2
C C C C
a) cosec = c cosec = 2R sin C cosec = 4R cos .
2 2 2 2
This is the length of the side of the second triangle which passes through the point
C; similar expressions hold for the other two sides.

Problem 217. Show that the sum of the squares on the sides of the triangle formed
as in the preceding Problem = 8R(4R + r).
Solution. By the preceding
{ Example the sum of the}squares
2 2 1 1 1
= 16R cos A + cos2 B + cos2 C
2 2 2
= 8R2 {3 + cos A + cos B + cos C}
{ }
A B C
= 8R2 4 + 4 sin sin sin , by Art. 114(page 328)
2 2 2
32R S2 2
= 32R2 + = 32R2 + 8Rr.
sabc

Problem 218. Reduce to its simplest form


cos 6 θ + 6 cos 4 θ + 15 cos 2 θ + 10
.
cos 5 θ + 5 cos 3 θ + 10 cos θ
CHAPTER XXV : Miscellaneous Examples 288

Solution. The numerator can be expressed in powers of θ; and it will be found to


reduce to 25 cos6 θ; in like manner the denominator will be found to reduce to 24 cos5 θ :
see Art. 280 (page 349). Hence the expression reduces to 2 cos θ.

π √
Problem 219. If θ be a positive angle less than , show that cos θ is less than
2
θ
cos √ .
2 ( )2
θ2 θ4 θ2 √
Solution. cos θ is less than 1 − + , that is less than 1 − , therefore cosθ is
2 16 4
θ2 π
less than 1 − ; this holds if θ lies between 0 and : see Art. 328 (page 361). Again,
4 ( )2 2
θ 1 θ θ2
cos √ is greater than 1 − √ , that is greater than 1 − ; this holds as long as
2 2 2 4
θ θ2
cos √ and 1 − remain both positive, and this certainly holds if θ lies between 0
2 4
π θ
and . Hence cos θ is less than cos √ .
2 2

Problem 220. If any point be taken within a regular polygon of an even number of
sides from which perpendiculars are drawn to the sides taken in order, then the sum
of one set of alternate perpendiculars is equal to the sum of the other set.
Solution. Suppose the polygon has n sides. Let O be the centre of the circle inscribed
in the polygon, and S the assumed point. Let OS = c; and suppose OS to be inclined at

an angle α to the first perpendicular which is drawn; put β for , and r for the radius
n
of the inscribed circle. Then the length of the first perpendicular will be r + c cos α,
that of the second r + c cos(α + β), that of the third r + c cos(α + 2β), and so on. Hence
the sum of one set of{perpendiculars }
nr n
= + c cos α + cos(α + 2β) + cos(α + 4β) + . . . to terms .
2 2
n
By Art. 304 (page 353) the sum of the series of cosines contains the factor sin β,
2
that is sin π, that is zero.
nr
Hence the sum of the set of perpendiculars = .
2
Similarly the sum of the other set of perpendiculars has the same value.


Problem 221. Show that rr1 r2 r3 = S.

S S S S
Solution. r= , r1 = , r2 = , ;
s s−a s−b s−c
S 4 S 4
therefore rr1 r2 r3 = = 2 = S2;
s(s − a)(s − b)(s − c) S

therefore rr1 r2 r3 = S.

π 1 3
Problem 222. Show that = 5 tan−1 + 2 tan−1 .
4 7 79
2
1 7 7
Solution. We have 2 tan−1 = tan−1 = tan−1 ;
7 1 24
1−
49
7
2 × 2 × 7 × 24
−1 1 −1 7
thus 4 tan
7
= 2 tan
24
= ( )2 = tan−1 (24 + 7)(24 − 7)
24
7
1−
24
336
= tan−1 .
527
CHAPTER XXV : Miscellaneous Examples 289

1 1 1 336 1
Then 5 tan−1 = 4 tan−1 + tan−1 = tan−1 + tan−1
7 7 7 527 7
336 1
+
= tan−1 527 7 = tan−1 2879 .
336 3353
1−
7 × 527
3
2× 2 × 3 × 79
−1 3 −1
Again 2 tan
79
= tan ( )2 = tan−1 (79 + 3)(79 − 3)
79
3
1−
79
−1 237
= tan .
3116
( ) 1 − 2879
π −1 2879 3353 = 474 = 237 ;
Finally tan − tan =
4 3353 2879 6232 3116
1+
3353
π 2879 237
so that − tan−1 = tan−1 .
4 3353 3116

Problem 223. Assuming the expression for tan nθ in terms of tan θ, show that if n
be an odd integer the following two series are numerically equal,
n(n − 1) n(n − 1)(n − 2)(n − 3)
1− + − ...,
2 4
n(n − 1)(n − 2) n(n − 1)(n − 2)(n − 3)(n − 4)
n− + − ...;
3 5
and if n be an even integer one of the two series is zero.
π
Solution. In the expression for tan nθ put for θ; then θ = 1.
4
n−1
If n is an odd number we have tan nθ = (−1) 2 , so that the numerator of the
expression is numerically equal to the denominator.
If n is an even number, tan θ is either zero or infinite; so that in the former case
the numerator of the expression must vanish must vanish, and in the latter case the
denominator must vanish.

Problem 224. Show that sin4 θ cos5 θ


1 1 3
= (cos 9 θ + cos 7 θ) − (cos 5 θ + cos 3 θ) + cos θ.
256 64 128
Solution. We have sin4 θ cos5 θ = (1 − cos2 θ)2 cos5 θ = cos9 θ − 2 cos7 θ + cos5 θ.
1
Now cos9 θ = 8 {cos 9θ + 9 cos 7θ + 36 cos 5θ + 84 cos 3θ + 126 cos θ},
2
1
cos7 θ = 6 {cos 7θ + 7 cos 5θ + 21 cos 3θ + 35 cos θ},
2
1
cos5 θ = 4 {cos 5θ + 5 cos 3θ + 10 cos θ}.
2
Hence
1 1 3
cos9 θ − 2 cos7 θ + cos5 θ = (cos 9θ + cos 7θ) − (cos 5θ + cos 3θ) + cos θ.
256 64 128
Or we may proceed thus :
1
sin4 θ cos5 θ = sin4 θ cos4 θ sin θ = (sin 2θ)4 cos θ
{ 16 }
1 1 1 3
= cos 8θ − cos 4θ + cos θ
16 8 2 8
1 1 3
= (cos 9θ + cos 7θ) − (cos 5θ + cos 3θ) + cos θ.
256 64 128
CHAPTER XXV : Miscellaneous Examples 290

Problem 225.{ Show that sin3 x }


3 32 − 1 3 34 − 1 5 3 2n − 1 2n+1
= x − x + . . . + (−1 )n−1 x + ... .
4 3 5 2n + 1
Solution. We have
1
sin3 x = (3 sin x − sin 3x)
4{ }
3 x3 x5 x7 (−1)n x2n+1
= x− + − + ... + + ...
4 3 5 7 2n + 1
{ }
1 (3x)3 (3x)5 (3x)7 (−1)n (3x)2n+1
− 3x − + − + ... + + ... ;
4 3 5 7 2n + 1
then by arranging according to powers of x we obtain the required result.

π
Problem 226. If ϕ = , show that
13 √
1+ 13
cos ϕ + cos 3ϕ + cos 9ϕ = ,
4√
131−
and cos 5ϕ + cos 7ϕ + cos 11ϕ = .
4
Solution. Put s for cos ϕ + cos 3ϕ + cos 9ϕ, and t for cos 5ϕ + cos 7ϕ + cos 11ϕ.
Then s + t = cos ϕ + cos 3ϕ + cos 5ϕ + cos 7ϕ + cos 9ϕ + cos 11ϕ
cos(ϕ + 5ϕ) sin 6ϕ sin 12ϕ sin ϕ 1
= (Art. 304) (page 353) = = = .
sin ϕ 2 sin ϕ 2 sin ϕ 2
And st = (cos ϕ + cos 3ϕ + cos 9ϕ)(cos 5ϕ + cos 7ϕ + cos 11ϕ)
= cos ϕ(cos 5ϕ + cos 7ϕ + cos 11ϕ) + . . .
Resolve each product into the sum of two cosines by Art. 84 (page 325); thus we
get
2st = cos 6ϕ + cos 4ϕ + cos 8ϕ + cos 6ϕ + cos 12ϕ + cos 10ϕ
+ cos 8ϕ + cos 2ϕ + cos 10ϕ + cos 4ϕ + cos 14ϕ + cos 8ϕ
+ cos 14ϕ + cos 4ϕ + cos 16ϕ + cos 2ϕ + cos 20ϕ + cos 2ϕ
= 3 cos 2ϕ + 3 cos 4ϕ + 2 cos 6ϕ + 3 cos 8ϕ + 2 cos 10ϕ
+ cos 12ϕ + 2 cos 14ϕ + cos 16ϕ + cos 20ϕ.
π
Now since ϕ = we have cos 14ϕ = cos 12ϕ, cos 20ϕ = − cos 7ϕ = cos 6ϕ, cos 16ϕ =
13
− cos 3ϕ = cos 10ϕ. Thus
2st = 3{cos 2ϕ + cos 4ϕ + cos 6ϕ + cos 8ϕ + cos 10ϕ + cos 12ϕ}
3 cos(2ϕ + 5ϕ) sin 6ϕ 3 cos 6ϕ sin 6ϕ 3 sin 12ϕ 3
= =− =− =− ;
sin ϕ sin ϕ 2 sin ϕ 2
3
therefore st = − .
4
1 3
Then, since s + t = , and st = − , we find by Algebra that
2 4
√ √
1 ± 13 1 ∓ 13
s= and t = ;
4 4
and it is obvious that the upper sign must be taken, because s is positive for cos ϕ and
cos 3ϕ, which are positive, are both numerically greater than cos 9ϕ, which is negative.

Problem 227. A point is taken inside a regular polygon and perpendiculars are
drawn from it to the sides of the polygon; a new polygon is formed by joining the
successive feet of the perpendiculars : find the sum of the squares on the sides of the
new polygon.
Solution. Suppose the polygon has n sides. Let O be the centre of the circle inscribed
in the polygon, and S the assumed point. Let OS = c; and suppose OS to be inclined at

an angle α to the first perpendicular which is drawn; put β for , and r for the radius
n
of the inscribed circle. Then the length of the first perpendicular will be r + c cos α,
CHAPTER XXV : Miscellaneous Examples 291

that of the second r + c cos(α + β), that of the third r + c cos(α + 2β), and so on.
Then for the squares on the sides of the new polygon we obtain the expressions
{r + c cos α}2 + {r + c cos(α + β)}2
− 2{r + c cos α}{r + c cos(α + β)} cos β,
{r + c cos(α + β)}2 + {r + c cos(α + 2β)}2
− 2{r + c cos(α + β)}{r + c cos(α + 2β)} cos β,
and so on.
Thus for the squares on the mth side of the new polygon we shall obtain
2r2 (1 − cos β) + 2rc{cos(α + mβ − β) + cos(α + mβ)}(1 − cos β)
+c2 {cos2 (α + mβ − β) + cos2 (α + mβ) − 2 cos β cos(α + mβ − β) cos(α + mβ)};
that is
2r2 (1 − cos β) + 2rc{cos(α + mβ − β) + cos(α + mβ)}(1 − cos β)
c2
+ {1 + cos(2α + 2mβ − 2β) + 1 + cos(2α + 2mβ)
2
−2 cos β[cos β + cos(2α + 2mβ − β)]}.
We have to obtain the sum formed from this expression by giving to m all integral
values from 1 to n, both inclusive; the result, by Art. 305 (page 353),
c2
= 2nr2 (1 − cos β) + {2n − 2n cos2 β}
2
β
= 4nr2 sin2 + nc2 sin2 β.
2

π
Problem 228. Show that if β = then cos 5 θ + sin 5 θ
20
= −2 sin(θ − 3 β) sin(θ + β) cos(θ + 3 β) cos(θ − β)(cos θ + sin θ).
4

√ ( )
π
Solution. We have cos 5θ + sin 5θ = 2 cos 5θ −
√ ( ) √4
π
= 2 cos 5 θ − = 2 cos 5(θ − β).
20

And by Art. 318 (page 358) we have cos 5(θ − β) sin
2
= 24 cos(θ − β) cos(θ − β + 2α) cos(θ − β + 4α) cos(θ − β + 6α) cos(θ − β + 8α),
π
where α = = 2β.
10
Thus cos 5(θ − β)
= 24 cos(θ − β) cos(θ + 3β) cos(θ + 7β) cos(θ + 11β) cos(θ + 15β).
Also cos(θ + 7β) = sin(3β − θ) = − sin(θ − 3β),
( )
π
cos(θ + 11β) = cos θ + β + = − sin(θ + β),
( ) 2
3π 1
cos(θ + 15β) = cos θ + = − √ (cos θ + sin θ).
4 2

Hence 2 cos 5(θ − β)

= −24 cos(θ − β) cos(θ + 3β) sin(θ − 3β) sin(θ + β)(sin θ + cos θ);
hence also cos 5θ + sin 5θ is equal to the last expression, which had to be shown.

{ } 12 { } 12
Problem 229. Given sin θ 1 + tan2 α tan2 β + cos θ 1 − tan2 α tan2 β
= tan α + tan β,
show how to determine θ by a formula suitable to logarithmic computation.
Solution. We√have √
sin θ (cos2 α cos2 β + sin2 α sin2 β) + cos θ (cos2 α cos2 β − sin2 α sin2 β)
= sin(α + β).

Assume r cos ϕ = (cos2 α cos2 β + sin2 α sin2 β),

and r sin ϕ = (cos2 α cos2 β − sin2 α sin2 β);
so that r = 2 cos2 α cos2 β
2
(39)
CHAPTER XXV : Miscellaneous Examples 292

cos2 α cos2 β − sin2 α sin2 β


and tan2 ϕ = (40)
cos2 α cos2 β + sin2 α sin2 β
Thus r sin(θ + ϕ) = sin(α + β) (41)
Now it is obvious that r may be found from (39) by logarithms. Also ϕ may be
determined by logarithms; for we have from (40)
1 − tan2 ϕ sin2 α sin2 β
= ,
1 + tan2 ϕ cos2 α cos2 β
2 2
that is cos 2ϕ = tan α tan β,
which is adapted to logarithms.
Thus θ can be found from (41) by logarithms.

Problem 230. If A, B, C are angles of a triangle, show that


sin A + sin B + sin C is never less than sin 2A + sin 2B + sin 2C .
Solution. If A, B and C are angles of a triangle, we have by Art. 114, (page 328) and
Chapter viii : P roblem 16,
sin A + sin B + sin C − (sin 2A + sin 2B + sin 2C)
A B C
= 4 cos cos cos − 4 sin A sin B sin C
2 2 2 { }
A B C A B C
= 4 cos cos cos 1 − 8 sin sin sin ;
2 2 2 2 2 2
and by Chapter xiii : P roblem 40 this expression can never be negative.

Problem 231. A regular polygon of n sides is inscribed in a circle, and from any
point in the circumference chords are drawn to the angular points; if these chords be
denoted by c1 , c2 , . . . cn , beginning with the chord drawn to the nearest angular point,
and taking the rest in order, show that the quantity
c1 c2 + c2 c3 + . . . + cn−1 cn − cn c1
is independent of the position of the point from which the chords are drawn.
Solution. Let A, B, C, . . . M, N denote the angular points of the polygon taken in
π
order; and let α = . Suppose P the point in the circumstance from which chords are
n
drawn, so that P is between N and A.
1
Then c1 c2 sin α = the area of the triangle P AB,
2
1
c2 c3 sin α = the area of the triangle P BC,
2
....................................
1
cn−1 cn sin α = the area of the triangle P M N.
2
1
Therefore (c1 c2 + c2 c3 + . . . + cn−1 cn ) sin α.
2
= the area of the triangles P AB, P BC, . . . P M N.
1
Also cn c1 sin α = the area of the triangle P N A.
2
1
Thus (c1 c2 + c2 c3 + . . . + cn−1 cn − cn c1 ) sin α
2
= the area of the regular polygon;
so that c1 c2 + c2 c3 + . . . + cn−1 cn − cn c1
2
= × the area of the regular polygon.
sin α
This result is the same for all positions of P on the circumstance of the circle.

Problem 232. Two circular sectors have a common chord and equal areas, and
their angles are as 2 is to 1 : show that one of the sectors must be a semicircle and
the other a quadrant.
Solution. Let θ be the angle of one sector, and 2θ the angle of the other. Let a and
θ 2θ
b be the corresponding radii. Then, since the areas are equal, a2 = b2 ; and since
2 2
CHAPTER XXV : Miscellaneous Examples 293

θ 2θ
there is a common chord, 2a sin = 2b sin .
2 2
θ θ θ θ a
Thus a sin = b sin θ = 2b sin cos ; therefore cos = ;
2 2 2 2 2b
θ a2 2b2 1 θ π
therefore cos2 = 2 = 2 = ; therefore = .
2 4b 4b 2 2 4
π
Therefore θ= and 2θ = π.
2


x 3 2x − k
Problem 233. If ϕ = tan−1 , and θ = tan−1 √ , show that one value of
2k − x k 3
π
ϕ − θ is .
6 √
x 3 2x − k
− √
tan ϕ − tan θ 2k − x k 3
Solution. We have tan(ϕ − θ) = = √
1 + tan ϕ tan θ x(2x − k) 3
1+ √
(2k − x)k 3
1 3kx − (2k − x)(2x − k) 1 2x2 − 2kx + 2k2 1
= √ · = √ . 2 = √ .
3 (2k − x)k + x(2x − k) 3 2x − 2kx + 2k2 3
π
Therefore one value of ϕ − θ is .
6

sin θ 863
Problem 234. If = , show that θ contains very nearly 5 ◦ .
θ 864
sin θ
Solution. We have here very nearly equal to unity; so we may infer that θ is
θ
θ3 θ2 863 θ2 1
small : hence sin θ = θ − nearly. Therefore 1 − = nearly; therefore =
6 6 864 6 864
1 1
nearly; therefore θ2 = nearly; therefore θ = nearly. Hence the number of
144 12
1 180
degrees in the angle is nearly · , so that is about 5.
12 π

Problem 235. ABC is a triangle, and DEF is another triangle formed by joining the
centres of the escribed circles of ABC . Show that the circle described round ABC is
the nine points circle of DEF.
Solution. Let ABC be any triangle; let D, E, F be the centers of the escrowed circles
opposite to A, B, C respectively.
Then AD bisected the angle of the triangle at A, and EF bisects the exterior angle
at A. Therefore AD is perpendicular to EF.
Similarly EB is perpendicular to F D, and F C is perpendicular to DE.
Therefore by Art. 332 (page 362) the circle described round ABC is the nine points
circle of DEF.

Problem 236. From the expansion of sin2n+1 θ in terms of the sines of the multiples
of θ, show that zero is the sum of n + 1 terms of the following series :
2n(2n − 3 ) 2n(2n − 1 )(2n − 5 )
1 − (2n − 1 ) + − + ....
2 3
Solution. As in Art. 283 (page 350) we have

22n (−1)n sin2n+1 θ


(2n + 1)2n
= sin(2n + 1)θ − (2n + 1) sin(2n − 1)θ + sin(2n − 3)θ − . . . .
2
Now suppose each side were to be expanded in powers of θ; on the left-hand side
{ }2n+1
θ3
we should have 22n (−1)n θ− + ... , by Art. 274 (page 348).
3
CHAPTER XXV : Miscellaneous Examples 294

On the right-hand side each sine gives rise to a series. Since the lowest power
of θ on the left-hand side is θ2n+1 it follows that the whole coefficient of every lower
power of θ on the right-hand side must be zero. The whole coefficient of θ is
(2n + 1)2n
2n + 1 − (2n + 1)(2n − 1) + (2n − 3) − . . . to n + 1 terms;
2
hence this is zero; and dividing by 2n + 1 we obtain the required result.
Similarly, supposing n to be greater than unity, we can obtain another result by
equating to zero the whole coefficient of θ3 on the right-hand side. And so on.

( √ ) √
Problem 237. If cos θ + ϕ −1 = cos α + −1 sin α, where the letters denote real
quantities, show that sin θ = ± sin α.
2

√ We have (
Solution. √ ) √ √
cos α + −1 sin α = cos θ + ϕ −1 = cos θ cos ϕ −1 − sin θ sin ϕ −1.
e−ϕ + eϕ e−ϕ − eϕ e−ϕ + eϕ e−ϕ − eϕ √
= cos θ − sin θ √ = cos θ + sin θ −1.
2 2 −1 2 2
Hence, by equating the possible and the impossible parts, we have
e−ϕ + eϕ e−ϕ − eϕ
cos θ = cos α, sin θ = sin α;
2 2
e−ϕ +e ϕ cos α e −ϕ −e ϕ sin α
so that = , = .
2 cos θ 2 sin θ
Square and subtract; thus
cos2 α sin2 α
1= − ;
cos2 θ sin2 θ
therefore sin θ cos α − cos θ sin α = sin2 θ cos2 θ;
2 2 2 2

therefore sin2 θ(1 − cos2 θ) = sin2 α;


therefore sin4 θ = sin2 α;
therefore sin2 θ = ± sin α.

Problem 238. Show that ( )


sin x − sin 3x + sin 5x − . . . to n terms n−1
= tan nx + π .
cos x − cos 3x + cos 5x − . . . to n terms 2
Solution. On the left-hand side the numerator
= sin x + sin(3x + π) + sin(5x + 2π) + . . . to n terms,
{ }
n−1 n
sin x + (2x + π) sin (2x + π)
2 2
= ;
1
sin (2x + π)
2
in like manner the denominator { }
n−1 n
cos x + (2x + π) sin (2x + π)
2 2
=
1
sin (2x + π)
2
Divide the former{by the latter and } we obtain ( )
n−1 n−1
tan x + (2x + π) , that is tan nx + π .
2 2

Problem 239. ABCP and DEFQ are two concentric circles, ABC and DEF being
any two equilateral triangles inscribed in them. If P and Q are any two points on the
circumferences of the circles, show that
QA2 + QB 2 + QC 2 = PD 2 + PE 2 + PF 2 .
Solution. Let O denote the centre of the circles, r the radius of the circle ABCP, and
R the radius of the circle DEF Q.

Suppose the angle QOA is equal to θ, then the angle QOB will be θ + , and
3

the angle QOC will be θ + ; or at least the angles may be so denoted by suitably
3
CHAPTER XXV : Miscellaneous Examples 295

choosing A, B, and C. Then


QA2 = QO2 + OA2 − 2QO · OA cos θ
= R2 + r2 − 2Rr cos θ,
( )

similarly QB 2 = R2 + r2 − 2Rr cos θ + ,
( 3 )

and QC 2 = R2 + r2 − 2Rr cos θ + .
3
Hence by addition, and Art. 305 (page 353), we have
QA2 + QB 2 + QC 2 = 3(R2 + r2 ).
In the same way we find that
P D 2 + P E 2 + P F 2 = 3(R2 + r2 ).

a sin cx
Problem 240. If tan θ = and r 2 = 1 − 2a cos cx + a 2 , show that
1 − a cos cx
n n(n − 1 ) 2
1 − a cos cx + a cos 2cx − . . . + (−1 )n a n cos ncx = r n cosnθ.
1 2
Solution. Put for each cosine its exponential value; then the proposed series
1 1
= (1 − aeιcx )n + (1 − ae−ιcx )n
2 2
1 1
= (1 − a cos cx − ιa sin cx)n + (1 − a cos cx + ιa sin cx)n .
2 2
Now assume 1 − a cos cx = r cos θ and a sin cx = r sin θ;
1 1
then the sum = (r cos θ − ιr sin θ)n + (r cos θ + ιr sin θ)n
2 2
rn rn
= (cos nθ − ι sin nθ) + (cos nθ + ι sin nθ)
2 2
= rn cos nθ.

Problem 241. Eliminate θ between


sin2 θ − p sin θ + 1 = 0 and cos2 θ − q cos θ + 1 = 0 .
Solution. By addition 3 − p sin θ − q cos θ = 0.
By subtraction cos2 θ − sin2 θ = −p sin θ + q cos θ.
Therefore 3(cos2 θ − sin2 θ) = q 2 cos2 θ − p2 sin2 θ;
therefore 3(2 cos2 θ − 1) = q 2 cos2 θ − p2 + p2 cos2 θ;
p2 − 3
therefore cos2 θ = 2 ;
p + q2 − 6
q2 − 3
therefore sin2 θ = 2 .
p + q2 − 6
Substitute in the equation 3 = p sin θ + q cos θ; thus
√ √ √
3 (p2 + q 2 − 6) = p (q 2 − 3) + q (p2 − 3).
This is the result of the elimination; the radicals are not necessarily positive. By
squaring, transposing, and squaring again, we obtain finally
{ }2
p2 q 2 − 6(p2 + q 2 ) + 27 = p2 q 2 (p2 − 3)(q 2 − 3).

Problem 242. Show that the area of a triangle


(a + b + c)2
= ( ).
A B C
4 cot + cot + cot
2 2 2
A B C
Solution. cot + cot + cot
2 √ 2 2 √ √
s(s − a) s(s − b) s(s − c)
= + +
(s − b)(s − c) (s − a)(s − c) (s − a)(s − b)
CHAPTER XXV : Miscellaneous Examples 296


s s2
= √ {s − a + s − b + s − c} = .
(s − a)(s − b)(s − c) S
s2
Hence the proposed expression = s2 ÷ = S.
S

1 π
Problem 243. If tan−1 ax + sec−1 bx = , then one solution is
2 4
1
x2 = .
2ab − a 2
π
Solution. Here 2 tan−1 ax + sec−1 bx = ;
2
2ax π −1 1 1
therefore sin−1 = − cos = sin−1 ;
1 + a2 x2 2 bx bx
2ax 1
therefore = ; therefore 2abx2 = 1 + a2 x2 ;
1 + a2 x2 bx
1
therefore x2 = .
2ab − a2

Problem 244. If O is the centre of the circle described round a triangle, and P
the point of intersection of the perpendiculars from the angles on the opposite sides,
show that ( )
3
OP 2 = 2R2 + cos 2A + cos 2B + cos 2C .
2
Solution. With the diagram of Art. 332 (page 362) we have OA = R, also the angle
π π
OAB = − C, and the angle BAP = − B; so that the angle OAP = C − B.
2 2
Hence OP 2 = R2 + AP 2 − 2R · AP cos(B − C).
a cos A
Now, as in P roblem 214, we have AP = = 2R cos A;
sin A
so that OP = R + 4R cos A − 4R cos A cos(B − C)
2 2 2 2 2

= R2 + 2R2 (1 + cos 2A) + 4R2 cos(B + C) cos(B − C)


= 3R2 + 2R2 cos 2A + 2R2 (cos2B + cos 2C)
= 3R2 + 2R2 (cos 2A + cos 2B + cos 2C).

Problem 245. If α, β, γ are the lengths of the straight lines joining the centres of
the escribed circles of a triangle with the centre of the inscribed circle, and x, y, z
the lengths of the straight lines joining the centres of the escribed circles, show that
βz + γy γx + αz αy + βx
= = .
x y z
Solution. The values of x, y, z are given in P roblem 216; and the values of α, β, γ in
Example xvi. 31. Hence
1 1 1 1
βz + γy 4R cos Cb sec B + 4R cos Bc sec C
= 2 2 2 2
x 1
4R cos A
( 2 )
1 1 1 1 1
4R cos C sin B + cos B sin C 4R sin (B + C)
2 2 2 2 2
= =
1 1
cos A cos A
2 2
= 4R.
Similarly the other expressions are also equal to 4R.

Problem 246. If π − θ denote the angle opposite to the side b of a triangle, and θ be
very small, show that approximately
{ ( ) }
a θ2 a 3a 2 3a 3 θ4
c = (b − a) 1+ − − 2 − 3 .
b 2 b b b 4
CHAPTER XXV : Miscellaneous Examples 297

Solution.
We have c = b cos A + a cos B = b cos A − a cos(π − B)
√( )
√ a2
=b (1 − sin2 A) − a cos θ = b 1 − 2 sin2 θ − a cos θ.
b
We wish to expand this in powers of θ, as far as terms involving θ4 .
√( )
a2 a2 a4
Now 1− sin2 θ =1− sin2 θ − 4 sin4 θ − . . .
b2 2b 2 8b
θ3
Put for sin θ its value θ − + . . . ; thus we obtain
6( )2 ( )4
a 2 θ3 a4 θ3
1− 2 θ− + ... − 4 θ− + ... ,
2b 6 8b 6
( )
a2 θ4 a4 4
that is 1− θ2 − − θ + ...
2b2 3 8b4
θ2 θ4
And cos θ = 1 − + − ...
2 4
Hence approximately ( ) ( )
a2 θ4 a4 θ2 θ4
c=b− θ −
2
− 3 θ4 − α 1− +
2b 3 8b 2 4
( ) ( )
a2 θ2 a2 a4 a
=b−a+ a− + − 3 − θ4
b 2 6b 8b 24
( )
(b − a)aθ2 θ4 4a2 3a4
=b−a+ + − 3 −a ;
2b 4 b b
( )
4a2 3a4 a2 a2 a4
and − 3 −a= −a+3 − 3
b b b b b
{ }
a(a − b) 3a2 (b2 − a2 ) a 3a2
= + = (a − b) − 3 (a + b) .
b b3 b b
Thus we obtain the required result.

Problem 247. Express sin5 θ cos6 θ in terms of sines of multiples of θ.


cos θ
Solution. sin5 θ cos6 θ = cos θ(sin θ cos θ)5 = (sin 2θ)5
25
cos θ 1
= × 4 {sin 10θ − 5 sin 6θ + 10 sin 2θ}
25 2
1
= 10 {sin 11θ + sin 9θ − 5(sin 7θ + sin 5θ) + 10(sin 3θ + sin θ)} .
2

( √ ) √
Problem 248. If tan θ + ϕ −1 = cos α + −1 sin α, where the letters denote real
π
quantities, show that θ = nπ ± where n is an integer.
4 ( √ )
√ sin θ + −1ϕ
Solution. We have cos α + −1 sin α = ( √ )
cos θ + −1ϕ
√ √ √
sin θ cos −1ϕ + cos θ sin −1ϕ sin θ(eϕ + e−ϕ ) − −1 cos θ(e−ϕ − eϕ )
= √ √ = √
cos θ cos −1ϕ − sin θ sin −1ϕ cos θ(eϕ + e−ϕ ) + −1 sin θ(e−ϕ − eϕ )

sin θ + −1k cos θ eϕ − e−ϕ
= √ , where k = ϕ .
cos θ − −1k sin θ e + e−ϕ
√ √ √
Hence sin θ + −1k cos θ = (cos α + −1 sin α)(cos θ − −1k sin θ)

= cos α cos θ + k sin α sin θ + −1(sin α cos θ − k sin θ cos α);
therefore sin θ = cos α cos θ + k sin α sin θ,
CHAPTER XXV : Miscellaneous Examples 298

and k cos θ = sin α cos θ − k sin θ cos α


sin θ − cos α cos θ sin α cos θ
therefore = .
sin α sin θ cos θ + sin θ cos α
Multiply up; thus we get cos α(sin θ − cos θ) = 0;
2 2
π
therefore tan2 θ = 1, and therefore θ = nπ ± .
4

Problem 249. Show that


1 1 π 1 π 1 π
= tan + tan + tan + ...
π 4 4 8 8 16 16
Solution. By Art. 309 (page 354) we have
1 1 x 1 x
− 2 cot 2x = tan x + tan + 2 tan 2 + . . . ,
x 2 2 2 2
and, since 2 cot 2x + tan x = cot x, we have
1 1 x 1 x
− cot x = tan + 2 tan 2 + . . . ;
x 2 2 2 2
π 1 1 π 1 π
then put for x, and divide by 2; thus = tan + tan + . . . .
2 π 4 4 8 8

Problem 250. Show that the coefficient of x n in the product


( )( )
x x π 2n
(1 + x) 1 + 2 1 + 2 . . . ad infinitum is .
2 3 2n + 1
( )n
θ2 θ2
Solution. Put − 2 for x; then we require the coefficient of − 2 , that is of
π π
(−1)n θ2n
in the development of
π 2n ( )( )( )
θ2 θ2 θ2
1− 2 1− 2 2 1 − 2 2 ....
π 2 π 3 π
(−1)n θ2n+1
Thus we require the coefficient of in the development of
( )( π 2n) ( )
θ 2 θ 2 θ2
θ 1− 2 1− 2 2 1 − 2 2 ....
π 2 π 3 π
that is in sin θ. See Art. 320 (page 359).
(−1)n θ2n+1
But the general term in the expansion of sin θ is .
2n + 1
1 1
Hence 2n × the required coefficient = ; so that the required coefficient is
π 2n + 1
π 2n
.
2n + 1

Problem 251. Show how to eliminate θ between


sin2 θ − p sin θ + m = 0 , and cos2 θ − q cos θ + n = 0 .
Solution. Proceed as in P roblem 241. We have
1 + m + n = p sin θ + q cos θ, cos2 θ − sin2 θ + n − m = q cos θ − p sin θ;
∴ (1 + m + n)(2 cos2 θ − 1 + n − m) = q 2 cos2 θ − p2 sin2 θ;
p2 + n2 − (1 + m)2
∴ cos2 θ = 2 ,
p + q 2 − 2(1 + m + n)
q 2 + m2 − (1 + n)2
and sin2 θ = 2 .
p + q 2 − 2(1 + m + n)
Substitute in 1 + m + n = p sin θ + q cos θ, and the elimination will be effected.

Problem 252. The internal bisectors of the angles of a triangle are produced to meet
the circumference of the circumscribing circle : show that the area of the triangle
CHAPTER XXV : Miscellaneous Examples 299

Rs
formed by joining the three points thus obtained = .
2
Solution. Let D, E, F be the points at which the bisectors of the angles A, B, C
1
respectively meet the circumference. Then the angle DAC = A, and the angle
2
1 1
CAE = the angle CBE = B; therefore DAE = (A + B); and therefore DE subtends
2 2
1
at the centre of the circle an angle equal to A + B : thus DE = 2R sin (A + B) =
2
1 1 1
2R cos C. Similarly EF = 2R cos A; and the angle DEF = (A + C); thus the area of
2 2 2
the triangle DEF
1 1 1 1 1 1 1
= · 4R2 cos A cos C sin (A + C) = 2R2 cos A cos B cos C
2 2 2 2 2 2 2
2R2 sS Rs
= = .
abc 2

x y c2
Problem 253. If sin−1 + sin−1 = sin−1 , then
a b ab
( ) 12
b2 x 2 + 2xy a 2 b2 − c 4 + a2 y 2 = c4 .

x y c2
Solution. Here sin−1+ sin−1 = sin−1 .
a b ab
Take the cosines
√ of both sides;
√ thus √
( ) ( ) ( )
x2 y2 xy c4
1− 1− − = 1− ;
a2 b2 ab a2 b2
√( ) √( ) √( )
x2 y2 xy c4
∴ 1− 2 1− 2 = + 1− 2 2 ;
a b ab a b
square both sides; thus
√( )
x2 y2 x2 y 2 x2 y 2 xy c4 c4
1− 2 − 2 + 2 2 = 2 2 +2 1− +1− ;
a b a b a b ab a2 b2 a2 b2

∴ b2 x2 + a2 y 2 + 2xy (a2 b2 − c4 ) = c4 .

Problem 254. From a point P each of two straight lines CA and CB, which are at
right angles, subtends an angle γ. If CA = a, and CB = b, show that
ab cos 2 γ
CP = √ .
sin γ a 2 + b2 − 2ab sin 2 γ
π
Solution. Let P CA = θ; then P CB = − θ;
2 ( )
π
sin −θ+γ
PC sin(θ + γ) PC 2
= , = ;
a sin γ ( b ) sin γ
π
thus a sin(θ + γ) = b sin − θ + γ = b cos(θ − γ);
2
∴ a(sin θ cos γ + cos θ sin γ) = b(cos θ cos γ + sin θ sin γ);
b cos γ − a sin γ
∴ tan θ = .
a cos γ − b sin γ
b cos γ − a sin γ
Hence sin θ = √
{(a cos γ − b sin γ)2 + (b cos γ − a sin γ)2 }
b cos γ − a sin γ
= √ ;
(a2 + b2 − 2ab sin 2γ)
CHAPTER XXV : Miscellaneous Examples 300

a cos γ − b sin γ
and cos θ = √ .
(a2 + b2 − 2ab sin 2γ)
a(sin θ cos γ + cos θ sin γ) ab cos 2γ
Then PC = = √ .
sin γ sin γ (a + b2 − 2ab sin 2γ)
2

√ of the equation x − x + x − x + 1 = 0 are


Problem√255. Show that the roots 4 3 2

cos 36 ◦ ± −1 sin 36 ◦ and cos 108 ◦ ± −1 sin 108 ◦ .


x5 + 1
Solution. x4 − x3 + x2 − x + 1 = . Hence we must find the roots of x5 + 1 = 0,
x+1
and omit the root −1. √
Now if x5 = −1 we may put x5 = cos nπ ± −1 sin nπ, where n is any odd integer.
√ 1 nπ √ nπ
Hence x = (cos nπ ± −1 sin nπ) 5 = cos ± −1 sin .
5 5
Put in succession 1 and 3 for n; thus we obtain the assigned values. If we put 5 for
n we obtain the root −1, which we had to omit.

Problem 256. If an angle of a triangle be 30 ◦ , one of the adjacent sides 1 foot,


and the opposite side 250 feet, find approximately the number of minutes in the other
acute angle.
Solution. Let θ denote the angle opposite to the side 1; then
sin θ 1 1
π = 250 ; therefore sin θ = 500 .
sin
6
1
As θ is very small we may put θ for sin θ; thus θ = approximately. Therefore the
500
1 180
number of degrees in the angle = × ; and therefore the number of minutes
500 π
60 180 3 180 3
= × = × = × 57.3 = 7 nearly.
500 π 25 π 25

Problem 257. Show that the area of the triangle formed by joining the points of
ρS
contact of the inscribed circle, or an escribed circle, of a triangle is , where ρ is
2R
the radius of the circle.
Solution. First take the inscribed circle : see Art. 248 (page 342).
A B
F E = 2r sin F OA = 2r cos ; similarly F D = 2r cos .
2 2
1 1
The angle EF A = (π − A); the angle DF B = (π − B); therefore the angle EF D =
2 2
1
(A + B).
2
Hence the area of the triangle DF E
1 A B A+B A B C 2r2 Ss rS
= · 4r2 cos cos sin = 2r2 cos cos cos = = .
2 2 2 2 2 2 2 abc 2R
Now take one of the escribed circles, as for instance that opposite to the angle A
: see Art. 250 (page 344).
B C
DF = 2r1 sin DOB = 2r1 sin ; similarly DE = 2r1 sin .
2 2
The angle F DE = the angle FDO + the angle EDO
1 1 1
= (π − B) + (π − C) = π − (B + C).
2 2 2
Hence the area of the triangle DF E
1 B C B+C
= · 4r12 sin sin sin
2 2 2 2
B C A 2r2 (s − a)S 2r1 S 2 r1 S
= 2r12 sin sin cos = 1 = = .
2 2 2 abc abc 2R
CHAPTER XXV : Miscellaneous Examples 301

( √ ) √
Problem 258. If tan θ + ϕ −1 = cos α + −1 sin α, where the letters denote real
( )
π α
quantities, show that e 2 ϕ = ± tan + .
4 2
Solution. Proceed as in P roblem 248; thus we obtain sin2 θ = cos2 θ.
If we take sin θ = cos θ we get 1 = cos α + k sin α; thus
1 − cos α α eϕ − e−ϕ α
k= = tan , that is ϕ = tan ;
sin α 2 e + e−ϕ 2
α
1 + tan ( )
2 π α
∴ e2ϕ = α = tan 4 + 2 .
1 − tan
2
If we take sin θ = − cos θ we get 1 = − cos α + k sin α; thus
1 + cos α α eϕ − e−ϕ α
k= = cot , that is ϕ = cot ;
sin α 2 e + e−ϕ 2
α α
1 + cot tan + 1 ( )
2 2 π α
∴ e2ϕ = α = α = − tan + .
1 − cot tan − 1 4 2
2 2
z2 z3
Problem 259. If s = 1 + z cos θ + cos 2θ + cos 3θ + . . . ,
2 3
z2 z3
and σ = z sin θ + sin 2θ + sin 3θ + . . . ,
2 3
σ 1 ( )
then z sin θ = tan−1 , and z cos θ = log s2 + σ 2 .
s 2
π
Find the values of s and σ when θ = .
2
Solution. Put the exponential values for the cosines in the series demoted by s : thus
1 z 2 ( 2ιθ ) z 3 ( 3ιθ )
s = 1 + z(eιθ + e−ιθ ) + e + e−2ιθ + e + e−3ιθ + . . . . . .
2 22 23
1 x y
= (e + e ),
2
where x = zeιθ = (cos θ + ι sin θ),
and y = ze−ιθ = z(cos θ − ι sin θ).
1
Thus s = ez cos θ (eιz sin θ + e−ιz sin θ ) = ez cos θ cos(z sin θ).
2
1 x
Similarly we find that σ = (e − ey ) = ez cos θ sin(z sin θ).

σ sin(z sin θ)
Therefore = = tan(z sin θ);
s cos(z sin θ)
σ
so that z sin θ = tan−1 .
{ s }
And s2 + σ 2 = e2z cos θ cos2 (z sin θ) + sin2 (z sin θ) = e2z cos θ ;
1 ( )
so that z cos θ = log s2 + σ 2 .
2
π σ
If θ = , we have = tan z and s2 + σ 2 = 1, so that σ = sin z and s = cos z.
2 s

Problem 260. Through a given point straight lines are drawn parallel to the sides
of a regular polygon; and from another given point perpendiculars are drawn to the
straight lines. Find the sum of the squares on the perpendiculars.
Solution. Let c be the distance of the two given points, n the number of sides in the

polygon; and put β = . Let α be the angle which the distance between the two
n
given points makes with the first straight line which is drawn. Then the numerical
values of the successive perpendiculars are
c sin α, c sin(α + β), c sin(α + 2β), . . .
CHAPTER XXV : Miscellaneous Examples 302

Hence the sum of the


{ squares on the perpendiculars }
= c2 sin2 α + sin2 (α + β) + sin2 (α + 2β) + . . . to n terms
c2
= {1 − cos 2a + 1 − cos 2(α + β) + 1 − cos 2(α + 2β) + . . .}
2
nc2
= · See Art. 305 (page 353).
2

Problem 261. Show how to eliminate θ between


a tan θ + b sec θ = h, and a cot θ + b cos θ = k.
Solution. We have
a sin θ + b = h cos θ, and cos θ(a + b sin θ) = k sin θ.
Find cos θ from the first equation, and substitute it in the second; thus we get
a2 + b2 − hk
sin2 θ + sin θ + 1 = 0.
ab
Again, find sin θ from the first equation, and substitute it in the second; thus we
get
a2 − b2 − hk k
cos2 θ + cos θ + = 0.
bh h
Then we may employ the process on P roblem 251.

Problem 262. Perpendiculars are drawn from the angles of an acute-angled trian-
gle on the opposite sides, and the feet of the perpendiculars joined. Show that the
2S
perimeter of the triangle thus formed = .
R
Solution. By Example xvi. 50 we know that the sides of the new triangle are respec-
tively a cos A, b cos B, and c cos C. Thus the perimeter
= a cos A + b cos B + c cos C = 4R sin A sin B sin C, by Example xvi. 22,
4R · 8S 3 2S
= = .
(abc)2 R

Problem 263. The internal bisectors of the angles of a triangle are produced to
meet the circumscribing circle. Show that the area of the triangle formed by joining
the points of intersection is never less than the area of the original triangle.
Solution. As in P roblem 252 we show that the area of the triangle thus formed is
1 1 1
2R2 cos A cos B cos C; denote this by Σ.
2 2 2
1
Also S = ab sin C = 2R2 sin A sin B sin C.
2
1 1 1
Σ cos A cos B cos C 1
Hence = 2 2 2 = .
S sin A sin B sin C A B C
8 sin sin sin
2 2 2
A B C
Now, as in Example xiii. 40 we see that 8 sin sin sin cannot be greater than unity;
2 2 2
and therefore S cannot be greater than Σ.

(r1 + r2 )(r2 + r3 )(r3 + r1 )


Problem 264. Show that 4R = .
r1 r2 + r2 r3 + r3 r1
S S S
Solution. r1 = , r2 = , r3 = .
s−a s−b s−c
Hence ( )( )
1 1 1 1
(r1 + r2 )(r2 + r3 )(r3 + r1 ) = S 3 + +
s(− a s−b )s − b s − c
1 1
+
s−c s−a
3
S abc s2 S 3 abc s2 abc
= = = .
(s − a) (s − b) (s − c)
2 2 2 S 4 S
CHAPTER XXV : Miscellaneous Examples 303

And { }
1 1 1
r1 r2 + r2 r3 + r3 r1 = S 2 + +
(s − a)(s − b) (s − b)(s − c) (s − c)(s − a)
S 2 (3s − a − b − c) S 2 s2
= = = s2 .
(s − a)(s − b)(s − c) S2
abc
Divide the first result by the second; and thus we get .
S

Problem 265. The shadows of two vertical walls which are inclined to each other at
an angle γ, and are a and a ′ feet in height, are observed when the Sun is due South
to be b and b′ feet in breadth. Show that if α be the Sun’s altitude above the horizon,
and β the inclination of the( first wall to
) the meridian,
b 2 b ′2 2bb′
cot2 α = + ′2 cosec 2 γ + cot γ cosec γ,
a2 a aa′
ab′
cot β = cosec γ + cot γ.
a′ b
Solution. The wall a feet high casts a shadow which extends a cot α feet from the
wall measured in the direction of the meridian; hence a cot α sin β is the breadth of
the shadow measured in the direction at right angles to the wall.
Thus b = a cot α sin β. Similarly b′ = a′ cot α sin(γ − β).
From these two equations we have to find α and β.
a sin β a′ sin(γ − β)
We get = ; so that
b b′
a a′
= ′ (sin γ cot β − cos γ);
b b
ab′
∴ cot β = cot γ + ′ cosec γ.
ab
{ ( )2 }
2 b2 b2 b2 ab′
Then cot α = 2 2
= 2 (1 + cot2 β) = 2 1 + cot γ + ′ cosec γ
a sin β a a ab
{ }
b2 a2 b′2 2ab′
= 1 + cot2 γ + ′2
cosec2 γ + ′ cot γ cosec γ
a2 a b 2 ab
( )
b2 b′2 2bb′
= + ′2 cosec 2 γ + cot γ cosec γ.
a2 a aa′

( √ )α+β √−1
Problem 266. Show that a + b −1 will be wholly real if
β ( 2 ) b
log a + b + α tan−1
2
2 a
π
is zero or an even multiple of .
2
b
Solution. Assume a = r cos θ, and b = r sin θ; so that r2 = a2 + b2 , and tan θ = . Also
a
β
assume α = ρ cos ϕ, β = ρ sin ϕ; so that ρ2 = α2 + β 2 , and tan ϕ = .
α
Then the proposed expression
= (r cos θ + ιr sin θ)ρ cos ϕ+ιρ sin ϕ = (reιθ )ρ cos ϕ+ιρ sin ϕ .
Denote this by u; then ( )
log u = (ρ cos ϕ + ιρ sin ϕ) log reιθ
= (ρ cos ϕ + ιρ sin ϕ)(ιθ + log r)
= ρ(cos ϕ log r − θ sin ϕ) + ρι(sin ϕ log r + θ cos ϕ)
= σ + ιτ say;
therefore u = eσ+ιτ = eσ eιτ = eσ (cos τ + ι sin τ ).
To make this wholly real the term involving ι must vanish, therefore sin τ must
vanish; therefore τ must be zero or a multiple of π; therefore ρ(sin ϕ log r +θ cos ϕ) must
CHAPTER XXV : Miscellaneous Examples 304

π β ( )
be zero or an even multiple of ; but ρ sin ϕ = β and ρ cos ϕ = α; so that log a2 + b2 +
2 2
b π
α tan−1 must be zero or an even multiple of .
a 2

Problem 267. Apply the exponential values of the sine and cosine to show that
{ }
a2 1 2 1 3
log 2 = 4 c sin 2
θ − c sin 2
2 θ + c sin 2
3 θ − . . .
a cos2 θ + b2 sin2 θ 2 3
a−b
when c = .
a+b
Solution.
a2 2a2
= 2
a2 cos2 θ + b2 sin2 θ a (1 + cos 2θ) + b2 (1 − cos 2θ)
2a2
= 2
a + b + (a2 − b2 ) cos 2θ
2

4a2 4a2
= −2θι
=
2(a + b ) + (a − b )(e
2 2 2 2 2θι +e ) (a + b) (1 + ce2θι )(1 + ce−2θι )
2

(1 + c)2
= .
(1 + ce2θι )(1 + ce−2θι )
Therefore
a2 ( ) ( )
log 2 2 2 2
= 2 log(1 + c) − log 1 + ce2θι − log 1 + ce−2θι
a cos θ + b sin θ { }
c2 c3 c4
=2 c− + − + ...
2 3 4
{ }
c2 4θι c3 6θι c4 8θι
− ce2θι − e + e − e + ...
2 3 4
{ }
−2θι c2 −4θι c3 −6θι c4 −8θι
− ce − e + e − e + ... .
2 3 4
The term which involves c is −c(eθι − e−θι )2 , that is 4c sin2 θ.
c2 2θι 4c2
The term which involves c2 is (e − e−2θι )2 , that is − sin2 2θ.
2 2
c 3 4c 3
The term which involves c3 is − (e3θι − e−3θι )2 , that is sin2 3θ.
3 3
And so on.
Thus we obtain the required result.

Problem 268. A triangle is formed by joining the centre of the inscribed circle of
a triangle with the centres of the escribed circles which are opposite to the angles A
abc C
and B. Show that the area of this triangle is cot .
2s 2
Solution. Let O denote the centre of the inscribed circle, D and E the centers of the
escribed circles. Then D, C, E are on a straight line which is at right angles to OC.
The area of the triangle ODE
1 1 C C
= OC · DE = r cosec (r1 + r2 ) sec
2 2 ( 2 )2
r(r1 + r2 ) S S S S2c
= = + =
sin C s sin C s − a s−b s(s − a)(s − b) sin C
abc cos2 1C
(s − c)c 2 = abc cot C .
= =
sin C s sin C 2s 2

Problem 269. If O be the centre of the circle inscribed in a triangle ABC , and
ra , rb , rc the radii of the circles inscribed in the triangles OBC , OCA, OAB, show that
( )
a b c A B C
+ + = 2 cot + cot + cot .
ra rb rc 4 4 4
CHAPTER XXV : Miscellaneous Examples 305

1 1
Solution. The angle OBC = B, and the angle OCB = C. Hence, as in Art. 249
2 2
(page 343), we have
( )
B C
ra cot + cot = a;
4 4
a B C
∴ = cot + cot .
ra 4 4
b C A c A B
Similarly = cot + cot , and = cot + cot .
rb 4 4 rc 4 4
Hence by addition we get the required result.

Problem 270. Sum the series


1 1 1
tan−1 + tan−1 + . . . + tan−1 .
2 8 2n 2
1
Solution. We easily see that tan−1 2 = tan−1 (2r + 1) − tan−1 (2r − 1).
2r
Resolve each of the given terms into two by this formula. Then by addition we find
n
that the sum = tan−1 (2n + 1) − tan−1 1 = tan−1 .
n+1

Problem 271. If a series of triangles be described of the same area, show that the
sum of the cotangents of the angles varies as the sum of the squares on the sides.

cos A b2 + c2 − a2 b2 + c2 − a2
Solution. cot A = = = ;
sin A 2bc sin A 4S
similar expressions hold for cot B and cot C. Thus
a2 + b2 + c2
cot A + cot B + cot C = .
4S
Hence if S be given the sum of the cotangents of the angles varies as the sum of
the squares of the sides.

Problem 272. Let I denote the centre of the circle inscribed in a triangle, O the
centre of the circumscribed circle, D, E, F the centres of the escribed circles : then
show that
OI 2 + OD 2 + OE 2 + OF 2 = 12R2 .
Solution. By Art. 253 (page 345) we have
OI 2 = R2 − 2Rr;
and OD 2 + OE 2 + OF 2 = 3R2 + 2R(r1 + r2 + r3 ).
Thus by addition we obtain
OI 2 + OD2 + OE 2 + OF 2 = 4R2 + 2R(r1 + r2 + r3 − r)
= 4R2 + 8R2 , by P roblem 201, = 12R2 .

(√ )
a−b x b + a cos x
Problem 273. Show that 2 tan−1 tan = cos−1 .
a+b 2 a + b cos x
(√ ) √
a−b x a−b x
Solution. Let θ = tan−1 tan , then tan θ = tan ,
a+b 2 a+b 2
a + b − (a − b) tan 2 x
1 − tan2 θ 2
cos 2θ = = x
1 + tan2 θ a + b + (a − b) tan2
( ) 2
x x
b + a cos 2 − sin 2
2 2 b + a cos x
= ( )= .
x x a + b cos x
a + b cos 2 − sin 2
2 2

Problem 274. A chord is drawn cutting two concentric circles whose radii are as 1
CHAPTER XXV : Miscellaneous Examples 306

to n, so that the intercepted portions subtend angles 2 α and 2 β at the centre. Show
that the chord is divided at either point of intersection with the inner circle in the
ratio of
n 2 − 2n cos(α − β) + 1 to n 2 − 1 .
Solution. Let O denote the centre of the circles. Let ABCD be a straight line cutting
the outer circumference at A and D, and the inner circumference at B and C. Let
OB = r, and OA = nr. Let the angle AOD = 2α, and the angle BOC = 2β; so that the
angle AOB = α − β.
Then AB 2 = n2 r2 + r2 − 2nr2 cos(α − β).
AB AB 2 AB 2
Now = = .
BD AB · BD AB · AC
But AB · AC = the square on the straight line drawn from A to touch the inner
circumference = (n2 − 1)r2 .
AB n2 − 2n cos(α − β) + 1
Therefore = .
BD n2 − 1

( √ )α+β √−1
Problem 275. Show that a + b −1 will be wholly imaginary if
β ( 2 ) b
log a + b2 + α tan−1
2 a
π
is an odd multiple of .
2
Solution. Proceed as in the solution of P roblem 266. That the expression may be
wholly imaginary we must have cos τ = 0, and therefore τ must be an odd multiple of
π π
, therefore ρ(sin ϕ log r + θ cos ϕ) must be an odd multiple of ; but ρ sin ϕ = β, and
2 2
β ( ) b π
ρ cos ϕ = α, so that log a2 + b2 + α tan−1 must be an odd multiple of .
2 a 2

Problem 276. Show that the area of the triangle formed by joining the centres of
the escribed circles of a triangle is
{( ) ( ) ( ) }
abc 1 1 C 1 1 A 1 1 B
+ tan + + tan + + tan .
2 a b 2 b c 2 c a 2
Solution.  C B
 A
( ) sin sin cos
1 C B 1 2 2 = 2
tan + tan = +
a 2 2 a C B C B
cos cos a cos cos
2 2 2 2
1 C B
= tan tan , by Art. 249 (page 343).
r 2 2
In this way we find that the proposed expression
{ }
abc A B B C C A
= tan tan + tan tan + tan tan
2r 2 2 2 2 2 2
abc
= , by Example viii. 15;
2r
and this is the area of the triangle by Example xvi. 34.
Or thus. Let I denote the centre of the inscribed circle, O the centre of the escribed
a a
circle opposite to A; then the area of the quadrilateral IBOC = (r + r1 ) = (s − a +
2 2
A a A
s) tan = (b + c) tan : see Arts. 249 (page 343) and 250 (page 344).
2 2 2
In this way we obtain for the whole required area the given expression.

Problem 277. Sum the following series to n terms( : )


log(1 + 2 cos θ) + log(1 + 2 cos 3 θ) + log 1 + 2 cos 3 2 θ + . . .
Solution.
CHAPTER XXV : Miscellaneous Examples 307

We have

sin
1 + 2 cos θ = 2 ,
θ
sin
2
32 θ
sin
1 + 2 cos 3θ = 2 ,

sin
2
and so on. Then, as the sum of the logarithms of any set of quantities is equal to
the logarithm of the product of those quantities, we see that the required sum is the
logarithm of
3θ 32 θ 3n θ 3n θ
sin sin sin sin
2 · 2 ...... 2 , that is the logarithm of 2 .
θ 3θ 3n−1 θ θ
sin sin sin sin
2 2 2 2

Problem 278. A regular polygon of n sides is placed with one of its sides in contact
with a fixed straight line, and is turned about one extremity of this side until the next
side is in contact with the straight line, and so on for a complete revolution. Show
that the length of the path described by any one of the angular points of the polygon
4 πR π
is cot , where R is the radius of the circle circumscribing the polygon.
n 2n
π
Solution. Put β for . The path consists of a set of arcs of circles, each of which
n
corresponds to the angle 2β, and the radii of which are the respective distances of
any assumed point from all the other angular points. The radii thus are 2R sin β,
2R sin 2β, 2R sin 3β, . . .
Hence the required sum
= 2R{sin β + sin 2β + sin 3β + . . . + sin nβ}2β.
The term sin nβ is zero, and may be omitted if we please.
By Art. 303 (page 352) this expression
( )
n−1 nβ n+1
sin β + β sin sin π
2 2 2n
= 4Rβ = 4Rβ π
1 sin
sin β
2 2n
π 4Rπ π
= 4Rβ cot = cot .
2n n 2n

Problem 279. Show that the sum of the areas of the sectors of the circles which
correspond to the path in the preceding Problem = 2 πR2 .
{ of the areas of all the sectors will be
Solution. The sum }
4R2 sin2 β + sin2 2β + sin2 3β + . . . + sin2 nβ β
= 2R2 β {1 − cos 2β + 1 − cos 4β + . . .}
{ ( ) }
cos 2β + n − 1β sin nβ
= 2R β2
n− = 2R2 nβ = 2R2 π.
sin β
If we wish to have the whole area of the figure bounded by the straight line and by
the arcs between two points where they cross the straight line, we must add to the
n 2π
above a set of triangles which make up the whole polygon, that is R2 sin .
2 n

Problem 280. Sum the following series to 2n terms :


sin 2 θ sin 4 θ sin 6 θ
− + − ...
sin θ sin 3 θ sin 3 θ sin 5 θ sin 5 θ sin 7 θ
CHAPTER XXV : Miscellaneous Examples 308

Solution. We have { }
sin 2rθ 1 1 1
= + .
sin(2r − 1)θ sin(2r + 1)θ 2 cos θ sin(2r − 1)θ sin(2r + 1)θ
If we resolve each term of the proposed { series into two by the
} aid of this formula
1 1 1
we find that the sum of 2n terms = − .
2 cos θ sin θ sin(4n + 1)θ

Problem 281. In an acute-angled triangle let P denote the point of intersection of


the perpendiculars from the angles on the opposite sides; and
let AP = a, BP = β, CP = γ : then
1
S = (aα + bβ + cγ) ,
4
2abc = a2 α cosec A + b2 β cosec B + c2 γ cosec C.
Solution. As in P roblem 214 we have
a cos A b cos B c cos C
α= , β= , γ= .
(sin A sin B sin)C
1 2
1 a cos A 2
b cosB 2
c cos C
Hence (aα + bβ + cγ) = + +
4 4 sin A sin B sin C
= R2 (sin A cos A + sin B cos B + sin C cos C)
R2
= (sin 2A + sin 2B + sin 2C) = 2R2 sin A sin B sin C, by Art. 114 (page 328),
2
1
= ab sin C, by Art. 252 (page 344), = S.
2
a3 cos A b3 cos B c3 cos C
Also, a2 α cosec A + b2 β cosec B + c2 γ cosec C = 2
+ 2
+
sin A sin B sin2 C
= 8R3 (sin A cos A + sin B cos B + sin C cos C)
= 4R3 (sin 2A + sin 2B + sin 2C) = 16R3 sin A sin B sin C
= 8RS, by the former part of the Problem, = 2abc.

Problem 282. Let R denote the radius of the circle circumscribing a triangle; and
let r ′ , r ′′ , r ′′′ denote the radii of the greatest circles which touch the former circle
and the sides of the triangle, being outside the triangle : then show that
( )2
abc
64Rr ′ r ′′ r ′′′ = .
a+b+c
Solution. We obtain immediately from a diagram
2r′ = R(1 − cos A), 2r′′ = R(1 − cos B), 2r′′′ = R(1 − cos C);
A B C 8R3 S 4
hence 8r′ r′′ r′′′ = 8R3 sin2 sin2 sin2 = .
2 2 2 ( a2 b2 c2 s2)
4 4 2 2 2 2
64R S a b c abc
∴ 64Rr′ r′′ r′′′ = 2 2 2 2 = 2
= .
a b c s 4s a+b+c

Problem 283. Show that one value of


√ √
x 2 − c2 c a2 − x 2 x 2 − ac
sin−1 √ − sin−1 √ is sin−1 .
a2 − c2 x a2 − c2 √ x(a − c)

(x2 − c2 ) c (a2 − x2 )
Solution. Let θ = sin−1 √ and ϕ = sin−1 √ ;
(a − c)
2 x (a2 − c2 )
√ √
(a2 − x2 ) α (x2 − c2 )
then cos θ = √ and cos ϕ = √ ;
(a2 − c2 ) x (a2 − c2 )
√ √ √ √
(x2 − c2 ) a (x2 − c2 ) (a2 − x2 ) c (a2 − x2 )
∴ sin(θ − ϕ) = √ · √ − √ · √
(a2 − c2 ) x (a2 − c2 ) (a2 − c2 ) x (a2 − c2 )
a(x2 − c2 ) −
c(a2 x2 ) x2 (a + c) − ac(a + c) − ac x2
= − = = .
x(a2 − c2 ) x(a2 − c2 ) x(a2 − c2 ) x(a − c)
CHAPTER XXV : Miscellaneous Examples 309

Problem 284. If the lengths of the three straight lines drawn from the angles of a
triangle to bisect the opposite sides be denoted by h, k, l, show that
4(h2 + k2 + l2 ) = 3(a2 + b2 + c2 ),
16(h2 k2 + k2 l2 + l2 h2 ) = 9(a2 b2 + b2 c2 + c2 a2 ),
16(h4 + k4 + l4 ) = 9(a4 + b4 + c4 ).
Solution. Suppose D the middle point of BC. Then
AB 2 = AD2 + BD2 − 2AD · BD cos ADB,
AC 2 = AD2 + CD 2 − 2AD · CD cos ADC;
a2 1 a2
therefore by addition b2 + c2 = 2h2 + ; so that h2 = (b2 + c2 ) − ;
2 2 4
1 b2 1 c 2
similarly k2 = (c2 + a2 ) − , and l2 = (a2 + b2 ) − .
2 4 2 4
2 2 2 2
Therefore by addition 4(h + k + l ) = 3(a + b + c ). 2 2

Also (4h2 )2 + (4k2 )2 + (4l2 )2


= (2b2 + 2c2 − a2 )2 + (2c2 + 2a2 − b2 )2 + (2a2 + 2b2 − c2 )2
= 9(a4 + b4 + c4 ), by development.
Again, from what has been already shewn,
16(h2 + k2 + l2 )2 = 9(a2 + b2 + c2 ),
and 16(h4 + k4 + l4 ) = 9(a4 + b4 + c4 );
subtract and divide by 2; thus
16(h2 k2 + k2 l2 + l2 h2 ) = 9(a2 b2 + b2 c2 + c2 a2 ).

Problem 285. The area of any triangle is to the area of the triangle whose sides
are respectively equal to the straight lines joining its angular points with the middle
points of the opposite sides, as 4 is to 3 .
Solution. The area of the triangle which can be formed with the straight lines h, k, l,
by Arts. 247 (page 342) and 218 (page 338),
1√
= (2h2 k2 + 2k2 l2 + 2l2 h2 − h4 − k4 − l4 )
4√
1 9
= (2a2 b2 + 2b2 c2 + 2c2 a2 − a4 − b4 − c4 )
4 16
3 √ 3
= (2a2 b2 + 2b2 c2 + 2c2 a2 − a4 − b4 − c4 ) = S.
16 4

Problem 286. Show that one value of


{ ( √ ) ( √ )} 1
a cos θ + −1 sin θ − b cos θ − α + −1 sin θ − α n
{ √ }
1 θ+β θ+β
is k n cos + −1 sin
n n
b sin α
when k = a + b − 2ab cos α, and tan β =
2 2 2
.
a − b cos α
Solution. a cos θ − b cos(θ − α) = cos θ(a − b cos α) − b sin α sin θ;
assume a − b cos α = k cos β, and b sin α = k sin β;
then a cos θ − b cos(θ − α) = k(cos θ cos β − sin θ sin β) = k cos(θ + β).
Similarly
a sin θ − b sin(θ − α) = sin θ(a − b cos α) + b sin α cos θ = k sin(β + θ).
Thus the proposed expression
{ √ }1
= k cos(θ + β) + k −1 sin(θ + β) n
{ √ }
1 θ+β θ+β
= k n cos + −1 sin .
n n

Problem 287. Any point is taken within a triangle ABC ; its distances from A, B, C
are h, k, l respectively, and its perpendicular distances from BC , CA, AB are α, β, γ
CHAPTER XXV : Miscellaneous Examples 310

respectively. Show that


h 2 α sin A + k 2 β sin B + l 2 γ sin C = aβγ + bγα + cαβ.
Solution. Denote the point by O; and let OD, OE, OF be the perpendiculars on
BC, CA, AB respectively. Then OA is the diameter of the circle which would go round
EF
OEAF ; so that OA = , by Art. 252 (page 344). Therefore
sin A
OA · α sin A = α · OA · EF = α(OE · F A + OF · AE), by Euclid vi. D,
2

= α · β · AF + α · γ · AE.
Transform the other two terms in like manner; thus we obtain
αβ(AF + BF ) + βγ(BD + CD) + γα(AE + EC) = αβc + βγα + γαb.

Problem 288. If D, E, F be the feet of the perpendiculars drawn from the point in
the preceding Problem on the sides of the triangle, show that
aβγ + bγα + cαβ = 4R × area of DEF.
c
Solution. We have αβc = αβ sin C = 2R × 2 area of OED.
sin C
Transform the other two terms similarly; thus we obtain
4R( area of OED + area of ODF + area of OF E).

Problem 289. Sum the following series to n terms :


sin θ sin 3 θ sin 5 θ
+ + + ...
cos2 θ cos2 2 θ cos2 θ cos2 3 θ cos2 2 θ
Solution. We have
1 1 cos2 A − cos2 B sin2 B − sin2 A sin(B − A) sin(B + A)
− = = = ;
cos2 B cos2 A cos2 A cos2 B cos2 {
B cos2 A } B cos A
cos 2 2
sin(B + A) 1 1 1
so that = −
cos2 B cos2 A sin(B − A) cos2 B cos2 A
Apply this transformation to every term of the proposed series; then we find that
the sum { } { }
1 1 1 1 1
= 2
− 2
= 2
− 1 = cosec θ tan2 nθ.
sin θ cos nθ cos 0 sin θ cos nθ

Problem( 290. Find the ) general


( value of θ which
) satisfies
( the equation
)
√ √ √
cos θ + −1 sin θ cos 2 θ + −1 sin 2 θ . . . cos nθ + −1 sin nθ = 1 .
Solution. By De Moivre’s Theorem the√equation becomes
cos(θ + 2θ + . . . + nθ) + −1 sin(θ + 2θ + . . . + nθ) = 1,
n(n + 1) √ n(n + 1)
that is cos θ + −1 sin θ = 1.
2 2
n(n + 1) n(n + 1) n(n + 1)
Hence we must have cos θ = 1, and sin θ = 0; so that θ =
2 2 2
2mπ, where m is zero or any integer.

Problem 291. D, E, F are the centres of the escribed circles of a triangle opposite
to A, B, C respectively : if r ′ , r ′′ , r ′′′ denote the radii of the circles described round
DBC , ECA, FAB, show that
r ′ r ′′ r ′′′ = 2R2 r.
BC α α
Solution. We have r′ = = = ; and similar expressions
2 sin BDC B+C A
2 sin 2 cos
2 2
hold for r′′ and r′′′ . Thus
abc a2 b 2 c2 16R 2S2 2R2 S
r′ r′′ r′′′ = = = = = 2R2 r.
A B C 8sS 8sS s
8 cos cos cos
2 2 2

Problem 292. If two sides a, b, and the included angle C of a triangle are given, and
a small error γ exist in C , the corresponding error in the radius of the circumscribed
CHAPTER XXV : Miscellaneous Examples 311

abγ
circle is cot A cot B.
2c
a a
Solution. We have R= , so that R sin A = (42)
2 sin A 2
Suppose that in consequence of the error γ in C there is an error α in A, and an
error ρ in R. Thus
a
(R + ρ) sin(A + α) = ;
2
therefore approximately by Art. 181 (page 334)
a
(R + ρ)(sin A + α cos A) = (43)
2
From (42) and (43) by subtraction, neglecting the product αρ,
αR cos A + ρ sin A = 0;
ρ
so that α = − tan A.
R
Similarly if β be the error in B arising from the error γ in C, we have
ρ
β = − tan B.
R
But α + β + γ = 0, since the sum of the three angles of a triangle is equal to a fixed
quantity, namely two right angles.
ρ
Thus γ − (tan A + tan B) = 0;
R
Rγ Rγ cos A cos B cγ cos A cos B
∴ρ= = = .
tan A + tan B sin(A + B) 2 sin2 C
c sin A c sin B abγ cot A cot B
And since sin C = and = ; we have ρ = .
a b 2c

Problem 293. A quadrilateral is formed by connecting two points in the produced


sides of a right-angled isosceles triangle, equidistant from the vertex, by a straight
line whose length is n times that of the base. Show that the angle between the diag-
n−1
onals of the quadrilateral is 2 tan−1 .
n+1
Solution. Let C denote the right angle, CA and CB the equal sides; produce CA to
D and CB to E; then since DE is n times AB, it follows that CD and CE are each n
times CA or CB. Let AE and BD intersect at O. Then the angle DOA = the sum of the
angles OBA and OAB, and these are equal; so that the angle DOA = twice the angle
OAB. But the angle OAB = the angle EAC− the angle BAC, so that
tan EAC − tan BAC n−1
tan OAB = tan(EAC − BAC) = = .
1 + tan EAC tan BAC 1+n

Problem 294. In the equation θ = cos θ, show that there must be a solution, and
π
only one; and that the value of θ is less than .
4
π
Solution. As θ continually increases from 0 to the value of cos θ continually de-
2
creases from 1 to 0; so that there must be one value of θ, and only one, in this range,
which makes θ = cos θ. Also as cos θ is greater than θ when θ = 0, and is less than θ
π π π
when θ = , this value is less than . As θ changes from 0 to − , the cosine is always
4 4 2
positive, and so we cannot have cos θ = θ.
π
When θ is numerically greater than it is numerically greater than unity, and so
2
cannot be equal to cos θ.
Hence there must be one, and only one, solution of the equation θ = cos θ.

Problem 295. If β is an approximate value of θ in the equation cos θ = θ, and too


β − cos β
large, show that β − is a closer value, and also too large.
1 + cos β
π
Solution. Suppose β the circular measure of an angle between 0 and , which is
2
greater than the solution of θ = cos θ, so that β − cos β is positive. Let β − α denote the
CHAPTER XXV : Miscellaneous Examples 312

β − cos β cos α
solution, so that β−α = cos(β − α) = cos β cos α+sin β sin α; therefore α = .
sin α
1 + sin β
α
sin α β − cos β
Now is less than unity, and so is cos α; hence is less than the true value
α 1 + sin β
β − cos β
of α, and is a positive quantity. Therefore β − is nearer than β to the solution
1 + sin β
of the equation, and is still too large.

( √ ) √
Problem 296. If tan θ + ϕ −1 = tan α + −1 sec α, where the letters denote real
α
quantities, show that e 2 ϕ = ± cot .
2
Solution. As in the solution of P roblem 248 we get

√ sin θ + −1k cos θ
tan α + −1 sec α = √ ;
cos θ − −1k sin θ
√ √ √
∴ (tan α + −1 sec α)(cos θ − −1k sin θ) = sin θ + −1k cos θ;
∴ sin θ = tan α cos θ + k sin θ sec α,
and k cos θ = sec α cos θ − k sin θ tan α;
∴ (sin θ − tan α cos θ)(cos θ + sin θ tan α) = sin θ cos θ sec2 α;
∴ sin θ cos θ(1 − sec2 α − tan2 α) = tan α(cos2 θ − sin2 θ);
cos 2θ
∴ − tan α = = cot 2θ.
( )sin 2θ
π π
Hence cot 2θ = cot + α ; ∴ 2θ = nπ + + α.
2 2
sin θ − tan α cos θ sin(θ − α)
And k= = ;
sin θ sec α (
sin θ )
1+k sin(θ − α) + sin θ α α
∴ = = tan θ − cot .
1−k sin θ − sin(θ − α) 2 2
( ) ( )
α nπ π
Now tan θ − = tan + = ±1;
2 2 4
1+k α α
thus = ± cot , that is e2ϕ = ± cot .
1−k 2 2

Problem 297. A regular pentagon and a regular hexagon are inscribed in a circle
of radius r, so as to have an angular point in common; and the other adjacent angular
4r sin 18 ◦ sin 15 ◦
points are joined. Show that the perimeter of the figure so formed is .
sin 3 ◦
Solution. When the figure is constructed it will be found to have ten sides, five of
which are respectively equal to the other five.
The sum of five sides will be found to be
2r {sin 30◦ + sin 6◦ + sin 24◦ + sin 12◦ + sin 18◦ } ;
2r sin(6◦ + 12◦ ) sin 15◦ 2r sin 18◦ sin 15◦
and by Art. 303 (page 352) this = ◦
=
sin 3 sin 3◦

Problem 298. Sum the following series to n terms :


1 3θ 3θ 32 θ
cos θ cos2 θ cosec 2 + cos 3θ cos2 cosec 2
2 2 2 2
32 θ 33 θ
+ cos 32 θ cos2 cosec 2 + ...
2 2
cos θ(1 + cos θ) cos θ(1 + cos θ)
Solution. The first term = =
1 − cos 3θ (1 − cos θ)(1 + 2 cos θ)2
1 1 1 1
cos θ(1 + cos θ) + −
= 4 4 = 4 − 4
(1 − cos θ)(1 + 2 cos θ)2 1 − cos θ (1 − cos θ)(1 + 2 cos θ)2
1 1
= 4 − 4 .
1 − cos θ 1 − cos 3θ
CHAPTER XXV : Miscellaneous Examples 313

Each term is to be resolved into{two in this manner; so}that the sum


1 1 1
= − .
4 1 − cos θ 1 − cos 3n θ

Problem 299. A series of radii divide the circumference of a circle into 2n equal
parts. Show that the product of the perpendiculars let fall from any point of the
rn
circumference on n successive radii = n−1 sin nϕ, where r is the radius, and ϕ the
2
angle between the radius to the given point and one of the extreme radii.
π
Solution. Put β for . The first perpendicular = r sin ϕ, the second perpendicular
n
= r sin(ϕ + β), the third = r sin(ϕ + 2β), and so on. Hence the product
= rn sin ϕ sin(ϕ + β) sin(ϕ + β) sin(ϕ + 2β) . . . . . . sin(ϕ + nβ − β);
rn
and this by Art. 318 (page 358) = n−1 sin nϕ.
2

Problem 300. There are n stones arranged at equal intervals round the circumfer-
ence of a circle : compare the labour of carrying them all to the centre with that of
heaping them all round one of the stones; and show that when the number of stones
is indefinitely increased the ratio is that of π to 4 .
Solution. Let r denote the radius. When all the stones are taken to the centre each
stone is carried over a length r, so that the labour may be denoted by nr. When all
the stones are taken to the position of one stone the labour in like manner may be
represented by the sum of the straight lines drawn from one corner of the polygon to
all the other corners.
π
Let β= : then this sum
n
= 2r{sin β + sin 2β + sin 3β + . . . + sin nβ}
( )
n−1 nβ n+1
2r sin β + β sin 2r sin β
2 2 2 β
= = = 2r cot .
β β 2
sin sin
2 2
nr n β n π
Hence the required ratio = = tan = tan .
β 2 2 2 2n
2r cot
2
To find the value of this when n is indefinitely increased we put it in the form
π
π tan 2n π
4 π ; then by Art. 118 (page 330) the limit is 4 .
2n
APPENDIX
Miscellaneous Articles and Propositions

Art. 8 To compare the number of degrees in any angle with the number of grades in
the same angle.
Let D be the number of degrees contained in any angle, G the number of grades
D
contained in the same angle. Then since there are 90 degrees in a right angle,
90
expresses the ratio of the given angle to a right angle; and since there are 100 grades
G
in a right angle, also expresses the ratio of the given angle to a right angle.
100
D G
Hence = ;
90 100
90 9 1
therefore D= G= G=G− G,
100 10 10
100 10 1
and G= D= D = D + D.
90 9 9
1
The formula D = G − G gives the following rule : From the number of grades
10
contained in any angle subtract one-tenth of that number, the remainder is the num-
ber of degrees contained in the angle.
1
The formula G = D + D gives the following rule : To the number of degrees
9
contained in any angle add one-ninth of that number, the sum is the number of grades
contained in the angle.

Art. 9 Again, let m be the number of English minutes contained in any angle, µ the
number of French minutes contained in the same angle. Then since there are 90 × 60
m
English minutes in a right angles, expresses the ratio of the given angle to a
90 × 60
µ
right angle; and since there are 100 × 100 French minutes in a right angle,
100 × 100
also expresses the ratio of the given angle to a right angle. Hence
m µ
= ;
90 × 60 100 × 100
9×6 27
therefore m= µ= µ,
10 × 10 50
50
and µ= m.
27
Similarly, if s be the number of English seconds contained in any angle, and σ the
number of French seconds contained in the same angle,
s σ
= ;
90 × 60 × 60 100 × 100 × 100
81
therefore s= σ,
250
250
and σ= s.
81

Art. 22 We will now show how to connect the circular measure of any angle with the
measure of the same angle in degrees.
Let x denote the number of degrees in any given angle, θ the circular measure of
x
the same angle. Since there are 180 degrees in two right angles, expresses the
180
ratio of the given angle to two right angles. And since π is the circular measure of
θ
two right angles, also expresses the ratio of the given angle to two right angles.
π
Hence
x θ
= ;
180 π
APPENDIX : Miscellaneous Articles and Propositions 316

180θ
thus x= ,
π
πx
and θ= .
180

Art. 24 Similarly we may connect the circular measure of any angle with the measure
of the same angle in grades.
Let y denote the number of grades in any given angle, θ the circular measure of
the same angle; then the ratio of the given angle to two right angles is expressed by
y θ
and also by .
200 π
y θ
Hence = ;
200 π
200θ
thus y= ;
π
πy
and θ= .
200
200
The number of grades in the angle which is the unit of circular measure is , that
π
is, 63·661977 . . .

Art. 32 To prove that (sin A)2 + (cos A)2 = 1 .


In the right-angled triangle AP M we have
P M 2 + AM 2 = AP 2 ;
P M 2 + AM 2
therefore 2
= 1;
( ) AP ( )
PM 2 AM 2
therefore + = 1;
AP AP
that is (sin A)2 + (cos A)2 = 1.

Art. 34 To prove that


(sec A)2 = 1 + (tan A)2 , and (cosec A)2 = 1 + (cot A)2 .
In the right-angled triangle AP M we have
AP 2 = AM 2 + P M 2 ;
AP 2 P M2
therefore 2
=1+ 2
,
( AM ) (AM )2
AP 2 PM
therefore =1+ ,
AM AM
2 2
that is (sec A) = 1 + (tan A) .
Again, since AP 2 = P M 2 + AM 2 ,
( ) ( )
AP 2 AM 2
=1+ ,
PM PM
2 2
that is (cosec A) = 1 + (cot A) .
The results here obtained are usually written thus,
sec2 A = 1 + tan2 A, cosec 2 A = 1 + cot2 A.

Art. 42 Let OB, OC be two straight lines which meet at right


APPENDIX : Miscellaneous Articles and Propositions 317

P
N

B′ O M B

C′

angles; produce BO to any point B ′ and CO to any point C ′ . Let P be any point in the
plane containing the two straight lines. The position of P will be known if we know
the distance of P from each of the straight lines BB ′ and CC ′ , and also know on which
side of each of these straight lines it is situated. Draw P M and P N perpendicular to
the straight lines BB ′ and CC ′ respectively. We shall adopt the following conventions
: the distance ON or P M will be denoted by a positive number when P is above the
straight line BB ′ , and by a negative number when P is below the straight line BB ′ ;
the distance OM or P N will be denoted by a positive number when P is to the right
of CC ′ , and by a negative number when P is to the left of CC ′ .

Art. 45 It follows immediately from the definitions, that if two angles differ by four
right angles or by any multiple of four right angles the Trigonometrical Ratios of the
two angles are the same.

Art. 48 To compare the Trigonometrical Ratios of any angle and of its supplement.
Let P AB be any angle, produce BA to B ′ and make P ′ AB ′ = P AB;

P′ P

B′ M′ A M B

take AP ′ = AP, and draw P M and P ′ M ′ perpendicular to BB ′ .


The angle P ′ AB = 180◦ − P ′ AB ′ = 180◦ − P AB; thus P ′ AB is the supplement of
P AB. The triangles P AM and P ′ AM ′ are geometrically equal in all respects; now
PM P ′M ′
sin A = , sin(180◦ − A) = ;
AP AP ′
APPENDIX : Miscellaneous Articles and Propositions 318

and since P M and P ′ M ′ are equal in magnitude and of the same sign, we have
sin A = sin(180◦ − A).
AM AM ′
Also cos A = , cos(180◦ − A) = ;
AP AP ′
now AM and AM ′ are equal in magnitude, but since they are measured in opposite
directions from A, they are of opposite sign; thus
cos A = − cos(180◦ − A).
The other Trigonometrical Ratios of the angle A may be compared with those of the
supplement either by direct use of the figure, or by employing the two results already
established; thus adopting the latter method,
sin(180◦ − A) sin A
tan(180◦ − A) = = = − tan A,
cos(180◦ − A) − cos A

cos(180 − A) − cos A
cot(180◦ − A) = = = − cot A,
sin(180◦ − A) sin A
1 1
sec(180◦ − A) = = = − sec A,
cos(180◦ − A) − cos A
1 1
cosec (180◦ − A) = = = cosec A,
sin(180◦ − A) sin A
vers (180◦ − A) = 1 − cos(180◦ − A) = 1 + cos A.
Thus the sine and the cosecant of any angle are respectively the same as the sine and
cosecant of the supplement of the angle; all the other Trigonometrical Ratios of any
angle, except the versed sine, are numerically equal to the corresponding Ratios of
the supplement of the angle, but are of opposite sign.
APPENDIX : Miscellaneous Articles and Propositions 319

Art. 49 To prove that sin(−A) = − sin A and cos(−A) = cos A.

M
B′ A B

P′

M
B′ A B

P′

Let P AB be any angle; draw P M perpendicular to BAB ′ , and produce it to P ′ so


that M P ′ may be equal in length to M P, and join AP ′ . Then the angles P ′ AB and
P AB which are measured in opposite directions from AB are numerically equal, and
if P AB be denoted by A, then P ′ AB will be denoted by −A. And
PM P ′M
sin A = , sin(−A) = ;
AP AP ′
and P ′ M is numerically equal to P M, but of opposite sign; thus
sin(−A) = − sin A.
AM AM
Also cos(−A) = = = cos A.
AP ′ AP
sin(−A) − sin A
Moreover, tan(−A) = = = − tan A;
cos(−A) cos A
cos(−A) cos A
cot(−A) = = = − cot A;
sin(−A) − sin A
1 1
sec(−A) = = = sec A;
cos(−A) cos A
1 1
cosec (−A) = = = − cosec A;
sin(−A) − sin A
vers (−A) = 1 − cos(−A) = 1 − cos A = vers A.
All these results may if we please be obtained by direct use of the figure.

Art. 50 To prove that


sin(180 ◦ + A) = − sin A and cos(180 ◦ + A) = − cos A.

Let P AB be any angle, produce P A to P ′ so that AP ′ may be equal in length to AP.


Draw P M and P ′ M ′ perpendicular to BAB ′ . Then if P AB be denoted by A, the angle
P ′ AB measured in the same direction from AB will be denoted by 180◦ + A.
APPENDIX : Miscellaneous Articles and Propositions 320

M′
′ A M B
B

P′

The triangles P AM and P ′ AM ′ are geometrically equal in all respects;


PM P ′M ′
and sin A = , sin(180◦ + A) = ;
AP AP ′
AM AM ′
cos A = , cos(180◦ + A) = .
AP AP ′
Now P M and P M are numerically equal but of opposite sign; also AM and AM ′ are
′ ′

numerically equal but of opposite sign; thus


sin(180◦ + A) = − sin A, cos(180◦ + A) = − cos A;
sin(180◦ + A) − sin A
moreover tan(180◦ + A) = = = tan A,
cos(180◦ + A) − cos A

cos(180 + A) − cos A
cot(180◦ + A) = = = cot A;
sin(180◦ + A) − sin A
similarly sec(180◦ + A) = − sec A, cosec (180◦ + A) = − cosec A.
vers (180◦ + A) = 1 − cos(180◦ + A) = 1 + cos A.
All these results may if we please be obtained by direct use of the figure.
It is obviously only another mode of expressing the two fundamental results if we
write
sin A = − sin(A − 180◦ ), cos A = − cos(A − 180◦ ).

Art. 52 To prove that sin(90 ◦ + A) = cos A, and cos(90 ◦ + A) = − sin A.

P′ P

B′ M′ A M B

Let P AB be any angle; let AP ′ be at right angles to AP and so situated that a


moveable straight line can pass from the position AP to the position AP ′ by revolving
round A in the positive direction through a right angle. Then if P AB be denoted by A
we can denote P ′ AB by 90◦ +A. Take AP ′ = AP and draw P M and P ′ M ′ perpendicular
to BAB ′ . Then the angle P AM is geometrically equal to the angle AP ′ M ′ , and the
triangles P AM and P ′ AM ′ are geometrically equal in all respects. And
P ′M ′ AM
sin(90◦ + A) = , cos A = ;
AP ′ AP
′ ′
now P M is numerically equal to AM and both are of the same sign (Art. 42) (page 316);
thus
sin(90◦ + A) = cos A.
AM ′ PM
Again cos(90◦ + A) = , sin A = ;
AP ′ AP
now AM ′ and P M are numerically equal but of opposite sign (Art. 42)(page 316); thus
cos(90◦ + A) = − sin A.
APPENDIX : Miscellaneous Articles and Propositions 321

Art. 66 To find an expression for all the angles which have a given sine.
Let BAC be the least positive angle which has the given sine;

C′ C

B′ A B

denote this angle by α. Produce BA to any point B ′ and make the angle B ′ AC ′ = BAC;
then BAC ′ = π − α.
Now it is obvious from the figure that the only positive angles which have the same
sine as α are π − α, and the angles formed by adding any multiple of four right angles
to α or to π − α; that is, angles included in the formulae 2nπ + α and 2nπ + π − α, where
n is zero or any positive integer. Also the only negative angles which have the same
sine as α are −(π + α), and −(2π − α), and the angles formed by adding to these any
multiple of four right angles taken negatively; that is angles included in the formulae
2nπ − (π + α), and 2nπ − (2π − α), where n is zero or any negative integer. All the
angles which have been indicated will be found on trial to be included in the formula
nπ + (−1)n α, where n is zero, or any integer positive or negative. Also all the angles
included in this formula will be found among the angles which have been indicated.
Thus the formula nπ + (−1)n α includes all the angles which have the same sine as
α, and all the angles which it includes have the same sine as α.
This formula also determines all the angles which have the same cosecant as α.
APPENDIX : Miscellaneous Articles and Propositions 322

Art. 67 To find an expression for all the angles which have a given cosine.

A B

C′

Let BAC be the least positive angle which has the given cosine; denote this angle by
α. Make the angle BAC ′ = BAC.
Now it is obvious from the figure, that the only positive angles which have the
same cosine as α are 2π − α, and the angles formed by adding any multiple of four
right angles to α or to 2π − α; that is, angles included in the formulae 2nπ + α and
2nπ + 2π − α, where n is zero or any positive integer. Also the only negative angles
which have the same cosine as α are −α, and −(2π − α), and the angles formed by
adding to these any multiple of four right angles taken negatively; that is, angles
included in the formulae 2nπ − α and 2nπ − (2π − α), where n is zero or any negative
integer.
All the angles which have been indicated will be found on trial to be included in
the formulae 2nπ ± α, where n is zero or any integer positive or negative. Also all
the angles included in this formula will be found among the angles which have been
indicated.
Thus the formula 2nπ ± α included all the angles which have the same cosine as α,
and all the angles which it includes have the same cosine as α.
This formula also determines all the angles which have the same secant or the
same versed sine as α.

Art. 68 To find an expression for all the angles which have a given tangent,
Let BAC be the least positive angle which has the given tangent; denote this angle
by α. Produce BA to any point B ′ and CA to any point C ′ .

B′ A B

C′

Now it is obvious from the figure that the only positive angles which have the same
tangent as α are π + α, and the angles formed by adding any multiple of four right
angles to α or to π + α; that is, angles included in the formulae 2nπ + α and 2nπ + π + α,
where n is zero or any positive integer. Also the only negative angles which have the
APPENDIX : Miscellaneous Articles and Propositions 323

same tangent as α are −(π − α), and −(2π − α), and the angles formed by adding to
these any multiple of four right angles taken negatively; that is, angles included in
the formulae 2nπ − (π − α) and 2nπ − (2π − α), where n is zero or any negative integer.
All the angles which have been indicated will be found on trial to be included in the
formula nπ +α, where n is zero, or any integer positive or negative. Also all the angles
included in this formula will be found among the angles which have been indicated.
Thus the formula nπ + α includes all the angles which have the same tangent as α,
and all the angles which it includes have the same tangent as α.
Thus the formula also determines all the angles which have the same cotangent
as α.

Art. 71 Before leaving this part of the subject we will recur to the definitions of the
Trigonometrical Ratios; we considered them as ratios formed by comparing the sides
of a right-angled triangle, but formerly they were differently defined, and it is advis-
able to notice the old definitions in order that the student may understand allusions
to them which will occur in his reading.
C t

T
P

A M B

Let A be the centre of any circle, AB a radius, BP any arc; draw the radius AC at
right angles to AB, and draw tangents to the circle at the points B and C; produce AP
to meet the first tangent at T and the second tangent at t; draw P M perpendicular to
AB. Then the old definitions are as follows, in which the straight lines of the figure
are considered to be functions of the arc BP. P M is the sine of the arc BP, AM is
its cosine, BT is its tangent, Ct is its cotangent, AT is its secant, At is its cosecant,
BM is its versed sine; also the straight line joining B and P is the chord of the arc
BP. Thus the terms sine, cosine, &c., formerly denoted certain straight lines and not
certain ratios. On the old system the lengths of the sine, cosine, &c. depended on
the radius of the circle considered, so that it became necessary to state what length
was ascribed to this radius in any investigation.

Art. 76 To express the sine and the cosine of the sum of two angles in terms of the
sines and the cosines of the angles themselves.
APPENDIX : Miscellaneous Articles and Propositions 324

R N

O C
M Q
Let the angle COD be denoted by A, and the angle DOE by B; then the angle COE
will be denoted by A + B. In OE take any point P, draw P M perpendicular to OC, and
P N perpendicular to OD; draw N R perpendicular to P M and N Q perpendicular to
OC.
Then the angle N P R is the complement of P N R, and is therefore equal to RN O,
which is equal to N OC or A.
PM RM + P R NQ PR
Now sin(A + B) = = = +
OP OP OP OP
N Q ON PR PN
= · + ·
ON OP P N OP
= sin A cos B + cos A sin B.
OM OQ − QM OQ NR
cos(A + B) = = = −
OP OP OP OP
OQ ON NR NP
= · − ·
ON OP N P OP
= cos A cos B − sin A sin B.

Art. 82 In the expressions for sin(A + B) and cos(A + B) put B = A; thus


sin 2A = 2 sin A cos A;
cos 2A = cos2 A − sin2 A = 1 − 2 sin2 A = 2 cos2 A − 1.
Thus 1 + cos 2A = 2 cos2 A,
1 − cos 2A = 2 sin2 A,
1 − cos 2A
and = tan2 A.
1 + cos 2A
sin 2A sin 2A
Also = tan A, = cot A.
1 + cos 2A 1 − cos 2A

Art. 83
sin(A + B) sin(A − B)
= (sin A cos B + cos A sin B)(sin A cos B − cos A sin B)
= sin2 A cos2 B − cos2 A sin2 B
( ) ( )
= sin2 A 1 − sin2 B − 1 − sin2 A sin2 B
= sin2 A − sin2 B.
This result is very important.

And cos(A + B) cos(A − B)


= (cos A cos B − sin A sin B)(cos A cos B + sin A sin B)
= cos2 A cos2 B − sin2 A sin2 B
APPENDIX : Miscellaneous Articles and Propositions 325

( ) ( )
= cos2 A 1 − sin2 B − 1 − cos2 A sin2 B
= cos2 A − sin2 B = cos2 B − sin2 A.

Art. 84 From the four fundamental formulae we have


sin(A + B) + sin(A − B) = 2 sin A cos B,
sin(A + B) − sin(A − B) = 2 cos A sin B,
cos(A + B) + cos(A − B) = 2 cos A cos B,
cos(A − B) − cos(A + B) = 2 sin A sin B.
Let A + B = C and A − B = D; therefore
1 1
A = (C + D) and B = (C − D); thus
2 2
C+D C−D
sin C + sin D = 2 sin cos ,
2 2
C+D C−D
sin C − sin D = 2 cos sin ,
2 2
C+D C−D
cos C + cos D = 2 cos cos ,
2 2
C+D C−D
cos D − cos C = 2 sin sin .
2 2
These formulae will be found to be extremely useful in mathematical investigations;
they enable us to put the sum or the difference of two sines or two cosines in the form
of a product; or to replace the product of a sine or a cosine into a sine or a cosine by
half the sum or half the difference of two such Ratios.

Art. 87
2 sin A cos A
sin 2A = 2 sin A cos A = (Arts. 82 (page 324) and 32 (page 316);
sin2 A + cos2 A
Divide both numerator and denominator of the last expression by cos2 A;
2 sin A
thus we get cos A ;
sin2 A
1+
cos2 A
2 tan A
therefore sin 2A = .
1 + tan2 A
cos2 A − sin2 A
Also cos 2A = cos2 A − sin2 A =
cos2 A + sin2 A
(Arts. 82 (page 324) and 32 (page 316)
sin2 A
1−
cos2 A = 1 − tan A .
2
= 2
sin A 1 + tan2 A
1+ 2
cos A

Art. 88
A+B A−B
sin A + sin B 2 sin cos
= 2 2 (Art. 84) (page 325)
sin A − sin B A+B A−B
2 cos sin
2 2
A+B
tan
= 2 ;
A−B
tan
2
A+B A−B
cos A + cos B 2 cos cos
= 2 2 (Art. 84) (page 325)
cos B − cos A A+B A−B
2 sin sin
2 2
A+B A−B
= cot cot ;
2 2
APPENDIX : Miscellaneous Articles and Propositions 326

A+B A−B
sin A + sin B 2 sin cos A+B
= 2 2 = tan .
cos A + cos B A+B A−B 2
2 cos cos
2 2
A+B A−B
sin A − sin B 2 cos sin A+B
= 2 2 = cot .
cos B − cos A A+B A−B 2
2 sin sin
2 2

Art. 91
sin 3A = sin(2A + A) = sin 2A cos A + cos 2A sin A (Art. 76)(page 323)
= 2 sin A cos2 A + (1 − 2 sin2 A) sin A
= 2 sin A(1 − sin2 A) + (1 − 2 sin2 A) sin A
= 3 sin A − 4 sin3 A.
cos 3A = cos(2A + A) = cos 2A cos A − sin 2A sin A (Art. 76)(page 323)
= (2 cos2 A − 1) cos A − 2 cos A sin2 A
= (2 cos2 A − 1) cos A − 2 cos A(1 − cos2 A)
= 4 cos3 A − 3 cos A.
sin 3A 3 sin A − 4 sin3 A
Hence tan 3A = = .
cos 3A 4 cos3 A − 3 cos A
Divide both numerator and denominator by cos3 A; thus
3 tan A
3
− 4 tan3 A
tan 3A = cos
3
4−
cos2 A
3 tan A(1 + tan2 A) − 4 tan3 A
= (Art. 34) (page 316)
4 − 3(1 + tan2 A)
3 tan A − tan3 A
= .
1 − 3 tan2 A

Art. 93 If sin A = sin B and cos A = cos B, then either A and B are equal, or they differ
by some multiple of four right angles.

For cos(A − B) = cos A cos B + sin A sin B


= cos2 A + sin2 A = 1
therefore A − B = 0, or a multiple of four right angles taken positively or negatively.
(Art. 67 : page 322.)

Art. 94 If cos A = cos B and sin A = − sin B, then A + B is zero, or a multiple of four
right angles positive or negative.

For the given relations may be written


cos A = cos(−B), sin A = sin(−B). (Art. 49 : page 319.)
Hence by the preceding Article A − (−B), that is A + B, is zero or some multiple of
four right angles taken positively or negatively.

Art. 98 By Art. 82 (page 324)


A A
sin A = 2 sin cos ,
2 2
A A
also 1 = sin2 + cos2 ,
2 2
( )
A A 2
thus sin + cos = 1 + sin A,
( 2 2
)2
A A
and sin − cos = 1 − sin A;
2 2
APPENDIX : Miscellaneous Articles and Propositions 327

A A √
therefore sin + cos = 1 + sin A (44)
2 2
A A √
and sin − cos = 1 − sin A (45)
2 2
A √ √
therefore 2 sin = 1 + sin A + 1 − sin A,
2
A √ √
and 2 cos = 1 + sin A − 1 − sin A.
2

Art. 100 If sin A only be given and nothing more be known respecting A, then the
ambiguities of sign which occur in Art. 98 (page 326) cannot be removed. If however
A itself be given, or if we merely know in which quadrant the angle A lies, we can
determine the proper signs; for in any particular case we may proceed thus. We have
A A √
sin + cos = ± 1 + sin A (46)
2 2
A A √
sin − cos = ± 1 − sin A (47)
2 2
A
Now suppose, for example, that A lies between 0 and 90◦ , then lies between 0 and
2
◦ A A A A
45 ; therefore cos and sin are both positive and cos is greater than sin ; hence
2 2 2 2
the left-hand member of (46) is a positive quantity, and we must therefore take the
positive sign in (46), and the left-member of (47) is a negative quantity, and we must
therefore take the negative sign in (47). Therefore if A lies between 0 and 90◦ , we
have
A A √ A A √
sin + cos = + 1 + sin A, sin − cos = − 1 − sin A;
2 2 2 2
A √ √
∴ 2 sin = + 1 + sin A − 1 − sin A,
2
A √ √
2 cos = + 1 + sin A + 1 − sin A.
2
A
For another example, suppose that A lies between 270◦ and 360◦ , then lies between
2
A A A
135◦ and 180◦ ; therefore cos is negative, and sin is positive, and cos is numer-
2 2 2
A
ically greater than sin ; hence the left-hand member of (46) is a negative quantity,
2
and we must therefore take the negative sign in (46), and the left-hand member of (47)
is a positive quantity, and we must therefore take the positive sign in (47). Therefore
if A lies between 270◦ and 360◦ , we have
A A √ A A √
sin + cos = − 1 + sin A, sin − cos = + 1 − sin A;
2 2 2 2
A √ √
∴ 2 sin = − 1 + sin A + 1 − sin A,
2
A √ √
2 cos = − 1 + sin A − 1 − sin A.
2

Art. 101 It is easy to give general formulae for determining the signs of
A A A A
sin + cos and sin − cos .
2 2 2 2
A A √ ( 1 A 1 A
) √ (
A π
)
For sin + cos = 2 √ sin + √ cos = 2 sin + ;
2( 2) 2 2 2 2 2 4
A π A π
Now sin + is positive if + lies between 2nπ and (2n + 1)π, and negative
2 4 2 4
A π
if + lies between (2n + 1)π and (2n + 2)π, where n is zero or any integer positive
2 4
A A A π 3π
or negative. Thus sin + cos is positive if lies between 2nπ − and 2nπ + ,
2 2 2 4 4
A 3π 7π
and negative if lies between 2nπ + and 2nπ + .
2 4 4
APPENDIX : Miscellaneous Articles and Propositions 328

√ ( )
A A A π A A
Similarly sin −cos = 2 sin − ; and hence we can infer that sin −cos
2 2 2 4 2 2
A π 5π A
is positive if lies between 2mπ + and 2mπ + , and negative if lies between
2 4 4 2
5π 9π
2mπ + and 2mπ + , where m is zero or any integer positive or negative.
4 4
A
We will apply this to an example : required the limits between which must lie
2
in order that
A √ √
2 sin = − 1 + sin A − 1 − sin A.
2
To obtain this result the lower sign must be taken in (46) and in (47) of Art. 100
A 3π 7π
(page 327); thus from (46) we infer that must lie between 2nπ + and 2nπ + ;
2 4 4
A 5π 9π
and from (47) we infer that must lie between 2mπ + and 2mπ + : hence,
2 4 4
A 5π 7π
combining these results, we see that must lie between 2nπ + and 2nπ + ,
2 4 4
where n is zero or any integer positive or negative.

Art. 108 To find the sine and the cosine of an angle of 36 ◦ .


(√ )2 √
◦ ◦ 5−1 6−2 5
cos 36 = 1 − 2 sin 18 = 1 − 2
2
=1−
4 8
√ √
3− 5 1+ 5
=1− = ,
4 4√
√ √
10 − 2 5
sin 36◦ = 1 − cos2 36◦ = .
4

Art. 113 It is easy to find expressions for the Trigonometrical Ratios of any compound
angle in terms of the Ratios of the component angles. For example,
sin(A + B + C) = sin(A + B) cos C + cos(A + B) sin C
= sin A cos B cos C + sin B cos C cos A + sin C cos A cos B − sin A sin B sin C.
cos(A + B + C) = cos(A + B) cos C − sin(A + B) sin C
= cos A cos B cos C − cos A sin B sin C − cos B sin A sin C − cos C sin A sin B.
sin(A + B + C)
tan(A + B + C) =
cos(A + B + C)
sin A cos B cos C + sin B cos C cos A + sin C cos A cos B − sin A sin B sin C
= ;
cos A cos B cos C − cos A sin B sin C − cos B sin A sin C − cos C sin A sin B
divide both numerator and denominator of the last expression by
cos A cos B cos C; thus we obtain
tan A + tan B + tan C − tan A tan B tan C
tan(A + B + C) = .
1 − tan B tan C − tan C tan A − tan A tan B
Suppose B and C each equal to A; thus we have
3 tan A − tan3 A
tan 3A = .
1 − 3 tan2 A

Art. 114 When three or more angles are connected by some relation, we may often
find that some simple relation exists among some of their Trigonometrical Ratios,
thus, for example,
if
A + B + C = 180◦ ,
then will
sin 2A + sin 2B + sin 2C = 4 sin A sin B sin C.
For
sin 2A + sin 2B = 2 sin(A + B) cos(A − B) = 2 sin C cos(A − B),
APPENDIX : Miscellaneous Articles and Propositions 329

and
sin 2C = 2 sin C cos C = −2 sin C cos(A + B),
∵ cos(C) = cos (180◦ − (A + B)) = − cos(A + B).
∴ sin 2A + sin 2B + sin 2C = 2 sin C {cos(A − B) − cos(A + B)}
= 4 sin C sin A sin B.
Again, if A + B + C = 180◦ , then will
tan A + tan B + tan C = tan A tan B tan C.
For tan 180◦ = 0, therefore tan(A + B + C) = 0; therefore by Art. 113, (page 328)
tan A + tan B + tan C − tan A tan B tan C = 0
Again, by Art. 113, (page 328)
1 − tan B tan C − tan C tan A − tan A tan B
cot(A + B + C) = ;
tan A + tan B + tan C − tan A tan B tan C
now cot 90◦ = 0; hence we have the following result,
if A + B + C = 90◦ , then will
1 = tan B tan C + tan C tan A + tan A tan B.

Art. 116. If θ be the circular measure of a positive angle less than a right angle, θ is
greater than sin θ and less than tan θ.
Let AOB be an angle less than a right angle and let OB = OA; from B draw BM
perpendicular to OA and produce it to C so that M C = M B; draw BT at right angles
to OB meeting OA produced at T, and join CT and OC. Then the triangles M OC and
M OB are equal in all respects, so that the angle T OC = the angle T OB; therefore the
triangles T OC and T OB are equal in all respects, so that T CO is a right angle, and
T C = T B.
With centre O and radius OB describe OB describe an arc of a circle BAC; this
will touch BT at B and CT at C.
Now we assume as an axiom that the straight line BC is less that the arc BAC;
BM
thus BM the half of BC is less than BA the half of the arc BAC; therefore is less
OB
BA
than ; that is, the sine of AOB is less than the circular measure of AOB.
OB
B

O T
M A

Again, we assume as an axiom that the arc BAC is less than the sum of the two
BA BT
exterior lines BT and T C; thus BA is less than BT ; therefore is less than ;
OB OB
that is, the circular measure of AOB is less than the tangent of AOB.
π
Hence sin θ, θ, and tan θ are in ascending order of magnitude if θ be less than .
2
APPENDIX : Miscellaneous Articles and Propositions 330

sin θ
Art. 118. The limit of when θ is indefinitely diminished is unity.
θ
For sin θ, θ, and tan θ are in ascending order of magnitude ; divide by sin θ; therefore
θ 1 θ
1, , and are in ascending order of magnitude. Thus lies in value be-
sin θ cos θ sin θ
1
tween 1 and ; but when θ is zero, cos θ is unity; hence as θ diminishes indefinitely
cos θ
θ sin θ
approaches the limit unity. Therefore also approaches the limit unity.
sin θ θ
α
Art. 119. From the preceding Article we see that the limit of m sin when m increases
m
indefinitely is α.
α α α α α
For m sin = α sin ÷ ; and when m is indefinitely great sin ÷ is unity.
m m m m m
α
Similarly the limit of m tan when m increases indefinitely is α.
m
It must be carefully remembered that in the important proposition of the preceding
Article, θ is the circular measure of the angle considered. If any other unit of angular
measurement be adopted instead of the unit of circular measure, the limit under
sin n◦
consideration will not be unity. For example, let us find the limit of when n is
n
indefinitely diminished. Let θ be the circular measure of an angle of n degree, then

θ= ; thus
180
sin n◦ sin θ π sin θ
= = · .
n 180 180 θ
θ
π
sin θ
Now when n diminishes indefinitely, θ does so also, and the limit of is unity;
θ
sin n◦ π
hence the limit of when n is diminished indefinitely is , which is the circular
n 180
sin n′
measure of an angle of one degree. Similarly we may prove that the limit of
n
when n is indefinitely diminished is the circular measure of an angle of one minute;
and so on. Thus we shall find that, whatever be the unit of angular measurement, the
limit of the ratio of the sine of an angle to the angle, when the angle is indefinitely
diminished, is the circular measure of the unit.

Art. 120. If θ be the circular measure of a positive angle less than a right angle, sin θ
θ3
is greater than θ − .
4
θ θ θ θ θ
For sin θ = 2 sin cos ; and tan is greater than , therefore sin is greater than
2 2 2 2 2
θ θ θ θ θ
cos ; therefore sin θ is greater than 2 cos2 , that is greater than θ cos2 , that is
2 2 ( ) 2 2 ( )2 2
θ θ θ
greater than θ 1 − sin2 . And sin2 is less than , therefore a fortiori sin θ is
( )2 2 2
θ2 θ3
greater than θ 1 − ; that is, sin θ is greater than θ − .
4 4

Art. 121. Thus we see that if θ lie between zero and a right angle sin θ is less than θ
θ3 θ θ θ θ3
and greater than θ − ; and therefore sin is less than and greater than − .
4 2 2 2 32
( )2
θ θ
Now cos θ = 1 − 2 sin2 . Thus cos θ is greater than 1 − 2 , that is greater than
2 ( )2 2 ( )2
θ2 θ θ3 θ2 θ4 θ3
1 − . Also cos θ is less than 1 − 2 − , that is less than 1 − + −2 ;
2 2 32 2 16 32
θ 2 θ 4
therefore a fortiori cos θ is less than 1 − + .
2 16
APPENDIX : Miscellaneous Articles and Propositions 331

x x x x
Art. 129. The limit of cos cos cos . . . cos n when the integer n is indefinitely in-
2 4 8 2
sin x
creased is .
x
For
x x
sin x = 2 sin cos
2 2
x x x
= 4 sin cos cos
4 4 2
x x x x
= 8 sin cos cos cos
8 8 4 2
............
x x x x x
= 2n sin n cos n . . . cos cos cos .
2 2 8 4 2
x x x x sin x
∴ cos cos cos . . . cos n = x .
2 4 8 2 2n sin n
2
Now
x
sin xsin x 2n
x = x x ,
2n sin n sin n
2 2
sin x
and the limit of this when n is indefinitely increased is , since by Art. 118 (page 330),
x
x
n
the limit of 2 x is unity. This result is sometimes cited as Euler’s Formula.
sin n
2

Art. 130. To prove that if x be the circular measure of a positive angle less than a
x3
right angle sin x is greater than x − .
6
We have, by Art. 91 (page 326),
x x
sin x = 3 sin − 4 sin3
( 3 3 )
x x x
= 3 3 sin 2 − 4 sin3 2 − 4 sin3
3 3 3
x x x
= 32 sin 2 − 4 sin3 − 4 × 3 sin3 2
( 3 3 ) 3
x x x x
= 33 3 sin 3 − 4 sin3 3 − 4 sin3 − 4 × 3 sin3 2
3 { 3 3 } 3
x x x x
= 33 sin 3 − 4 sin3 + 3 sin3 2 + 32 sin3 3 .
3 3 3 3
Proceeding in this way we see { that }
x x x x
sin x = 3n sin n − 4 sin3 + 3 sin3 2 + . . . + 3n−1 sin3 n .
3 3 3 3
Hence, by Art. 116 (page 329), sin x is greater than
{ }
x 4x3 1 1 1
3n sin n − 3 1 + 2 + 4 + . . . + 2n−2 ,
3 3 3 3 3
1
x 4x3 1 − 32n
that is greater than 3 sin n −
n . Thus sin x exceeds the last expression;
3 3 32 − 1
and the excess does not vanish however great n may be : therefore sin x exceeds the
limit to which the last expression approaches when n is made infinite.
x 1
But the limit of 3n sin n is x by Art. 119 (page 330); and the limit of 1 − 2n is 1 :
3 3
thus
x3
sin x is greater than x − .
6
By proceeding as in Art. 121 (page 330), we may now show that
APPENDIX : Miscellaneous Articles and Propositions 332

x2 x4
cos x is less than 1 − + .
2 24
(Le Cointe’s Trigonometry, and Messenger of Mathematics, iii. 101.)

Art. 135. The logarithm of the base itself is unity.


For ax = a when x = 1.

Art. 139. To find the relation between the logarithms of the same number to different
bases.
Let
x = loga m, y = logb m;
∴ m = ax and = by ;
∴ ax = by ;
x y
∴ a y = b, and b x = a;
x y
∴ = loga b, and = logb a.
y x
x
∴ y = x logb a, and = .
loga b
Hence the logarithm of a number to the base b may be found by multiplying the log-
arithm of the number to the base a by
1
logb a, or by .
loga b
We may notice that logb a × loga b = 1.

Art. 143. To expand ax in a series of ascending powers of x; that is, to expand a number
in a series of ascending powers of its logarithm to a given base.
x(x − 1)
ax = {1 + (a − 1)}x = 1 + x(a − 1) + (a − 1)2
1·2
x(x − 1)(x − 2) x(x − 1)(x − 2)(x − 3)
+ (a − 1)3 + (a − 1)4 + . . .
·2·3
1{ 1·2·3·4 }
1 1 1
= 1 + x a − 1 − (a − 1)2 + (a − 1)3 − (a − 1)4 + . . .
2 3 4
+ terms involving x2 , x3 , . . . .
This shows that ax can be expanded in a series beginning with 1 and proceeding in
ascending powers of x; we may therefore suppose that
ax = 1 + c1 x + c2 x2 + c3 x3 + c4 x4 + . . .
where c1 , c2 , c3 , . . . are quantities which do not depend on x, and which therefore
remain unchanged however x may be changed; also
1 1 1
c1 = a − 1 − (a − 1)2 + (a − 1)3 − (a − 1)4 + . . .
2 3 4
while c2 , c3 , . . . are at present unknown; we proceed to find their values. Changing x
into x + y we have
ax+y = 1 + c1 (x + y) + c2 (x + y)2 + c3 (x + y)3 + . . . ;
{ }
but ax+y = ax ay = ay 1 + c1 x + c2 x2 + c3 x3 + . . . .
Since the two expressions for ax+y are identically equal, we may assume that the
coefficients of x in the two expressions are equal, thus
c1 + 2c2 y + 3c3 y 2 + 4c4 y 3 + . . . = c1 ay
{ }
= c1 1 + c1 y + c2 y 2 + c3 y 3 + . . . .
In this identity we may assume the coefficients of the corresponding powers of y are
equal; thus
c2
2c2 = c21 ; ∴ c2 = 1 ,
2
c1 c2 c31
3c3 = c1 c2 ; ∴ c3 = = ,
3 1·2·3
c14
c1 c3
4c4 = c1 c3 ; ∴ c4 = = .
4 1·2·3·4
APPENDIX : Miscellaneous Articles and Propositions 333

............
c21 x2 c31 x3 c41 x4
Thus, ax = 1 + c1 x + + + + ...
2 3 4
1
Since this result is true for all values of x, take x such that c1 x = 1, then x = and
c1
1 1 1 1
a c1 = 1 + 1 + + + + ...;
2 3 4
1
this series is usually denoted by e; thus a c1 = e, therefore a = ec1 and c1 = loge a;
hence,
(loge a)2 x2 (loge a)3 x3
ax = 1 + (loge a) x + + + ...
2 3
This result is called the Exponential Theorem.
Put e for a then loge a becomes loge e, that is, unity, (Art. 135) (page 332); thus,
x2 x3 x4
ex = 1 + x + + + + ...
2 3 4
With respect to the assumption which has been made twice in the course of this
article, the student is referred to the chapter on Indeterminate Coefficients in the
Algebra.

Art. 145. To expand loge (1 + x) in a series of ascending powers of x.


We have seen in Art. 143 (page 332), that c1 = loge a; that is, by the same Article,
1 1 1
loge a = a − 1 − (a − 1)2 + (a − 1)3 − (a − 1)4 + . . .
2 3 4
For a put 1 + x; hence,
x2 x3 x4
loge (1 + x) = x − + − + ...
2 3 4
This series may be applied to calculate loge (1 + x) if x is a proper fraction; but unless
x be very small, the terms diminish so slowly that we shall have to retain a large
number of them ; if x be greater than unity, the series is altogether unsuitable. We
shall therefore deduce some more convenient formulae.

Art. 146. We have


x2 x3 x4
loge (1 + x) = x − + − + ...
2 3 4
x 2 x 3 x 4
∴ loge (1 − x) = −x − − − − ...,
2 3 4
1+x
by subtraction we obtain the value of loge (1 + x) − loge (1 − x), that is, of loge ;
{ } 1−x
1+x x3 x5
∴ loge =2 x+ + + ... ...
1−x 3 5
m−n m 1+x
In this series write for x, and therefore for , thus
m +{n n 1−x }
( )3 ( )
m m−n 1 m−n 1 m−n 5
loge =2 + + + ... ... (48)
n m+n 3 m+n 5 m+n
Put n = 1, then { }
( ) ( )
m−1 1 m−1 3 1 m−1 5
loge m = 2 + + + ... ... (49)
m+1 3 m+1 5 m+1
n+1
Again in (48) put m = n + 1, thus we obtain the value of loge ; therefore
{ n }
1 1 1
loge (n + 1) − loge n = 2 + + + . . . ... (50)
2n + 1 3(2n + 1)3 5(2n + 1)5

Art. 148. From Art. 139 (page 332), we see that the logarithm of a number to the
1
base 10 can be found by multiplying the Napierian logarithm by , that is, by
loge 10
APPENDIX : Miscellaneous Articles and Propositions 334

1
, or by .43429448; this multiplier is called the modulus of the common sys-
2.30258509
tem.
The series in Art. 146 (page 333), may be so adjusted as to give common logarithms;
for example, take the series (50), multiply throughout by the modulus which we shall
denote by µ; thus { }
1 1 1
µ loge (n + 1) − µ loge n = 2µ + 3
+ 5
+ ... ;
{2n + 1 3(2n + 1) 5(2n + 1)
}
1 1 1
∴ log10 (n + 1) − log10 n = 2µ + + + . . . ;
2n + 1 3(2n + 1)3 5(2n + 1)5
Similarly from Art. 145 (page 333), we { have }
x2 x3 x4
log10 (1 + x) = µ x− + − + ... .
2 3 4

( )n
α
Art. 150. To find the limit of cos when n is increased indefinitely.
n
( )n ( ) n2
α α
Let u = cos = 1 − sin2 ; then
n n
( ) n2 ( )
n α α
log u = log 1 − sin2 log 1 − sin2 =
( 2 n n )
n α 1 α 1 α
=− sin2 + sin4 + sin6 + . . . .
2 n 2 n 3 n
α
α sin
Now n sin = α α n = α when n is increased indefinitely (Art. 118) (page 334);
n
n
α α α α
therefore n sin2 = α sin = 0 ultimately; and similarly n sin4 , n sin6 , . . . vanish
n n n n
ultimately. Therefore log u = 0; therefore u = 1. Thus the required limit is unity.

Art. 181. To prove that in general the change of the sine of an angle is approximately
proportional to the change of the angle.

We have sin(θ + h) − sin θ = sin h cos θ − sin θ(1 − cos h)


( )
1 − cos h
= sin h cos θ 1 − tan θ
( sin h)
h
= sin h cos θ 1 − tan θ tan .
2
Let us now suppose that h is the circular measure of a very small angle so that
sin h = h approximately; thus, approximately,
( )
h
sin(θ + h) − sin θ = h cos θ 1 − tan θ tan ;
2
π
Let us also suppose that θ is not very nearly equal to so that tan θ is not very
2
h
large, and thus tan θ tan may be neglected.
2
We have then, approximately,
sin(θ + h) − sin θ = h cos θ,
and this establishes the proposition.

Similarly sin(θ − h) − sin θ = −h cos θ approximately.

Art. 188. To prove that in general the change of the tangent of an angle is approxi-
mately proportional to the change of the angle.
We have
sin(θ + h) sin θ
tan(θ + h) − tan θ = −
cos(θ + h) cos θ
APPENDIX : Miscellaneous Articles and Propositions 335

sin(θ + h) cos θ − cos(θ + h) sin θ sin(θ + h − θ) sin h


= = =
cos(θ + h) cos θ cos(θ + h) cos θ cos(θ + h) cos θ
sin h tan h
= = .
cos2 θ(cos h − sin h tan θ) cos2 θ(1 − tan θ tan h)
Let us now suppose that h is so small that we may put h for tan h, and also that θ is not
π
nearly equal to so that tan θ tan h may be neglected. We have then, approximately,
2
h
tan(θ + h) − tan θ = = h sec2 θ,
cos2 θ
also by changing the sign of h
tan(θ − h) − tan θ = −h sec2 θ;
this establishes the proposition.

Art. 194.
We have shown that approximately
sec(θ + h) − sec θ = h sin θ sec2 θ;
π
change θ into − θ′ , thus
2( ) ( ) ( ) ( )
π π π π
sec − θ′ + h − sec − θ′ = h sin − θ′ sec2 − θ′ ,
2 ( ) 2 2 2
that is cosec θ′ − h − cosec θ′ = h cos θ′ cosec 2 θ′ ,
and by changing the sign of h
( )
cosec θ′ + h − cosec θ′ = −h cos θ′ cosec 2 θ′ .
This may also be proved independently.

Art. 196. To prove that in general the change of the tabular logarithmic sine of an
angle is approximately proportional to the change of the angle.
We have approximately
sin(θ + h) = sin θ + h cos θ,
sin(θ + h)
∴ = 1 + h cot θ;
sin θ
sin(θ + h)
∴ log sin(θ + h) − log sin θ = log = log(1 + h cot θ), and
sin θ
log(1 + h cot θ) = µh cot θ approximately (Art. 148) (page 333),
where µ is the modulus; thus approximately
log sin(θ + h) − log sin θ = µh cot θ,
also by changing the sign of h,
log sin(θ − h) − log sin θ = −µh cot θ.
If L stand for tabular logarithm, we have
L sin(θ + h) = 10 + log sin(θ + h),
L sin θ = 10 + log sin θ;
∴ L sin(θ ± h) − L sin θ = ±µh cot θ.
This establishes the proposition.

Art. 208. If we have to determine an angle from its natural sine or cosine it will be
advisable to employ the natural sine if the angle be less than 45◦ , and the natural
cosine if the angle be greater than 45◦ . For the differences of consecutive sines vary
approximately as the cosine of the angle, and the differences of consecutive cosines
vary approximately as the sine of the angle; thus the differences of consecutive sines
are greater or less than the differences of consecutive cosines according as the angle
is less or greater than 45◦ . A similar remark holds for the logarithmic sine and cosine.

Art. 212. In a right-angled triangle each side is equal to the product of the hypotenuse
into the cosine of the adjacent angle.
APPENDIX : Miscellaneous Articles and Propositions 336

Let ABC be a triangle having a right angle at C; then


AC BC
= cos A, = cos B;
AB AB
∴ b = c cos A, a = c cos B.
Since cos A = sin B and cos B = sin A,
we may also enunciate the proposition
thus : in a right-angled triangle each
A
side is equal to the product of the hy-
potenuse into the sine of the opposite
angle.

Art. 214. In any triangle the sides are


proportional to the sines of the opposite
angles.
Let ABC be any triangle, and from A B C
draw AD perpendicular to the opposite
side meeting that side, or that side pro-
duced, at D.
If B and C are acute angles we have from the left-hand figure,
AD = AB sin B, and AD = AC sin C;
∴ AB sin B = AC sin C,
c sin C
∴ = .
b sin B
A A

B D C B C D
If the angle C be obtuse we have from the right-handed figure,
AD = AB sin B, and AD = AC sin ACD = AC sin (180◦ − C) = AC sin C;
∴ AB sin B = AC sin C,
c sin C
∴ = .
b sin B
If the angle C be a right angle, we have from the figure of Art. 212 (page 335),
AC = AB sin B,
c 1 sin C
∴ = = .
b sin B sin B
Thus it is proved that in every case
c sin C
= .
b sin B
Similarly
a sin A a sin A
= , and = .
b sin B c sin C
The results may be written symmetrically thus,
sin A sin B sin C
= = ;
a b c
1
and we shall show hereafter that each of these is equal to , where R is the radius
2R
of the circle described round the triangle.

Art. 215. To express the cosine of an angle of a triangle in terms of the sides.
Let ABC be a triangle, and suppose C an acute angle. (See the left-hand figure of
the preceding Article.) Then by Euclid ii. 13,
AB 2 = BC 2 + AC 2 − 2BC · CD, and
APPENDIX : Miscellaneous Articles and Propositions 337

CD = AC cos C;
∴ c2 = a2 + b2 − 2ab cos C.
Next suppose C an obtuse angle. (See the right-hand figure of the preceding Arti-
cle.) Then by Euclid ii. 12,
AB 2 = BC 2 + AC 2 + 2BC · CD, and
CD = AC cos(180◦ − C) = −AC cos C,
∴ c2 = a2 + b2 − 2ab cos C.
a2 + b2 − c2
Thus in both cases we have cos C = .
2ab
Moreover when C is a right angle, a2 + b2 = c2 and cos C is zero; thus the formula
just found for cos C is true in every case.
Similarly
b2 + c2 − a2 c2 + a2 − b2
cos A = , cos B = .
2bc 2ca

Art. 216. In every triangle each side is equal to the sum of the product of each of the
other sides into the cosine of the angle which it makes with the first side.
From the left-hand figure in Art. 214 (page 336), we have
BC = BD + DC = AB cos B + AC cos C,
∴ a = c cos B + b cos C.
From the right-hand figure in Art. 214 (page 336), we have
BC = BD − DC = AB cos B − AC cos(180◦ − C),
= AB cos B + AC cos C,
∴ a = c cos B + b cos C.
Similarly we shall have b = a cos C + c cos A, and c = b cos A + a cos B.

Art. 217. To express the sine, the cosine, and the tangent of half an angle of a triangle
of the sides.
We have by Art. 215 (page 336),
b2 + c2 − a2
cos A = ,
2bc
b2 + c2 − a2 a2 − (b − c)2
∴ 1 − cos A = 1 − = ;
2bc 2bc
A (a + b − c)(a + c − b)
∴ sin2 = .
2 4bc
Let 2s = a + b + c, so that s is half the sum of the sides of the triangle; then
a + b − c = a + b + c − 2c = 2(s − c),
a + c − b = a + b + c − 2b = 2(s − b).

A (s − b)(s − c) A (s − b)(s − c)
∴ sin
2
= , ∴ sin = .
2 bc 2 bc
Also
b2 + c2 − a2 (b + c)2 − a2
1 + cos A = 1 + = ;
2bc 2bc
A (a + b + c)(b + c − a) s(s − a)
∴ cos2 = = , and
2 4bc √ bc
A s(s − a)
∴ cos = .
2 bc
A A
From the values of sin and cos we deduce
2 2 √
A (s − b)(s − c)
tan = ,
2 s(s − a)
The positive sign must be given to the radicals which occur in this Article, because
A
is less than a right angle, and therefore its sine, cosine, and tangent are all positive.
2
APPENDIX : Miscellaneous Articles and Propositions 338

Similar expressions hold for the sine, the cosine, and the tangents of half of each
of the other angles.

Art. 218. To express the sine of an angle of a triangle in terms of the sides.
A A
Since sin A = 2 sin cos , we obtain
2 2 √ √
(s − b)(s − c) s(s − a)
sin A = 2 ·
bc bc
2√
s(s − a)(s − b)(s − c).
=
bc
Or we may find sin A directly from the known value of cos A;
( )2
b2 + c2 − a2 2b2 c2 + 2c2 a2 + 2a2 b2 − a4 − b4 − c4
∴ sin A = 1 −
2
2 c2
= ;
4b
√ 4b2 c2
2b c + 2c a + 2a b − a − b − c
2 2 2 2 2 2 4 4 4
∴ sin A = ;
2bc
the former expression may be shown to agree with this by forming the product of the
factors s, s − a, s − b, and s − c.
Similar expressions hold for sin B and sin C.

Art. 228. To solve a triangle having given two angles and a side.
Suppose A and C the given angles, and b the given side;
then B = 180◦ − A − C;
a sin A b sin A
= , therefore a = ,
b sin B sin B
therefore log a = log b + log sin A − log sin B = log b + L sin A − L sin B;
similarly log c = log b + L sin C − L sin B.
Thus B, a, and c are determined.
If A and B are the given angles then
C = 180◦ − B − A,
and we may proceed as before to find a and c.

Art. 229. To solve a triangle having given two sides and the included angle.
Suppose b and c the given sides and A the included angle.
sin B b sin(A + C) b
We have = ; therefore = ;
sin C c sin C c
sin A cos C + cos A sin C b b
therefore = ; that is sin A cot C + cos A = :
sin C c c
thus cot C is determined, and therefore C can be found; and then B.
But as this process is not adapted to logarithmic computation another is usually
given :
sin B b sin B − sin C b−c
we have = , therefore = ,
sin C c sin B + sin C b+c
tan 12 (B − C) b−c
therefore = , (Art. 88) (page 325),
tan 12 (B + C) b+c
1 1 A
and tan (B + C) = tan (180◦ − A) = cot ,
2 2 2
1 b−c A
therefore tan (B − C) = cot ,
2 b+c 2
1 A
therefore log tan (B − C) = log(b − c) − log(b + c) + log cot ,
2 2
1 A
therefore L tan (B − C) = log(b − c) − log(b + c) + L cot ;
2 2
A
this formula determines 21 (B − C); and 21 (B + C) is known since it is 90◦ − ; thus
2
B and C can be immediately found.
APPENDIX : Miscellaneous Articles and Propositions 339

a sin A
Also = , from which a can be found.
c sin C
We have supposed b and c unequal; if however b = c then B = C, and all the angles
will be known, so that a can be found as in Art. 228 (page 338).

Art. 233. To solve a triangle having given two sides and the angle opposite to one of
them.
Let a and b be the given sides, and A the given angle;
sin B b b
then = ; therefore sin B = sin A;
sin A a a
therefore L sin B = log b − log a + L sin A.
b sin A
If is less than unity, two different angles may be found less than 180◦ which
a
b sin A
have for sine, one of these angles being less than a right angle, and the other
a
greater. If a be greater than b, then A must be greater than B, and therefore B must
be an acute angle; thus only the smaller value is admissible for B. If a be less than b,
then either value may be taken for B. When B is determined, C is known since it is
180◦ − A − B, and then c can be found from
c sin C
= .
a sin A
Thus if two values are admissible for B we obtain two corresponding values for C and
c, so that two triangles can be found from the given parts.
b sin A
If = 1, then B is a right angle, so that only one triangle can be found from
a
the given parts.
b sin A
If is greater than unity, no triangle exists with the given parts.
a
Thus, when two sides are given and the angle opposite the less we can generally
find two triangles from the given parts, and this case in the solution of triangles is
therefore called the ambiguous case. We say that two triangles can be generally
found in order to have regard to the exceptions; for the triangle may be right angled,
and then only one triangle can be found, or the triangle may be impossible.

Art. 234. The ambiguous case may be illustrated by figures.

A B B′ D

B′ A B D

Let CAD be the given angle A, and AC the given side b; suppose a circle described
from C as a centre with radius equal to a. The perpendicular from C on AD is equal
APPENDIX : Miscellaneous Articles and Propositions 340

to b sin A; therefore if a be greater than b sin A, the circle will meet the straight line
AD at two points, which we will denote by B and B ′ . If a be less than b, then B and
B ′ are on the same side of A, as in the first figure; thus two triangles, namely ABC
and AB ′ C, can be obtained, each having the given parts a, b, A. If a be greater than
b, then B ′ and B are on opposite sides of A, as in the second figure; thus only one
triangle, namely CAB, can be obtained having the given parts a, b, A; the triangle
CAB ′ has an angle CAB ′ which is 180◦ − A instead of A.
If a be equal to b sin A, the circle touches the straight line AD, and the two points
B and B ′ in the first figure coincide; thus one triangle in obtained which has a right
angle at B.
If a be less than b sin A the circle does not meet the straight line AD, and no triangle
exists with the given parts a, b, A.

Art. 238. We have seen in Chapter xii. that the Tables of trigonometrical functions
cannot always be used with advantage; this circumstance guides us in selecting the
method of solution of a triangle to be adopted when more than one method is theo-
retically applicable, and leads us to modify the method of solution in some cases. For
example, suppose we have to find A from the equation sin A = n, where n is nearly
equal to unity; this is an inconvenient equation for determining A, because the differ-
ence of consecutive sines is nearly insensible when the angles are nearly right angles.
We have however
( )√
A 1 − cos(90◦ − A)
sin 45◦ − =
2 2
√ √
1 − sin A 1−n
= = ;
2 2
and this formula is free from the objection.
Similarly if we have to find A from the equation
cos A = n,
where n is nearly equal to unity, we may advantageously transform the equation thus
√ √
A 1 − cos A 1−n
sin = = ;
2 2 2
1 − cos A 1−n
or thus = ;
1 + cos A √ 1 + n
A 1−n
therefore tan = .
2 1+n

Art. 240. To find the height and the distance of an inaccessible object on a horizontal
plane.

A B C

Let P be the top of an object, and let it be required to find its height P C, and the
distance of the object from a point A in the horizontal plane through C. At A observe
the angle P AC; then measure any length AB directly towards the object, and at B
observe the angle P BC. Then in the triangle AP B the side AB is known, and the angle
P AB; also the angle P BA is known, since it is the supplement of P BC; therefore AP
can be found. Then P C = AP sin P AC, and AC = AP cos P AC; thus the height P C and
the distance AC are determined.
APPENDIX : Miscellaneous Articles and Propositions 341

A C

If however it is not convenient to measure the length AB directly towards the


object, we may proceed thus; measure the length AB in any direction from A; at A
observe the angles P AC and P AB, and at B observe the angle P BA. Then in the
triangle AP B the side AB and the angles P AB and P BA are known; therefore AP
can be found. Then, as before, P C = AP sin P AC, and AC = AP cos P AC.

Art. 242. The lengths of the straight lines which join three points A, B, C , are known;
at any point P in the same plane as A, B, C , the angles APC and BPC are observed:
it is required to find the distance of P from each of the points A, B, C .
Let the angle AP C be denoted by α, the angle BP C by β, the angle P AC by x, and
the angle P BC by y; then α and β are

A B

known, and when x and y are found the required distances P A, P B, P C can be found;
for in each of the triangles P AC and P BC two angles and a side will then be known.
We will show how x and y may be found.
Since the angles of the triangles AP C and BP C are together equal to four right
angles, we have
x + y = 2π − α − β − C;
thus the sum of x and y is known.
From the triangle ACP we have
AC sin P AC b sin x
PC = = ;
sin P AC sin α
from the triangle BCP we have
BC sin P BC a sin y
PC = = ;
sin BP C sin β
b sin x a sin y
therefore = ;
sin α sin β
APPENDIX : Miscellaneous Articles and Propositions 342

sin x a sin α
therefore = .
sin y b sin β
a sin α
Now assume tan ϕ = , then the value of ϕ can be found from the Trigonometrical
b sin β
Tables; thus
sin x
= tan ϕ;
sin y
( )
sin x − sin y tan ϕ − 1 π
therefore = = tan ϕ − ;
sin x + sin y tan ϕ + 1 4
tan 12 (x − y)
( )
π
∴ (Art. 88 : (page 325)) 1
= tan ϕ − ;
tan 2 (x + y) 4
from the last equation we can determine x − y, since x + y is known; thus x and y can
be found.

Art. 247. To find expressions for the area of a triangle.


A triangle is half a rectangle on the same base and altitude; thus if ABC be any
triangle, and AD the perpendicular from A on the opposite side, we have (see the
figures in Art. 214 : page 336)
1
area of triangle = BC · AD, and
2
AD = AB sin B,
1
∴ area of triangle = ac sin B (51)
2
thus the area of a triangle is half the product of two sides into the sine of the included
angle.
By Art. 218 (page 338),
2√
sin B = {s(s − a)(s − b)(s − c)};
ac
substitute the value of sin B in (51) and we obtain

area of triangle = {s(s − a)(s − b)(s − c)} (52)
this furnishes
√ a convenient expression for the area when all the sides are known; the
expression {s(s − a)(s − b)(s − c)} is often for abbreviation denoted by S.
b sin A b sin C
By Art. 214 (page 336), a = , c= ,
sin B sin B
substitute these values in (51); thus we obtain
b2 sin A sin C
area of triangle = (53)
2 sin B
thus we can find the area when a side and two angles are given, for if two angles are
given the third angle is also known.

Art. 248. To find the radius of the circle inscribed in a triangle.


Let ABC be a triangle, O the centre of the circle inscribed in the triangle and
touching the sides at the points D, E, F. Join OD, OE, and OF. The angles at D, E,
and F are right angles by Euclid iii. 18. Let r denote the radius of the circle; then
APPENDIX : Miscellaneous Articles and Propositions 343

F
E

B D C

1 ar
area of triangle BOC = BC · OD = ,
2 2
1 br
area of triangle COA = CA · OE = ,
2 2
! cr
area of triangle AOB = AB · OF = ;
2 2
therefore, by addition,
r
(a + b + c) = area of triangle ABC = S, (Art. 247) (page 342),
2
S
that is rs = S, therefore r = .
s
The radius of the inscribed circle is thus equal to the area of the triangle divided
by half the sum of the sides; and hence different forms can be obtained for the radius
by employing the different expressions already given for the area of the triangle.
It is easy to show by Geometry that
AE = AF = s − a, BF = BD = s − b, CD = CE = s − c.

Art. 249. We may also obtain the value of r in another form, which will be often useful.
By Euclid iv. 4, the straight lines OA, OB, OC bisect the angles A, B, C respec-
tively. Thus
( )
B C B C
BD = r cot , CD = r cot , ∴ r cot + cot = a,
2 2 2 2
B C
B+C B C a sin sin
∴ r sin = a sin sin , ∴ r = 2 2 .
2 2 2 A
cos
2
APPENDIX : Miscellaneous Articles and Propositions 344

A A
Or thus : r = AE tan = (s − a) tan .
2 2

Art. 250. To find the radius of the circle which touches one side of a triangle and the
other sides produced.
Let ABC be a triangle, and let O be the centre
of the circle which touches the side BC, and the A
other sides produced at the points D, E, F. Join
OD, OE, and OF. The angles at D, E, and F are
right angles by Euclid iii. 18. Let r1 denote the
radius of the circle.
The quadrilateral OBAC may be divided into
the two triangles OAB, OAC; therefore the area
c b D
of this quadrilateral is r1 + r1 . B C
2 2
Again, the same quadrilateral may be divided E
into the triangles OBC and ABC; therefore the
a F
area of this quadrilateral is r1 + S. Thus
2
c b a O
r1 + r1 = r1 + S;
2 2 2
r1
∴ (c + b − a) = S,
2
∴ r1 (s − a) = S,
S
∴ r1 = .
s−a
It is easy to show by Geometry that
AF = AE = s, BD = BF = s − c, CD = CE = s − b.
The centre of the inscribed circle is also on AO, and the distance between it and
O subtends a right angle at B and at C.
Similarly, if r2 be the radius of the circle which touches CA and the other sides
produced, and r3 the radius of the circle which touches AB and the other sides pro-
duced,
S S
r2 = , r3 = .
s−b s−c
A circle which touches one side of a triangle and the other sides produced is called
an escribed circle.

Art. 251. We may also obtain an expression for the radius of an escribed circle similar
to that in Art. 249 (page 343) for the radius of the inscribed circle.
For, in the figure of Art. 250 (page 344), the straight line OB bisects the angle
which is the supplement of B, and the straight line OC bisects the angle which is the
supplement of C. Thus ( ) ( )
B C
BD = r1 cot 90◦ − , CD = r1 cot 90◦ − ;
( 2 ) 2
B C
∴ r1 tan + tan = a;
2 2
B C B C
a cos cos a cos cos
∴ r1 = 2 2 = 2 2 .
B+C A
sin cos
2 2
A A
Or thus: r1 = AF tan = s tan .
2 2

Art. 252. To find the radius of the circle described round a triangle.
APPENDIX : Miscellaneous Articles and Propositions 345

B C
D

Let ABC be a triangle, and O the centre of the circle described round it. Draw OD
perpendicular to BC, then BC is bisected at D by Euclid iii. 3. Let R denote the radius
of the circle.
The angle BOC is double the angle BAC, by Euclid iii. 20;
therefore BOD = A;
a a
and BD = R sin A = ; ∴ R = ;
2 2 sin A
thus R is expressed in terms of a side and the opposite angle.
2S abc
By Art. 218 (page 338), sin A = , ∴R= .
bc 4S

Art. 253. Many theorems have been demonstrated with respect to the circles which
have been noticed in Arts. 248 . . . 252; as an example we will find an expression for the
distance between the centres of the inscribed and circumscribed circles of a triangle.
A

O I

D
B C

E
Let ABC be a triangle, let O be the centre of the circumscribed circle. From O draw
a perpendicular OD on BC, and produce it to meet the circumference of the circle at
E. Then the arc BE is equal to the arc CE; and therefore the straight line AE bisects
the angle BAC. Thus the centre of the inscribed circle will be on AE; let the point I
denote it. Join OI and IC.
The angle EIC = 21 (A + C) by Euclid i. 32; and the angle ECI = ECB + BCI =
1
2
(A + C) : therefore the angle EIC = the angle ECI : and therefore EI = EC.
A A
And EC = 2R sin ; therefore EI = 2R sin .
2 2
A r
Hence EI × IA = 2R sin × = 2Rr.
2 A
sin
2
And (R − OI)(R + OI) = 2Rr, by Euclid iii. 35;
therefore OI 2 = R2 − 2Rr.
APPENDIX : Miscellaneous Articles and Propositions 346

If we suppose IE produced through E to a point J such that EJ = EI, the point J will
be the centre of the escribed circle which is opposite the angle A; and we shall have
OJ 2 = R2 + 2Rr.

Art. 254. To find the area of a quadrilateral which can be inscribed in a circle.
Let ABCD be the quadrilateral; let
AB = a, BC = b, CD = c, DA = d.
B

D
The figure can be divided into the triangles ABC, ADC; its area therefore
1 1
= (ab sin B + cd sin D) = (ab + cd) sin B,
2 2
for the angles B and D are supplemental by Euclid iii. 22.
Now from the triangle ABC,
AC 2 = a2 + b2 − 2ab cos B,
and from the triangle CDA,
AC 2 = c2 + d2 − 2cd cos D = c2 + d2 + 2cd cos B;
therefore c2 + d2 + 2cd cos B = a2 + b2 − 2ab cos B,
a2 + b2 − c2 − d2
therefore cos B = ;
2(ab + cd)
(a + b − c − d )
2 2 2 2 2
∴ sin2 B = 1 −
4(ab + cd)2
{ }{ }
2(ab + cd) + c2 + d2 − a2 − b2 2(ab + cd) − c2 − d2 + a2 + b2
=
4(ab + cd)2
{ }{ }
(c + d)2 − (a − b)2 (a + b)2 − (c − d)2
=
4(ab + cd)2
(c + b + d − a)(a + c + d − b)(a + b + d − c)(a + b + c − d)
=
4(ab + cd)2
1
Now let 2 (a + b + c + d) = s; thus
16(s − a)(s − b)(s − c)(s − d)
sin2 B = .
4(ab + cd)2
Hence the area of the quadrilateral

= (s − a)(s − b)(s − c)(s − d).
If we substitute the value of cos B in the expression for AC 2 , we obtain
2cd(a2 + b2 − c2 − d2 )
AC 2 = c2 + d2 +
2(ab + cd)
2 2 cd(a2 + b2 − c2 − d2 )
=c +d +
ab + cd
(ac + bd)(ad + bc)
= .
ab + cd
Similarly it may be shown that
a2 + d2 − b2 − c2
cos A = ,
2(ad + bc)
APPENDIX : Miscellaneous Articles and Propositions 347

(ac + bd)(ab + cd)


BD2 = .
ad + bc
The radius of the circle described round the quadrilateral may be easily expressed;
for this circle passes round the triangle ABC, hence by Art. 252 (page 344), its radius

AC 1 (ab + cd)(ac + bd)(ad + bc)
= = .
2 sin B 4 (s − a)(s − b)(s − c)(s − d)

Art. 255. To find the radii of the inscribed and circumscribed circles of a regular
polygon, that is of a polygon which has all its sides equal and all its angles equal.
Let AB be the side of a regular polygon of n sides; let O be the centre of the circles,
OD the radius of the inscribed circle, OA the radius of the circumscribed circle.

A B
D

Let AB = a, OA = R, OD = r.
The angle AOB is the nth part of 4 right angles, that is,
2π π
AOB = , AOD = .
n n
a π π
AD = = R sin = r tan ;
2 n n
a a
therefore R= π , r = π.
2 sin 2 tan
n n

Art. 258. To find the area of a sector of a circle.


Let θ be the circular measure of the angle of the sector; then
area of sector θ
= , (Euclid vi. 33);
area of circle 2π
θ r2 θ
therefore the area of the sector = πr 2 × = .
2π 2
Since θ is the circular measure of the angle of the sector, the length of the arc of
the sector is rθ; hence the area of a sector is equal to half the product of the length
of the arc into the radius.
The area of a segment of a circle can now be found. For a segment of a circle which
is less than a semicircle is equal to the difference between a sector and a triangle; so
that if θ be the circular measure of the angle of the sector the area of the segment is
r2
(θ − sin θ). A segment of a circle which is greater than a semicircle is equal to the
2
difference between the circle, and a segment less than a semicircle.

Art. 267. De Moivre’s Theorem : Whatever be the value of n { }n


positive or negative,
√ √
integral or fractional, cos nθ + −1 sin nθ is one of the values of cos θ + −1 sin θ .
√ √
Multiply cos α + −1 sin α by cos β + −1 sin β;
the product is

cos α cos β − sin α sin β + −1 {sin α cos β + cos α sin β} ,

that is, cos(α + β) + −1 sin(α + β);
APPENDIX : Miscellaneous Articles and Propositions 348


multiply the last expression by cos γ + −1 √ sin γ; the product is
cos(α + β + γ) + −1 sin(α + β + γ).
By proceeding √ in this way we obtain the product of any number of factors of the
form
√ cos α + −1 sin α. Suppose there are n of these factors, each factor being cos θ +
−1 sin θ; we then have { }n
√ √
cos θ + −1 sin θ = cos nθ + −1 sin nθ.
This proves De Moivre’s theorem when n is a positive integer.
Next, let n be a negative integer; suppose n = −m, then
{ √ }n { √ }−m
cos θ + −1 sin θ = cos θ + −1 sin θ
1 1
= { √ }m = √ ;
cos θ + −1 sin θ cos mθ + −1 sin mθ
multiply both numerator and denominator √ by
cos mθ − −1 sin mθ,

cos mθ − −1 sin mθ
thus we obtain ;
2
cos mθ √ + sin2 mθ
that is cos mθ − −1 sin mθ;

that is cos(−mθ) + −1 sin(−mθ),

or cos nθ + −1 sin nθ.
This proves De Moivre’s theorem when n is a negative integer.
Thus, since when n is any integer,
{ √ }n √
cos θ + −1 sin θ = cos nθ + −1 sin nθ,
√ { √ }1
it follows that cos θ + −1 sin θ is one of the values of cos nθ + −1 sin nθ n , when n
is any integer.
p
Lastly, let n be a fraction; suppose n = , then
q
p
{ √ }n { √ }
cos θ + −1 sin θ = cos θ + −1 sin θ q
{ √ }1
= cos pθ + −1 sin pθ q ,
and, by what has just been shown, one of the values of the last expression is
pθ √ pθ
cos + −1 sin .
q q
Thus De Moivre’s theorem is completely established.

Art. 270. We proceed to deduce some important results from De Moivre’s theorem.
In the equation { }n
√ √
cos nθ + −1 sin nθ = cos θ + −1 sin θ ,
suppose n a positive integer. Expand the right-hand member by the Binomial Theo-
rem, and equate the possible and impossible parts of the two members; thus
n(n − 1)
cos nθ = cosn θ − cosn−2 θ sin2 θ
1·2
n(n − 1)(n − 2)(n − 3)
+ cosn−4 θ sin4 θ − . . .
4
n(n − 1)(n − 2)
sin nθ = n cosn−1 θ sin θ − cosn−3 θ sin3 θ
3
n(n − 1)(n − 2)(n − 3)(n − 4)
+ cosn−5 θ sin5 θ − . . .
5

Art. 274. We shall now prove formulae for the expansion of sin α and cos α in series of
powers of α.
We have, when n is a positive integer,
n(n − 1)
cos nθ = cosn θ − cosn−2 θ sin2 θ
1·2
APPENDIX : Miscellaneous Articles and Propositions 349

n(n − 1)(n − 2)(n − 3)


+ cosn−4 θ sin4 θ − . . . .
4
Let nθ = α; and suppose n to increase without limit, and let θ so change that n may
remain a positive integer and nθ be always equal to α; thus θ must diminish without
limit. The preceding equation may be written
( )
α(α − θ) sin θ 2
cos α = cosn θ − cosn−2 θ
1·2 θ
( )
α(α − θ)(α − 2θ)(α − 3θ) sin θ 4
+ cosn−4 θ − ....
4 θ
Now when n increases without limit, and, therefore, θ( diminishes ) without limit,
sin θ sin θ sin θ n
is equal to unity, and so is every power of up to ; also cos θ is unity
θ θ θ
and so is every power of cos θ up to cosn θ (Art. 150 : page 334). Hence the above
formula becomes
α2 α4 α6
cos α = 1 − + − + ....
1·2 4 6
n(n − 1)(n − 2)
Also sin nθ = n cosn−1 θ sin θ − cosn−3 θ sin3 θ + . . .
3
( )
sin θ α(α − θ)(α − 2θ) sin θ 3
thus sin α = α cosn−1 θ − cosn−3 θ + ....
θ 3 θ
Hence, by supposing n to increase without limit, we obtain
α3 α5 α7
sin α = α − + − + ....
3 5 7
The results of this Article are of the greatest importance; we shall make some
remarks upon them in the next three Articles.

Art. 280. To express cosn θ in terms of cosines of multiples of θ when n is a positive


integer.
( )n
1 1 n(n − 1) n−2 1
2n cosn θ = x+ = xn + nxn−1 ·
+ x · 2 + ...
x x 1·2 x
n(n − 1) 2 1 1 1
+ x · n−2 + nx · n−1 + n .
1·2 x x x
Now rearrange the terms on the right-hand side, putting together the first term
and the last, the second (
and the last but
) one, and so( on; thus we obtain
)
n 1 n−2 1 n(n − 1) 1
x + n +n x + n−2 + xn−4 + n−4 + . . . ;
x x 1·2 x
1 1
but xn + n = 2 cos nθ, xn−2 + n−2 = 2 cos(n − 2)θ, and so on;
x x
therefore
n(n − 1)
2n−1 cosn θ = cos nθ + n cos(n − 2)θ + cos(n − 4)θ + . . .
1·2
n(n − 1) . . . (n − r + 1)
+ cos(n − 2r)θ + . . .
r
(
The last term of the series on the right-hand ) will take different forms according
side
1 n
as n is even or odd. In the expansion of x + by the Binomial Theorem there are
x
( )th
n
n + 1 terms; thus when n is even, there will be a middle term, namely the +1 ,
2
which is ( ) ( )
n(n − 1) . . . n − 12 n + 1 n 1 n(n − 1) . . . 21 n + 1
x 2 · n ; that is n .
1 x2 x2
2
n
APPENDIX : Miscellaneous Articles and Propositions 350

Hence, when n is even, the last term of 2(n−1 cosn ) θ is


n(n − 1) . . . 12 n + 1
.
2 12 n

)nn is odd suppose it = 2m + 1; there are two middle terms in the expansion of
( When
1
x+ , namely, the (m + 1)th and (m + 2)th : their sum is
x ( )
n(n − 1) . . . (n − m + 1) 1
x+ .
m x
Hence when n is odd, the last term of 2n−1 cosn θ is
n(n − 1) . . . 12 (n + 3)
cos θ.
1
2
(n − 1)

Art. 281. We shall find that sinn θ can be expressed in terms of cosines of multiples of
θ if n be an even positive integer, and in terms of sines of multiples of θ if n be an odd
positive integer; this will appear in the following two Articles.

Art. 282. To express sinn θ in terms of cosines of multiples of θ, when n is an even


positive integer.
( )
n 1 n 1 n(n − 1) n−2 1
2n (−1) 2 sinn θ = x−
= xn − nxn−1 · + x · 2 + ...
x
( )n−2 x
( 1)·n−1 2
( x)n
n(n − 1) 2 1 1 1
+ x · − + nx − + − .
1·2 x x x
Now rearrange the terms on the right-hand side, putting together the first term and
the last, the second and the
( last but one,
) and so on;(thus we obtain )
1 1 n(n − 1) 1
x + n −n x
n n−2
+ n−2 + xn−4 + n−4 − . . .
x x 1·2 (x )
n
n(n − 1) . . . +1
n 2
+ (−1) 2 .
n
2
Therefore
n n(n − 1)
2n−1 (−1) 2 sinn θ = cos nθ − n cos(n − 2)θ + cos(n − 4)θ − . . .
1·2
n(n − 1) . . . (n − r + 1)
+ (−1)n−r cos(n − 2r)θ + . . .
r
( )
n
n(n − 1) . . . +1
n 2
+ (−1) 2 .
n
2
2

Art. 283. To express sinn θ in terms of sines of multiples of θ when n is an odd positive
integer.
( )n
n 1 1 n(n − 1) n−2 1
2n (−1) 2 sinn θ = x− = xn − nxn−1 ·
+ x · 2
x x 1·2 x
n(n − 1) 2 1 1 1
− ... − x · n−2 + nx · n−1 − n .
1·2 x x x
Now rearrange the terms on the right-hand side, putting together the first term
and the last, the second and
( the last but
) one, and so(on; thus we obtain
)
1 1 n(n − 1) 1
xn − n − n xn−2 − n−2 + xn−4 − n−4 − . . .
x x 1·2 x
APPENDIX : Miscellaneous Articles and Propositions 351

(n + 3)
n(n − 1) . . . ( )
n−1
2 1
+ (−1) 2 x− ;
(n − 1) x
2
1 √
but xn − n
= 2 (−1) sin nθ,
x
1 √
xn−2 − n−2 = 2 (−1) sin(n − 2)θ,
x
and so on; therefore
n−1 n(n − 1)
2n−1 (−1) 2 sinn θ = sin nθ − n sin(n − 2)θ + sin(n − 4)θ
1·2
(n + 3)
n(n − 1)(n − 2) n−1
n(n − 1) . . .
− sin(n − 6)θ + (−1) 2
2 sin θ.
3 (n − 1)
2

Art. 293. Moreover tan−1 x has an infinite number of values corresponding to the
same value of x, so that one member of what appears as an equation admits of more
values than the other; this point is left unexplained in the investigation which has
been given.
The subject of series cannot be adequately treated without using the Differential
Calculus. The student must therefore be referred to treatises on that subject for a
satisfactory demonstration of Gregory’s Series. It is there shown that so long as θ lies
π π 1 1
between − and , the result θ = tan θ − tan3 θ + tan5 θ − . . . is absolutely true.
4 4 3 5
(See Differential Calculus, Chapter VII.)
π π
If, however, θ = nπ + ϕ, where ϕ lies between − and , then
4 4
1 1
ϕ = tan ϕ − tan3 ϕ + tan5 ϕ − . . . ;
3 5
1 1
that is, θ − nπ = tan θ − tan3 θ + tan5 θ − . . . ;
3 5

π π
Art. 294. In Gregory’s Series put θ = ; then since tan = 1,
4 4
π 1 1 1 1
= 1 − + − + − ...
4 3 5 7 9
This series might be used for calculating the value of π; but it is very slowly con-
vergent, so that a large number of terms would have to be taken to calculate π to a
close approximation.

Art. 296. Machin’s Series. We shall first show that


π 1 1
= 4 tan−1 − tan−1 .
4 5 239
2
1 10 5
2 tan−1 = tan−1 5 = tan−1 = tan−1 ,
5 1 24 12
1−
25
10
−1 1 −1 5 −1 120
4 tan = 2 tan = tan 12 = tan−1 .
5 12 25 119
1−
144
1 π
Hence 4 tan−1 is a little greater than ; suppose
5 4
1 π
4 tan−1 = + tan−1 x,
(5 4 )
120 π 1+x
then = tan + tan−1 x = ;
119 4 1−x
APPENDIX : Miscellaneous Articles and Propositions 352

1
from this we find x = ;
239
π 1 1
therefore = 4 tan−1
− tan−1 .
{4 5 239 }
π 1 1 1 1
Therefore =4 − + − + . . .
{4 5 3 · 53 5 · 55 7 · 57 }
1 1 1 1
− − + − + . . . .
239 3(239)3 5(239)5 7(239)7

Art. 300. To find the coefficient of x n in the expansion of e ax cos bx in powers of x.

1 ax ( bxι ) 1 1
Here eax cos bx = e e + e−bxι = e(a+bι)x + e(a−bι)x .
2 2 2
Expand these two exponential expressions by the exponential theorem; then the
n
coefficient of x is {( ) ( ) }
1 rn a b n a b n
{(a + bι)n + (a − bι)n } = + ι + − ι .
2n 2n r r r r
a b
Now suppose = cos θ, = sin θ, so that r2 = a2 + b2 .
r r
n
Thus the coefficient of x becomes
( 2 2 ) n2
a +b
{(cos θ + ι sin θ)n + (cos θ − ι sin θ)n }
2n
( ) n2
a2 + b2
= (cos nθ + ι sin nθ + cos nθ − ι sin nθ)
2n
( )n
a2 + b 2 2
= cos nθ.
n
Similarly the coefficient of x in the expansion of eax sin bx in powers of x is
n
( ) n2
a2 + b2
sin nθ.
n

Art. 303. To find the sum of the sines of a series of angles which are in arithmetical
progression.
Let the proposed series consist of the following n terms,
sin α + sin(α + β) + sin(α + 2β) + . . . + sin {α + (n − 1)β} .
We have ( ) ( )
1 1 1
cos α − β − cos α + β = 2 sin β sin α,
( 2 ) ( 2 ) 2
1 3 1
cos α + β − cos α + β = 2 sin β sin(α + β),
( 2 ) ( 2 ) 2
3 5 1
cos α + β − cos α + β = 2 sin β sin(α + 2β),
2 2 2
...........................
( ) ( )
2n − 3 2n − 1 1
cos α + β − cos α + β = 2 sin β sin {α + (n − 1)β} .
2 2 2
Let S denote the proposed
( series;
) then,( by addition,)
1 2n − 1 1
cos α − β − cos α + β = 2S sin β;
2 2
( ) ( )2
1 2n − 1
cos α − β − cos α + β
2 2
therefore S=
1
2 sin β
( ) 2
n−1 nβ
sin α + β sin
2 2
= .
1
sin β
2
APPENDIX : Miscellaneous Articles and Propositions 353

Art. 304. To find the sum of the cosines of a series of angles which are in arithmetical
progression.
Let the proposed series consist of the following n terms,
cos α + cos(α + β) + cos(α + 2β) + . . . + cos {α + (n − 1)β} .

We have ( ) ( )
1 1 1
sin α + β − sin α − β = 2 sin β cos α,
( 2 ) ( 2 ) 2
3 1 1
sin α + β − sin α + β = 2 sin β cos(α + β),
( 2 ) ( 2 ) 2
5 3 1
sin α + β − sin α + β = 2 sin β cos(α + 2β),
2 2 2
....................................
( ) ( )
2n − 1 2n − 3 1
sin α + β − sin α + β = 2 sin β cos {α + (n − 1)β} .
2 2 2
Let S denote the proposed
( series;
) then by
( addition,
)
2n − 1 1 1
sin α + β − sin α − β = 2S sin β;
2 2 2
therefore ( ) ( )
2n − 1 1
sin α + β − sin α − β
2 2
S=
1
2 sin β
( )2
n−1 nβ
cos α + β sin
2 2
= .
1
sin β
2


Art. 305. Suppose in Arts. 303 (page 352) and 304 (page 353) that β = ; then since
n

sin = sin π = 0, then sum of the sines or the sum of the cosines of the series of
2
2π 4π 2(n − 1)π
angles α, α + , α+ ,...α + is zero.
n n n
This is a very important result, and the student should pay great attention to it.
Moreover we may give this wide extension to our result : let m and n be positive

integers, m being less than n, and β = , then the following sum is a number inde-
n
pendent of angles, ( )
sinm α + sinm (α + β) + sinm (α + 2β) + . . . + sinm α + n − 1β .
The same theorem is true when sine is changed into cosine. The theorem is estab-
lished by the aid of Arts. 280 . . . 283 (page 349, 350, 350, 350). Suppose, for example,
we take m = 4. We have
1
sin4 α = {cos 4α − 4 cos 2α + 3} ,
8
1
sin4 (α + β) = {cos(4α + 4β) − 4 cos(2α + 2β) + 3} ,
8
and so on.
Thus the proposed series can be replaced by other series; the sum to n terms
of cos 4α + cos(4α + 4β) + . . . is zero by Art. 304 (page 353); the sum to n terms of
cos 2α + cos(2α + 2β) + . . . is zero by the same Article; thus the proposed series reduces
3n
to .
8
The condition that m is less than n ensures that the denominators in the expres-
sions for the sums of the sines and cosines do not vanish.

Art. 307. We may now deduce the sum of the following n terms:
sin α − sin(α + β) + sin(α + 2 β) − . . . + (−1 )n−1 sin {α + (n − 1 )β} .
APPENDIX : Miscellaneous Articles and Propositions 354

This series may be written


sin α + sin(α + β + π) + sin(α + 2β + 2π) + . . . + sin {α + (n − 1)(β + π)} .
We have then only to change β into β + π in the result of Art. 303 (page 352).
Hence the required sum is
{ }
(n − 1)(β + π) n(β + π)
sin α + sin
2 2
.
β+π
sin
2
Similarly
cos α − cos(α + β) + cos(α + 2β) − . . . + (−1)n−1 cos {α + (n − 1)β}
{ }
(n − 1)(β + π) n(β + π)
cos α + sin
2 2
= .
β+π
sin
2

Art. 309. To find the sum of the following n terms :


1 x 1 x 1 x
tan x + tan + 2 tan 2 + . . . + n−1 tan n−1
2 2 2 2 2 2 .

We have tan x = cot x − 2 cot 2x,


1 x 1 x
tan = cot − cot x,
2 2 2 2
1 x 1 x 1 x
tan 2 = 2 cot 2 − cot ,
22 2 2 2 2 2
...........................
1 x 1 x 1 x
tan n−1 = n−1 cot n−1 − n−2 cot n−2 .
2n−1 2 2 2 2 2
Let S denote the proposed series; then, by addition,
1 x
S = n−1 cot n−1 − 2 cot 2x.
2 2
1 x 1 β x
The term n−1 cot n−1 = cos β , where β = n−1 ; if we suppose n to in-
2 2 x sin β 2
β
crease indefinitely, cos β = 1, and = 1.
sin β
1
Thus the limit of the proposed series, when n is indefinitely increased, is −2 cot 2x.
x

Art. 310. To find the sum of the following n terms :


sin α + c sin(α + β) + c2 sin(α + 2β) + . . . + cn−1 sin {α + (n − 1)β} .

Let S denote the proposed series; substitute for the sines their exponential values
: thus
2ιS = eαι + ce(α+β)ι + c2 e(α+2β)ι + . . . + cn−1 e(α+nβ−β)ι
− e−αι − ce−(α+β)ι − c2 e−(α+2β)ι − . . . − cn−1 e−(α+nβ−β)ι
We have now two geometrical progressions; thus
1 − cn enβι 1 − cn e−nβι
2ιS = eαι − e−αι
1 − ce {
βι 1 − ce−βι} { }
eαι − e−αι − c e(α−β)ι − e−(α−β)ι − cn e(α+nβ)ι − e−(α+nβ)ι
{ }
+ cn+1 e(nβ+α−β)ι − e−(nβ+α−β)ι
= ( )
1 − c eβι + e−βι + c2

therefore
sin α − c sin(α − β) − cn sin(α + nβ) + cn+1 sin {α + (n − 1)β}
S= .
1 − 2c cos β + c2
If c be less than unity, then when n is indefinitely increased cn and cn+1 diminish
without limit; hence if c be less than unity, the limit of the proposed series when n is
APPENDIX : Miscellaneous Articles and Propositions 355

indefinitely increased is
sin α − c sin(α − β)
.
1 − 2c cos β + c2
Similarly we can show that
cos α + c cos(α + β) + c2 cos(α + 2β) + . . . + cn−−1 cos {α + (n − 1)β}
cos α − c cos(α − β) − cn cos(α + nβ) + cn+1 cos {α + (n − 1)β}
= .
1 − 2c cos β + c2
π
This result may also be obtained from the preceding by changing α into α + . If c be
2
less than unity the limit of the proposed series, when n is indefinitely increased, is
cos α − c cos(α − β)
.
1 − 2c cos β + c2

Art. 311. To sum the infinite series


c2 c3
c sin(α + β) + sin(α + 2β) + sin(α + 3β) + . . . ,
2 3
c2 c3
and c cos(α + β) + cos(α + 2β) + cos(α + 3β) + . . .
2 3

Denote the former series by S and the latter by C; multiply the former by ι and
add it to the latter : thus
c2 ι(α+2β) c3 ι(α+3β)
C + ιS = ceι(α+β) + e + e + ...
2 3
( ιβ
) ( )
= eια ece −1 = eια ec cos β+ιc sin β − 1 = ec cos β eι(α+c sin β) − eια

= ec cos β {cos(α + c sin β) + ι sin(α + c sin β)} − (cos α + ι sin α).


Equate the real and imaginary parts : thus
C = ec cos β cos (α + c sin β) − cos α.
S = ec sin β sin (α + c sin β) − sin α.
The method of this Article might be used in Art. 310 (page 354); or the method of that
Article might be used here.

Art. 313. It is known from the treatises on the Theory of Equations that the expression
xn − 1, where n is a positive integer, can be resolved into n factors, each√ of the form
x − a, where a is either a real quantity or an expression of the form α + β −1, where α
and β are real : and there is only one such set of factors. We proceed now to resolve
the expression xn − 1, and some similar expressions, into component factors. The
factors of the expression xn − 1 are found by solving the equation xn − 1 = 0; every
root of the equation determines one factor of the expression : thus if a denote a root
the corresponding factor is x − a.

Art. 314. To resolve x n − 1 into factors.


2rπ √ 2rπ
The expression cos ± (−1) sin , where r is any integer, is a root of the
n n
equation √ xn = 1; for the nth power of this expression is by De Moivre’s Theorem
cos 2rπ ± (−1) sin 2rπ, that is 1.
First, suppose n is even. If we put r = 0 we obtain a real root 1, and the correspond-
n
ing factor is x − 1; if we put r = , we obtain a real root −1, and the corresponding
2
factor is x + 1.
n
If we put for r in succession the values 1, 2, 3, . . . − 1 we obtain n − 2 additional
2
roots, since each value of r gives rise to two roots. These roots are all different, for the
2rπ
angles are less than π and all different, and thus cos cannot have two coincident
n
values. Therefore x − 1 = (x − 1)(x + 1)P, where P is the product of n − 2 factors
n
APPENDIX : Miscellaneous Articles and Propositions 356

n
obtained by ascribing to r in succession the values 1, 2, 3, . . . − 1 in the expression
2rπ √
2
2rπ
x − cos ∓ (−1) sin .
n n
2rπ √ 2rπ 2rπ
The product of the two factors x − cos − (−1) sin , and x − cos +
√ (n ) n n
2rπ 2rπ 2 2rπ
(−1) sin , is the real quadratic factor x − cos + sin2 , that is, x2 −
n n n
2rπ
2x cos + 1.
n
Hence when n is even ( )( )
2π 4π
xn − 1 = (x − 1)(x + 1) x2 − 2x cos +1 x2 − 2x cos + 1 ...
{ } {n n}
n−4 n−2
. . . x − 2x cos
2
π+1 x − 2x cos
2
π+1 (54)
n n
Secondly, suppose n is odd. The only real root of x = 1 is now 1; the other n −
n
n−1
1 roots are obtained by giving to r in succession the values 1, 2, 3, . . . in the
2rπ √ 2rπ
2
expression cos ± (−1) sin .
n n
Hence when n is odd ( )( )
2π 4π
xn − 1 = (x − 1) x2 − 2x cos +1 x2 − 2x cos + 1 ...
{ n }{ n }
n−3 n−1
. . . x − 2x cos
2
π+1 x − 2x cos
2
π+1 (55)
n n

Art. 315. To resolve x n + 1 into factors.


2r + 1 √ 2r + 1
The expression cos π ± −1 sin π, where r is any integer, is a root of the
n n √
equation x = −1; for the n power of this expression is cos(2r + 1)π± −1 sin(2r + 1)π,
n th

by De Moivre’s Theorem, that is −1.


First, suppose n is even; there is no real root of the equation xn = −1; the n roots
n
are all imaginary, and are found by giving to r in succession the values 0, 1, 2, 3, . . . −
2
2r + 1 √ 2r + 1
1, in the expression cos π ± −1 sin π.
n n
2r + 1 √ 2r + 1 2r + 1
The product of the two factors x−cos π− −1 sin π, and x−cos π+
n ( n) n
√ 2r + 1 2r + 1 2 2r + 1
−1 sin π, is the real quadratic factor x − cos π + sin2 π, that is,
n n n
2r + 1
x2 − 2x cos π + 1.
n
Hence when (n is even )( )( )
π 3π 5π
xn + 1 = x2 − 2x cos + 1 x2 − 2x cos +1 x2 − 2x cos + 1 ...
( n )( n ) n
n−3 n−1
. . . x2 − 2x cos π+1 x2 − 2x cos π+1 (56)
n n
Secondly, suppose n is odd. The only real root of x = −1 is −1; the other n − 1
n
n−3
roots are obtained by giving to r in succession the values 0, 1, 2, 3, . . . in the
2
2r + 1 √ 2r + 1
expression cos π ± −1 sin π.
n n
Hence when n is odd ( )( )
π 3π
xn + 1 = (x + 1) x2 − 2x cos + 1 x2 − 2x cos + 1 ...
( n )( n )
n−4 n−2
. . . x2 − 2x cos π+1 x2 − 2x cos π+1 (57)
n n

Art. 316. The four formulae established in the two preceding Articles are identically
true; we may deduce many particular results by supposing particular values assigned
to x. Thus in (54) of Art. 314 (page 355), divide both sides by x − 1; the quotient on
APPENDIX : Miscellaneous Articles and Propositions 357

the left-hand side will be xn−1 + xn−2 + . . . + x + 1. Now put x = 1; thus when n is even
( )( ) ( )( )
n 2π 4π n−4 n−2
n = 2 2 1 − cos 1 − cos . . . 1 − cos π 1 − cos π ;
n n n n
and by extracting the square root,
√ n−1 2π 4π n−4 n−2
n = 2 2 sin sin . . . sin π sin π (58)
2n 2n 2n 2n
The positive sign of the radical must be taken on the left-hand side, because the right-
hand side is obviously positive.
Again in (55) of Art. 314 (page 355), divide both sides by x − 1, and afterwards put
x = 1; thus when ( n is odd ) ( ) ( )( )
n−1 2π 4π n−3 n−1
n=2 2 1 − cos 1 − cos . . . 1 − cos π 1 − cos π ;
n n n n
and by extracting the square root,
√ n−1 2π 4π n−3 n−1
n = 2 2 sin sin . . . sin π sin π (59)
2n 2n 2n 2n
Again, in (56) of( Art. 315 (page
)( 356), put
) ( x = 1; thus when n
)( is even )
n π 3π n−3 n−1
2 = 2 2 1 − cos 1 − cos . . . 1 − cos π 1 − cos π ;
n n n n
and by extracting the square root,
n−1 π 3π n−3 n−1
1 = 2 2 sin sin . . . sin π sin π (60)
2n 2n 2n 2n
Again, in (57) of(Art. 315 (page)( 356), put
) ( x = 1; thus when n)(is odd )
n+1 π 3π n−4 n−2
2=2 2 1 − cos 1 − cos . . . 1 − cos π 1 − cos π ;
n n n n
and by extracting the square root,
n−1 π 3π n−4 n−2
1 = 2 2 sin sin . . . sin π sin π (61)
2n 2n 2n 2n
Four other results may apparently be deduced from the four formulae of the two
preceding Articles by putting x = −1; but it will be found on trial that these results
do not differ really from those already deduced. Thus, for example, in (54) of Art. 314
(page 355) , divide both sides by x + 1, afterwards put x = −1, and extract the square
root; thus when n is even
√ n−1 2π 4π n−4 n−2
n = 2 2 cos cos . . . cos π cos π;
2n 2n 2n 2n
this however is the same result as that in (58) of the present Article, the factors on
the right-hand side being merely differently arranged; for
2π n−2 4π n−4
cos = sin π, cos = sin π, . . .
2n 2n 2n 2n

Art. 317. To resolve x 2n − 2x n cos θ + 1 into factors.


If cos θ = 1 the expression becomes (xn − 1)2 , and if cos θ = −1 it becomes (xn + 1)2 ;
in these cases the resolution into factors is effected by what has already been given
in Arts. 314 (page 355) and 315 (page 356), and we will therefore suppose these cases
excluded from what follows.
If we put
x2n − 2xn cos θ + 1 = 0,
√ √
we obtain xn = cos θ ± −1 sin θ; hence x is an nth root of cos θ + −1 sin θ; the nth
2rπ + θ √ 2rπ + θ
roots are found from the expression cos ± −1 sin by ascribing integral
n n
values to r, for it is obvious√from De Moivre’s Theorem that the nth power of the last
expression is cos(2rπ + θ)± −1 sin(2rπ + θ), and if r be an integer this reduces to cos θ±

−1 sin θ. If we ascribe to r in succession the values 0, 1, 2, . . . n − 1 in the expression
2rπ + θ √ 2rπ + θ
cos ± −1 sin we obtain 2n different values for the expression. For if
n n
r = p and r = q could give the same value to the expression we should have
2pπ + θ √ 2pπ + θ 2qπ + θ √ 2qπ + θ
cos ± −1 sin = cos ± −1 sin ;
n n n n
2pπ + θ 2qπ + θ 2pπ + θ
now by Art. 93 (page 326) we cannot have cos = cos and sin =
n n n
APPENDIX : Miscellaneous Articles and Propositions 358

2qπ + θ 2pπ + θ 2qπ + θ 2pπ + θ 2qπ + θ


sin ; nor can cos = cos and sin = − sin , for that,
n n n n n
2pπ + θ 2qπ + θ
by Art. 94 (page 326), would require + to be a multiple of 2π, so that θ
n n
would be a multiple of π, and this value of θ has been expressly excluded above. Thus
we obtain 2n different values of x.
2rπ + θ √ 2rπ + θ 2rπ + θ
The product of the two factors x−cos − −1 sin , and x−cos +
n( n) n
√ 2rπ + θ 2rπ + θ 2
2rπ + θ
−1 sin , is the real quadratic factor x − cos + sin2 , that is,
n n n
2rπ + θ
x2 − 2x cos + 1.
n
Thus x2n( − 2x n cos θ + 1
)( )( )
θ 2π + θ 4π + θ
= x2 − 2x cos + 1 x2 − 2x cos +1 x2 − 2x cos +1
{ n } {n n }
(2n − 4)π + θ (2n − 2)π + θ
. . . x − 2x cos
2
+1 x − 2x cos
2
+1 .
n n

Art. 318. We shall now deduce some important results from the preceding general
theorem. Suppose x = ( 1; then )( )( )
θ 2π + θ 4π + θ
2(1 − cos θ) = 2n 1 − cos 1 − cos 1 − cos ...
n n ( n )
2nπ − 2π + θ
. . . 1 − cos .
n
π
Let θ = 2nϕ and = α; extract the square root; thus
2n
sin nϕ = 2n−1 sin ϕ sin(2α + ϕ) sin(4α + ϕ) . . . sin(2nα − 2α + ϕ).
We shall now prove that the upper sign must always be taken on the left-hand side.
First, suppose ϕ to lie between 0 and 2α; then every factor on the right-hand side is
positive, and so is sin nϕ. Next suppose ϕ to lie between 2α and 4α; then every factor
on the right-hand side is positive except the last, and sin nϕ is negative. Next suppose
ϕ to lie between 4α and 6α, then every factor on the right-hand side is positive except
the last two, and sin nϕ is positive. By proceeding in this way we see that for every
value of ϕ between 0 and 2nα, the upper sign must be taken, so that we have for all
values of ϕ between 0 and π
sin nϕ = 2n−1 sin ϕ sin(2α + ϕ) sin(4α + ϕ) . . . sin(2nα − 2α + ϕ).
We shall next show that this formula is true for all values of ϕ; for suppose ϕ = mπ + ψ
where m is any integer, positive or negative, and ψ is between 0 and π; then we know
that
sin nψ = 2n−1 sin ψ sin(2α + ψ) sin(4α + ψ) . . . sin(2nα − 2α + ψ);
but sin nψ = sin(nϕ − nmπ) = sin nϕ cos nmπ = (−1)nm sin nϕ,
sin ψ = sin(ϕ − mπ) = sin ϕ cos mπ = (−1)m sin ϕ,
sin(2α + ψ) = sin(2α + ϕ − mπ) = sin(2α + ϕ) cos mπ = (−1)m sin(2α + ϕ),
and so on.
Substitute these values of sin nψ, sin ψ, sin(2α + ψ), . . . in the formula which ex-
presses sin nψ in factors; then divide both sides by (−1)nm and we obtain the required
formula for sin nϕ, whatever may be the value of ϕ.
π
In the expression for sin nϕ change ϕ into ϕ + α; then nϕ is changed into nϕ + ;
2
hence
cos nϕ = 2n−1 sin(ϕ + α) sin(ϕ + 3α) sin(ϕ + 5α) . . . sin(2nα − α + ϕ).
In the last result put ϕ = 0; thus
1 = 2n−1 sin α sin 3α sin 5α . . . sin(2nα − α),
π
where α= .
2n
Again we have
sin nϕ
= 2n−1 sin(2α + ϕ) sin(4α + ϕ) . . . sin(2nα − 2α + ϕ);
sin ϕ
APPENDIX : Miscellaneous Articles and Propositions 359

sin nϕ
now let ϕ diminish without limit; then since the limit of is n, we obtain
sin ϕ
n=2 n−1
sin 2α sin 4α sin 6α . . . sin(2nα − 2α).
These two formulae are sometimes useful; the first includes (60) and (61) of Art. 316
(page 356), and the second included (58) and (59) of Art. 316 (page 356).
If we divide the expression for sin nϕ by that for cos nϕ we obtain an expression for
tan nϕ; when n is odd this takes a simple form which we may obtain more readily thus
π
: in the expression for sin nϕ, change ϕ into ϕ + ; we obtain
2

cos nϕ sin = 2n−1 cos ϕ cos(2α + ϕ) . . . cos(2nα − 2α + ϕ).
2
Divide the expression for sin nϕ by this; hence
( when
) n is(odd )
n−1 π n−1
tan nϕ = (−1) 2 tan ϕ tan ϕ + . . . tan ϕ + π .
n n

Art. 319. The expression for sin nϕ in Art. 318 (page 358) may be put into a different
form; for
sin(2nα − 2α + ϕ) = sin(π − 2α + ϕ) = sin(2α − ϕ),
sin(2nα − 4α + ϕ) = sin(π − 4α + ϕ) = sin(4α − ϕ).
and so on.
Then by multiplying together the second factor and the last, the third and the last
but one, and so on, we have ( )( )
sin nϕ = 2n−1 sin ϕ sin2 2α − sin2 ϕ sin2 4α − sin2 ϕ . . .
It will be necessary to examine separately the cases hen n is even and when n is
odd.
First suppose n is even; then the factor sin(nα + ϕ), that is, cos ϕ, will occur without
any factor to multiply it : hence if (n be even, we have
)( )
sin nϕ = 2n−1 sin ϕ cos ϕ sin2 2α − sin2 ϕ sin2 4α − sin2 ϕ . . .
{ }{ }
. . . sin2 (n − 4)α − sin 2ϕ sin2 (n − 2)α − sin2 ϕ .
Next suppose n is odd; then we( have )( )
sin nϕ = 2n−1 sin ϕ sin2 2α − sin2 ϕ sin2 4α − sin2 ϕ . . .
{ }{ }
. . . sin2 (n − 3)α − sin2 ϕ sin2 (n − 1)α − sin2 ϕ .
Similarly from the formula
cos nϕ = 2n−1 sin(ϕ + α) sin(ϕ + 3α) sin(ϕ + 5α) . . . sin(2nα − α + ϕ)
We obtain if n be even ( )( )
cos nϕ = 2n−1 sin2 α − sin2 ϕ sin2 3α − sin2 ϕ . . .
{ }{ }
. . . sin2 (n − 3)α − sin2 ϕ sin2 (n − 1)α − sin2 ϕ .
and if n be odd ( )( )
cos nϕ = 2n−1 cos ϕ sin2 α − sin2 ϕ sin2 3α − sin2 ϕ . . .
{ }{ }
. . . sin2 (n − 4)α − sin2 ϕ sin2 (n − 2)α − sin2 ϕ .

Art. 320. We can now resolve sin θ and cos θ into their factors. Suppose nϕ = θ and
that n is odd; then by the preceding Article
( )( )
θ θ θ
sin θ = 2n−1 sin sin2 2α − sin2 sin2 4α − sin2 ...
n n n
θ
Divide both sides by sin , and then diminish θ indefinitely; since the limit of sin θ ÷
n
θ
sin is n we obtain
n
n = 2n−1 sin2 2α sin2 4α . . . ;
therefore by division,
 θ
 θ

θ  sin2 sin2
sin θ = n sin 1− n  1 − n ...
n sin2 2α sin2 4α
APPENDIX : Miscellaneous Articles and Propositions 360

θ
π sin θ
Now suppose n to increase without limit; then since α = , the limit of n is ,
2n sin 2α π
θ
sin θ
the limit of n is , and so on.
sin 4α 2π
Thus finally, ( )( )( )
θ2 θ2 θ2
sin θ = θ 2
1−
1− 2 2 1 − 2 2 ...
π 2 π 3 π
We shall obtain the same result if we begin by supposing n even.
Similarly we may show (that )( )( )
4θ2 4θ2 4θ2
cos θ = 1− 1− 1− ...
π2 32 π 2 52 π 2

Art. 321. In the same way as x2n − 2xn cos θ + 1 was decomposed in Art. 317 (page 357)
we may decompose x2n − 2xn an cos θ + a2n , and each quadratic factor of the last ex-
2rπ + θ
pression will be of the form x2 − 2xa cos + a2 , where r is an integer; and all the
n
factors are found by giving to r in succession the values 0, 1, 2, . . . n − 1.
2(n − 1)π + θ 2π − θ 2(n − 2)π + θ 4π − θ
And cos = cos , cos = cos ,
n n n n
2rπ ± θ
and so on; thus all the factors will be found if we take x2 − 2xa cos + a2 , and
n
n−1
use both signs and give to r in succession the values 0, 1, 2, . . . up to if n be odd,
2
n n
and up to if n be even; in the latter case when r = we must take only one factor
2 2
nπ + θ
x2 − 2xa cos + a2 .
n
z z
Now suppose x = 1 + , and a = 1 − ; thus
(
2n
)2n ( 2n )n ( )
z z 2 z 2n
1+ −2 1− 2
cos θ + 1 −
2n 4n 2n
is the expression to be decomposed into factors; and the general form of the factors
is
( ) ( ) ( )
z 2 z2 2rπ ± θ z 2
1+ −2 1− cos + 1 − ,
2n 4n2 n 2n
( ) ( )
z2 z2 2rπ ± θ
that is, 2 1+ −2 1− cos ,
4n2 4n2 n
( )
2rπ ± θ z2 2rπ ± θ
that is, 4 sin2 1+ cot2 .
2n 4n2 2n
Suppose n to increase indefinitely; then
( ) ( )
z 2n z 2n
1+ = ez , 1− = e−z , (Algebra, Art. 552)(page 365),
2n 2n
z2 2rπ ± θ z2
also cot2 = ;
4n2 2n (2rπ ± θ)2
and by putting z = 0 we obtain
θ θ 2π ± θ 4π ± θ
4 sin2 = 4 sin2 4 sin2 4 sin2 ...;
2 2n 2n 2n
thus finally
{ }{ }{ }
θ z2 z2 z2
ez − 2 cos θ + e−z = 4 sin2 1+ 1+ 1+ ...
2 θ2 (2π ± θ)2 (4π ± θ)2

Let ι stand for −1; then we may put
θ + ιz θ − ιz
ez − 2 cos θ + e−z = 2(cos ιz − cos θ) = 4 sin sin :
2 2
APPENDIX : Miscellaneous Articles and Propositions 361

θ + ιz θ − ιz
it will be a useful exercise to resolve sin and sin into factors by Art. 320
2 2
(page 359), and to show that the result agrees with that which has just been obtained.
For θ put π + ϕ; thus we can obtain a formula for resolving ez + 2 cos ϕ + e−z into
factors.

Art. 328. The propositions which are given in Chapter ix. admit of some extensions
beyond the enunciations to which, for the sake of simplicity, we have there confined
ourselves. It will be sufficient if we consider only positive angles.
We have shown in Art. 116 (page 329) that sin θ is less than θ so long as θ is less
π
than ; it is obvious then that sin θ is less than θ for every value of θ.
2
Now consider Art. 120 (page 330). The demonstration there given depends on the
θ θ
fact that tan is greater than . Thus it is really shown that sin θ is algebraically
2 2 ( )
θ3 θ3
greater than θ − as long as θ is less than π; that is, sin θ − θ − is always
4 4
θ 3
positive. For we find, by calculation, that − θ is greater than unity when θ = π,
4
θ3
and it increases beyond this value : thus − θ + sin θ is always positive. And sin θ
4
θ 3
is arithmetically greater than θ − certainly as long as both are positive, that is
4
π
certainly up to θ = 2, which is beyond θ = .
2
Next consider Art. 121 (page 330). From that article combined with the extension
just given to Art. 116 (page 329), it follows that cos theta is always algebraically greater
θ2
than 1 − . And from Art. 121 (page 330) combined with the extension just given to
2 ( )2
θ2
Art. 120 (page 330), it follows that cos θ is algebraically less than 1 − , certainly
4
θ
as long as is less than 2 : hence it follows that cos θ is always algebraically less than
( )2 2
θ2 θ
1− , for this expression is greater than unity if is not less than 2. But cos θ
4 2
θ2 π
is not always arithmetically greater than 1 − , even if θ is less than . On the other
( 2 )2 2
θ2
hand cos θ is arithmetically less than 1 − , while θ lies between 0 and some
4
π
value which is greater than and less than π.
2
Now consider Art. 130 (page 331). In the same manner as we extended Art. 120
θ3
(page 330) we can show that sin θ is algebraically greater than θ − for every value
6
θ3 √
of θ, and that sin θ is arithmetically greater than θ − , certainly up to θ = 6. And
6
θ2 θ4 θ √
cos θ is algebraically less than 1 − + , certainly as long as is less than 6 :
2 24 2
θ2 θ4
hence it will follow that cos θ is always algebraically less than 1 − + . And cos θ
2 24
θ 2 θ 4 π
is arithmetically less than 1 − + , certainly while θ lies between 0 and ; for
2 24 2
θ2 θ4 √ √
1− + is positive throughout this range, and until θ = 6 − 12.
2 24

Art. 331. The following theorem is given for the sake of an important application :
EDH is a triangle having the sides ED and DH equal.
Produce DH to any point N ; and EH to a point I, such that EI 2 = 4DH · DN.
APPENDIX : Miscellaneous Articles and Propositions 362

Draw DM at right angles to DE, and IM parallel to DE. Then the circle which has
the centre I and the radius IM will touch the circle which has the centre N and the
radius N D.
Then DH = h, EI = i, DN = n; ∠DEH = θ.
Then EH = 2h cos θ; and from the triangle IHN
IN 2 = (i − 2h cos θ)2 + (n − h)2 − 2(n − h)(i − 2h cos θ) cos θ
= i2 + (n − h)2 − 2i(n + h) cos θ + 4nh cos2 θ
= (n + h)2 − 2i(n + h) cos θ + i2 cos2 θ
= (n + h − i cos θ)2 .

I
N

D
M

Thus IN = DN − IM ; therefore DN = IN + IM, which demonstrates the theorem.

Art. 332. The application of the preceding theorem which we propose to make is this
: the nine points circle of any triangle touches the inscribed circle and the escribed
circles of the triangle.
A

O
H
P

B C
D G

E
For an account of the nine points circle the student is referred to the Appendix to
Euclid, pages 317, 318, where the following theorems are demonstrated : ABC is a
triangle, and P is the intersection of the perpendiculars from A, B, C on the oppo-
site sides; the circle which passes through the middle points of P A, P B, P C passes
through the feet of the perpendiculars and through the middle points of the sides
of the triangle; the diameter of the nine points circle is equal to the radius of the
circumscribed circle of the triangle.
Let ABC be a triangle, O the centre of the circumscribed circle, D the middle point
of BC; let AG be perpendicular to BC, let P be the intersection of the perpendiculars,
APPENDIX : Miscellaneous Articles and Propositions 363

F the middle point of P A. Let OD be produced to meet the circumference of the


circumscribed circle at E; join OA, AE, and F D.
Since the nine points circle passes through D, F, G it follows that DF is a diameter;
c cos A
and therefore DF = OA. Also OD = AF, for it may shown that each = . Hence,
2 sin C
since the opposite sides of OAF D are equal, DF is parallel to OA. This if H be the
point of intersection of EA and F D we have ED = DH.
Suppose that in Art. 331 (page 361) the letters D, E, I indicate the same points as
in Art. 253 (page 345). Let
A A R
i = 2R sin , h = 2R sin2 ; then n = ;
2 2 2
thus N is the centre of the nine points circle of the triangle; and therefore the nine
points circle touches the inscribed circle.
Again, suppose that in Art. 331 (page 361) the letters D, E indicate the same points
as in Art. 253 (page 345); and let I now denote what was denoted by J in Art. 253
(page 345); then we see that the nine points circle touches the escribed circle which
is opposite the angle A.
Since the nine points circle of the triangle ABC passes through the middle points
of AB, BP, and P A, it is also the nine points circle of the triangle AP B; and so it
touches the inscribed and escribed circles of that triangle. A similar remark holds
with regard to the triangles BP C and CP A.

[Algebra] Art. 342. Some particular cases of the equation ax2 + bx + c = 0 may now
be investigated. The roots of the equation are
√ √
−b + b2 − 4ac −b − b2 − 4ac
and ;
2a 2a
we will first examine the results of supposing a = 0.
The numerator of the first root becomes −b + b, that is, 0; thus this root takes the
0
form . The numerator of the second root becomes −2b; thus this root takes the form
0
−2b c
. If in the original equation we put a = 0, it becomes bx + c = 0, so that x = − ; and
0 b
0
we may arrive at this result from the expression which takes the form by a suitable
0 √
−b + b2 − 4ac
transformation. For multiply both numerator and denominator of by
2a
√ −2c −2c
b + b2 − 4ac; thus we obtain √ , and if we now put a = 0, we obtain ,
√b + b − 4ac
2 2b
−c −b − b − 4ac
2
that is, . If the root be transformed by multiplying its numerator
b 2a
√ −2c
and denominator by b − b2 − 4ac it becomes √ , and the smaller a is the
b − b2 − 4ac
smaller is the denominator of this fraction, and the greater the fraction itself : and
equivalent result may obviously be obtained without effecting any transformation of
the root. Thus we may enunciate our results as follows : in the equation ax2 +bx+c = 0,
if a be very small compared with b and c, one root is very large and the other root is
c
nearly equal to − , and the smaller a is, the larger one root becomes, and the nearer
b
c
the other root approaches to − .
b

[Algebra] Art. 385. Suppose that we have three unknown quantities x, y, z connected
by the two equations
ax + by + cz = 0, a′ x + b′ y + c′ z = 0;
these equations are not sufficient to determine the unknown quantities, but they will
determine the ratios subsisting between them. For multiply the first equation by c′ ,
and the second by c, and subtract : thus
(ac′ − a′ c)x + (bc′ − b′ c)y = 0;
x y
therefore = ′ .
bc′ − b′ c ca − c′ a
APPENDIX : Miscellaneous Articles and Propositions 364

Again, multiply the first equation by b′ , and the second by b, and subtract : thus we
obtain
x z
= ′ .
bc′ − b′ c ab − a′ b
Hence we may write the results in this form :
x y z
= ′ = ′ .
bc′ − b′ c ca − c′ a ab − a′ b
These results are very important, and should be carefully remembered; the second
denominator may be derived from the first, and the third from the second.
Denote the common value of these fractions by k; then
x = k(bc′ − b′ c), y = k(ca′ − c′ a), z = k(ab′ − a′ b).
Now suppose that we have also a third equation connecting the unknown quantities
x, y, z; then by substituting in it for x, y, z; the expressions just given, we shall obtain
an equation which will determine k : thus the values of x, y, z become known.
Suppose, for example, the third equation is
lx2 + my 2 + nz 2 = 1,
then k is determined{ by }
k2 l(bc′ − b′ c)2 + m(ca′ − c′ a)2 + n(ab′ − a′ b)2 = 1.

[Algebra] Art. 526. Following is the third example which illustrates the use of the
Binomial Theorem.
Example (3) : Required approximate values of the roots of the quadratic equation
ax2 + bx + c = 0, when √
ac is very small compared with b2 .
−b ± b2 − 4ac
The roots are .
2a
And by the Binomial Theorem,
√ ( )1
4ac 2
b2 − 4ac = b 1 −
{ b2 }
( ) ( )
1 4ac 1 4ac 2 1 4ac 3
=b 1− − − − . . . . . . .
2 b2 8 b2 16 b2
Thus for the root with the upper sign we get
c ac2 2a2 c3
− − 3 − − ......
b b b5
and for the root with the lower sign we get
b c ac2 2a2 c3
− + + 3 + + ......
a b b b5
If a be very small, while b and c are not small, the former root does not differ much
c
from − , and the latter root is numerically very large. See Art. 342 (page 363).
b
It is deserving of notice that the approximate value of the root in the former case
coincides with what we shall obtain in the following way. Write the equation thus,
bx + c = −ax2 .
c
For an approximate result neglect the term ax2 as small; thus we obtain x = − . Then
b
substitute this approximate value of x in the term ax2 ; thus we obtain
ac2
bx + c = − 2 ,
b
c ac2
that is, x=− − 3 .
b b
Again, substitute this new approximate value of x in the term ax2 , and preserve the
terms involving a and a2 ; thus we obtain
ac2 2a2 c3
bx + c = − 2 − ,
b b4
c ac 2 2a2 c3
that is, x=− − 3 − ,
b b b5
and so on.

[Algebra] Art. 550. We will give another method of arriving at the exponential theo-
APPENDIX : Miscellaneous Articles and Propositions 365

rem. By the Binomial Theorem


( )
1 nx 1 nx(nx − 1) 1 nx(nx − 1)(nx − 2) 1
1+ = 1 + nx + +
n n 2 n2 3 n3
nx(nx − 1)(nx − 2)(nx − 3) 1
+
4 n4
+ ......
( ) ( )( )
1 1 2
( )nx x x− x x− x−
1 n n n
that is, 1+ =1+x+ +
n 2 3
( )( )( )
1 2 3
x x− x− x−
n n n
+ + ......
4
( )
1 n
Put x = 1, then 1 +
( n )( ) ( )( )( )
1 1 2 1 2 3
1− 1− 1− 1− 1− 1−
n + n n n n n
=1+1+ + + ...
2 3 4
( )nx {( )n }x
1 1
But 1+ = 1+ ;
n n
( ) ( )( )
1 1 2
x x− x x− x−
n n n
hence 1+x+ + + ......
2 3
 ( )( ) x
1 1 2
 1− 1− 1− 
n + n n
= 1+1+ + ......
 2 3 
Now this being true however large n may be, will be true when n is mae infinite; then
1
vanishes and we obtain
n { }x
x2 x3 x4 1 1 1
1+x+ + + + ...... = 1 + 1 + + + + ...... ,
2 3 4 2 3 4
that is, = ex .
We have thus obtained the expansion of ex in powers of x; to find the expansion of ax
suppose a = ec so that c = loge a, thus
c 2 x2 c 3 x3 c 4 x4
ax = ecx = 1 + cx + + + + ......
2 3 4

) We have found in Art. 550 (page 364), that when n increases with-
[Algebra](Art. 552.
1 nx
out limit 1 + ultimately becomes ex ; in the same way we may show that when
n ( )
r nx
n increases without limit 1 + ultimately becomes erx .
n
Th
is
pa
ge
is
in
te
nt
ion
al
ly
le
ft
bl
an
k .

Potrebbero piacerti anche